You are on page 1of 113

MATRIC HR. SEC.

SCHOOL
SIDDHAR KOVIL MAIN ROAD, SIVATHAPURAM, SALEM -636307

NEW
SYLLABUS

MATHEMATICS
STUDY MATERIAL
M.MANI., M.Sc.,B.Ed.,

S.MANIKANDAN., M.Sc.,B.Ed.,
DEPARTMENT OF MATHEMATICS
PV MATRIC HR SEC SCHOOL
SIDDHAR KOVIL MAIN ROAD, SIVATHAPURAM, SALEM -636307
P.V MATRIC HR. SEC. SCHOOL (iv) Let A be the set consisting of all the female members
of a family. The relation R defined by
MATHEMATICS - XI STD “aRb if a is not a sister of b”.
Solution : Given relations is “aRb if a is not a sister of b”.
let a,b,c ߳ A.
EXERCISE 1.2 Reflexivity: No person can be a sister of himself/herself.
aRa for all a ߳ A. ∴ ܴ is reflexive.
1. Discuss the following relations for reflexivity, Symmetricity: Given family will consisting only female
symmetricity and transitivity: members. So that if “a is not a sister of b” and also
(i) The relation R defined on the set of all positive “b also not a sister of a” i.e., aRb = bRa. ∴ ܴ is Symmetric.
integers by “mRn if m divides n”.
Solution : : Given relation is “mRn if m divides Transitivity: If “aRb if a is not a sister of b” and
n”. “bRc if a is not a sister of b”
Reflexivity: mRm since m divides m for all positive But “aRc a is not a sister of c” is need not be true.
integers m. ∴ ܴ is reflexive. [refer below example]
Symmetricity: If mRn, m divides n for all m≤n. i.e., aRb = bRc But aRc. ∴ ܴ is not
Then nRm, n cannot divides m for m≤n. transitive.
i.e., mRn ⇏ nRm. ∴ ܴ is Not Symmetric. ∴ ܴ is reflexive, Symmetric, and not
௡ ௡
Transitivity: if mRn ⟹ Let ௠ = ݇ ⇒ m = ௞ ; ----(1) transitive

if nRp ⟹ Let ௡ = ݈ ⇒ p = nl ;----(2) Illustrative Example for (iv) :
(ଶ) ௣ ௡௟ Mother : Sumathi
⇒ = ೙ = ݈݇
(ଵ) ௠

ೖ Daughters : Kala
i.e., =kl so that “m divides p” Mala

so that mRp also true. aRb : Kala is not a sister of Sumathi - True
i.e., m divides n and n divides p ⇒ m dives p. bRc : Sumathi is not a sister of mala - True
∴ ܴ is transitive. aRc : kala is not a sister of mala - Not True
∴ ܴ is reflexive, Not Symmetric, and transitive so it is not transitive.
(ii) Let P denote the set of all straight lines in a plane. (v) On the set of natural numbers the relation R defined
The relation R defined by “lRm if l is perpendicular tom”. by “xRy if x + 2y = 1”.
Solution : let l,m,n ߳ P Solution ; Where x+2y=1 i.e., ‫ = ݕ‬ଵି௫ if x߳ܰ , ‫ܰ ∉ ݕ‬

Reflexivity: A straight line is
So that R={} . R is an empty set.
not perpendicular to itself.
∴ ܴ is Not reflexive, Symmetric, and transitive.
∴ ܴ is not reflexive.
Symmetricity: 2. Let X = {a, b, c, d} and R = f(a, a), (b, b), (a, c)}. Write
if line l is perpendicular to m ⇒ m also perpendicular to l. down the minimum number of ordered pairs to be
i.e., If lRm ⇒ mRl. ∴ ܴ is Symmetric. included to R to make it (i) reflexive (ii) symmetric
Transitivity: If lRm and mRn but lRn (iii) transitive (iv) equivalence.
If l is perpendicular to m and m is perpendicular to n. Solution : Given X = {a, b, c, d} and R = f(a, a), (b, b), (a, c)}.
Then l and n are parallel to each other. i.e., mRn⇏ lRn . (i) Minimum number of ordered pairs are
∴ ܴ is not transitive. (c,c)&(d,d)
Should be included to R to make it reflexive.
∴ ܴ is not reflexive, Symmetric, and not transitive
(ii) (a,c) ߳R so that (c,a) should be included to R to
(iii) Let A be the set consisting of all the members of a make it symmetric.
family. The relation R defined by “aRb if a is not a sister (iii) Already R is transitive. Nothing to include with
of b”. R to make it transitive.
Solution : Given relations is “aRb if a is not a sister of b”. (iv) (c,c),(d,d) and (c,a) should be included to R to
let a,b,c ߳ A. make it equivalence.
Reflexivity: No person can be a sister of himself/herself. 3. Let A = {a, b, c} and R = {(a, a), (b, b), (a, c)}. Write
aRa for all a ߳ A. ∴ ܴ is reflexive. down the minimum number of ordered pairs to be
Symmetricity: included to R to make it
A family will consisting male and female members. (i) reflexive (ii) symmetric (iii) transitive (iv) equivalence
if “a is not a sister of b” ⇏ “b is not a sister of a” (i) Minimum number of ordered pairs are (c,c)
i.e., aRb ⇏ bRa. ∴ ܴ is not Symmetric. Should be included to R to make it reflexive.
Transitivity: If aRb then “a is not a sister of b” and (ii) (a,c) ߳R so that (c,a) should be included to R to
If bRc then “b is not a sister of c” make it symmetric.
aRc “a is not a sister of c” is not possible. (iii) Already R is transitive. Nothing to include with
i.e., aRb and bRc But aRc. ∴ ܴ is not transitive. R to make it transitive.
∴ ܴ is reflexive, Not Symmetric, and not transitive. (iv) (c,c) and (c,a) should be included to R to make it
equivalence.
4. Let P be the set of all triangles in a plane and R be the Reflexivity : (4,4)(5,5) ∉ ܴ.
relation defined on P as aRb if a is similar to b. Prove ∴ ܴ is not reflexive.
that R is an equivalence relation. Symmetricity : For every elements of
Solution : Let P be the set of all triangles in a plane and R be (1,2)(1,3)(1,4)(1,5)(2,3)(2,4)∈ ܴ. There exist
the relation defined on P as aRb if a is similar to b. (2,1)(3,1)(4,1)(5,1)(3,2)(4,2) ∈ R respectively.
Reflexivity: Since every triangle in P is similar to itself. ∴ ܴ is symmetric.
So aRa is possible. ∴ ܴ is reflexive. Transitivity : It is not transitive.
For example (4,1)(1,5) ∈ R but (4,5) ∉ ܴ
∴ R is not transitive .
Equivalence : R is not equivalence.
8. Let A = {a, b, c}. What is the equivalence relation of
Symmetricity: If a is similar to b. smallest cardinality on A? What is the
then b is similar to a. equivalence relation of largest cardinality on A?
i.e., aRb⇒ bRa . ∴ ܴ is Symmetric. Solution : Given A={a,b,c}.
(i) Let R = {(a,a)(b,b)(c,c)}
Where R is reflexive, symmetric and equivalence.
This is the equivalence relation of smallest
Transitivity: cardinality on A.
a is similar to b and b is similar to c. Where n(R)=3.
we know that a and c are similar. Let aRb and bRc then aRc. (ii) Let R=AXA
i.e., ∴ ܴ is transitive. R={(a,a)(a,b)(a,c)(b,a)(b,b)(b,c)(c,a)(c,b)c,c)}
∴ ܴ is reflexive, Symmetric, and transitive. ∴R is equivalence. Where R is reflexive, symmetric and equivalence.
This is the equivalence relation of largest
5. On the set of natural numbers let R be the relation cardinality on A.
defined by aRb if 2a + 3b = 30. Write down Where n(R)=9.
the relation by listing all the pairs. Check whether it is __________________________________________________
(i) reflexive (ii) symmetric (iii) transitive (iv) equivalence
Solution: Given relation is 2a+3b = 30 for all a,b ߳ N. 9. In the set Z of integers, define mRn if m - n is divisible
Where = ଶ .
ଷ଴ିଷ௕ by 7. Prove that R is an equivalence relation.
a 3 6 9 12 Solution:
b 8 6 4 2 Reflexivity : As m - m = 0 which is divisible by 7.,
hence mRm proving that R is reflexive.
Now R={(3,8)(6,6)(9,4)(12,2)} Symmetricity : Let mRn.Then m - n = 7k
Reflexivity : (3,3)(9,9)(12,12) ∉ ܴ. for some integer k; thus n - m = 7(-k) and hence nRm.
∴ ܴ is not reflexive. This shows that R is symmetric.
Symmetricity : (8,3)(4,9)(2,12) ∉ ܴ. ∴ ܴ is not symmetric.
Transitivity : There is no relation like aRb and bRc. Transitivity : if mRn, m-n=7k ⇒ m=7k+n
∴ R is transitive . if nRp, n-p=7l ⇒ p=n-7l
Equivalence : R is not equivalence. now m-p = (7k+n)-(n-7l)
6. Prove that the relation “friendship” is not an m-p = 7k+n-n+7l
equivalence relation on the set of all people in Chennai. m-p = 7(k+l). ∴mRp also Exist in R.
Solution : Let a,b,c are people in Chennai. ∴ R is transitive. Thus R is equivalence relation.
Reflexivity : a person cannot be a friend of himself/herself.
So that aRa. ∴ ܴ is not reflexive.
Symmetric : if “a is a friend of b” then clearly,
“b is a friend of a”.
i.e., aRb = bRa. ∴ ܴ is Symmetric .
Transitivity : a is friend of b also b is a friend of c. but
a need not to be a friend of c.
i.e.,Let aRb and bRc but aRc. ∴ R is not transitive.
So that R is not an equivalence set.
7. On the set of natural numbers let R be the relation
defined by aRb if a + b ≤ 6. Write down the
relation by listing all the pairs. Check whether it is
(i) reflexive (ii) symmetric (iii) transitive (iv) equivalence
Solution:The relation defined by aRb if a + b ≤ 6 for all a,b∈ ܰ
a 1 1 1 1 1 2 2 2 2 3 3 3 4 4 5
b 1 2 3 4 5 1 2 3 4 1 2 3 1 2 1
R={(1,1)(1,2)(1,3)(1,4)(1,5)(2,1)(2,2)(2,3)(2,4)(3,1)(3,2)(3,3)
(4,1)(4,2)(5,1)}
f: X →X ; f = {(x, y), (x, z), (z, x)};
P.V MATRIC HR. SEC. SCHOOL (ii) Let X = {x, y, z}

MATHEMATICS - XI STD Solution : Since the element x in domain


have two images namely y and z. also
EXERCISE 1.3 element y in domain not related to any
1. Suppose that 120 students are studying in 4 sections of element in codomain. It is not a function.
eleventh standard in a school. Let A denote the set of ___________________________________________________

from A → B for each of the following:


students and B denote the set of the sections. Define a 5.Let A = {1, 2, 3, 4} and B = {a, b, c, d}. Give a function
relation from A to B as “x related to y if the student x
belongs to the section y”. Is this relation a function? (i) neither one-to-one nor onto.
What can you say about the inverse relation? (ii) not one-to-one but onto.
Explain your answer. (iii) one-to-one but not onto. (iv) one-to-one and onto.
Solution: Solution :
Let A= set of students = {1,2,3,…..120} and n(A)=120 (i) neither one-to-one nor onto.
B=Set of sections = {P, Q, R, S} and n(B) = 4 Let f = {(1,a), (2,a), (3,a), (4,a)}
R: xRy⇒“if the student x belongs to the section y”. Clearly, f is not onto, as b, c and d does not have
(i) For every student in A, there corresponds a any pre image in A.
unique class in B. Hence the relation defined f is not an one to one function, as a has four pre
from A to B as “x is related to y if the student x images in A.

∴f is a function from A→B.


belongs to the section y” is a function.. (ii) not one-to-one but onto
It is possible to define a many to one onto function from a set A
(ii) The inverse relation is f-1 : B→A is not a function to B if n(A)>n(B). Since n(A)=n(B), it is not possible to
because every section in B will be related to more define function which is not one to one but onto.
than one student belongs to set A. (iii) one-to-one but not onto.
_______________________________________________ It is possible to define a one-to-one into function from a set A to


4   ∞   3
2.Write the values of f at -4,1,- B if n(A)<n(B). Here n(A)=n(B). Hence it is


4   3   2
not possible to define function which is one

to one but not onto.
2,7,0ifF(x)=    2 1

(iv)one-to-one and onto.
⎪   1 7
 Let f = {(1, a), (2, b), (3, c), (4, d)}
⎩ 0 ℎ 
∴f(-4) = 8
It is one to one but onto.

∴f(1)=0 %
Solution: f(-4) = -(-4) +4 = 8

f(-2)=(-2)2-(-2)=4+2=6 ∴f(-2)=6
f(1) = 1-(1)2=0 6.Find the domain of%&'()* +.

∴ f(7)=0 1  2 sin / 0
Where denominator is 0,

∴ f(0)=0
f(7)=0
f(0)=02-0=0
0 0
Sin x = ½


5  ∈ !∞, 0#
3. Write the values of f at -3 , 5 , 2 , - 1, 0 if Possible x = 1 (or) n2+(-1)n 1
 0
⎪ 
3  2  ∈ !3, ∞#
Domain is R-{n2+(-1)n 1 } ; n∈Z.

⎨  ∈ !0,2#
f(x)= 
⎪   3
7. Find the largest possible domain of the real valued
34&+'
⎩  $ℎ  function f(x)=
3+'&5
.

∴ f(-3) = 1
Solution: ( ) indicates the end points of the intervals are not Solution :

f(5)=52+3(5)-2 = 25+15-2 = 38 ∴ f(5) = 38 Since 4   is negative


exist. f(-3)=(-3)2+(-3)-5 = 9-3-5 =1 X values f(x)

∴ f(2) = 1 If x 2 and x >2


∴ √4   is imaginary
∴ f(-1) = -5
f(2)=(2)2-3 =4-3 = 1

∴ f(0) = -3
f(-1)=(-1)2+(-1)-5 = 1-1-5 = -5 If x=-2 and x =2 f(x)=0

Since +'  5 is negative


f(0)= 02-3 =-3 If x= 3 and x= -3 f(x) is not defined

∴ √+'  5 is imaginary
4.State whether the following relations are functions or not. If If x>-3 and x <3
it is a function check for one-to-oneness

(i) If A = {a, b, c} and f = {(a, c), (b, c), (c, b)}; (f : A →A).
andontoness. If it is not a function, state why?
∴ Domain R is Empty set (or) ∅.
For no real values of x , f(x) is defined.
(ii) If X = {x, y, z} and f = {(x, y), (x, z), (z, x)}; (f : X → X). %
8.Find the range of the function' 89( +&%.
(i) Let A={a,b,c}, f: A →A ; f = {(a, c), (b, c), (c, b)}; 1  :$  1
Solution:

2  2:$  2 [∵ <=> ?>@ A@ 2


3  2:$  1  1 [∵ B=AC:D A@ 1
 G G1
E E E
F HIJK&E HIJK&E
f is a function.
LCM NO P: ?$:C> $N A$ℎ  D
and [∵
But f is not a one to one function as the
Range is Q∞,  R ST1, ∞#
image ‘c’ has two pre images ‘b’ and ‘c’. E
And f is not an onto function as the image F
‘a’ has no pre-image.
9. Show that the relation xy = -2 is a function for a 14.The distance of an object falling is a function of time t
suitable domain. Find the domain and the range and can be expressed as s(t) = -16t2.

Solution : Let U / V!+# / . Then f(x) is defined for all real


&'
of the function. Graph the function and determine if it is one-to-one.
+
Solution : s(t) = -16t2
numbers except 0. Hence the domain of f(x) is the set of all non Let s(t1) = -16t12
Let s(t2) = -16t22
∴ Suitable Domain is R-{0}.
zero-real numbers.
Suppose s(t1) = s(t2)
-16t12= -16t22

10.If f,g : R → R are defined by f(x) = |x|+ x and


Also Range is R-{0}.
⇒t1= t2ort1= - t2
t12= t22

g(x) = |x|- x, find g o f and f o g. B N:  < :CNN$ A NOC ].Take t1= t2
Solution :f(x) = |x|+x Hence it is one to one.
That is f(x) = -x+x = o if x 0
g(x)=|x|-x
t 0 1 2 3 …
0   0
= x+x = 2x if xG0 Now f(x) = W
s 0 -16 -64 -144 …
2 G 0

g(x)=-x-x = = -2x if x 0
2   0
Now g(x) = W
0 G0
= x-x = 0 if xG0
Let  0
Then fog(x)=f(g(x))=f(-2x)= 0

Let G 0
Also gof(x) = g(f(x))=g(0)=-2(0)=0

Then fog(x)=f(g(x))=f(0)=2(0)=0
Also gof(x) = g(f(x))=g(2x)=0 for all x∈R.
11.If f, g, h are real valued functions defined on R,
then prove that (f +g) o h = f o h+g o h.
What can you say about f o (g + h)? Justify your answer.
Solution :
To Prove :(f +g) o h = f o h + g o h.
Proof : L.H.S (f +g) o h :
[(f+g)oh](x) = f+g(h(x)) 15. The total cost of airfare on a given route is comprised
= f(h(x))+g(h(x)) of the base cost C and the fuel surcharge S in rupee. Both
=foh(x) +goh(x) C and S are functions of the mileage m; C(m) = 0.4m + 50
(f+g)oh = foh + gof and S(m) = 0.03m. Determine a function for the total cost
of a ticket in terms of the mileage and find the airfare for
Now fo(g+h)(x) = f((g+h)(x)) flying 1600 miles.

X f(g(x))+f(h(x))
= f(g(x)+h(x)) Solution :Given cost function and fuel surcharge function are

X fog(x) + foh(x)
as follows: C(m) = 0.4m + 50 and S(m) = 0.03m.
Total cost of a ticket T(m) = c(m) + S(m)
12.Iff :R→R is defined by f(x) = 3x - 5, prove that f is a T(m) = 0.4m + 50 + 0.03m
bijection and find its inverse. T(m) = 0.43 m + 50
Given m = 1600
Let y = 3x-5 ⟹ y+5 = 3x ⟹ /
Solution :f(x)= 3x-5
Z[\ T(1600) = 0.43 (1600) + 50
F
Z[\
= Rs. 738
F
Now g(y)=
#  5= y
Z[\ Z[\
16.A salesperson whose annual earnings can be
F F
fog(y)=f(g(y))=f( )=3( represented by the function A(x) = 30,000+0.04x,
/ F /
FK&\[\ FK where x is the rupee value of the merchandise he sells.
gof(x) = g(f(x)) = g(3x-5) = F
His son is also in sales and his earnings are represented
Thus fog = Iyand gof=Ix . Hence by the function S(x) = 25,000 + 0.05x. Find (A + S)(x) and
Z[\
f and g are bijection and inverse to each other. f is bijective. determine the total family income if they each sell
F
Hence f is bijection and f-1 (y) = . Rupees 1,50,00,000 worth of merchandise.
K[\ Solution :Given A(x) = 30,000+0.04x, and
F
Replacing y by x we get f-1 (x) = .
S(x) = 25,000 + 0.05x
13.The weight of the muscles of a man is a function of his (A+S)(x)= A(x)+S(x)
body weight x and can be expressed as = 30,000+0.04x + 25,000 + 0.05x
W(x) = 0:35x. Determine the domain of this function. (A+S)(x) = 55,000 + 0.09x
Solution : x is the body weight of a man. Given x = 1,50,00,000
Here body weight of a man is not zero. (A+S)(x) = 55,000 + 0.09(1,50,00,000)

∴ Domain is x>0 ; ie., (0,∞).


For all x>0 ; W(x) = 0:35x is defined. =55,000+1,350,000
Total Family Income = 1,405,000.
17.The function for exchanging American dollars for
Singapore Dollar on a given day is f(x) = 1.23x, where x 20.A simple cipher takes a number and codes it, using the
represents the number of American dollars. On the same function f(x) = 3x-4. Find the inverse of this function,
day the function for exchanging Singapore Dollar to determine whether the inverse is also a function and
Indian Rupee is g(y) = 50.50y, where y represents the verify the symmetrical property about the line y = x

Solution :f(x) = 3x-4⇒ y = 3x-4 ⇒ y+4=3x


number of Singapore dollars. Write a function which will (by drawing the lines).

Z[m
give the exchange rate of American dollars in
terms of Indian rupee. x= F
Solution :Given x is number of American dollor. K[m
F
f-1(x)= which is also a function
f(x) is Singapore dollor.
f(x)=1.23x, Let y= 3x-4
x 0 1 2 -1 -2
y -4 -1 2 -7 -10
K[m
y is the Singapore dollor.
F
g(y) is Indian rupee. Let y=
g(y)=50.50y x 0 1 2 -1 -2
y 4/3 5/3 2 1 2/3
Now The function which will give the exchange rate of
American dollars in Indian rupee.
gof(x)= g(f(x))
= g(1.23x)
= 50.50(1.23x) = 62.115x
gof(x)=62.115x
Where x is American dollar, gof(x) is Indian Rupee.
18.The owner of a small restaurant can prepare a
particular meal at a cost of Rupees 100. He estimates
that if the menu price of the meal is x rupees, then the
number of customers who will order that meal at that
price in an evening is given by the function D(x) = 200-x.
Express his day revenue, total cost and profit on this
meal as functions of x.
Solution :
Number of customers = 200 – x
Cost of one meal = Rs.100
Total Cost = Rs. 100(200-x)
Revenue on one meal = x
Total revenue = x ( 200 –x)
Profit = Revenue - cost
= x(200-x) – 100(200-x)
=Rs. (200-x)(x-100)

temperatures is y=  . Find the inverse of this


19. The formula for converting from Fahrenheit to Celsius
_+ %`a
It is symmetric with respect to y=x.
5 5
function and determine whether the inverse is also a
function.

y= b  b ⇒
Solution :
\K E1c \K&E1c
b
⇒f(x) =
\K&E1c _+&%`a
b 5
Where y =
⇒ 9y = 5x-160 ⇒ 9y+160= 5x
\K&E1c
b
y=
⟹g(y)=
bZ[E1c 5U[%`a
\ _
x=
defghi
b! #[E1c
#= /
\K&E1c j \K
b \ \
gof(x) = g(f(x))= g( =
jklghi
\! #&E1c
#= / /@
bZ[E1c d \Z
\ b \
fog(x)=f(g(x))=f(
gof(x)= Ix and fog(x)=Iy
This implies that f and g are bijections, and inverses to each
5+[%`a
other. f -1(x)= _
⟹f -1(x)=
32.
bK
\
PV MATRIC HIGHER SECONDARY SCHOOL
XI STD – MATHEMATICS - EXERCISE 1.4
1. For the curve y = x3 given figure (i)    (ii)     
and draw the following X -2 -1 0 1 2 X -2 -1 0 1 2
y 8 1 0 -1 -8 y -7 0 1 2 9
(i)   

(ii)     

(iii)     

(iv)      with same scale.

   is the reflection of the      this is the graph of f(x)


graph of f about the x-axis. shifts to the upward for one unit.
(iii)      (iv)     
X -2 -1 0 1 2 X -2 -1 0 1 2
y -9 -2 -1 0 7 y -1 0 1 8 27

     this is the graph of f(x)       it causes the graph of


shifts to the downward for one unit. y=f(x) shift to the left by one unit.
2. For the given curve y=/ try to (i)     
 
(ii)      
draw the following graphs.
 x -1 0 1 x -1 0 1
(i)      y 1 0 -1 y 0 1 2

(ii)      

(iii)      

(iv)      


     ,It is the reflection of

      it causes the graph of

    about the x axis.

    the shift upwards by a unit.
 
(iii)       (iv)      
x -1 0 1 x -1 0 1
y 1 0 -1 y 0 1 2


     , it causes the graph 
      it causes the graph f(x)
 shift downwards by one unit. Shifts to the left by one unit.
3. Graph the functions f(x) = x3 and g(x) = √ on the same coordinate plane. Find f o g and graph it on the plane as
well. Explain your results.
Solution :          

    
 

   √
      /
 
 
  x
Where      so that (i)  is bijective
(ii) both f(x) and g(x) also bijective.
(iii) f and g are symmetrical about y=x.
__________________________________________________________________________________________________________
4. Write the steps to obtain the graph of the function y=      from the graph    .
Solution :
Step 1 Step 2 Step 3 Step 4
Draw the graph of    y=    causes the shift y=    stretches y=      causes the
to the right to the one towards y axis since shift to the left by 5 units.
unit. multiply factor is 3, which
greater than 1.


5. From the curve y = sin x, graph the functions (i) y = sin(-x) (ii) y = - sin(-x) (iii) y= sin(  


(iv) sin(   which is also cosx ( refer trigonometry )

Solution :
(i) y = sin(-x) (ii) y = - sin(-x)
   3    3
X -2 - - - 0  2 X -2 - - - 0  2
  2 2   2 2
y 0 -1 0 1 0 -1 0 1 0 y 0 1 0 -1 0 1 0 -1 0

 
(iii) y = sin(    (iv) y=sin(    (or) y = cos x
   3    3
X -2 - - - 0  2 X -2 - - - 0  2
 2 2  2 2
y -1 0 1 0 1 0 -1 0 y 1 0 -1 0 1 0 -1 0 1

6. From the curve y = x, draw (i) y = -x (ii) y = 2x (iii) y = x + 1 (iv) y= x+1 (v) 2x + y + 3 = 0.
y=x (i) y = -x (ii) y = 2x

(iii) y = x + 1  (v) 2x + y + 3 = 0.
(iv) y= x+1


7.From the curve y = |x|, draw (i) y = |x – 1| + 1 (ii) y = |x + 1| - 1 (iii) y = |x + 2| - 3.


y = |x|, (i) y = |x – 1| + 1
X -2 -1 0 1 2 X -2 -1 0 1 2
y 2 1 0 1 2 y 4 1 2 1 2

  |  1| !ℎ#$% %& %ℎ' (#)ℎ%


  |  1|  1 !ℎ#$%! *+,-(.!
(ii) y = |x + 1| - 1 (iii) y = |x + 2| - 3.
X -2 -1 0 1 2 X -2 -1 0 1 2
y -2 1 0 1 2 y -3 -2 -1 0 1

  |  1| !ℎ#$% %& %ℎ' /'$%


  |  1|  1 !ℎ#$%! .&,0,-(.!
  |  2| !ℎ#$% %& %ℎ' /'$%
  |  2|  3 !ℎ#$%! .&,0,-(.!

8.From the curve y = sin x, draw y = sin |x| (Hint: sin(-x) = -sinx).
y = sin x
   3
X -2 - - - 0  2
 2 2
y 0 1 0 -1 0 1 0 -1 0

 2  3 4 627  2  3 4
Now y = Sin |x| ; We know that |x|= 1 Now Sin|x|=1
 2  5 4 827 2  5 4

 #$  3 0
y = sin |x| Now y=Sin|x|; We know that |x|= 1
 #$  5 0
X -2 -

- -

0


3
2
:#0  #$  3 0
2 Now Sinx =1
  2 :#0 #$  5 0
y 0 -1 0 1 0 1 0 -1 0

PV MATRIC HR. SEC. SCHOOL 5. Find a positive number smaller than
 . Justify.
Solution :
MATHEMATICS - XI STD 1000 < 1001
2AAA < 2AA
 
BCCC
> BCCB
EXERCISE 2.1 Hence a positive number smaller than

is



BCCC BCCB
1. Classify each element of {√, , , . , , }as a
 
member of N,Q,R-Q or Z. EXERCISE 2.2
Solution : 1.solve for x
 √7 ∈  −  (i) | − F| < 

 ∈ Solution :

 0 ∈ ,  |3-x|< 7
 3.14 ∈  -7<3-x<7
 4 ∈ , ,  -7-3<-x<7-3

  ∈ 
-10<-x<4
 10>x>-4 [∵ HIJK LJM ;M − 1
2. Prove that √ is an irrational number. ⇒ -4 < x < 10
Solution : (ii) |F − N| ≥ −

Suppose that √ is a rational number. Solution :


! where m and n are positive integers with |4P − 5| ≥ −2
Let √ = #
" no common factors greater than 1. n ≠ 0 For all the values of x, |4P − 5| is positive.
6 i.e., ∀ P> |4P − 5| ≥ −2
√3 =
7 P> S JJ TUK TVM KℎX U;YX 7XZIUK Y7.
√37 = 6  
(iii) [ −  F[ ≤ 
37 = 6 ---------------- (1)
Since 37 is divisible by 3,it shows that 6 is multiple of 3.
 Solution :
] 
Also m is multiple of 3. So that m = 3k (k is any constant ) [3 − P[ ≤
 
(1) ⇒ 37 = 39 
- ≤ 3− P ≤
] 

37 = 99  


]


7 = 39  - − 3 ≤ −P ≤ 
−3
] ] 
Since 39  is divisible by 3,it shows that 7 is multiple of 3. - ≤ −P ≤
 
n also multiple of 3. So that n = 3l (l is any constant ) ]  
- ≤ −P ≤ [∵ HIJK LJM ;M
It follows, that m and n are having a common factor 3. ] ] ]
] 
Thus, we arrived at a contradiction. ]
≥ P ≥ ]
[∵ HIJK LJM ;M − 1
 
Hence, √3 is an irrational number. ≤ F ≤
 
3. Are there two distinct irrational numbers such that (iv) |F| −  < −
their difference is a rational number? Justify. Solution :
Solution : |x| − 10 < −3
Let a+√; & < + √; > R-Q |P| < −3 + 10
|P|<7
Difference = (a+√;) - (< + √;) -7< x < 7

= a+√; – c-√; = a-c > Q 2.Solve < _ and express the solution using the
|
F|
So that difference between two distinct irrational numbers interval notation.
may be a rational number. Solution :

4. Find two irrational numbers such that their sum is a <6
|`|
rational number. Can you find two irrational numbers 
whose product is a rational number. |2P − 1| >
b
 
Solution : 2P − 1 > b 2P − 1 < b
Let 4+√2 & 4 − √2 > R-Q  
2P > + 1 2P < +1
(i)
Sum = (4+√2) + (4 − √2) b
 c
b
2P > b 2P < b
= 4+√2 + 4 − √2 = 8 > Q So that sum of two
 c
distinct irrational numbers may be a rational numbers. P> P<
 
(i) Let 4+√2 & 4 − √2 > R-Q
Product = (4+√2) (4 − √2)
= 4 − √2 = 16-4=12 > Q
So that Product of two distinct irrational numbers may be N 
a rational numbers. F ∈ d−∞, fg d , ∞f



3. Solve -3|x|+5≤ −
and graph the solution set in a 2.Solve 23x < 100 when (i) x is a natural number,
number line. (ii) x is an integer.
Solution : Solution:
-3|x|+5≤ −2 23x < 100
AA
-3|x|≤ −2-5 x<
]
-3|x|≤ −7 (i) If x is a natural number x= { 1,2,3, 4}
3|x|≥ 7 [∵ HIJK LJM ;M − 1 (ii) If x is an integer x ={ ….-4,-3,-2,-1,0,1,2,3,4}

|x|≥ ] ___________________________________________________
 
P ≥ (or) P ≤ 3. Solve -2x ≥ 9 when (i) x is a real number,
] ]
(ii) x is an integer, (iii) x is a natural number.
Solution:
−∞ −
7
0
7
∞ -2P ≥ 9
3 3
  2x ≤ -9
F ∈ d−∞, − h g i , ∞f s
  x≤-
4. Solve 2|x+1|-6≤  and graph the solution set in a s
(i) If x is real number x∈ −∞, − h
number line. 
Solution : (ii) If x is an integer x={…..-7,-6,-5}
2|x+1|-6≤ 7 (iii) If x is a natural number x has no solution.
2|x+1|≤ 7+6 4. Solve:
2|x+1|≤ 13  F

(i) N ≤ 
N
F
]
|x+1|≤  Solution:
] ]
- ≤ x+1 ≤ ⇒ 9 P − 2 ≤ 25 2 − P
 
] ] ⇒ 9P − 18 ≤ 50 − 25P
- −1≤x≤ −1 ⇒ 34P ≤ 68
 
c 
- ≤x≤ ⇒ 34P ≤ 68
 
P ≤2 Ans: x u −∞,
]
NF F
15 11 (ii)  <
− 
−∞ − 0 ∞
2 2 Solution:
−15 11 ⇒ 2 5 − P < 3P − 24
F∈ j , k
2 2 ⇒ 10 − 2P < 3P − 24

5.Solve N |F −
| < . ⇒ 34 < 5P
⇒ 5P > 34
Solution : ] 
 ⇒x> Ans: x u , ∞
|10P − 2| < 1 c N
c
|10P − 2| < 5 5. To secure A grade one must obtain an average of 90
-5< 10x-2 < 5 marks or more in 5 subjects each of maximum
-5+2< 10x < 5+2 100 marks. If one scored 84, 87, 95, 91 in first four
-3< 10x < 7 subjects, what is the minimum mark one scored
]  in the fifth subject to get A grade in the course?
- <x<
A A Solution:
Marks in first four subjects are 84, 87, 95, 91
3 7 Let Marks in fifth subject be “x”.
−∞ − 0 ∞ vwvw scw s ]cw`
10 10 Average mark = =
c c
From given data required inequation to score A grade:
6. Solve |NF − 
| < −
]cw`
⟹ ≥ 90
Solution : c
|5P − 12| < −2 357+x ≥ 450
Absolute value cannot be in negative. X ≥ 450 – 357
So x does not have any suitable solution to satisfy the X ≥ 93
above inequation. X has no solution. Minimum marks to score in fifth subject for getting A
Grade is 93.
EXERCISE 2.3 6.A manufacturer has 600 litres of a 12 percent solution
1. Represent the following inequalities in the interval of acid. How many litres of a 30 percent acid solution
notation: must be added to it so that the acid content in the
Question Interval Notation resulting mixture will be more than 15 percent but less
l F ≥ − l"m F <  P ∈ [−1,4 than 18 percent?
(b) F ≤ N l"m F ≥ − P ∈ [−3,5] Solution:
(c) F < − pq F <  P ∈ −∞, 3 Initial Amount of Solution = 600 litres.
m −
F >  pq F −  <  % of Acid present in initial solution = 12%
P < 0 Yr 3P < 15 P ∈ −∞, 5 
Amount of acid present in initial solution = AA y 600
x<5 = 72 litres.
.
Let Amount of mixing solution = x liters 9. A plumber can be paid according to the following
% Acid present in mixing solution = 30% schemes: In the first scheme he will be paid rupees 500
]A ]A `
Amount of acid present in mixing solution = yP= plus rupees 70 per hour, and in the second scheme he
AA AA
will paid rupees 120 per hour. If he works x hours, then
for what value of x does the first scheme give better
Total Amount of Solution = 600 + x
]A ` wages?
Total Amount of Acid present = 72 +
AA Solution:
Let no of working hours be “x”.
Total % Acid present in mixture solution : From given data,
z{|}~ €{‚| {ƒ „…† ‡ˆ‰Š‰‚|
= y 100 First Scheme : 500+70x
z{|}~ €{‚| {ƒ ‹{~|…{‚
ŒC  Second Scheme : 120x
 w
= BCC
bAA w Ž
y 100 Required linear inequation is
AAw]A` 500+70x > 120x
=
bAAw`
500 > 120x – 70x
From given data
AAw]A` 500 > 50 x
15< < 18
bAAw` 10 > x
For less than 10 working hours, first scheme gives better
Case (i): Case (ii) wages.
AAw]A` AAw]A`
15< bAAw` bAAw`
< 18 10. A and B are working on similar jobs but their annual
15(600+x) < 7200+30x 7200+30x < 18(600+x) salaries differ by more than Rs 6000. If B earns rupees
9000+15x < 7200+ 30x 7200+30x < 10800+18x 27000 per month, then what are the possibilities of A’s
9000-7200 < 30x-15x 30x – 18 x < 10800- 7200 salary per month?
1800 < 15 x 12 x < 3600 Solution:
120 < x x < 300 B’s Monthly Salary = 27000
Let A’s Monthly Salary = x
Hence x is must be 120 < x < 300 . Difference in Annual Income = 6000 Rs
bAAA
Difference in monthly salary = = 500

7.Find all pairs of consecutive odd natural numbers both Required linear inequation is
of which are larger than 10 and their sum is less than 40. |A’s Salary-B’s Salary|> 500
Solution: |x-27000|> 500
Let x and x+2 are the consecutive odd numbers. x – 27000 > 500 x-27000<-500
It is given that both natural numbers are greater than 10. x>27500 x<26500
x > 10 ----------------------------- (1) Possible Salary of A is either less than 26500 or more than
also given that their sum is less than 40. 27500 . Which gives more than 6000 difference in annual
i.e., x + ( x+2 ) < 40 income.
2x + 2 < 40
2x < 38 EXERCISE 2.4
x < 19 ------------------- (2) 1.Construct a quadratic equation with roots 7 and -3:
from (1) and (2) Solution:
10 < x < 19 Method 1 : Factors of quadratic equation is (x-7) & (x+3)
Possible Values of x is 11 , 13 , 15 , 17 Required quadratic Equation is (x-7)(x+3)=0
Respective x+1 values are 13, 15 , 17 , 19 X2-7x+3x-21=0
The required pairs of consecutive odd numbers are X2-4x-21=0
(11,13) , (13,15) , (15,17) and (17,19). Method 2: Sum of factors = 7+(-3) = 4
8. A model rocket is launched from the ground. The Product of factors = 7 x (-3) = -21
height h reached by the rocket after t seconds from lift Required quadratic Equation is x2-(S.R)x+P.R=0
off is given by h(t) = -5t2+100t; 0 ≤ t ≤ 20. At what time X2-4x-21=0
the rocket is 495 feet above the ground? 2. A quadratic polynomial has one of its zeros 1+√N and
Solution: it satisfies p(1) = 2. Find the quadratic polynomial.
From give data , Solution:
h(t) > 495 Always irrational roots occur in conjugate pairs.
(t-9) (t-11) 5t -100t+495
2
-5t2+100t > 495 If 1+√5 is a root then 1-√5 also a root to the equation.
-5t2+100t-495 > 0 t<9 - - +
Sum of roots = 1+√5 + 1-√5 = 2
5t2-100t+495 < 0 9<t<11 + - -
Product of roots = (1+√5)(1-√5) = 1-5 = -4
t2- 20t + 99 < 0 t>11 + + + Required Quadratic polynomial
(t-9) (t-11) < 0
P(x) = k(x2-(S.R)x+P.R)
P(x) = k(x2-2x-4) ------------- (1)
At 9<t<11 the rocket is 495 feet above the ground
Also given P(1)= 2

P(1) = k((1)2-2(1)-4) = 2 ⟹ k = −
c

(1) ⟹ P(x) = − c (x2-2x-4)
3. If  and  are the roots of the quadratic equation 6. Find the condition that one of the roots of
F
+ √
F +  = , form a quadratic polynomial with ax2 + bx + c may be (i) negative of the other,
 
zeroes  ,  . (ii) thrice the other, (iii) reciprocal of the other
Solution:
Solution: Let ax2 + bx + c =0
‘ and ’ are the roots of the quadratic equation (i) Let the roots negative of other.
P  + √2P + 3 = 0. i.e., the roots ‘ and - ‘.
Now ‘ + ’ = - √2 Now Sum = ‘ + −‘ = ™

‘’=3 š
0= ⇒ ;=0
Required quadratic polynomial is : ™
   
x2-( + )x+d . f = 0 (ii) Let the roots is thrice the other.
“w” 
i.e., the roots ‘ and 3 ‘.
x2-( )x+d f=0 Now Sum = ‘ + 3‘ =

” ”
™
 √  √  š
x2-( )x+d f = 0 ⟹ P + x + d f = 0 4‘ =
    ™

3P + √2P + 1 = 0.

‘=

›
4. If one root of k(x - 1)2 = 5x - 7 is double the other root, Product = ‘ 3‘ =
™
›
show that k = 2 or -25. 3‘  =
Solution: ™
š  ›
k(x - 1)2 = 5x – 7 3 d ™ f = ™
]š ˜ › ]š˜
= ⟹ =<
2
k(x -2x+1)-5x+7=0
b™˜ ™ b™
kx2-2kx+k-5x+7=0
kx2-(2k+5)x+(k+7)=0 ⟹ 3; = 16U<


from given data if ‘ is a root then 2 ‘ is another root of (iii) Let the roots is reciprocal of the other.

kx2-(2k+5)x+(k+7)=0 i.e., the roots ‘ and .
“
 › ›
Where Sum of roots ‘ + 2 ‘ = 2k+5 Product = ‘ x “ = ™ ⟹ 1 = ™ ⟹ < = U
3 ‘ = 2k+5
•wc 7. If the equations x2 -ax+b = 0 and x2 -ex+f = 0 have one
‘= ]–
---------------(1) root in common and if the second equation has equal
–w
Also product of the roots ‘.(2‘ = roots, then prove that ae = 2(b + f).
–
–w Solution:
2‘  = – x2 -ax+b = 0 ---------(I) x2 -ex+f = 0----------(II)
•wc  –w
2 = [∵ —rY6 1 ‘ & ’ are the roots of (I) ‘ & ‘ are the roots of (II)
]– –
– ˜ wA–wc –w Where ‘ is the common
2 s– ˜
= –
Where roots of (II) are
–w root of (I) & (II) equal
2 49 + 209 + 25 = 99  d – f

Sum = ‘ + ’ = a --------(1) Sum ‘ + ‘ = X
89  + 409 + 50 = 99  + 639 ⟹ 2‘ = X -------------(2)
−9  − 239 + 50 = 0 Product = ‘ ’ = b ---- (3) Product ‘ . ‘ = V
9  + 239 − 50 = 0 2 ‘ = f -----(4)
(k-2)(k+25) =0 To prove : ae = 2(b + f)
k=2 or k = -25 Hence proved. LHS = ae = (‘ + ’) 2‘ [∵ —rY6 1& 2
5.If the difference of the roots of the equation =2(‘  + ‘’
2x2 - (a + 1)x + a - 1 = 0 is equal to their product, then = 2(b+f) [∵ —rY6 3& 4
prove that a = 2. = R.H.S Hence Proved.
Solution:
8.Discuss the nature of roots of (i) -x2 + 3x + 1 = 0,
‘ and ’ are the roots of the quadratic equation
2x2 - (a + 1)x + a - 1 = 0 (ii) 4x2 - x - 2 = 0, (iii) 9x2 + 5x = 0.
™w
Here ‘ + ’ =  Solution:
™
Values 
− lž Nature
‘’ = of a,b,c & Sign Of roots

From given data a =-1 3 − 4 −1 1
=9+4=13 Real &
Difference of the roots = product of the roots (i)-x2 + 3x + 1 = 0 b =3
‘-’=‘’ Positive Distinct
i.e., c =1
Now ‘ − ’ = ‘’ a =4 −1 
− 4 4 −2
‘ + ’ − 4‘’= ‘’ 2
(ii)4x - x - 2 = 0 b =-1 =1+32=33 Real &
™ 
™w  ™ Positive Distinct
 − 4d f= d f c =-2
  
a =9 5
− 4 9 0
™˜ w™w ™˜ ™w

− 2 U − 1 = 
2
(iii)9x + 5x = 0 b =5 =25 Real &
Positive Distinct
Multiply by 4 c =0
U + 2U + 1 − 8 U − 1 = U − 2U + 1
U + 2U + 1 − 8U + 8 = U − 2U + 1
-4a+8 = 0 ⟹ U = 2 Hence proved.
9.Without sketching the graphs, find whether the graphs
of the following functions will intersect the x-axis and if
EXERCISE 2.6
so in how many points. 01.Find the zeros of the polynomial function
(i) y = x2 + x + 2, (ii) y = x2 - 3x - 7, (iii) y = x2 + 6x + 9. f(x) = 4x2 - 25.
Solution: Solution:
Results on f(x) = 4x2 - 25 =0

− lž
Values
of Intersection 2P − 5 = 0
a,b,c
& Sign (2x-5)(2x+5) = 0
Of x axis c c
a = 1 1 − 4 1 2 Zeros of the polynomials are x= and x=  .
=1-8=-7 does not
(i) y = x2 + x + 2 b =1 02. If x = -2 is one root of x3 - x2 - 17x = 22, then find the
Negative meet x-axis
c =2 other roots of equation.

a =1 −3 − 4 1 −7 intersects x- Solution:
(ii)y = x2 - 3x - 7 b =-3 =9+28=37 axis at two By the synthetic division
Positive −2 1 −1 −17 −22
c =-7 points [
a =1 6 − 4 1 9 touches x- 0 −2 6 22
(iii)y = x2 + 6x + 9 b =6 =36-36=0 axis at one 1 -3 -11 ⌊ 0
c =9 zero point Q=P  − 3P − 11 = 0
]±√sw ]±√c]
10.Write f(x)=x2+5x+4 in complete square form. X= 
= 
Solution: ]w√c] ]√c]
f(x) = x2+5x+4 Other two roots are and
 
c  c 
= x2+5x+d f − d f +4 03. Find the real roots of x4 = 16.
  Solution:
c  c
= dP + f − 
+4 x4 – 16 = 0
c  s P   − 4 = 0
= dP + f −  P  − 4 P  + 4 = 0
c  ]  (x+2) P − 2 P  + 4 = 0
= dP + f − df
X=-2 , x = 2 other two roots are complex roots.
EXERCISE 2.5 04. Solve (2x + 1)2 - (3x + 2)2 = 0
1.Solve + x - 15 ≤ 0.
2x2 Solution:
2x2 + x - 15 ≤ 0 (2x + 1)2 - (3x + 2)2 = 0
(X+3)(2X-5) ≤ 0 {(2x + 1)+(3x + 2)} {(2x + 1)-(3x + 2)} = 0
The critical points are -3 and 5/2 . [∵ U − ;  = U + ; U − ;
Sign of Sign of Sign of (5x+3) (-x-1) = 0
Interval ]
(x+3) (2x-5) 2x2 + x - 15 X=− and x=-1 are the required roots.
c
(-∞, −3 - - + ive ___________________________________________________
c
(-3,  + - - ive EXERCISE 2.7
c
( , ∞ + + + ive 1. Factorize: x4 + 1. (Hint: Try completing the square.)

Solution:
x4 + 1 = 0
−∞ ⟵ + X ⟶ −3 0
5
⟵ + X ⟶ ∞ P   + 2P  + 1 − 2P  = 0
2 
⟵ − X ⟶
P  + 1 − £√2P ¤ = 0
c (P  + 1 + √2P £P  + 1 − √2P¤ = 0
X ∈ i−3, h is satisfies 2x2 + x - 15 ≤ 0
(P  + √2P + 1 £P  − √2P + 1¤ = 0
2. Solve –x2+3x-2 ≥0 2. If x2 + x + 1 is a factor of the polynomial
Solution: 3x3 + 8x2 + 8x + a then find the value of a.
–x2+3x-2 ≥0 Solution:
x2-3x+2 ≤0 F + N
(x-1)(x-2) ≤0
F
+ ¥ + ⌋ F + §F
+ §F + l
Critical number are 1, 2
F + F
+ F
Sign of Sign of
Interval Sign of (x-1)
(x-2) x2-3x+2 NF
+ NF + l
NF
+ NF + N
(-∞, 1 - - + ive
(1,2) + - - ive 
∴U−5=0
(2, ∞) + + + ive
U=5

−∞ ⟵ + X ⟶ 1 ⟵ − X ⟶ 2 ⟵ + X ⟶ ∞

X ∈ [1,2] is satisfies –x2+3x-2 ≥0


EXERCISE 2.8
F F
1.Find all values of x for which > .
F

Solution :
P ] P − 1
>0
P−2
] P
P − 1 = 0
x = 0 or x=1 and x≠ 2
Interval P ] P − 1
P ] P − 1 P ] P − 1 (x-2)
P−2
−∞,  - - + - -
(0,1) + - - - +
(1,2) + + + - -
(2,∞) + + + + +
` Π`
`
> 0 in (0,1) U (2,∞)
2. Find all values of x that satisfies the inequality All the points above the F ≤ ¬ and all the points
`]
< 0. below F ≥ ¬ is required region. Darkly shaded area will
` `
represents the solution set of the given linear inequalities.
Solution :
`]
. ¬ ≥
F , −
F + ¬ ≤ _
<0
` ` Solution :
2P − 3 < 0 Let y=2x X 0 1
]
⟹P= U7© P ≠ 2 U7© P ≠ 4 By joining (0,0) and (1,2)
 y 0 2
2P − 3 we get y=2x
Interval 2P − 3 P − 2 P − 4
P − 2 P − 4
 - + -
ª−∞, « Let -2x+3y=6 X 0 -3

 + + + By Joining (0,2) and (-3,0) y 2 0


ª ,
«

we get -2x+3y=6

,  + - -
, ∞ + + +
`]
< 0 in d−∞, 
f g
, 
` `
F

3. Solve F

FN
≤
Solution :
F
− 
≤
F

F − N
F
 Fw

≤
FN Fw
P = 2 Yr P = −2 U7© P ≠ −3 , P ≠ 5
F
− 
Intervals F −
 F +
 F − N F + 
F −
F − N

(-∞, −3 + + +
(-3,-2] + - -
[-2,2) - - +
[2,5) + - - All the points above the ¬ ≥
F and all the points
(5,∞) + + + below −
F + ¬ ≤ _ is required region. Darkly shaded
F
 area will represents the solution set of the given linear
F

FN
≤  in (-3,-2] U [2,5)
inequalities.
EXERCISE 2.10 3. 3x+5y≥ N , F ≥  , ¬ ≥ 
Solution : X 0 15
Determine the region in the plane determined by the
Let 3x+5y=45 By joining (0,9)
inequalities: 1.F ≤ ¬ , F ≥ ¬.
y 9 0
and (15,0) we get 3x+5y=45 .
Solution :
` x= 0 represents y axis ; and y=0 represents x axis.
Let x=3y ⇒ y =
] All points bounded
X 0 3 above x=0 , y=0
y 0 1 and 3x+5y=45 is
joining the points (0,0) and (3,1) to get x=3y. required region. .
Let y=x Darkly shaded area
X 0 1 will represents the
y 0 1 solution set of the
Joining the points (0,0) and (1,1 ) to get x=y given linear
inequalities.
4. 2x+3y≤ N , F ≥ N , ¬ ≥

Solution :
Let 2x+3y=35 X 1 16
By joining (1,11) and (16,1) we y 11 1
get 2x+3y=35 .
Draw x=5 and y=2 .

No common Solution region occur to the given


inequalities 2x+3y≤ 6 , x + 4y ≤ 4 , P ≥ 0 , M ≥ 0
7. 2x+3y≥ § , ¥ +
­ ≥ § , ¥ + ­ ≤ _
Solution :
Let 2x+3y=8 . X 0 4
By joining (0,8) and (4,0) we get y 8 0
All points bounded between x=5 , y=2 and 2x+3y=35 2x+3y=8 .
is required region. . Darkly shaded area will represents
the solution set of the given linear inequalities. Let x+2y=8 X 0 8
5. 2x+3y≤ _ , ¥ + ­ ≤  , F ≥  , ¬ ≥  By joining (o,4) and (8,0) we get y 4 0
Solution : x+2y=8.
Let 2x+3y=6 X 0 3
By joining (0,2) and (3,0) we get y 2 0 Let x+y=6
2x+3y=6. By joining (0,6) and (6,0) we get X 0 6
x+y=6 y 6 0
Let x+4y=4 X 0 4
By joining (0,1) and (4,0) we get y 1 0
x+4y=4.

All points bounded between 2x+3y= § , ¥ +


­ = § ,
and ¥ + ­ = _ is required region. . Darkly shaded area
will represents the solution set of the given linear
inequalities.

All points bounded between x=0 , y=0 , x+4y=4 and


2x+3y=6 is required region. Darkly shaded area will
represents the solution set of the given linear inequalities.
6. x-2y≥  ,
¥ − ­ ≤ −
, F ≥  , ¬ ≥ 
Solution :
x-2y=0; By joining (0,0) , (2,1) and X 0 2 4
(4,2) to get x-2y=0 . y 0 1 2

2x-y=2 ; by joining (0,2) ,


X 0 -1
(-1,0) we get 2x-y=2
y 2 0
P.V MATRIC HR. SEC. SCHOOL 5.Find the radius of the spherical tank whose volume is
32𝝅/3 units.
MATHEMATICS - XI STD Solution : Volume of the sphere ⟹ 𝜋𝑟 3 =
4 32 𝜋
3 3
32 𝜋 3
𝑟3 = 𝑋
3 4 𝜋

EXERCISE 2.11 𝑟3 = 8
𝑟 3 = 23
1. Simplify:
𝟐 𝑟=2
(i) (𝟏𝟐𝟓)𝟑 𝟕+√𝟔
Solution : 6. Simplify by rationalising the denominator .
𝟑−√𝟐
2 2 Solution :
(125) = (5 ) = 52 = 25
3 3 3
−𝟑 𝟕 + √𝟔 𝟕 + √𝟔 𝟑 + √𝟐
(ii) 𝟏𝟔 𝟒 = 𝑿
𝟑 − √𝟐 𝟑 − √𝟐 𝟑 + √𝟐
Solution : 𝟐𝟏 + 𝟕√𝟐 + 𝟑√𝟔 + √𝟏𝟐
−3 −3
1 1 =
(16) 4 = (24 ) 4 = (2)−3 = = 𝟗−𝟐
23 8 𝟐𝟏 + 𝟕√𝟐 + 𝟑√𝟔 + 𝟐√𝟑
−𝟐 =
(𝒊𝒊𝒊)(−𝟏𝟎𝟎𝟎) 𝟑 𝟕
Solution : 𝟏 𝟏 𝟏 𝟏 𝟏
−𝟐 −𝟐 7. Simplify
𝟑−√𝟖
− + − +
𝟏 𝟏 √𝟖−√𝟕 √𝟕−√𝟔 √𝟔−√𝟓 √𝟓−𝟐
(−𝟏𝟎𝟎𝟎) 𝟑 = ((−𝟏𝟎)𝟑 ) 𝟑 = (−𝟏𝟎) −𝟐
= =
(−𝟏𝟎)𝟐 𝟏𝟎𝟎 Solution :
𝟏
(iv) (𝟑 ) −𝟔 𝟑 By rationalizing denominator of each terms we get,
Solution : 𝟏 𝟏 𝟑 + √𝟖 𝟑 + √𝟖
𝟏 = 𝑿 = = 𝟑 + √𝟖
(𝟑−𝟔 )𝟑 = 𝟑−𝟐 =
𝟏
=
𝟏 𝟑 − √𝟖 𝟑 − √𝟖 𝟑 + √𝟖 𝟗−𝟖
𝟑𝟐 𝟗
−𝟐 𝟏 𝟏 √𝟖 + √𝟕 √𝟖 + √𝟕
(𝟐𝟕) 𝟑 = 𝑿 = = √𝟖 + √𝟕
(iv) −𝟏 √𝟖 − √𝟕 √𝟖 − √𝟕 √𝟖 + √𝟕 𝟖−𝟕
(𝟐𝟕) 𝟑 𝟏 𝟏 √𝟕 + √𝟔 √𝟕 + √𝟔
Solution : = 𝑿 = = √𝟕 + √𝟔
−𝟐 √𝟕 − √𝟔 √𝟕 − √𝟔 √𝟕 + √𝟔 𝟕−𝟔
−𝟐 𝟏 −𝟏
(𝟐𝟕) 𝟑 𝟏 𝟏 𝟏 𝟏 𝟏
= (𝟐𝟕) 𝟑 +𝟑 = (𝟐𝟕) 𝟑 = = = √𝟔 + √𝟓 √𝟔 + √𝟓
−𝟏
(𝟐𝟕) 𝟑
(𝟐𝟕)𝟏/𝟑 𝟑
√𝟐𝟕 𝟑 = 𝑿 = = √𝟔 + √𝟓
√𝟔 − √𝟓 √𝟔 − √𝟓 √𝟔 + √𝟓 𝟔−𝟓
𝟏 −𝟏
2. Evaluate: (((𝟐𝟓𝟔)−𝟐 ) 𝟒 )𝟑 . 𝟏 𝟏 √𝟓 + 𝟐 √𝟓 + 𝟐
= 𝑿 = = √𝟓 + 𝟐
Solution : √𝟓 − 𝟐 √𝟓 − 𝟐 √𝟓 + 𝟐 𝟓−𝟒
−1 3 𝟏 𝟏 𝟏 𝟏 𝟏
(((256) ) ) = (256)−2

1 4 1 1
𝑋 (− ) 𝑋 3 − + − +
2 4 𝟑−√𝟖 √𝟖−√𝟕 √𝟕−√𝟔 √𝟔−√𝟓 √𝟓−𝟐
3
= (𝟑 + √𝟖) − (√𝟖 + √𝟕) +(√𝟖 + √𝟕)-( √𝟔 + √𝟓)+( √𝟓 + 𝟐)
= (28 )8 = 𝟑 + √𝟖 − √𝟖 − √𝟕 + √𝟖 + √𝟕 − √𝟔 − √𝟓 + √𝟓 + 𝟐
= 23 = 8 = 3+2 = 5
𝟏 𝟏 𝟗 𝟏 𝟏
𝒙𝟐 +𝟏
3. If (𝒙𝟐 + 𝒙−𝟐 )𝟐 = then find the value of (𝒙𝟐 − 𝒙−𝟐 ) 8. If x=√𝟐+√𝟑 find .
𝟐 𝒙𝟐 −𝟐
for x>1. Solution : x=√2+√3
Solution :
Now 𝑥 = (√2 + √3)2 = 2 + 2√6 + 3 = 5 + 2√6
2
We know that
𝑥2 + 1 5 + 2√6 + 1 6 + 2√6 3 − 2√6
(𝑎 − 𝑏)2 = (𝑎 + 𝑏)2 − 4𝑎𝑏 = = 𝑋
1

1 1

1 1

1 𝑥2 − 2 5 + 2√6 − 2 3 + 2√6
3 − 2√6
(𝑥 − 𝑥 ) 2 = (𝑥 + 𝑥 ) 2 − 4 𝑥 . 𝑥
2 2 2 2 2 2
1 1 1 1 18−12√6+6√6−24
= (𝑥 + 𝑥 ) 2 − 4 𝑥
2

2
( +(− ))
2 2 =
1 1 9−24
= (𝑥 2 + 𝑥 −2 ) 2 − 4 −6−6√6 −3(2+2√6)
1 1 9 1 = ⇒
(𝑥 2 − 𝑥 −2 ) 2 = − 4 = −15 −15
2 2
1 1 1 2+2√6
𝑥 − 𝑥
2

2 = for x>1. =
√2 5
4. Simplify and hence find the value of n:
𝟑𝟐𝒏 𝟗𝟐 𝟑−𝒏
EXERCISE 2.12
= 𝟐𝟕
𝟑𝟑𝒏 1.Let b > 0 and b ≠ 1. Express y = bx in logarithmic form.
𝟑𝟐𝒏 𝟗𝟐 𝟑−𝒏 𝟑𝟐𝒏 (𝟑𝟐 )𝟐 𝟑−𝒏
Solution : = 𝟐𝟕 ⇒ = 𝟐𝟕 Also state the domain and range of the logarithmic
𝟑𝟑𝒏 𝟑𝟑𝒏
𝟑𝟐𝒏+𝟒−𝒏 function.
⇒ = 𝟐𝟕 Solution :
𝟑𝟑𝒏
⇒ 𝟑 𝒏+𝟒−𝟑𝒏
= 𝟐𝟕 logarithmic form : log 𝑏 𝑦 = 𝑥
⇒ 𝟑𝟒−𝟐𝒏 = 𝟑𝟑 Domain : x∈𝑅
⇒ 𝟒 − 𝟐𝒏 = 𝟑 Range : (0,∞)
⇒ −𝟐𝒏 = −𝟏 ⇒ 𝒏 = 𝟏/𝟐
2. Compute 𝐥𝐨𝐠 𝟗 𝟐𝟕 − 𝐥𝐨𝐠 𝟐𝟕 𝟗. 6.Prove that 𝐥𝐨𝐠
𝒂𝟐
+ 𝐥𝐨𝐠
𝒃𝟐
+ 𝐥𝐨𝐠
𝒄𝟐
= 𝟎.
Solution: 𝒃𝒄 𝒄𝒂 𝒂𝒃
Solution:
log 9 27 = log 9 33 ⇒ 3 log 9 3 [ ∵ 𝑃𝑜𝑤𝑒𝑟 𝑟𝑢𝑙𝑒
𝑎2 𝑏2 𝑐2 𝑎2 𝑏 2 𝑐 2
⇒ 3 log 9 91/2 log + log + log = log ( . . )
3 3 𝑏𝑐 𝑐𝑎 𝑎𝑏 𝑏𝑐 𝑐𝑎 𝑎𝑏
⇒ log 9 9 = 𝑎2 𝑏 2 𝑐 2
2 2
= log ( ) = log 1 = 0
3 𝑎2 𝑏 2 𝑐 2
i.e., log 9 27 =
2 𝟏𝟔 𝟐𝟓 𝟖𝟏
1 7.Prove that 𝒍𝒐𝒈𝟐 + 𝟏𝟔 𝒍𝒐𝒈 + 𝟏𝟐 𝒍𝒐𝒈 + 𝟕 𝒍𝒐𝒈 =𝟏
𝟏𝟓 𝟐𝟒 𝟖𝟎
also log 27 9 = [ ∵ 𝐶ℎ𝑎𝑛𝑔𝑒 𝑜𝑓 𝑏𝑎𝑠𝑒 𝑟𝑢𝑙𝑒
log9 27 Solution:
1 2
log 27 9 = 3⁄ = 3
16 25 81
2 𝐿𝐻𝑆: 𝑙𝑜𝑔2 + 16 𝑙𝑜𝑔 + 12 log + 7 𝑙𝑜𝑔
2 15 24 80
i.e., log 27 9 = 16 16 25 12 81 7
3 = 𝑙𝑜𝑔2 + 𝑙𝑜𝑔 ( ) + log ( ) + 𝑙𝑜𝑔 ( )
3 2 9−4 5 15 24 80
Now log 9 27 − log 27 9 = − = = 16 16 25 12 81 7
2 3 6 6 = 𝑙𝑜𝑔2 + 𝑙𝑜𝑔 [( ) . ( ) . ( ) ]
15 24 80
3. Solve 𝐥𝐨𝐠 𝟖 𝒙 + 𝐥𝐨𝐠 𝟒 𝒙 + 𝐥𝐨𝐠 𝟐 𝒙 = 𝟏𝟏 16 12 7
24 52 34
Solution : = 𝑙𝑜𝑔2 + log [( ) . ( ) .( ) ]
5.3 3.23 5.24
log 8 𝑥 + log 4 𝑥 + log 2 𝑥 = 11 264 .524 .328
1 1 1 = 𝑙𝑜𝑔2 + log [ ] = 𝑙𝑜𝑔2 + 𝑙𝑜𝑔5
+ + = 11 516 .316 .316 .236 .57.228
log𝑥 8 log𝑥 4 log𝑥 2 = log (2X5) = log 10 = log10 10=1. Proved.
1 1 1
log𝑥 23
+ log 22 + log 2 = 11 8.Prove 𝐥𝐨𝐠 𝒂𝟐 𝒂 𝐥𝐨𝐠 𝒃𝟐 𝒃 𝐥𝐨𝐠 𝒄𝟐 𝒄 =
𝟏
𝑥 𝑥 𝟖
1 1 1 1 1 1
+ + = 11 Solution : L.H.S : log 𝑎2 𝑎 log 𝑏2 𝑏 log 𝑐 2 𝑐 = log 2
.
log𝑏 𝑏2 log𝑐 𝑐 2
3 log 𝑥 2 2 log 𝑥 2 log 𝑥 2 𝑎𝑎

1 1 1 1 1 1
= . .
( + + 1) = 11 2log 𝑎 𝑎 2log 𝑏 𝑏 2log 𝑐 𝑐
log 𝑥 2 3 2 1 1 1 1
1 11 = . . = [𝑹. 𝑯. 𝑺] Hence proved.
2 2 2 8
( ) = 11 ________________________________________________________________
log 𝑥 2 6
𝒏(𝒏+𝟏)
1 6 9.Prove 𝐥𝐨𝐠 𝒂 + 𝐥𝐨𝐠 𝒂𝟐 + 𝐥𝐨𝐠 𝒂𝟑 + ⋯ … . + 𝐥𝐨𝐠 𝒂𝒏 = 𝟐
𝐥𝐨𝐠 𝒂
= 11 𝑋 ( )
log 𝑥 2 11 Solution : L.H.S : log 𝑎 + log 𝑎 + log 𝑎 + ⋯ … . + log 𝑎𝑛 2 3
1
=6 ⇒ log 𝑎 + 2 log 𝑎 + 3 log 𝑎 + ⋯ … . + n log 𝑎
log 𝑥 2
⟹ log 𝑎 [1 + 2 + 3 + ⋯ . . +𝑛]
log 2 𝑥 = 6 ⇒ 𝑥 = 26 = 64. 𝑛(𝑛+1)
⇒ log 𝑎 [R.H.S] Hence Proved.
4. Solve 𝐥𝐨𝐠 𝟒 𝟐𝟖𝒙 = 𝟐𝐥𝐨𝐠𝟐 𝟖 2
𝐥𝐨𝐠 𝒙 𝐥𝐨𝐠 𝒚 𝐥𝐨𝐠 𝒛
Solution : 10. If = = then prove that xyz=1
𝒚−𝒛 𝒛−𝒙 𝒙−𝒚
log 4 28𝑥 = 2log2 8 log 𝑥 log 𝑦 log 𝑧
3 Solution : Let = = =𝑘
8𝑥 log 4 2 = 2log2 2 𝑦−𝑧 𝑧−𝑥 𝑥−𝑦
8𝑥 log 4 41/2 = 23 log2 2 log 𝑥 log 𝑦 log 𝑧
8𝑥 𝑦−𝑧 =𝑘 | 𝑧−𝑥=𝑘
|
𝑥−𝑦 = 𝑘
log 4 4 = 23 log2 2 log 𝑥 = 𝑘(𝑦 − 𝑧) log 𝑦 = 𝑘(𝑧 − 𝑥) log 𝑧 = 𝑘(𝑥 − 𝑦)
2 log 𝑥 = 𝑘𝑦 − 𝑘𝑧 log 𝑦 = 𝑘𝑧 − 𝑘𝑥 log 𝑧 = 𝑘𝑥 − 𝑘𝑦
4𝑥 = 23 Now log 𝑥 + log 𝑦 + log 𝑧 = 𝑘𝑦 − 𝑘𝑧 + 𝑘𝑧 − 𝑘𝑥 + 𝑘𝑥 − 𝑘𝑦
8
𝑥= log 𝑥𝑦𝑧 = 0 ⇒ i.e., 𝑥𝑦𝑧 = 1 [∵ log 1 = 0
4
𝑥=2 11. Solve 𝐥𝐨𝐠 𝟐 𝒙 − 𝟑 𝐥𝐨𝐠 𝟏 𝒙 = 𝟔
𝟐
𝒂+𝒃 𝟏 Solution :
5. If 𝒂𝟐 + 𝒃𝟐 = 𝟕𝒂𝒃.Show that 𝐥𝐨𝐠 ( ) = (𝐥𝐨𝐠 𝒂 + 𝐥𝐨𝐠 𝒃) Given log 2 𝑥 − 3 log1 𝑥 = 6
𝟑 𝟐 Let log 2 𝑥 = 𝑡 2
Solution : Now log 𝑥 = 1
1 𝑡 − 3(−𝑡) = 6
log𝑥 1/2
Given 𝑎2 + 𝑏 2 = 7𝑎𝑏 2
1
4𝑡 = 6
= 6
Adding 2ab on both sides log 𝑥 1 − log 𝑥 2 𝑡=
1 4
𝑎2 + 𝑏 2 + 2𝑎𝑏 = 7𝑎𝑏 + 2𝑎𝑏 =
1
3
log 𝑥 1 − 𝑡=
(𝑎 + 𝑏)2 = 9𝑎𝑏 log 2 𝑥 2
1 log 2 𝑥 = 3/2
(𝑎+𝑏)2 = = −𝑡
⇒ = 𝑎𝑏 0−
1 𝑥 = 23/2 = 81/2 = √8 = 2√2
9 𝑡
𝑎+𝑏 2 𝑖. 𝑒. , log1 𝑥 = −𝑡 𝑥 = 2√2
⇒ ( ) = 𝑎𝑏 2
3

(
𝑎+𝑏
) = (𝑎𝑏)1/2
12.Solve 𝐥𝐨𝐠 𝟓−𝒙 (𝒙𝟐 − 𝟔𝒙 + 𝟔𝟓) = 𝟐
3 Solution : log 5−𝑥 (𝑥 2 − 6𝑥 + 65) = 2
𝑡𝑎𝑘𝑒 log 𝑜𝑛 𝑏𝑜𝑡ℎ 𝑠𝑖𝑑𝑒𝑠
𝑎+𝑏 𝑥 2 − 6𝑥 + 65 = (5 − 𝑥)2
𝑙𝑜𝑔 ( ) = 𝑙𝑜𝑔(𝑎𝑏)1/2 𝑥 2 − 6𝑥 + 65 = 25 − 10𝑥 + 𝑥 2
3
𝑎+𝑏 1 4𝑥 = −40
𝑙𝑜𝑔 ( ) = (𝑙𝑜𝑔 𝑎 𝑏)
3 2 𝑥 = −10
𝑎+𝑏 1
𝑙𝑜𝑔 ( ) = (𝑙𝑜𝑔 𝑎 + 𝑙𝑜𝑔 𝑏)
3 2
𝐻𝑒𝑛𝑐𝑒 𝑃𝑟𝑜𝑣𝑒𝑑.
P.V MATRIC HR. SEC. SCHOOL (𝑎 sin 𝜃 + 𝑏 𝑐𝑜𝑠 𝜃)2 = 𝑎2 + 𝑏 2 - 𝑐 2
𝑎 sin 𝜃 + 𝑏 𝑐𝑜𝑠 𝜃 = ±√𝑎2 + 𝑏 2 − 𝑐 2
MATHEMATICS - XI STD Hence Proved
4.If 𝐬𝐢𝐧 𝜽 + 𝐜𝐨𝐬 𝜽 = 𝒎, show that
𝟒−𝟑(𝒎𝟐 −𝟏)𝟐
EXERCISE 3.1 𝐜𝐨𝐬 𝟔 𝜽 + 𝐬𝐢𝐧𝟔 𝜽 =
𝟒
where 𝒎𝟐 ≤ 𝟐
1. Identify the quadrant in which an angle of each given Solution : L.H.S
measure lies cos 6 𝜃 + sin6 𝜃 = (cos 2 𝜃)3 + (sin2 𝜃)3
(i) 25° (ii)825° (iii) -55° (iv) 328° (v) -230° = (cos2 𝜃 + sin2 𝜃)3 − 3 sin2 𝜃 cos2 𝜃 ( cos2 𝜃 + sin2 𝜃)
[∵ 𝑎3 + 𝑏 3 = (𝑎 + 𝑏)3 − 3𝑎𝑏(𝑎 + 𝑏)
(i) 25° 25° is in Ist 6 6
cos 𝜃 + sin 𝜃 = 1- 3 sin 𝜃 cos2 𝜃 --------------------(1)
2
quadrant
R.H.S
4−3(m2 −1)2 𝟒−𝟑 ((𝐬𝐢𝐧 𝜽+𝐜𝐨𝐬 𝜽)𝟐 −𝟏)𝟐
= 𝟒
4
(ii)825° 825° 4−3(sin2 𝜃+cos2 𝜃+2 sin 𝜃 cos 𝜃−1)2
=
=(2x360°)+105° 𝟒
is in IInd Quadrant 4−3(2 sin 𝜃 cos 𝜃)2
=
4
4−3(4 sin2 𝜃 cos2 𝜃)
=
(iii) -55° -55° is in IVth 4
Quadrant 4−12 sin2 𝜃 cos2 𝜃
=
4
4 12 sin2 𝜃 cos2 𝜃
= −
4 4
(iv) 328° 328° is in IVth
Quadrant
= 1- 3 sin2 𝜃 cos2 𝜃 --------------------(2)
From (1) and (2) L.H.S = R.H.S
4−3(𝑚2 −1)2
i.e., cos6 𝜃 + sin6 𝜃 =
4
𝐜𝐨𝐬 𝟒 𝜶 𝐬𝐢𝐧 𝟒 𝜶
5.If + = 𝟏 prove that
(v) -230° 230° is in IInd 𝐜𝐨𝐬 𝟐 𝜷 𝐬𝐢𝐧𝟐 𝜷
Quadrant 𝐜𝐨𝐬 𝟒 𝜷 𝐬𝐢𝐧 𝟒 𝜷
(i) 𝐬𝐢𝐧 𝟒 𝜶 + 𝐬𝐢𝐧 𝟒 𝜷 = 𝟐 𝐬𝐢𝐧 𝟐 𝜶 𝐬𝐢𝐧 𝟐 𝜷 (ii) 𝟐 + =𝟏
𝐜𝐨𝐬 𝜶 𝐬𝐢𝐧𝟐 𝜶
Solution :
cos4 𝛼 sin 4 𝛼
(i) Given 2 + =1
cos 𝛽 sin2 𝛽
cos4 𝛼 sin2 𝛽+ sin 4 𝛼 cos2 𝛽
cos2 𝛽 sin2 𝛽
=1
2. For each given angle, find a coterminal angle with
cos 𝛼 sin 𝛽 + sin 𝛼 cos 𝛽 = cos 2 𝛽 sin2 𝛽
4 2 4 2
measure of 𝜽 such that
(1 − sin2 𝛼)2 sin2 𝛽 + sin 4 𝛼 (1 − sin2 𝛽) = (1 − sin2 𝛽) sin2 𝛽
(i) 395° 395° = (1 X 360) + 35°
(1 − 2 sin2 𝛼+sin 4 𝛼 ) sin2 𝛽+sin 4 𝛼 − sin 4 𝛼 sin2 𝛽
Conterminal angle for 395° is 35°
(ii)525°
= sin2 𝛽-sin4 𝛽
525° = (1 X 360) + 165°
sin2 𝛽 − 2 sin2 𝛼 sin2 𝛽 + sin 4 𝛼 sin2 𝛽+sin 4 𝛼 − sin 4 𝛼 sin2 𝛽
Conterminal angle for 525° is 1655°
(iii) 1150° 1150° = (3 X 360) + 70° = sin2 𝛽-sin4 𝛽
Conterminal angle for 1150° is 70° sin2 𝛽 − 2 sin 2
𝛼 sin2 𝛽 + sin 4 𝛼 sin2 𝛽+sin 4 𝛼 − sin 4 𝛼 sin2 𝛽
(iv) -270° -270° = 360+(-270) = 90° = sin2 𝛽 −sin4 𝛽
Conterminal angle for -270°is 90° sin 4 𝛼 + sin4 𝛽 = 2 sin2 𝛼 sin2 𝛽 ----------- (I)
(v) -450° -450°= -(1x360+90)=-90 (i) is proved
= 360 +(-90) = 270° Now (I) ⇒ sin 4 𝛼 + sin4 𝛽 − 2 sin2 𝛼 sin2 𝛽 = 0
Conterminal angle for -450°is 270° (sin2 𝛼 − sin2 𝛽 )2 = 0
3. If 𝒂 𝐜𝐨𝐬 𝜽 − 𝒃 𝒔𝒊𝒏 𝜽 = 𝒄, Show that sin2 𝛼 − sin2 𝛽 = 0
𝒂 𝐬𝐢𝐧 𝜽 + 𝒃 𝒄𝒐𝒔 𝜽 = ±√𝒂𝟐 + 𝒃𝟐 − 𝒄𝟐 sin2 𝛼 = sin2 𝛽 -----------(1)
Solution : given 𝑎 cos 𝜃 − 𝑏 𝑠𝑖𝑛 𝜃 = 𝑐 ---------------(1) 1-cos 2 𝛼 = 1 - cos 2 𝛽
(𝑎 cos 𝜃 − 𝑏 𝑠𝑖𝑛 𝜃)2 + (𝑎 sin 𝜃 + 𝑏 𝑐𝑜𝑠 𝜃)2 =
cos 2 𝛼 = cos 2 𝛽 -----------(2)
𝑎2 cos 2 𝜃 − 2𝑎𝑏 cos 𝜃 𝑠𝑖𝑛 𝜃 + 𝑏 2 sin2 𝜃 +
cos4 𝛽sin 4 𝛽 cos2 𝛽 cos2 𝛽 sin 2 𝛽 sin 2 𝛽
𝑎2 sin2 𝜃 + 2𝑎𝑏 cos 𝜃 𝑠𝑖𝑛 𝜃 + 𝑏 2 Cos 2 𝜃 (ii)L.H.S: + = + sin2 𝛼
cos2 𝛼 sin2 𝛼 cos2 𝛼
= 𝑎 cos 𝜃+𝑏 2 sin2 𝜃+𝑎2 sin2 𝜃+𝑏 2 Cos 2 𝜃
2 2
cos2 𝛽 cos2 𝛼 sin 2 𝛽 sin 2 𝛼
= 𝑎2 (cos 2 𝜃 + sin2 𝜃) + 𝑏 2 (cos 2 𝜃 + sin2 𝜃) = + sin2 𝛼 [∵ 𝐹𝑟𝑜𝑚 (1)& (2)
cos2 𝛼
= 𝑎2 + 𝑏 2 cos2 𝛽 cos2 𝛼 sin 2 𝛽 sin 2 𝛼
= 2 + =cos2 𝛽 + sin2 𝛽 = 1 [𝑅. 𝐻. 𝑆]
cos 𝛼 sin2 𝛼
i.e., (𝑎 cos 𝜃 − 𝑏 𝑠𝑖𝑛 𝜃)2 + (𝑎 sin 𝜃 + 𝑏 𝑐𝑜𝑠 𝜃)2 = 𝑎2 + 𝑏 2
Hence (ii) is proved
Now 𝑐 2 + (𝑎 sin 𝜃 + 𝑏 𝑐𝑜𝑠 𝜃)2 = 𝑎2 + 𝑏 2 [from (1)
6. If 𝒚 =
𝟐 𝒔𝒊𝒏 𝜶
then prove that
𝟏−𝒄𝒐𝒔𝜶+𝒔𝒊𝒏𝜶
=𝒚 1 sin3 𝜃 1
𝟏+𝒄𝒐𝒔𝜶+𝒔𝒊𝒏𝜶 𝟏+𝒔𝒊𝒏𝜶 Now 𝑠𝑒𝑐 𝜃 + tan3 𝜃 𝑐𝑜𝑠𝑒𝑐𝜃 = + .
cos 𝜃 cos3 𝜃 sin 𝜃
Solution : 1 sin2 𝜃
2 𝑠𝑖𝑛 𝛼 = cos 𝜃
+ cos3 𝜃
𝑦=
1 + 𝑐𝑜𝑠𝛼 + 𝑠𝑖𝑛𝛼 cos2 𝜃+sin2 𝜃 1
2 𝑠𝑖𝑛 𝛼 1+𝑠𝑖𝑛𝛼−𝑐𝑜𝑠𝛼 = cos3 𝜃
= cos3 𝜃
= 𝑥 3
= sec 𝜃= (sec 𝜃) 3
1+𝑠𝑖𝑛𝛼+𝑐𝑜𝑠𝛼 1+𝑠𝑖𝑛𝛼−𝑐𝑜𝑠𝛼
2 𝑠𝑖𝑛 𝛼(1+𝑠𝑖𝑛𝛼−𝑐𝑜𝑠𝛼) 1/2 3
= = ((2 − 𝑘2 ) )
(1+𝑠𝑖𝑛𝛼)2 −cos2 𝛼 3 2 3/2
2 𝑠𝑖𝑛 𝛼(1+𝑠𝑖𝑛𝛼−𝑐𝑜𝑠𝛼) 𝑠𝑒𝑐 𝜃 + tan 𝜃 𝑐𝑜𝑠𝑒𝑐𝜃 = (2 − 𝑘 )
= Hence proved.
1+2𝑠𝑖𝑛𝛼+sin2 𝛼−(1−sin2 𝛼)
2 𝑠𝑖𝑛 𝛼(1+𝑠𝑖𝑛𝛼−𝑐𝑜𝑠𝛼) If 𝜃 = 0 tan 𝜃 = 0 ; 0= 1 − 𝑘 2 ⟹ 𝑘 = ±1
= If 𝜃 = 45° tan 45° = 1 ; 1= 1 − 𝑘 2 ⟹ 𝑘 = 0
1+2𝑠𝑖𝑛𝛼+sin2 𝛼−1+sin2 𝛼
2 𝑠𝑖𝑛 𝛼(1+𝑠𝑖𝑛𝛼−𝑐𝑜𝑠𝛼) If 𝜃 = 90° tan 𝜃 = ∞ ; k is not defined
= So that k∈ [−1,1]
2𝑠𝑖𝑛𝛼+2sin2 𝛼
2 𝑠𝑖𝑛 𝛼(1+𝑠𝑖𝑛𝛼−𝑐𝑜𝑠𝛼) 9. If 𝐬𝐞𝐜 𝜽+tan𝜽 = p obtain the values of 𝐬𝐞𝐜 𝜽 , 𝐭𝐚𝐧 𝜽 and
=
2𝑠𝑖𝑛𝛼(1+𝑠𝑖𝑛𝛼) 𝐬𝐢𝐧 𝜽 in terms of P.
2 𝑠𝑖𝑛 𝛼(1+𝑠𝑖𝑛𝛼−𝑐𝑜𝑠𝛼) Solution :
=
2𝑠𝑖𝑛𝛼(1+𝑠𝑖𝑛𝛼) Given 𝑠𝑒𝑐 𝜃 + 𝑡𝑎𝑛 𝜃 = 𝑝 -------------(1)
1+𝑠𝑖𝑛𝛼−𝑐𝑜𝑠𝛼 We know that sec 2 𝜃 − tan2 𝜃 = 1
= Hence Proved.
1+𝑠𝑖𝑛𝛼 Now (𝑠𝑒𝑐 𝜃 + 𝑡𝑎𝑛 𝜃)(𝑠𝑒𝑐 𝜃 − 𝑡𝑎𝑛 𝜃) = 1
7.If x = ∑∞
𝒏=𝒐 𝐜𝐨𝐬
𝟐𝒏
𝜽 , y = ∑∞
𝒏=𝒐 𝐬𝐢𝐧
𝟐𝒏
𝜽 and 𝑝 (𝑠𝑒𝑐 𝜃 − 𝑡𝑎𝑛 𝜃) = 1
𝝅
Z= ∑𝒏=𝒐 𝐜𝐨𝐬 𝜽 𝐬𝐢𝐧 𝜽 , 𝟎 < 𝜽 < then show that
∞ 𝟐𝒏 𝟐𝒏
𝑠𝑒𝑐 𝜃 − 𝑡𝑎𝑛 𝜃 = 1/𝑝 --------------(2)
𝟐
xyz=x+y+z . [𝑯𝒊𝒏𝒕: 𝟏 + 𝒙 + 𝒙𝟐 + 𝒙𝟑 + ⋯ . . =
𝟏
] Adding (1) and (2)
𝟏−𝒙 𝑠𝑒𝑐 𝜃 + 𝑡𝑎𝑛 𝜃 = 𝑝
Solution : 𝑠𝑒𝑐 𝜃 − 𝑡𝑎𝑛 𝜃 = 1/𝑝
If x = ∑∞
𝑛=𝑜 cos
2𝑛
𝜃
2 𝑠𝑒𝑐 𝜃 = 𝑝 + 1/𝑝
𝑥 = 1 + cos2 𝜃 + cos 4 𝜃 + cos 6 𝜃 + ⋯ … . . ∞ 𝑝2 + 1
1 1 1 1 2 2 𝑠𝑒𝑐 𝜃 =
𝑥= 1−𝐜𝐨𝐬 𝟐 𝜽
= 𝐬𝐢𝐧𝟐 𝜽
i.e., 𝑥 = 𝐬𝐢𝐧𝟐 𝜽
also 𝑥
= sin 𝜃 𝑝
𝑝2 +1
If y = ∑∞
𝑛=𝑜 sin
2𝑛
𝜃 𝑠𝑒𝑐 𝜃 =
2𝑝
𝑥 = 1 + sin2 𝜃 + sin4 𝜃 + sin6 𝜃 + ⋯ … . . ∞ Subtracting (1) and (2)
1 1 1 1
𝑥 = 1−𝐬𝐢𝐧𝟐 𝜽 = 𝐜𝐨𝐬 𝟐 𝜽
i.e., 𝑦 = 𝐜𝐨𝐬𝟐 𝜽 also 𝑦
= cos 2 𝜃 𝑠𝑒𝑐 𝜃 + 𝑡𝑎𝑛 𝜃 = 𝑝
𝑠𝑒𝑐 𝜃 − 𝑡𝑎𝑛 𝜃 = 1/𝑝
1 1 𝐜𝐨𝐬 𝟐 𝜽+𝐬𝐢𝐧𝟐 𝜽
Now 𝑥 + 𝑦 = 𝐬𝐢𝐧𝟐 𝜽
+ 𝐜𝐨𝐬𝟐 𝜽 = 𝐬𝐢𝐧𝟐 𝜽 𝐜𝐨𝐬 𝟐 𝜽
2 tan 𝜃 = 𝑝 − 1/𝑝
𝟏 1 1 𝑝2 −1
= = . = 𝑥 .𝑦 𝑡𝑎𝑛 𝜃 =
𝐬𝐢𝐧𝟐 𝜽 𝐜𝐨𝐬 𝟐 𝜽 𝐬𝐢𝐧𝟐 𝜽 𝐜𝐨𝐬 𝟐 𝜽 2𝑝
i.e., 𝑥 + 𝑦 = 𝑥𝑦 ------------------ (1) 𝑝2 −1
tan 𝜃 2𝑝 𝑝2 −1
Z= ∑∞𝑛=𝑜 cos
2𝑛
𝜃 sin2𝑛 𝜃 Now sin 𝜃 = = 𝑝2 +1
=
sin 𝜃 𝑝2 +1
2𝑝
𝑧 = 1 + cos 2 𝜃 sin2 𝜃 + cos 4 𝜃 sin4 𝜃 + ⋯ … … . . ∞
𝑝2 −1
1 1 𝑥𝑦 𝑠𝑖𝑛 𝜃 =
𝑧= = = 𝑝2 +1
1 − cos 2 𝜃 sin2 𝜃 1 1 𝑥𝑦 −1
1− . ___________________________________________________
𝑥 𝑦
Where 𝑧 =
𝑥𝑦 10. If cot𝜽(1+sin𝜽)=4m and cot𝜽(1-sin𝜽)=4n , then prove
𝑥𝑦−1
that (𝒎𝟐 − 𝒏𝟐 )𝟐 = 𝒎𝒏 .
𝑧(𝑥𝑦 − 1) = 𝑥𝑦
𝑥𝑦𝑧 − 𝑧 = 𝑥𝑦 Solution : 4𝑚 = 𝑐𝑜𝑡 𝜃 (1 + 𝑠𝑖𝑛 𝜃)
𝑥𝑦𝑧 = 𝑥𝑦 + 𝑧 16𝑚2 = cot 2 𝜃 (1 + 𝑠𝑖𝑛 𝜃)2 ---------(1)
𝑥𝑦𝑧 = (𝑥 + 𝑦) + 𝑧 [∵ 𝐹𝑟𝑜𝑚 (1) 4𝑛 = 𝑐𝑜𝑡 𝜃 (1 − 𝑠𝑖𝑛 𝜃)
𝑥𝑦𝑧 = 𝑥 + 𝑦 + 𝑧 Hence Proved. 16𝑛2 = cot 2 𝜃 (1 − 𝑠𝑖𝑛 𝜃)2 ---------(2)
*We may prove by different methods also. (1)-(2)
16𝑚2 − 16𝑛2 = cot 2 𝜃 (1 + 𝑠𝑖𝑛 𝜃)2 − cot 2 𝜃 (1 − 𝑠𝑖𝑛 𝜃)2
8. If 𝐭𝐚𝐧 𝜽 = 𝟏 − 𝒌 , 𝒔𝒉𝒐𝒘 𝒕𝒉𝒂𝒕
𝟐 𝟐
16(𝑚2 − 𝑛2 ) = cot 2 𝜃( (1 + 𝑠𝑖𝑛 𝜃)2 − (1 − 𝑠𝑖𝑛 𝜃)2 )
𝒔𝒆𝒄 𝜽 + 𝐭𝐚𝐧𝟑 𝜽 𝒄𝒐𝒔𝒆𝒄𝜽 = (𝟐 − 𝒌𝟐 )𝟑/𝟐 . Also , find the
= cot 2 𝜃( (1 + 2𝑠𝑖𝑛𝜃 + sin2 𝜃) − (1 + 2𝑠𝑖𝑛𝜃 + sin2 𝜃) )
value of k for which this result holds.
= cot 2 𝜃( 1 + 2𝑠𝑖𝑛𝜃 + sin2 𝜃 − 1 + 2𝑠𝑖𝑛𝜃 − sin2 𝜃) )
Solution :
16(𝑚2 − 𝑛2 ) = cot 2 𝜃( 4 𝑠𝑖𝑛 𝜃)
Given tan2 𝜃 = 1 − 𝑘 2
cos 2 𝜃 4 𝑠𝑖𝑛 𝜃 cos 2 𝜃
1 + tan2 𝜃 = 2 − 𝑘 2 [∵ 𝐴𝑑𝑑𝑖𝑛𝑔 1 𝑜𝑛 𝑏𝑜𝑡ℎ 𝑠𝑖𝑑𝑒𝑠 𝑚2 − 𝑛2 = 2
. =
sin 𝜃 16 4 sin 𝜃
Sec 2 𝜃 = 2 − 𝑘 2 2
cos2 𝜃 cos4 𝜃
sec 𝜃 = (2 − 𝑘 2 )1/2 ---------------(1) (𝑚2 − 𝑛2 )2 = ( ) = ----------(3)
4 sin2 𝜃 16 sin2 𝜃
4𝑚 . 4𝑛 = 𝑐𝑜𝑡 𝜃 (1 + 𝑠𝑖𝑛 𝜃). 𝑐𝑜𝑡 𝜃 (1 − 𝑠𝑖𝑛 𝜃)
16 𝑚𝑛 = cot 2 𝜃 (1 − sin2 𝜃)
EXERCISE 3.2
cos2 𝜃 1.Express each of the following angles in radian
= . cos 2 𝜃 measure:
sin2 𝜃
cos4 𝜃 (i)30° 𝜋 𝜋
𝑚𝑛 = ---------------- (4) 30° =30 x = radians
16 sin2 𝜃 180 6
From (3) and (4) (𝑚 − 𝑛2 )2 = 𝑚𝑛.
2
Hence proved (ii) 135° 135°=135 x
𝜋
=
3𝜋
radians
180 4
___________________________________________________ (iii)-205° -205°=-205 x
𝜋
=-
41𝜋
radians
180 36
11. If 𝐜𝐨𝐬𝐞𝐜 𝜽 − 𝒔𝒊𝒏 𝜽 = 𝒂𝟑 and 𝐬𝐞𝐜 𝜽 − 𝒄𝒐𝒔 𝜽 = 𝒃𝟑 then
(iv)150° 150°=150 x
𝜋 5𝜋
= radians
prove that 𝒂𝟐 𝒃𝟐 (𝒂𝟐 + 𝒃𝟐 ) = 𝟏 180 6
Solution : (v)330° 330°=330 x
𝜋
=
11𝜋
radians
180 6
𝑎3 = cosec 𝜃 − 𝑠𝑖𝑛 𝜃
1
= − 𝑠𝑖𝑛 𝜃 2. Find the degree measure corresponding to the
𝑠𝑖𝑛 𝜃 following radian measures
2
1−sin 𝜃
= (i)
𝝅 𝜋 𝜋 180
𝑠𝑖𝑛 𝜃 𝟑 = x = 60°
cos 2 𝜃 3 3 𝜋
𝑎3 = (ii)
𝝅 𝜋 𝜋 180
𝑠𝑖𝑛 𝜃 𝟗 = x = 20°
9 9 𝜋
cos 2/3 𝜃 sin2/3 𝜃 𝟐𝝅 2𝜋 2𝜋 180
𝑎= 1/3
𝑆𝑖𝑚𝑖𝑙𝑎𝑟𝑙𝑦 𝑏 = (iii) = x = 72°
sin 𝜃 cos1/3 𝜃 𝟓
5 5 𝜋
2 2 2 2
cos 2/3 𝜃 sin2/3 𝜃 cos 2/3 𝜃 sin2/3 𝜃 (iv)
𝟕𝝅 7𝜋 7𝜋 180
𝑎2 𝑏2 (𝑎2 + 𝑏2 ) = ( 1/3
) ( 1/3 ) [( 1/3 ) + ( 1/3 ) ] 𝟑 = x = 420°
sin 𝜃 cos 𝜃 sin 𝜃 cos 𝜃 3 3 𝜋
cos 4/3 𝜃 sin4/3 𝜃 cos 4/3 𝜃 sin4/3 𝜃 (iv)
𝟏𝟎𝝅 10𝜋 10𝜋 180
= . [ + ] 𝟗 = x = 200°
sin2/3 𝜃 cos 2/3 𝜃 sin2/3 𝜃 cos 2/3 𝜃 9 9 𝜋
3.What must be the radius of a circular running path,
cos 2 𝜃 + sin2 𝜃 around which an athlete must run 5 times in order to
= sin2/3 𝜃 cos 2/3 𝜃 [ 2/3 ]
sin 𝜃 cos 2/3 𝜃 describe 1 km?
= cos 2 𝜃 + sin2 𝜃 Solution :
2 2 (𝑎 2
𝑎 𝑏 + 𝑏 = 1 Hence Proved.
2)
Let r be the radius . r
________________________________________________________________ From given data
12. Eliminate 𝜽 from the equation 𝒂 𝒔𝒆𝒄 𝜽 − 𝒄 𝒕𝒂𝒏 𝜽 = 𝒃 5 (circumference of the circle) = 1 km
and 𝒃 𝒔𝒆𝒄 𝜽 + 𝒅 𝒕𝒂𝒏 𝜽 = 𝒄 . 5 ( 2 𝜋𝑟) = 1000 m
Solution : 1000 1000 x 27
r= 10𝜋
= 10 x 22
= 31.818𝑚
𝑎 𝑠𝑒𝑐 𝜃 − 𝑐 𝑡𝑎𝑛 𝜃 = 𝑏 ------------ (1)
𝑟 = 31.82 𝑚
𝑏 𝑠𝑒𝑐 𝜃 + 𝑑 𝑡𝑎𝑛 𝜃 = 𝑐 ------------ (2)
4. In a circle of diameter 40 cm, a chord is of length
(1) 𝑥 𝑏 ⇒ 𝑎𝑏 𝑠𝑒𝑐 𝜃 − 𝑏𝑐 𝑡𝑎𝑛 𝜃 = 𝑏 2
20 cm. Find the length of the minor arc of the chord.
(2) 𝑥 𝑎 ⇒ 𝑎𝑏 𝑠𝑒𝑐 𝜃 + 𝑎𝑑 𝑡𝑎𝑛 𝜃 = 𝑎𝑐
Solution :
subtracting above two
Diameter = 40 cm
− 𝑏𝑐 𝑡𝑎𝑛 𝜃 − 𝑎𝑑 𝑡𝑎𝑛 𝜃 = 𝑏 2 − 𝑎𝑐 Radius =OA=OB=20 cm
⇒ 𝑏𝑐 𝑡𝑎𝑛 𝜃 + 𝑎𝑑 𝑡𝑎𝑛 𝜃 = 𝑎𝑐 − 𝑏 2 O
Length of the Chord =AB = 20 cm o
𝜽
⇒ 𝑡𝑎𝑛 𝜃(𝑏𝑐 + 𝑎𝑑 ) = 𝑎𝑐 − 𝑏 2 In ∆𝑂𝐴𝐵 , OA=OB=AB= 20 20 20
𝑎𝑐 − 𝑏 2 ∴ ∆𝑂𝐴𝐵 is an equilateral triangle.
𝑡𝑎𝑛 𝜃 = 𝜋 A 20 B
(𝑏𝑐 + 𝑎𝑑 ) i.e., ∟𝐴𝑂𝐵 = 𝜃 = 60° =
3
(1) 𝑥 𝑑 ⇒ 𝑎𝑑 𝑠𝑒𝑐 𝜃 − 𝑐𝑑 𝑡𝑎𝑛 𝜃 = 𝑏𝑑 length of arc AB= s = r 𝜃
𝜋 22 1
(2) 𝑥 𝑐 ⇒ 𝑏𝑐 𝑠𝑒𝑐 𝜃 + 𝑐𝑑 𝑡𝑎𝑛 𝜃 = 𝑐 2 s = 20 x = 20 x x = 20.95𝑐𝑚
3 7 3
Adding above two __________________________________________________
𝑎𝑑 𝑠𝑒𝑐 𝜃 + 𝑏𝑐 𝑠𝑒𝑐 𝜃 = 𝑏𝑑 − 𝑐 2
5. Find the degree measure of the angle subtended at
⇒ 𝑠𝑒𝑐 𝜃 (𝑎𝑑 + 𝑏𝑐) = 𝑏𝑑 − 𝑐 2
the centre of circle of radius 100 cm by an arc of length
𝑏𝑑− 𝑐 2
⇒ 𝑠𝑒𝑐 𝜃 = 22 cm.
𝑎𝑑+𝑏𝑐
Solution :
sec 2 𝜃 − tan2 𝜃 = 1 Where r =100 cm and s=22cm
2 2
𝑏𝑑− 𝑐2 𝑎𝑐−𝑏2 s = r𝜃 ⇒ 𝜃= =
𝑠 22
( 𝑎𝑑+𝑏𝑐 ) − ((𝑏𝑐+𝑎𝑑 )) = 1 𝑟 100
22
2 ⇒ 𝜃= 𝑟𝑎𝑑𝑖𝑎𝑛𝑠
(𝑏𝑑− 𝑐2 )2 (𝑎𝑐−𝑏2 ) 100
(𝑎𝑑+𝑏𝑐)2
− (𝑎𝑑+𝑏𝑐)2 =1 ⇒ 𝜃=
22 180 22 x 180 x 7
x 𝜋 = 100 x 22 =
126
100 10
2
(𝑏𝑑− 𝑐2 )2 −(𝑎𝑐−𝑏 2 ) ⇒ 𝜃 = 12.6° = 12° 36′
=1 __________________________________________________
(𝑎𝑑+𝑏𝑐)2
(𝑏𝑑 − 𝑐2 )2 − (𝑎𝑐 − 𝑏 2 )2 = (𝑎𝑑 + 𝑏𝑐)2
(𝑏𝑑 − 𝑐2 )2 = (𝑎𝑑 + 𝑏𝑐)2 + (𝑎𝑐 − 𝑏 2 )2
___________________________________________________
6.What is the length of the arc intercepted by a central 10. A train is moving on a circular track of 1500 m radius
angle of measure 41° in a circle of radius 10 ft? at the rate of 66 km/hr. What angle will it turn in 20
Solution : seconds?
𝐺𝑖𝑣𝑒𝑛 𝜃 = 41° 𝑎𝑛𝑑 𝑟 = 10 𝑓𝑡 ; 𝑠 =? Solution :
We have to convert 𝜃 𝑖𝑛 𝑟𝑎𝑑𝑖𝑎𝑛 𝑚𝑒𝑎𝑠𝑢𝑟𝑒.
𝜋 41 𝜋 O
𝜃 = 41° = 41 x = o 1500
𝜽
180 180 r
𝑠 =𝑟x𝜃
41 𝜋 41 𝜋 41 22 A B
= 10 x 180 = = 18 x
18 7 l
𝑆 = 7.16 𝑓𝑡 Length covered in 1 hr By the train = 66 km
7.If in two circles, arcs of the same length subtend Length covered in (60x60) sec = 66000 m
angles 60° and 75° at the centre, find the ratio of 66000
their radii. Length covered in 1 sec =
60 x 60
Solution : 66000
Length covered in 20 sec = 60 x 60 x 20 =
1100
Length Radius Angle subtended at 3
𝑠
of arc centre Angle made in 20 sec 𝜃 =
𝜋 𝜋
𝑟
First circle 𝑠1 𝑟1 𝜃1 =60° = 60𝑋
180
=
3
1100
3
Second 𝑠2 𝑟2 𝜋 5𝜋 𝜃= 𝑟𝑎𝑑𝑖𝑎𝑛𝑠
𝜃2 = 75° = 75𝑋 = 1500
180 12 1100 1 180
Circle 𝜃= x x
1500 3 𝜋
1100 1 180 x 7
Where 𝑠1 = 𝑠2 𝜃 = 1500 x 3 x 22
𝜃 = 14°
𝑟1 𝜃1 = 𝑟2 𝜃2
𝜋 5𝜋
__________________________________________________
𝑟1 = 𝑟2 11. A circular metallic plate of radius 8 cm and thickness
3 12
5𝜋 6 mm is melted and molded into a pie (a sector of the
𝑟1 12 5 circle with thickness) of radius 16 cm and thickness 4
= 𝜋 = mm. Find the angle of the sector.
𝑟2 4
3 Solution :
𝑟1 ∶ 𝑟2 = 5: 4 Let 𝑟1 = Radius of circular metallic plate
_________________________________________________________ 𝑟2 = Radius of the sector
8. The perimeter of a certain sector of a circle is equal to Volume of the circular plate = Volume of the sector
the length of the arc of a semi-circle having 𝑟2 2
the same radius. Express the angle of the sector in 𝜋 𝑟1 2 x 𝑡ℎ𝑖𝑐𝑘𝑛𝑒𝑠𝑠 = 𝜃 x 𝑡ℎ𝑖𝑐𝑘𝑛𝑒𝑠𝑠
2
degrees, minutes and seconds. 6 (16)2 4
𝜋 (8)2 x = 𝜃x
Solution: 10 2 10
64 𝜋 x 6 = 128 𝜃 x 4
64 𝜋 x 6 3𝜋
O r r 𝜃 = 128 x 4 = 4 (𝑜𝑟)135°
o
𝜽
r r _______________________________________________
A B
r𝜽
Perimeter of the sector = length of arc of semi circle
𝑟 + 𝑟 + 𝑟𝜃 = 𝜋𝑟
2𝑟 + 𝑟𝜃 = 𝜋𝑟
2+𝜃 = 𝜋
𝜃 = 𝜋−2
22 8
𝜃 = 7 − 2= 7 radians
8 180 8 x 180 x 7 720
𝜃 = x = =
7 𝜋 7 x 22 11
720
𝜃 = = 65° 27′ 16′′
11
___________________________________________________
9. An airplane propeller rotates 1000 times per minute.
Find the number of degrees that a point on
the edge of the propeller will rotate in 1 second.
Solution :
No of rotation per minute = 1000
1000 50
No of rotation per second = = 3 rotations
60
50
= 3 x 360 = 6000°
___________________________________________________
P.V MATRIC HR. SEC. SCHOOL 3. Find the values of other five trigonometric functions
for the following:
MATHEMATICS - XI STD 𝟏
(i)𝒄𝒐𝒔 𝜽 = − 𝜽 𝒍𝒊𝒆𝒔 𝒊𝒏 𝒕𝒉𝒆 𝑰𝑰𝑰 quadrant
𝟐
X=1 o
Where r=√𝑥 2 + 𝑦2 A 𝜃
EXERCISE 3.3 y=√𝑟 2 − 𝑥 2 = √4 − 1 = √3
Y=-√3 r=2
1.Find the values of the following angles. o.s : y=√3 adj.s: x=1 hyp : r=2
(i)𝒔𝒊𝒏(𝟒𝟖𝟎°) ⇒ 𝒔𝒊𝒏(𝟑𝟔𝟎° + 𝟏𝟐𝟎°) In III quadrant only 𝑡𝑎𝑛𝜃 𝑎𝑛𝑑 𝑐𝑜𝑡𝜃 B
⇒ 𝒔𝒊𝒏 𝟏𝟐𝟎° will positive (ASTC rule) (1,-2)
⇒ 𝒔𝒊𝒏 (𝟏𝟖𝟎 − 𝟔𝟎)° 𝑜.𝑠 𝑦 −√3 2
sin 𝜃 = = = ; cosec 𝜃 = −
√𝟑 ℎ𝑦𝑝 𝑟 2 √3
⇒ 𝒔𝒊𝒏 𝟔𝟎° = 𝑎𝑑𝑗.𝑠 𝑥 −1
𝟐 cos 𝜃 = = = ; sec 𝜃 = −2
(ii) 𝒔𝒊𝒏(−𝟏𝟏𝟏𝟎°) ⇒ −𝒔𝒊𝒏 𝟏𝟏𝟏𝟎° ℎ𝑦𝑝 𝑟 2
𝒐.𝒔 𝒚 √3 𝟏
⇒ − 𝒔𝒊𝒏(𝟑𝒙𝟑𝟔𝟎° + 𝟑𝟎°) tan 𝜃 = = = = √𝟑 ; cot 𝜽 =
𝒂𝒅𝒋.𝒔 𝒙 𝟏 √3
⇒ −𝒔𝒊𝒏𝟑𝟎°
-----------------------------------------------------------------------------
𝟏 𝟐
⇒− (ii)𝒄𝒐𝒔 𝜽 = 𝜽 𝒍𝒊𝒆𝒔 𝒊𝒏 𝒕𝒉𝒆 𝑰 quadrant
𝟐 𝟑
(iii)𝒄𝒐𝒔(𝟑𝟎𝟎°) ⇒ 𝒄𝒐𝒔 (𝟑𝟔𝟎° − 𝟔𝟎°) B
⇒ 𝒄𝒐𝒔 𝟔𝟎° Where r=√𝑥 2 + 𝑦 2
𝟏 y=√𝑟 2 − 𝑥 2 = √9 − 4 = √5 r=3 y=√5

𝟐 o.s : y=√5 adj.s: x=2 hyp : r=3
(iv)𝒕𝒂𝒏(𝟏𝟎𝟓𝟎°) ⇒ 𝒕𝒂𝒏(𝟐𝒙𝟑𝟔𝟎° + 𝟑𝟑𝟎°) In I quadrant only 𝑎𝑙𝑙 𝑓𝑢𝑛𝑐𝑡𝑖𝑜𝑛𝑠
𝜃
⇒ 𝒕𝒂𝒏 𝟑𝟑𝟎° O x=2 A
will positive (ASTC rule)
⇒ 𝒕𝒂𝒏 (𝟑𝟔𝟎 − 𝟑𝟎°) 𝑜.𝑠 𝑦 √5 3
⇒ −𝒕𝒂𝒏 𝟑𝟎° = −
𝟏 sin 𝜃 = = = ; cosec 𝜃 =
ℎ𝑦𝑝 𝑟 3 √5
√𝟑 𝑎𝑑𝑗.𝑠 𝑥 2 3
(v) cot (660°) ⇒ 𝒄𝒐𝒕(𝟏𝒙𝟑𝟔𝟎° + 𝟑𝟎𝟎°) cos 𝜃 = = = ; sec 𝜃 =
ℎ𝑦𝑝 𝑟 3 2
⇒ 𝒄𝒐𝒕 𝟑𝟎𝟎° 𝑜.𝑠 𝑦 √5 2
⇒ 𝒄𝒐𝒕(𝟐𝟕𝟎° + 𝟑𝟎°) tan 𝜃 = = = ; cot 𝜃 =
𝑎𝑑𝑗.𝑠 𝑥 2 √5
𝟏 -----------------------------------------------------------------------------
⇒ −𝒕𝒂𝒏 𝟑𝟎° = − 𝟐
√𝟑 (iii)𝒔𝒊𝒏 𝜽 = − 𝜽 𝒍𝒊𝒆𝒔 𝒊𝒏 𝒕𝒉𝒆 𝑰𝑽 quadrant
𝟑
𝟏𝟗 𝝅 ⇒ 𝒕𝒂𝒏 (𝟏𝟗𝒙𝟔𝟎)°
(vi) tan ( ) Where r=√𝑥 2 + 𝑦 2 O x=√5 A
𝟑 ⇒ 𝒕𝒂𝒏 𝟏𝟏𝟒𝟎°
x=√𝑟 2 − 𝑦 2 = √9 − 4 = √5 𝜃
⇒ 𝒕𝒂𝒏 (𝟑𝒙𝟑𝟔𝟎° + 𝟔𝟎°)
o.s : y=2 adj.s: x=√5 hyp : r=3 r=3 y=2
⇒ 𝒕𝒂𝒏 𝟔𝟎° = √𝟑
𝟏𝟏𝝅 ⇒ 𝒔𝒊𝒏(−𝟏𝟏𝒙𝟔𝟎°) = 𝒔𝒊𝒏 (−𝟔𝟔𝟎°) In IV quadrant only 𝒄𝒐𝒔 𝜽 & 𝒔𝒆𝒄 𝜽
(vii)𝒔𝒊𝒏 (− ) B
𝟑 ⇒ −𝒔𝒊𝒏 (𝟔𝟔𝟎°) will positive (ASTC rule)
𝑜.𝑠 𝑦 2 3
⇒ −𝒔𝒊𝒏(𝟏𝒙𝟑𝟔𝟎° + 𝟑𝟎𝟎°) sin 𝜃 = = = − ; cosec 𝜃 = −
ℎ𝑦𝑝 𝑟 3 2
⇒ −𝒔𝒊𝒏 𝟑𝟎𝟎° ⇒ −𝒔𝒊𝒏(𝟑𝟔𝟎° − 𝟔𝟎°) 𝑎𝑑𝑗.𝑠 𝑥 √5 3
√𝟑 cos 𝜃 = = = ; sec 𝜃 =
ℎ𝑦𝑝 𝑟 3 √5
⟹ −(−𝒔𝒊𝒏𝟔𝟎°) = 𝒔𝒊𝒏𝟔𝟎° = 𝑜.𝑠 𝑦 2 √5
𝟐 tan 𝜃 = = =− ; cot 𝜃 = −
𝟓 𝟐√𝟔 𝑎𝑑𝑗.𝑠 𝑥 √5 2
2.( , )is a point on the terminal side of an angle 𝜽 in -----------------------------------------------------------------------------
𝟕 𝟕
standard position. Determine the trigonometric function (iv)𝒕𝒂𝒏 𝜽 = −𝟐 𝜽 𝒍𝒊𝒆𝒔 𝒊𝒏 𝒕𝒉𝒆 𝑰𝑰 quadrant
values of angle 𝜽. Where r=√𝑥 2 + 𝑦 2 == √4 + 1 = √5
Solution : B
o.s : y=2 adj.s: x=1 hyp : r=√5
𝟓 𝟐√𝟔
Since B( , )is the point on the terminal side of an In II quadrant only 𝒔𝒊𝒏 𝜽 & 𝒄𝒐𝒔𝒆𝒄 𝜽 y=2 r=√5
𝟕 𝟕
angle 𝜃. 𝟓 𝟐√𝟔 will positive (ASTC rule)
𝑩( , ) 𝜃
0𝐵2 = 𝑂𝐴2 + 𝐴𝐵2 𝟕 𝟕
sin 𝜃 =
𝑜.𝑠
= =
𝑦 2
; cosec 𝜃 =
√5

𝑂𝐵 = √𝑂𝐴2 + 𝐴𝐵2
ℎ𝑦𝑝 𝑟 √5 2 A x=-1
𝑎𝑑𝑗.𝑠 𝑥 1 √5
2 1 𝟐√𝟔
cos 𝜃 = = =− ; sec 𝜃 = − = −√5
5 2 2√6 25 24 𝟕 ℎ𝑦𝑝 𝑟 √5 1
𝑂𝐵 = √( ) + ( ) =√ + =1 tan 𝜃 =
𝑜.𝑠
=
𝑦
= −2 ; cot 𝜃 = −
1
7 7 49 49 𝜃 𝑎𝑑𝑗.𝑠 𝑥 2
𝟐√ 𝟔 O 𝟓 A -----------------------------------------------------------------------------
𝑜𝑝𝑝𝑜𝑠𝑖𝑡𝑒 𝑠𝑖𝑑𝑒 𝟐√ 𝟔 𝟕
(v)𝒔𝒆𝒄 𝜽 =
𝟏𝟑
𝜽 𝒍𝒊𝒆𝒔 𝒊𝒏 𝒕𝒉𝒆 𝑰𝑽 quadrant
sin 𝜃 = = 𝟕 = 𝟓
ℎ𝑦𝑝𝑜𝑡𝑒𝑛𝑢𝑠𝑒 1 𝟕
𝐴𝑑𝑗𝑎𝑐𝑒𝑛𝑡 𝑠𝑖𝑑𝑒 5/7 5 Where r=√𝑥 2 + 𝑦 2 O x=5 A
cos 𝜃 = = = y=√𝑟 2 − 𝑥 2 = √169 − 25 = √144 = 12 𝜃
𝐻𝑦𝑝𝑜𝑡𝑒𝑛𝑢𝑠𝑒 1 7 y=12
𝟐√ 𝟔
o.s : y=12 adj.s: x=5 hyp : r=13
𝑜𝑝𝑝𝑜𝑠𝑖𝑡𝑒 𝑠𝑖𝑑𝑒 𝟐√𝟔 In IV quadrant only 𝒄𝒐𝒔 𝜽 & 𝒔𝒆𝒄 𝜽 r=13
tan 𝜃 = = 𝟕 = will positive (ASTC rule) B
𝑎𝑑𝑗𝑎𝑐𝑒𝑛𝑡 𝑠𝑖𝑑𝑒 5 𝟓
𝑜.𝑠 𝑦 12 13
7 sin 𝜃 = = =− ; cosec 𝜃 = −
7 7 5 ℎ𝑦𝑝 𝑟 13 12
Similarly 𝑐𝑜𝑠𝑒𝑐 𝜃=2 ; 𝑠𝑒𝑐 𝜃 = ; 𝑐𝑜𝑡 𝜃 = 𝑎𝑑𝑗.𝑠 𝑥 5 𝑜.𝑠 𝑦 12 5
√6 5 2√6 cos 𝜃 = = = ; tan 𝜃 = = =− ; cot 𝜃 = −
ℎ𝑦𝑝 𝑟 13 𝑎𝑑𝑗.𝑠 𝑥 5 12
𝒄𝒐𝒕(𝟏𝟖𝟎° + 𝜽)𝒔𝒊𝒏(𝟗𝟎° − 𝜽)𝐜𝐨𝐬(−𝜽)
𝟒. 𝑷𝒓𝒐𝒗𝒆 𝒕𝒉𝒂𝒕 = 𝐜𝐨𝐬 𝟐 𝜽 𝒄𝒐𝒕𝜽
𝒔𝒊𝒏(𝟐𝟕𝟎° + 𝜽)𝒕𝒂𝒏(−𝜽)𝒄𝒐𝒔𝒆𝒄 (𝟑𝟔𝟎° + 𝜽)
Solution :
cos 𝜃
𝑐𝑜𝑡(180° + 𝜃) = cot 𝜃 =
sin 𝜃
𝑠𝑖𝑛(90° − 𝜃) = 𝑐𝑜𝑠𝜃
cos(−𝜃) = cos 𝜃
𝑠𝑖𝑛(270° + 𝜃) = − 𝑐𝑜𝑠 𝜃
− sin 𝜃
𝑡𝑎𝑛(−𝜃) = −𝑡𝑎𝑛𝜃 =
cos 𝜃
1
𝑐𝑜𝑠𝑒𝑐 (360° + 𝜃) = 𝑐𝑜𝑠𝑒𝑐 𝜃 =
sin 𝜃
L.H.S:
cos 𝜃 . 𝑐𝑜𝑠𝜃 . cos 𝜃
𝒄𝒐𝒕(𝟏𝟖𝟎° + 𝜽)𝒔𝒊𝒏(𝟗𝟎° − 𝜽)𝐜𝐨𝐬(−𝜽)sin 𝜃
=
− 𝑐𝑜𝑠 𝜃 . −cos
𝒔𝒊𝒏(𝟐𝟕𝟎° + 𝜽)𝒕𝒂𝒏(−𝜽)𝒄𝒐𝒔𝒆𝒄 (𝟑𝟔𝟎° + 𝜽) sin 𝜃 1
.
𝜃 sin 𝜃
= cos 2 𝜃 𝑐𝑜𝑡𝜃 R.H.S:
Hence proved
5.Find all the angles between 𝟎° and 360° which satisfy
𝟑
the equation 𝐬𝐢𝐧𝟐 𝜽 = .
𝟒
Solution :
3
Sin2 𝜃 =
4
√3
⇒ sin 𝜃 = ±
2
√3 √3
⇒ sin 𝜃 = and sin 𝜃 = −
2 2
√3
Case (i) : sin 𝜃 =
2
Where 𝜃 = 60° 𝑎𝑛𝑑 120° 0 < 𝜃 < 180°
−√3
Case (ii) : sin 𝜃 =
2
Where 𝜃 = 240° 𝑎𝑛𝑑 300° 180 < 𝜃 < 360°
𝝅 𝝅 𝟕𝝅 𝟒𝝅
7.Prove that 𝐬𝐢𝐧𝟐 + 𝐬𝐢𝐧𝟐 + 𝐬𝐢𝐧𝟐
+ 𝐬𝐢𝐧𝟐 =𝟐
𝟏𝟖 𝟗 𝟏𝟖 𝟗
Solution :
L.H.S
𝜋 𝜋 7𝜋 4𝜋
= sin2 + sin2 + sin2 + sin2
18 9 18 9
𝜋 180 𝜋 180 7𝜋 180 4𝜋 180
= sin2 ( x ) + sin2 ( x ) + sin2 ( x ) + sin2 ( x )
18 𝜋 9 𝜋 18 𝜋 9 𝜋
= sin2 (10°) + sin2 (20°) + sin2 (70°) + sin2 (80°)
=(𝑠𝑖𝑛 10°)2 + (𝑠𝑖𝑛 20°)2 + (𝑠𝑖𝑛 70°)2 + (𝑠𝑖𝑛 80°)2
= (𝑠𝑖𝑛 (90 − 10)°)2 + (𝑠𝑖𝑛 (90 − 70)°)2 + (𝑠𝑖𝑛 70°)2 + (𝑠𝑖𝑛 80°)2
= (𝑐𝑜𝑠 80°)2 + (𝑐𝑜𝑠 70°)2 + (𝑠𝑖𝑛 70°)2 + (𝑠𝑖𝑛 80°)2
=(𝑐𝑜𝑠 80°)2 + (𝑠𝑖𝑛 80°)2 + (𝑐𝑜𝑠 70°)2 + (𝑠𝑖𝑛 70°)2
= cos 2 80 + sin2 80 + cos 2 70 + sin2 70
=1+1=2 [ R.H.S ] Hence Proved.
Scanned by CamScanner
Scanned by CamScanner
Scanned by CamScanner
Scanned by CamScanner
Scanned by CamScanner
Scanned by CamScanner
Scanned by CamScanner
Scanned by CamScanner
Scanned by CamScanner
Scanned by CamScanner
Scanned by CamScanner
Scanned by CamScanner
PV MATRIC HR SEC SCHOOL 4.Prove that 𝒄𝒐𝒔 𝟓𝜽 = 𝟏𝟔 𝐜𝐨𝐬 𝟓 𝜽 − 𝟐𝟎 𝐜𝐨𝐬 𝟑 𝜽 + 𝟓 𝒄𝒐𝒔𝜽
Solution: L.H.S
MATHEMATICS - XI STD 𝑐𝑜𝑠 5𝜃 = cos(3𝜃 + 2𝜃) = cos 3𝜃 cos 2𝜃 − sin 3𝜃 sin 2𝜃
= (4 cos3 𝜃 − 3 cos 𝜃)(2 cos 2 𝜃 − 1) − (3 sin 𝜃 − 4 sin3 𝜃)(2𝑠𝑖𝑛𝜃 cos 𝜃)
= 8 cos5 𝜃 − 6 cos 3 𝜃 − 4 cos 3 𝜃 + 3 cos 𝜃 − 6 sin2 𝜃 cos 𝜃 + 8 sin4 𝜃 cos 𝜃
= 8 cos 5 𝜃 − 10 cos 3 𝜃 + 3 cos 𝜃 − 6(1 − cos 2 𝜃) cos 𝜃 + 8(1 − cos 2 𝜃)2 cos 𝜃
EXERCISE 3.5 = 8 cos 5 𝜃 − 10 cos 3 𝜃 + 3 cos 𝜃 − 6 cos 𝜃 + 6 cos 3 𝜃 + 8 cos 𝜃 − 16 cos 3 𝜃 + 8 cos 5 𝜃
1. Find the value of cos 2A , A lies in the first quadrant, = 16 cos5 𝜃 − 20 cos 3 𝜃 + 5 𝑐𝑜𝑠𝜃 = R.H.S Hence proved.
𝟏𝟓 𝟒 𝟏𝟔 𝟏−𝐭𝐚𝐧𝟐 𝜶
when (i) 𝒄𝒐𝒔 𝑨 = ; (𝒊𝒊)𝒔𝒊𝒏 𝑨 = (𝒊𝒊𝒊)𝒕𝒂𝒏 𝑨 = 5.Prove that 𝒔𝒊𝒏 𝟒𝜶 = 𝟒 𝒕𝒂𝒏𝜶 𝟐
𝟏𝟕 𝟓 𝟔𝟑 (𝟏+𝐭𝐚𝐧𝟐 𝜶)
Solution: Solution :
𝟏𝟓
(i) 𝒄𝒐𝒔 𝑨 = 𝑐𝑜𝑠 2𝐴 = 2 cos2 𝐴 − 1 L.H.S sin 4𝛼 = sin 2(2𝛼) = 2 sin 2𝛼 cos 2𝛼
𝟏𝟕
15 2 225 2 𝑡𝑎𝑛𝛼 1−tan2 𝛼
= 2( ) − 1 ⟹ 2( )−1 = 2( )( )
17 289 1+tan2 𝛼 1+tan2 𝛼
450 450−289 161 1−tan2 𝛼
= − 1= = = 4 𝑡𝑎𝑛𝛼 R.H.S
289 289 289 (1+tan2 𝛼)2
𝟒
(𝒊𝒊) 𝒔𝒊𝒏 𝑨 = 𝑐𝑜𝑠 2𝐴 = 1 − 2 sin2 𝐴
𝟓 6.If A+B=45° , show that (𝟏 + 𝒕𝒂𝒏 𝑨)(𝟏 + 𝒕𝒂𝒏 𝑩) = 𝟐
4 2 16
= 1 −2( ) = 1 −2( )
5 25
Solution : 𝐴 + 𝐵 = 45°
=1−
32
=
−7 tan(𝐴 + 𝐵) = tan 45°
25 25 tan 𝐴 +tan 𝐵
𝟏𝟔 1−tan2 𝐴 =1
(iii) 𝐭𝐚𝐧 𝐀 = 𝑐𝑜𝑠 2𝐴 = 1−tan 𝐴 tan 𝐵
𝟔𝟑 1+tan2 𝐴 tan 𝐴 + tan 𝐵 = 1 − tan 𝐴 tan 𝐵
16 2 256 3969 − 256 tan 𝐴 + tan 𝐵 + tan 𝐴 tan 𝐵 = 1
1−( )
63 = 1 − 3969 = 3969 3713
=
16 2 256
=
3969 + 256 4225 Adding 1 on both sides
1+( ) 1+
63 3969 3969 tan 𝐴 + tan 𝐵 + tan 𝐴 tan 𝐵 + 1 = 2
2. If 𝜽 is an acute angle, then find 1+ tan 𝐴 + tan 𝐵 + tan 𝐴 tan 𝐵 = 2
𝝅 𝜽 𝟏 1+ tan 𝐴 + tan 𝐵 (1 + tan 𝐴 ) = 2
(i) 𝒔𝒊𝒏 ( − ) , 𝒘𝒉𝒆𝒏 𝒔𝒊𝒏𝜽 =
𝟒 𝟐 𝟐𝟓 (1 + tan 𝐴 )(1 + tan 𝐵 ) = 2 Hence Proved.
𝜋 𝜃 𝜋 𝜃 𝜋 𝜃
𝑠𝑖𝑛 ( − ) = 𝑠𝑖𝑛 𝑐𝑜𝑠 − cos 𝑠𝑖𝑛 7.Prove that
4 2 4 4 2 2
=
1
𝑐𝑜𝑠
𝜃

1
𝑠𝑖𝑛
𝜃 (𝟏 + 𝒕𝒂𝒏𝟏°)(𝟏 + 𝒕𝒂𝒏𝟐°)(𝟏 + 𝒕𝒂𝒏𝟑°) … . (𝟏 + 𝒕𝒂𝒏𝟒𝟒°)
√2 2 √2 2
1 𝜃 𝜃 is a multiple of 4.
= (𝑐𝑜𝑠 − 𝑠𝑖𝑛 )
√2 2 2 Solution : 𝑖𝑓 𝐴 + 𝐵 = 45° 𝑡ℎ𝑒𝑛 (1 + 𝑡𝑎𝑛𝐴)(1 + 𝑡𝑎𝑛 𝐵) = 2
𝜋 𝜃 1 𝜃 𝜃 2
sin2 ( − ) = (𝑐𝑜𝑠 − 𝑠𝑖𝑛 ) Given (1 + 𝑡𝑎𝑛1°)(1 + 𝑡𝑎𝑛2°)(1 + 𝑡𝑎𝑛3°) … . (1 + 𝑡𝑎𝑛44°)
4 2 2 2 2
=
1 𝜃 2𝜃 𝜃
(cos2 + sin − 2𝑠𝑖𝑛 𝑐𝑜𝑠 )
𝜃 Let change the order and written as
2 2 2 2 2 (1 + 𝑡𝑎𝑛1°)(1 + 𝑡𝑎𝑛44°)(1 + 𝑡𝑎𝑛1°)(1 + 𝑡𝑎𝑛44°) … … (1 + 𝑡𝑎𝑛1°)(1 + 𝑡𝑎𝑛44°)
1
= (1 − sin 𝜃)
2
1 𝟏 1 24 12 = 2 X 2 X … .. X 2 (22times)
= (1 − ) ⟹ ( )=
2 𝟐𝟓 2 25 25 =222 = (22 )11 = 411 [𝑀𝑢𝑙𝑡𝑖𝑝𝑙𝑒 𝑜𝑓 4] Hence proved.
𝜋 𝜃 12 2√3 𝝅 𝝅
𝑠𝑖𝑛 ( − ) = √ = 8.Prove that 𝒕𝒂𝒏 ( + 𝜽) − 𝒕𝒂𝒏 ( − 𝜽) = 𝟐 𝒕𝒂𝒏 𝟐𝜽
4 2 25 5 𝟒 𝟒
𝝅 𝜽
(i) 𝒄𝒐𝒔 ( + ) , 𝒘𝒉𝒆𝒏 𝒔𝒊𝒏𝜽 =
𝟖 Solution :
𝟒 𝟐 𝟗 𝜋 𝜋
𝜋 𝜋 𝑡𝑎𝑛 + tan 𝜃 𝑡𝑎𝑛 − tan 𝜃
𝜋 𝜃 𝜋 𝜃 𝜋 𝜃 𝑡𝑎𝑛 ( + 𝜃) − 𝑡𝑎𝑛 ( − 𝜃) = 4 − 4
𝑐𝑜𝑠 ( + ) = 𝑐𝑜𝑠 𝑐𝑜𝑠 − sin 𝑠𝑖𝑛 4 4 𝜋 𝜋
1 − 𝑡𝑎𝑛 tan 𝜃 1 + 𝑡𝑎𝑛 tan 𝜃
4 2 4 2 4 2 4 4
1 𝜃 1 𝜃
=
√2
𝑐𝑜𝑠 2
− √2
𝑠𝑖𝑛 2 =
1+tan 𝜃

1−tan 𝜃

1 𝜃 𝜃 1− tan 𝜃 1+ tan 𝜃
= (𝑐𝑜𝑠 − 𝑠𝑖𝑛 ) =
(1+tan 𝜃)2 −(1−tan 𝜃)2
√2 2 2
1−tan2 𝜃
𝜋 𝜃 1 𝜃 𝜃 2
cos2 ( + ) = 2 (𝑐𝑜𝑠 2 − 𝑠𝑖𝑛 2 ) 1+2 tan 𝜃+tan2 𝜃−1+2 tan 𝜃−tan2 𝜃
4 2 =
1 𝜃 2𝜃 𝜃 𝜃 1−tan2 𝜃
= (cos2 + sin − 2𝑠𝑖𝑛 𝑐𝑜𝑠 ) 4 𝑡𝑎𝑛𝜃 2 𝑡𝑎𝑛𝜃
2 2 2 2 2 = ⟹ 2( ) = 2𝑡𝑎𝑛2𝜃
1 1−tan2 𝜃 1−tan2 𝜃
= (1 − sin 𝜃)
2 𝟏
1 𝟖 1 1 1 9.Prove that 𝒄𝒐𝒕 (𝟕 °) = √𝟐 + √𝟑 + √𝟒 + √𝟔
= (1 − ) ⟹ ( )= 𝟐
2 𝟗 2 9 18 𝟏
Solution : Let 𝟕 ° = 𝜶
𝜋 𝜃 1 1 𝟐
𝑐𝑜𝑠 ( + ) = √ = 1 𝑐𝑜𝑠 𝛼
4 2 18 3√2 L.H.S : 𝑐𝑜𝑡 (7 °) = 𝑐𝑜𝑡 𝛼 =
2 𝑠𝑖𝑛 𝛼
𝟏 𝟏 𝟏 𝟏
3. If 𝒄𝒐𝒔 𝜽 = (𝒂 + ) , 𝒔𝒉𝒐𝒘 𝒕𝒉𝒂𝒕 𝒄𝒐𝒔 𝟑𝜽 = (𝒂𝟑 + ). Multiply both numerator and denominator by 2 𝑐𝑜𝑠 𝛼
𝟐 𝒂 𝟐 𝒂𝟑
𝑐𝑜𝑠 𝛼 2 𝑐𝑜𝑠 𝛼 2 cos2 𝛼 1+cos 2𝛼
Solution : 𝑐𝑜𝑠 3𝜃 = 4 cos 3 𝜃 − 3 𝑐𝑜𝑠𝜃 Now 𝑐𝑜𝑡 𝛼 = 𝑋 = =
𝑠𝑖𝑛 𝛼 2 𝑐𝑜𝑠 𝛼 2 𝑠𝑖𝑛 𝛼 𝑐𝑜𝑠 𝛼 sin 2𝛼
1 3 1+cos 2𝛼 𝟏
1
= 4 ( (𝑎 + ) ) − 3 ( (𝑎 + ))
1 1
Now 𝑐𝑜𝑡 𝛼 = 𝑃𝑢𝑡 𝛼 = 𝟕 °
8 𝑎 2 𝑎 sin 2𝛼 𝟐
𝑊𝑒 𝑚𝑢𝑠𝑡 𝑘𝑛𝑜𝑤
1 1 3 3 1 1+
√3+1
= (𝑎 + ) − (𝑎 + ) 𝟏 1+cos 15° 2√2+√3+1 √3+1
2 𝑎 2 𝑎 𝑐𝑜𝑡 𝟕 ° = = 2√2
√3−1
= cos 15° =
2√2
𝟐 sin 15° √3−1
1 1 3 1 √3−1
= [(𝑎 + ) − 3 (𝑎 + )] 2√2 [ sin 15° = 2√2
2 𝑎 𝑎
2 2+ 3+1
√ √ √ 3+1 2 6+ 3+2 2+ 3+1
√ √ √ √
1 1 1 1 1 1 1 = 𝑋 = = √𝟐 + √𝟑 + √𝟒 + √𝟔
= [𝑎3 + + 3𝑎 ( ) (𝑎 + ) − 3 (𝑎 + )] = (𝑎3 + ) √ 3−1 √ 3+1 2
2 𝑎3 𝑎 𝑎 𝑎 2 𝑎3
10. Prove that
(𝟏 + 𝒔𝒆𝒄𝟐𝜽)(𝟏 + 𝒔𝒆𝒄𝟒𝜽) … . . (𝟏 + 𝒔𝒆𝒄𝟐𝒏 𝜽) = 𝐭𝐚𝐧 𝟐𝒏 𝜽 𝒄𝒐𝒕 𝜽
Solution :
(1 + 𝑠𝑒𝑐2𝜃)(1 + 𝑠𝑒𝑐4𝜃) … . . (1 + 𝑠𝑒𝑐2𝑛 𝜃)
1 1 1
=(1 + ) (1 + cos 4𝜃) … . (1 + cos 2𝑛 𝜃)
cos 2𝜃
𝑛
(1 + cos 2𝜃)(1 + cos 4𝜃) … . (1 + cos 2 𝜃)
=
cos 2𝜃 . cos 4𝜃 … … . cos 2𝑛 𝜃
2 cos 𝜃 . 2 cos2 2𝜃 … . 2 cos 2 2𝑛−1 𝜃
2
=
cos2𝑛 𝜃
2 cos 𝜃 {cos 𝜃 . cos 2𝜃 … . cos 2𝑛−1 𝜃}
𝑛
=
cos 2𝑛 𝜃
sin 2𝑛 𝐴
cos 𝐴 cos 2𝐴 cos 22 𝐴 … . . cos 2𝑛−1 𝐴 = ------(1)
2𝑛 sin 𝐴
By using (1)
sin 2𝑛 𝜃
2𝑛 cos 𝜃 𝑋 ( 𝑛 ) 2𝑛 cos 𝜃 sin 2𝑛 𝜃 2𝑛 cos 𝜃 sin 2𝑛 𝜃
2 sin 𝜃
= cos 2𝑛 𝜃
= cos 2𝑛 𝜃 2𝑛 sin 𝜃
= 2𝑛 sin 𝜃
x cos 2𝑛 𝜃
= tan 2𝑛 𝜃 𝑐𝑜𝑡 𝜃
Hence Proved.
𝝅 𝝅 𝝅 𝝅 𝝅
11.Prove that 𝟑𝟐(√𝟑)𝒔𝒊𝒏 𝟒𝟖
𝒄𝒐𝒔
𝟒𝟖
𝒄𝒐𝒔
𝟐𝟒
𝒄𝒐𝒔
𝟏𝟐
𝒄𝒐𝒔
𝟔
=𝟑
Solution :
𝜋 𝜋 𝜋 𝜋 𝜋
Given 32(√3)𝑠𝑖𝑛 48
𝑐𝑜𝑠
48
𝑐𝑜𝑠
24
𝑐𝑜𝑠
12
𝑐𝑜𝑠
6
𝜋 𝜋 𝜋 𝜋 𝜋
= 16√3 (2 𝑠𝑖𝑛 𝑐𝑜𝑠 ) 𝑐𝑜𝑠 𝑐𝑜𝑠 𝑐𝑜𝑠
48 48 24 12 6
𝜋 𝜋 𝜋 𝜋
= 16√3 (𝑆𝑖𝑛 2𝑥 48) 𝑐𝑜𝑠 24
𝑐𝑜𝑠
12
𝑐𝑜𝑠
6
𝜋 𝜋 𝜋 𝜋
= 16√3 (𝑆𝑖𝑛 ) 𝑐𝑜𝑠 24
𝑐𝑜𝑠
12
𝑐𝑜𝑠
6
24
Continuing this process
𝜋 𝜋 𝜋 𝜋
= 8√3 (2 𝑆𝑖𝑛 𝑐𝑜𝑠 ) 𝑐𝑜𝑠 12
𝑐𝑜𝑠
6
24 24
𝜋 𝜋 𝜋
= 8√3 ( 𝑆𝑖𝑛 ) 𝑐𝑜𝑠 12
𝑐𝑜𝑠
6
12
𝜋 𝜋 𝜋
= 4√3 (2 𝑆𝑖𝑛 𝑐𝑜𝑠 ) 𝑐𝑜𝑠
12 12 6
𝜋 𝜋
= 4√3 (𝑠𝑖𝑛 6) 𝑐𝑜𝑠 6
𝜋 𝜋
= 2√3 (2𝑠𝑖𝑛 𝑐𝑜𝑠 )
6 6
𝜋 √3
= 2√3 𝑠𝑖𝑛 3
= 2√3 x 2
=3.
Scanned by CamScanner
Scanned by CamScanner
Scanned by CamScanner
Scanned by CamScanner
PV MATRIC HR. SEC. SCHOOL (𝒊𝒊𝒊) 𝐬𝐢𝐧𝟐 𝑨 + 𝐬𝐢𝐧𝟐 𝑩 + 𝐬𝐢𝐧𝟐 𝑪 = 𝟐 + 𝟐𝑪𝒐𝒔𝑨 𝑪𝒐𝒔𝑩 𝒄𝒐𝒔𝑪
MATHEMATICS - XI STD L.H.S : (𝐬𝐢𝐧𝟐 𝑨 + 𝐬𝐢𝐧𝟐 𝑩) + 𝐬𝐢𝐧𝟐 𝑪
1−𝑐𝑜𝑠2𝐴 1−𝑐𝑜𝑠2𝐵
=( + ) + sin2 𝐶 [∵by *13
2 2
Formulae to Remember 1
= [1 − 𝑐𝑜𝑠2𝐴 + 1 − 𝑐𝑜𝑠2𝐵] + sin2 𝐶
2
Formula Identities 1
1 = [1 − 𝑐𝑜𝑠2𝐴 + 1 − 𝑐𝑜𝑠2𝐵] + (1 − cos2 𝐶)
2
*1 sinA cosB [sin(𝐴 + 𝐵) + sin(𝐴 − 𝐵)] 1
2 = [2 − (𝑐𝑜𝑠2𝐴 + 𝑐𝑜𝑠2𝐵)] + (1 − cos2 𝐶)
1 2
*2 cosA sinB [sin(𝐴 + 𝐵) − sin(𝐴 − 𝐵)] 1
= 1 − (𝑐𝑜𝑠2𝐴 + 𝑐𝑜𝑠2𝐵) + 1 − cos2 𝐶
2 2
1 1 2𝐴+2𝐵 2𝐴−2𝐵
*3 cosA cosB [cos(𝐴 + 𝐵) + cos(𝐴 − 𝐵)] = 1 − (2 𝑐𝑜𝑠 ( ) 𝑐𝑜𝑠 ( )) + 1 − cos2 𝐶 [∵by *7
2 2 2
2
1 = 2 − cos(𝐴 + 𝐵) cos(𝐴 − 𝐵) − cos2 𝐶
*4 sinA sinB [cos(𝐴 − 𝐵) − cos(𝐴 + 𝐵)] = 2 − cos(𝐴 + 𝐵) cos(𝐴 − 𝐵) − cos 𝐶 𝐶𝑜𝑠𝐶
2
*5 sin C+ sin D 2 sin (
𝐶+𝐷
) cos(
𝐶−𝐷
) = 2 − cos(180 − 𝑐) cos(𝐴 − 𝐵) − cos 𝐶 𝐶𝑜𝑠(180 − (𝐴 + 𝐵))
2 2
𝐶+𝐷 𝐶−𝐷
= 2 + cos 𝐶 cos(𝐴 − 𝐵) + cos 𝐶 𝐶𝑜𝑠(𝐴 + 𝐵)
*6 sin C – sin D 2 cos ( ) sin( ) = 2 + cos 𝐶 [cos(𝐴 − 𝐵) + cos(𝐴 + 𝐵)]
2 2
𝐶+𝐷 𝐶−𝐷 𝐴−𝐵+𝐴+𝐵 𝐴−𝐵−(𝐴+𝐵)
*7 cos C + cosD 2 cos ( ) cos( ) = 2 + cos C [2 𝑐𝑜𝑠 ( ) cos ( ) [∵by *7
2 2 2 2
𝐶+𝐷 𝑫−𝑪 = 2+ cos C [2 cos A cos B]
*8 cos C - cosD 2 sin ( ) cos( )
2 𝟐
= 𝟐 + 𝟐𝑪𝒐𝒔𝑨 𝑪𝒐𝒔𝑩 𝒄𝒐𝒔𝑪 R.H.S Hence Proved
*9 Sin 2A 2 sinA cosA
𝟐 𝟐 𝟐
*10 Sin A 2 sin cos
𝐴 𝐴 (iv) 𝐬𝐢𝐧 𝑨 + 𝐬𝐢𝐧 𝑩 − 𝐬𝐢𝐧 𝑪 = 𝟐 𝒔𝒊𝒏 𝑨 𝒔𝒊𝒏𝑩 𝒄𝒐𝒔 𝑪
2 2 L.H.S : (𝐬𝐢𝐧𝟐 𝑨 + 𝐬𝐢𝐧𝟐 𝑩) − 𝐬𝐢𝐧𝟐 𝑪
*11 Cos A 1-2sin 2𝐴
1−𝑐𝑜𝑠2𝐴 1−𝑐𝑜𝑠2𝐵
2 =( + ) − sin2 𝐶 [∵by *13
*12 Cos A 2cos2 -1
𝐴
1
2 2
2
2 = [1 − 𝑐𝑜𝑠2𝐴 + 1 − 𝑐𝑜𝑠2𝐵] − sin 𝐶
*13 2 1 − 𝑐𝑜𝑠2𝐴 2
sin 𝐴 1
= [1 − 𝑐𝑜𝑠2𝐴 + 1 − 𝑐𝑜𝑠2𝐵] − (1 − cos2 𝐶)
2 2
*14 𝑡𝑎𝑛𝐴 + 𝑡𝑎𝑛𝐵 1
= [2 − (𝑐𝑜𝑠2𝐴 + 𝑐𝑜𝑠2𝐵)] − (1 − cos2 𝐶)
tan(A+B) 2
1 − 𝑡𝑎𝑛𝐴 𝑡𝑎𝑛𝐵 1
EXERCISE 3.7 = 1 − (𝑐𝑜𝑠2𝐴 + 𝑐𝑜𝑠2𝐵) − 1 − cos2 𝐶
2
01.If A+B+C=180°, Prove that, 1 2𝐴+2𝐵 2𝐴−2𝐵
= 1 − (2 𝑐𝑜𝑠 ( ) 𝑐𝑜𝑠 ( )) − 1 + cos2 𝐶 [∵by *7
(i)𝒔𝒊𝒏𝟐𝑨 + 𝒔𝒊𝒏𝟐𝑩 + 𝑺𝒊𝒏𝟐𝑪 = 𝟒 𝒔𝒊𝒏𝑨 𝑺𝒊𝒏𝑩 𝑺𝒊𝒏𝑪 2 2 2

Solution : A + B + C = 180° = − cos(𝐴 + 𝐵) cos(𝐴 − 𝐵) 𝐶 + cos2


A+B = 180°-C = − cos(𝐴 + 𝐵) cos(𝐴 − 𝐵) + cos 𝐶 𝐶𝑜𝑠𝐶
C = 180°-(A+B) = − cos(180 − 𝑐) cos(𝐴 − 𝐵) + cos 𝐶 𝐶𝑜𝑠(180 − (𝐴 + 𝐵))
L.H.S : 𝑠𝑖𝑛2𝐴 + 𝑠𝑖𝑛2𝐵 + 𝑆𝑖𝑛2𝐶 = cos 𝐶 cos(𝐴 − 𝐵) − cos 𝐶 𝐶𝑜𝑠(𝐴 + 𝐵)
= (𝑠𝑖𝑛2𝐴 + 𝑠𝑖𝑛2𝐵) + 𝑆𝑖𝑛2𝐶 = cos 𝐶 [cos(𝐴 − 𝐵) − cos(𝐴 + 𝐵)]
2𝐴+2𝐵 2𝐴−2𝐵 𝐴−𝐵+𝐴+𝐵 𝐴+𝐵−(𝐴−𝐵)
= {2 𝑠𝑖𝑛 ( ) 𝑐𝑜𝑠 ( )} + 𝑆𝑖𝑛2𝐶 [∵by *5 = cos C [2 𝑠𝑖𝑛 ( ) sin ( ) [∵by *8
2 2 2 2
= 2 sin(A+B) cos(A-B) + 2 sinC cosC [∵by *9 = cos C [2 sin A sin B]
= 2 sin(180° − 𝐶) cos(A-B) + 2 sinC cosC = 𝟐 𝑺𝒊𝒏 𝑨 𝑺𝒊𝒏𝑩 𝒄𝒐𝒔𝑪 R.H.S Hence Proved
= 2 sinC cos(A-B) + 2 sinC cosC 𝑨 𝑩 𝑩 𝑪 𝑪 𝑨
(𝒗)𝒕𝒂𝒏 𝒕𝒂𝒏 + 𝒕𝒂𝒏 𝒕𝒂𝒏 + 𝒕𝒂𝒏 𝒕𝒂𝒏 = 𝟏
= 2 sinC [cos(A-B) + cos C] 𝟐 𝟐 𝟐 𝟐 𝟐 𝟐
= 2 sinC [cos(A-B) + cos (180-(A+B))] 𝐴 + 𝐵 + 𝐶 = 180
= 2 sinC [cos(A-B) - cos (A+B)] 𝐴 𝐵 𝐶 180
+ + =
= 2 sinC [2 sin(
𝐴−𝐵+(𝐴+𝐵)
) sin (
𝐴+𝐵−(𝐴−𝐵)
)] [∵by *8 2 2 2 2
2 2 𝐴 𝐵 𝐶
= 2 sin C [2 sin A sin B] + = 90 −
2 2 2
= 4 𝑠𝑖𝑛𝐴 𝑆𝑖𝑛𝐵 𝑆𝑖𝑛𝐶 = R.H.S Hence proved 𝐴 𝐵 𝐶
𝑡𝑎𝑛 ( + ) = 𝑡𝑎𝑛 (90 − )
(ii)cosA+cosB-CosC=-1+4 cos cos sin
𝑨 𝑩 𝑪 2 2 2
𝟐 𝟐 𝟐 𝐴 𝐵 𝐶
Solution:L.H.S : cosA+cosB-CosC 𝑡𝑎𝑛 ( + ) = 𝑐𝑜𝑡
2 2 2
= (cosA+cosB)-CosC 𝐴
𝑡𝑎𝑛 +𝑡𝑎𝑛
𝐵
2 2 1
= (2 𝑐𝑜𝑠 (
𝐴+𝐵
) 𝑐𝑜𝑠 (
𝐴−𝐵
)) − cos 𝐶 [∵by *7 𝐴 𝐵 = 𝑐 [∵by *14
2 2 1−𝑡𝑎𝑛 +𝑡𝑎𝑛 𝑡𝑎𝑛
2 2 2

=(2 𝑐𝑜𝑠 (90 − ) 𝑐𝑜𝑠 (


𝑐 𝐴−𝐵
)) − (1 − 2 sin2 )
𝐶
[∵by *11 𝐴 𝐶 𝐵 𝐶 𝐴 𝐵
2 2 2 𝑡𝑎𝑛 𝑡𝑎𝑛 + 𝑡𝑎𝑛 𝑡𝑎𝑛 = 1 − 𝑡𝑎𝑛 𝑡𝑎𝑛
𝐶 𝐴−𝐵 𝐶
2 2 2 2 2 2
=2 𝑠𝑖𝑛 𝑐𝑜𝑠 ( ) − 1 + 2 sin2 𝐴 𝐶 𝐵 𝐶 𝐴 𝐵
2 2 2 𝑡𝑎𝑛 𝑡𝑎𝑛 + 𝑡𝑎𝑛 𝑡𝑎𝑛 + 𝑡𝑎𝑛 𝑡𝑎𝑛 = 1
𝐶 𝐴−𝐵 𝐶 2 2 2 2 2 2
=−1 + 2 𝑠𝑖𝑛 𝑐𝑜𝑠 ( ) + 2 sin2
2 2 2 𝑨 𝑩 𝑪
𝐶 𝐴−𝐵 𝐶 (𝒗𝒊)𝒔𝒊𝒏𝑨 + 𝒔𝒊𝒏𝑩 + 𝒔𝒊𝒏𝒄 = 𝟒 𝒄𝒐𝒔 𝒄𝒐𝒔 𝒄𝒐𝒔
=−1 + 2 𝑠𝑖𝑛 (𝑐𝑜𝑠 ( ) + 𝑠𝑖𝑛 ) 𝟐 𝟐 𝟐
2 2 2
𝐶 𝐴−𝐵 𝐴+𝐵 L.H.S : (𝒔𝒊𝒏𝑨 + 𝒔𝒊𝒏𝑩) + 𝒔𝒊𝒏𝒄
=−1 + 2 𝑠𝑖𝑛 (𝑐𝑜𝑠 ( ) + 𝑠𝑖𝑛 (90 − )) 𝐴+𝐵 𝐴−𝐵
2 2 2 = (2 𝑠𝑖𝑛 𝑐𝑜𝑠 )+sinC [∵by *5
2 2
𝐶 𝐴−𝐵 𝐴+𝐵 𝑐 𝐴−𝐵 𝑐 𝑐
=−1 + 2 𝑠𝑖𝑛 (𝑐𝑜𝑠 ( ) + 𝑐𝑜𝑠 ( )) = 2 sin (90 − ) 𝑐𝑜𝑠 + 2 sin cos [∵by *10
2 2 2 2 2 2 2
(
𝐴−𝐵
)+(
𝐴+𝐵
)
𝐴−𝐵
( )+(
𝐴+𝐵
) 𝑐 𝐴−𝐵 𝑐 𝑐
𝐶 2 2 2 2 = 2 sin 𝑐𝑜𝑠 + 2 sin cos
=−1 + 2 𝑠𝑖𝑛 (2𝑐𝑜𝑠 ( ) 𝑐𝑜𝑠 ( ))[∵by *7 2 2 2 2
2 2 2 𝑐 𝐴−𝐵 𝑐
= 2 sin (𝑐𝑜𝑠 + sin )
𝐶 𝐴 𝐵 2 2 2
=−1 + 2 𝑠𝑖𝑛 (2𝑐𝑜𝑠 𝑐𝑜𝑠 ) 𝑐 𝐴−𝐵 𝐴+𝐵
2 2 2 = 2 sin (𝑐𝑜𝑠 + sin (90 − ))
𝐴 𝐵 𝐶 2 2 2
= -1+4 cos cos sin [ R.H.S Hence Proved
2 2 2
𝑐
= 2 sin (𝑐𝑜𝑠
𝐴−𝐵
+ cos
𝐴+𝐵
) 2𝑥 2𝑦 2𝑧 2𝑥 2𝑦 2𝑧
2 2 2 2 + 2 + 2 = . .
𝐴−𝐵 𝐴+𝐵 𝐴−𝐵 𝐴+𝐵 1−𝑥 1−𝑦 1−𝑧 1 − 𝑥 1 − 𝑦 1 − 𝑧2
2 2
𝑐 + −
= 2 sin (2 𝑐𝑜𝑠 ( 2 2
) cos ( 2 2
)) [∵by *7 Hence Proved
2 2 2
𝝅
𝑐 𝐴 𝐵 4.𝑰𝒇 𝑨 + 𝑩 + 𝑪 = , 𝑷𝒓𝒐𝒗𝒆 𝒕𝒉𝒂𝒕 𝒕𝒉𝒆 𝒇𝒐𝒍𝒍𝒐𝒘𝒊𝒏𝒈
= 2 sin (2 𝑐𝑜𝑠 cos ) 𝟐
2 2 2 (𝒊)𝒔𝒊𝒏𝟐𝑨 + 𝒔𝒊𝒏𝟐𝑩 + 𝒔𝒊𝒏𝟐𝑪 = 𝟒 𝒄𝒐𝒔𝑨 𝒄𝒐𝒔𝑩 𝒄𝒐𝒔𝑪
𝑨 𝑩 𝑪
= 𝟒 𝒄𝒐𝒔 𝒄𝒐𝒔 𝒄𝒐𝒔 R.H.S Hence Proved 𝑠𝑖𝑛2𝐴 + 𝑠𝑖𝑛2𝐵 + 𝑆𝑖𝑛2𝐶
𝟐 𝟐 𝟐
(𝒗𝒊𝒊)𝒔𝒊𝒏(𝑩 + 𝑪 − 𝑨) + 𝒔𝒊𝒏(𝑪 + 𝑨 − 𝑩) + 𝒔𝒊𝒏(𝑨 + 𝑩 − 𝑪) = (𝑠𝑖𝑛2𝐴 + 𝑠𝑖𝑛2𝐵) + 𝑆𝑖𝑛2𝐶
2𝐴+2𝐵 2𝐴−2𝐵
= 𝟒 𝒔𝒊𝒏 𝑨 𝒔𝒊𝒏 𝑩 𝒔𝒊𝒏𝑪 = {2 𝑠𝑖𝑛 ( ) 𝑐𝑜𝑠 ( )} + 𝑆𝑖𝑛2𝐶 [∵by *5
2 2
L.H.S : 𝑠𝑖𝑛(𝐵 + 𝐶 − 𝐴) + 𝑠𝑖𝑛(𝐶 + 𝐴 − 𝐵) + 𝑠𝑖𝑛(𝐴 + 𝐵 − 𝐶) = 2 sin(A+B) cos(A-B) + 2 sinC cosC [∵by *9
= 𝑠𝑖𝑛(180 − 𝐴 − 𝐴) + 𝑠𝑖𝑛(180 − 𝐵 − 𝐵) + 𝑠𝑖𝑛(180 − 𝐶 − 𝐶) 𝜋
= 2 sin( − 𝐶) cos(A-B) + 2 sinC cosC
= 𝑠𝑖𝑛(180 − 2𝐴) + 𝑠𝑖𝑛(180 − 2𝐵) + 𝑠𝑖𝑛(180 − 2𝐶) 2
= 2 cosC cos(A-B) + 2 sinC cosC
= sin 2A+ sin 2B+ sin 2C
= 2 cosC [cos(A-B) + sin C]
= (𝑠𝑖𝑛2𝐴 + 𝑠𝑖𝑛2𝐵) + 𝑆𝑖𝑛2𝐶 𝜋
2𝐴+2𝐵 2𝐴−2𝐵 = 2 cosC [cos(A-B) + sin ( -(A+B))]
= {2 𝑠𝑖𝑛 ( ) 𝑐𝑜𝑠 ( )} + 𝑆𝑖𝑛2𝐶 [∵by *5 2
2 2 = 2 cosC [cos(A-B) + cos (A+B)]
= 2 sin(A+B) cos(A-B) + 2 sinC cosC [∵by *9 𝐴−𝐵+(𝐴+𝐵) 𝐴−𝐵−(𝐴+𝐵)
= 2 sin(180° − 𝐶) cos(A-B) + 2 sinC cosC = 2 cosC [2 cos( ) cos ( )] [∵by *7
2 2
= 2 sinC cos(A-B) + 2 sinC cosC = 2 sin C [2 cos A cos B]
= 2 sinC [cos(A-B) + cos C] = 4 𝑐𝑜𝑠𝐴 𝑐𝑜𝑠𝐵 𝑐𝑜𝑠𝐶
= 2 sinC [cos(A-B) + cos (180-(A+B))] (𝒊𝒊)𝒄𝒐𝒔𝟐𝑨 + 𝒄𝒐𝒔 𝟐𝑩 + 𝒄𝒐𝒔 𝟐𝒄 = 𝟏 + 𝟒𝒔𝒊𝒏𝑨 𝒔𝒊𝒏𝑩 𝒔𝒊𝒏𝑪
= 2 sinC [cos(A-B) - cos (A+B)] L.H.S : (𝑐𝑜𝑠2𝐴 + 𝑐𝑜𝑠 2𝐵) + 𝑐𝑜𝑠 2𝑐
𝐴−𝐵+(𝐴+𝐵) 𝐴+𝐵−(𝐴−𝐵) 2𝐴+2𝐵 2𝐴−2𝐵
= 2 sinC [2 sin( ) sin ( )] [∵by *8 =2𝑐𝑜𝑠 ( ) 𝑐𝑜𝑠 ( ) + cos 2𝐶 [∵by *7
2 2 2 2
= 2 sin C [2 sin A sin B] =2 cos(𝐴 + 𝐵) 𝑐𝑜𝑠(𝐴 − 𝐵) + 1 − 2 sin2 𝐶
= 4 𝑠𝑖𝑛𝐴 𝑆𝑖𝑛𝐵 𝑆𝑖𝑛𝐶 = R.H.S Hence proved 𝜋
=1+2 cos ( − 𝐶) 𝑐𝑜𝑠(𝐴 − 𝐵) − 2 sin2 𝐶
2
𝟐. 𝑰𝒇 𝑨 + 𝑩 + 𝑪 = 𝟐𝒔, 𝑷𝒓𝒐𝒗𝒆 𝒕𝒉𝒂𝒕
=1+2 sin 𝐶 𝑐𝑜𝑠(𝐴 − 𝐵) − 2 sin2 𝐶
𝑺𝒊𝒏(𝒔 − 𝑨)𝒔𝒊𝒏(𝒔 − 𝑩) + 𝒔𝒊𝒏 𝒔 𝒔𝒊𝒏(𝒔 − 𝑪) = 𝒔𝒊𝒏𝑨 𝒔𝒊𝒏𝑩 =1+2 sin 𝐶 [𝑐𝑜𝑠(𝐴 − 𝐵) − 𝑆𝑖𝑛 𝐶]
L.H.S : 𝑆𝑖𝑛(𝑠 − 𝐴)𝑠𝑖𝑛(𝑠 − 𝐵) + 𝑠𝑖𝑛 𝑠 𝑠𝑖𝑛(𝑠 − 𝐶) 𝜋
1 =1+2 sin 𝐶 [𝑐𝑜𝑠(𝐴 − 𝐵) − 𝑆𝑖𝑛 ( − (A + B)]
= [𝑐𝑜𝑠(𝑠 − 𝐵 − (𝑠 − 𝐴)) − cos(𝑠 − 𝐴 + 𝑠 − 𝐵)] + 2
2 =1+2 sin 𝐶 [𝑐𝑜𝑠(𝐴 − 𝐵) − 𝑐𝑜𝑠(A + B)]
1
[𝑐𝑜𝑠(𝑠 − (𝑠 − 𝐶)) − cos(𝑠 + 𝑠 − 𝐶)] [∵by *4 =1+2 sin 𝐶 [2 𝑠𝑖𝑛 (
𝐴−𝐵+𝐴+𝐵
) 𝑠𝑖𝑛 (
𝐴+𝐵−(𝐴−𝐵)
)]
2
1 2 2
= {𝑐𝑜𝑠(−(𝐴 + 𝐵)) − cos(2𝑠 − 𝐴 − 𝐵) + cos 𝐶 − cos(2𝑠 − 𝐶)} =1+2 sin 𝐶 [2 𝑠𝑖𝑛𝐴 𝑠𝑖𝑛𝐵]
2
1 =1+4 sinA sin B sin C Hence proved.
= {𝑐𝑜𝑠(𝐴 + 𝐵) − cos(2𝑠 − 𝐴 − 𝐵) + cos 𝐶 − cos(2𝑠 − 𝐶)}
2
1 𝝅
= {𝑐𝑜𝑠(𝐴 + 𝐵) − cos 𝐶 + cos 𝐶 − cos(𝐴 + 𝐵)} 𝟓. 𝑰𝒇 ∆𝑨𝑩𝑪 𝒊𝒔 𝒂 𝒓𝒊𝒈𝒉𝒕 𝒂𝒏𝒈𝒍𝒆 𝒕𝒓𝒊𝒂𝒏𝒈𝒍𝒆 𝒂𝒏𝒅 𝒊𝒇 ∠𝑨 = ,
2 𝟐
[∵ 2𝑠 = 𝐴 + 𝐵 + 𝐶 𝒕𝒉𝒆𝒏 𝒑𝒓𝒐𝒗𝒆 𝒕𝒉𝒂𝒕 (𝒊) 𝐜𝐨𝐬 𝟐 𝑩 + 𝐜𝐨𝐬 𝟐 𝑪 = 𝟏
2𝑠 − 𝐴 − 𝐵 = 𝐶 (ii) 𝐬𝐢𝐧𝟐 𝑩 + 𝐬𝐢𝐧𝟐 𝑪 = 𝟏 (iii) cosB-cos C=-1+√𝟐cos sin
𝑩 𝑩
2𝑠 − 𝐶 = 𝐴 + 𝐵 𝟐 𝟐
𝝅 𝝅
1 Since ∠𝑨 = ⟹ ∠𝑩 + ∠𝑪 =
= {𝑐𝑜𝑠(𝐴 + 𝐵) − cos(𝐴 + 𝐵)} 𝟐 𝟐
2 𝜋 𝐵+𝐶 𝜋/2 𝜋
1 i.e, C= − 𝐵 ; = =
= {2𝑠𝑖𝑛𝐴 𝑠𝑖𝑛𝐵} 2 2 2 4
2 𝜋
=sinA sinB R.H.S Hence proved (𝑖) cos2 𝐵 + cos2 𝐶 = cos2 𝐵 + cos2 ( − 𝐵)
2
𝟑. 𝑰𝒇 𝒙 + 𝒚 + 𝒛 = 𝒙𝒚𝒛 𝒑𝒓𝒐𝒗𝒆 𝒕𝒉𝒂𝒕 = cos2 𝐵 + sin2 𝐵 = 1
𝟐𝒙 𝟐𝒚 𝟐𝒛 𝟐𝒙 𝟐𝒚 𝟐𝒛 𝜋
+ + = . . (ii) sin2 𝐵 + sin2 𝐶 = sin2 𝐵 + sin2 ( − 𝐵)
2
𝟏 − 𝒙𝟐 𝟏 − 𝒚𝟐 𝟏 − 𝒛𝟐 𝟏 − 𝒙𝟐 𝟏 − 𝒚𝟐 𝟏 − 𝒛𝟐 = sin2 𝐵 + cos2 𝐵 = 1
Solution : 𝑙𝑒𝑡 𝑥 = 𝑡𝑎𝑛𝐴 ; 𝑦 = 𝑡𝑎𝑛𝐵 ; 𝑧 = 𝑡𝑎𝑛𝐶 𝐵+𝐶 𝐶−𝐵
(iii)cos B- cos C= 2 𝑠𝑖𝑛 ( ) sin ( )
Given 𝑥 + 𝑦 + 𝑧 = 𝑥𝑦𝑧 2 2
𝜋 𝐶−𝐵
𝑡𝑎𝑛𝐴 + 𝑡𝑎𝑛𝐵 + 𝑡𝑎𝑛𝐶 = tan 𝐴 tan 𝐵 tan 𝐶 = 2 𝑠𝑖𝑛 sin ( )+1−1
4 2
𝑡𝑎𝑛𝐴 + 𝑡𝑎𝑛𝐵 = tan 𝐴 tan 𝐵 tan 𝐶 − 𝑡𝑎𝑛𝐶 𝜋 𝐶−𝐵 √2
𝑡𝑎𝑛𝐴 + 𝑡𝑎𝑛𝐵 = −𝑡𝑎𝑛𝐶(1 − tan 𝐴 tan 𝐵) = 2 𝑠𝑖𝑛 sin ( )+1 −1
4 2 √2
𝑡𝑎𝑛𝐴 + 𝑡𝑎𝑛𝐵 𝜋 𝐶 𝐵 1
= −𝑡𝑎𝑛𝐶 =2 𝑠𝑖𝑛 sin ( − ) + √2 −1
4 2 2 √2
1 − tan 𝐴 tan 𝐵 1 𝐶 𝐵 𝜋
tan(𝐴 + 𝐵) = tan(−𝐶) =2 sin ( − ) + √2𝑠𝑖𝑛 − 1
√2 2 2 4
𝐴 + 𝐵 = 𝑛𝜋 + (−𝐶) 𝐶
=√2 sin ( − ) + √2𝑠𝑖𝑛 − 1
𝐵 𝜋
𝐴 + 𝐵 + 𝐶 = 𝑛𝜋 , Where n∈ 𝐼 2 2 4
𝐶 𝐵 𝐶 𝐵
𝐴+𝐵+𝐶 = 𝜋 =-1+√2 [𝑠𝑖𝑛 ( + ) + sin ( − )]
2 2 2 2
2𝐴 + 2𝐵 + 2𝐶 = 2𝜋 𝐶 𝐵 𝐶 𝐵
+ + −
𝐶 𝐵 𝐶 𝐵
+ − +
2 2 2 2 2 2 2 2
2𝐴 + 2𝐵 = 2𝜋 − 2𝐶 =-1 + √2 [2 𝑠𝑖𝑛 ( ) cos ( )]
2 2
tan(2𝐴 + 2𝐵)=tan(180 − 2𝐶) 𝐶 𝐵
tan 2𝐴+𝑡𝑎𝑛2𝐵 = -1 + √2 [2 𝑠𝑖𝑛 cos ]
=-tan2C 2 2
1−𝑡𝑎𝑛2𝐴 𝑡𝑎𝑛2𝐵 𝐵 𝐶
= -1 + 2√2 cos 𝑠𝑖𝑛 Hence Proved
tan 2𝐴 + 𝑡𝑎𝑛2𝐵 = −𝑡𝑎𝑛2𝑐 + 𝑡𝑎𝑛2𝐴 𝑡𝑎𝑛2𝐵 𝑡𝑎𝑛2𝑐 2 2
tan 2𝐴 + 𝑡𝑎𝑛2𝐵 + 𝑡𝑎𝑛2𝑐 = 𝑡𝑎𝑛2𝐴 𝑡𝑎𝑛2𝐵 𝑡𝑎𝑛2𝑐
2𝑡𝑎𝑛𝐴 2𝑡𝑎𝑛𝐵 2𝑡𝑎𝑛𝐶
+ +
1 − tan2 𝐴 1 − tan2 𝐵 1 − tan2 𝐶
2𝑡𝑎𝑛𝐴 2𝑡𝑎𝑛𝐵 2𝑡𝑎𝑛𝐶
= . .
1 − tan2 𝐴 1 − tan2 𝐵 1 − tan2 𝐶
2.Solve the following equations for which solutions lies in the
PV MATRIC HR SEC SCHOOL interval 𝟎° ≤ 𝜽 < 𝟑𝟔𝟎°
MATHEMATICS - XI STD (i)𝐬𝐢𝐧𝟒 𝒙 = 𝐬𝐢𝐧𝟐 𝒙
Solution : sin 𝑥 = sin 𝑥
sin 𝑥 − sin 𝑥 = 0
sin 𝑥 (sin 𝑥 − 1) = 0
sin 𝑥 (sin 𝑥 + 1)(sin 𝑥 − 1) = 0
sin 𝑥 = 0 , sin 𝑥 + 1 = 0 , sin 𝑥 − 1 = 0
(a) sin 𝑥 = 0
sin 𝑥 = 0
𝑥 = 𝑛𝜋 𝑊ℎ𝑒𝑟𝑒 𝑛𝜖𝑍
𝐼𝑓 𝑛 = 0,1 𝑤𝑒 𝑔𝑒𝑡 𝑥 = 0, 𝜋 𝜖 0° ≤ 𝑥 < 360°
(b) 𝑠𝑖𝑛𝑥 + 1 = 0
𝑠𝑖𝑛𝑥 = −1
𝑠𝑖𝑛𝑥 = −𝑠𝑖𝑛
𝑠𝑖𝑛𝑥 = 𝑠𝑖𝑛 −
Principal value 𝛼 = −
EXERCISE 3.8 Now 𝑥 = 𝑛𝜋 + (−1) (− )
1. Find the principal solution and general solutions of the
3𝜋
following: 𝐼𝑓 𝑛 = 1 𝑤𝑒 𝑔𝑒𝑡 𝑥 = 𝜖 0° ≤ 𝑥 < 360°
𝟏 2
(i) 𝑺𝒊𝒏 𝜽 = −
√𝟐 (c) 𝑠𝑖𝑛𝑥 − 1 = 0
𝝅 𝝅
Solution : Principal Value lies in − , 𝑠𝑖𝑛𝑥 = 1
𝟐 𝟐
𝟏 𝑠𝑖𝑛𝑥 = 𝑠𝑖𝑛
sin 𝜃 = −
√𝟐
Principal value 𝛼 =
sin 𝜃 = − sin
𝜋 Now 𝑥 = 𝑛𝜋 + (−1) ( )
sin 𝜃 = sin − 𝜋
4 𝐼𝑓 𝑛 = 0 𝑤𝑒 𝑔𝑒𝑡 𝑥 = 𝜖 0° ≤ 𝑥 < 360°
𝑷𝒓𝒊𝒏𝒄𝒊𝒑𝒂𝒍 𝒔𝒐𝒍𝒖𝒕𝒊𝒐𝒏: ∴ 𝛼 = − 𝑖𝑠 𝑡ℎ𝑒 𝑃𝑟𝑖𝑛𝑐𝑖𝑝𝑎𝑙 𝑠𝑜𝑙𝑢𝑡𝑖𝑜𝑛. 2
𝑮𝒆𝒏𝒆𝒓𝒂𝒍 𝑺𝒐𝒍𝒖𝒕𝒊𝒐𝒏 ∶ Finally 𝑥 = 0, , 𝜋,
𝐼𝑓 𝑆𝑖𝑛 𝜃 = 𝑠𝑖𝑛 𝛼 ⇒ 𝜃 = 𝑛𝜋 + (−1) 𝛼 ; 𝑊ℎ𝑒𝑟𝑒 𝑛 ∈ 𝑍 (𝒊𝒊)𝟐 𝐜𝐨𝐬𝟐 𝒙 + 𝟏 = −𝟑𝒄𝒐𝒔 𝒙
𝜋 Solution : 2 cos 𝑥 + 1 = −3𝑐𝑜𝑠 𝑥
2
sin 𝜃 = sin −
4 2 cos 𝑥 + 3𝑐𝑜𝑠 𝑥 + 1 =0 3
𝜃 = 𝑛𝜋 + (−1) − where 𝑛 ∈ 𝑍
(cos 𝑥 + 1) cos 𝑥 + =0
(ii) 𝒄𝒐𝒕 𝜽 = √𝟑 (a) cos 𝑥 + 1 = 0
Solution : 𝑐𝑜𝑡 𝜃 = √3 cos 𝑥 = −1 (b) cos 𝑥 + = 0
tan 𝜃 = cos 𝑥 = − cos 0 cos 𝑥 = −

𝝅 𝝅 cos 𝑥 = cos(𝜋 − 0) cos 𝑥 = − cos
Principal Value lies in − , cos 𝑥 = cos 𝜋
𝟐 𝟐
Principal solution 𝛼 = 𝜋 cos 𝑥 = cos(𝜋 − )
tan 𝜃 =

General solution: cos 𝑥 = cos
tan 𝜃 = tan
𝑥 = 2𝑛𝜋 ± 𝜋 ; n∈ 𝑍 Principal solution 𝛼 =
𝑷𝒓𝒊𝒏𝒄𝒊𝒑𝒂𝒍 𝒔𝒐𝒍𝒖𝒕𝒊𝒐𝒏: ∴ 𝛼= 𝑖𝑠 𝑡ℎ𝑒 𝑃𝑟𝑖𝑛𝑐𝑖𝑝𝑎𝑙 𝑠𝑜𝑙𝑢𝑡𝑖𝑜𝑛. If n=0 𝑥 = ±𝜋
𝑮𝒆𝒏𝒆𝒓𝒂𝒍 𝑺𝒐𝒍𝒖𝒕𝒊𝒐𝒏 ∶ General solution:
𝐼𝑓 𝑛 = 1 𝑥 = 2𝜋 ± 𝜋
𝐼𝑓 𝑡𝑎𝑛 𝜃 = 𝑡𝑎𝑛 𝛼 ⇒ 𝜃 = 𝑛𝜋 + 𝛼 ; 𝑊ℎ𝑒𝑟𝑒 𝑛 ∈ 𝑍 𝑊ℎ𝑒𝑟𝑒 𝑥 = 𝜋 ∈ [0,2𝜋) 𝑥 = 2𝑛𝜋 ± ; n∈ 𝑍
𝜋 If n=0 𝑥=±
tan 𝜃 = tan
6 2𝜋
𝜃 = 𝑛𝜋 + ; 𝑊ℎ𝑒𝑟𝑒 𝑛 ∈ 𝑍 𝐼𝑓 𝑛 = 1 𝑥 = 2𝜋 ±
3
(ii)tan 𝜽 = − 2𝜋 4𝜋
√ 𝑊ℎ𝑒𝑟𝑒 𝑥 = , ∈ [0,2𝜋)
3 3
Solution : tan 𝜃 = −
√ (𝒊𝒊𝒊) 𝟐 𝐬𝐢𝐧𝟐 𝒙 + 𝟏 = 𝟑 𝒔𝒊𝒏 𝒙
𝝅 𝝅 2
Principal Value lies in − , Solution : 2 sin 𝑥 + 1 = 3 𝑠𝑖𝑛 𝑥
𝟐 𝟐 1  3  2
2 sin 𝑥 − 3 𝑠𝑖𝑛 𝑥 + 1 = 0 1 2
tan 𝜃 = −  
√ 1 2 2
tan 𝜃 = − tan 𝑠𝑖𝑛𝑥 − (sin 𝑥 − 1) = 0 1
2  1
a) 𝑠𝑖𝑛𝑥 − = 0 2
tan 𝜃 = tan(− ) b) 𝑠𝑖𝑛𝑥 − 1 = 0
𝑷𝒓𝒊𝒏𝒄𝒊𝒑𝒂𝒍 𝒔𝒐𝒍𝒖𝒕𝒊𝒐𝒏: ∴ 𝛼=− 𝑖𝑠 𝑡ℎ𝑒 𝑃𝑟𝑖𝑛𝑐𝑖𝑝𝑎𝑙 𝑠𝑜𝑙𝑢𝑡𝑖𝑜𝑛. 1 𝑠𝑖𝑛𝑥 = 1
𝑠𝑖𝑛𝑥 =
2 𝑠𝑖𝑛𝑥 = sin
𝑮𝒆𝒏𝒆𝒓𝒂𝒍 𝑺𝒐𝒍𝒖𝒕𝒊𝒐𝒏 ∶
𝑠𝑖𝑛𝑥 = sin
𝐼𝑓 𝑡𝑎𝑛 𝜃 = 𝑡𝑎𝑛 𝛼 ⇒ 𝜃 = 𝑛𝜋 + 𝛼 ; 𝑊ℎ𝑒𝑟𝑒 𝑛 ∈ 𝑍 𝑃𝑟. 𝑆𝑜𝑙. 𝛼 = ;
𝜋 𝑃𝑟. 𝑆𝑜𝑙. 𝛼 = ; G. Sol. x=𝑛𝜋 + (−1)
tan 𝜃 = tan(− ) G. Sol.x=𝑛𝜋 + (−1)
6 𝜋 5𝜋
𝑖𝑓 𝑛 = 0,1 𝑥 = , ∈ [0,2𝜋) 𝜋
𝜃 = 𝑛𝜋 − ; 𝑊ℎ𝑒𝑟𝑒 𝑛 ∈ 𝑍 6 6 𝑖𝑓 𝑛 = 0,1 𝑥 = ∈ [0,2𝜋)
2
(𝒊𝒗) 𝒄𝒐𝒔 𝟐𝒙 = 𝟏 − 𝟑𝒔𝒊𝒏𝒙 (v) 𝒔𝒊𝒏 𝟐𝜽 − 𝒄𝒐𝒔𝟐𝜽 − 𝒔𝒊𝒏𝜽 + 𝒄𝒐𝒔𝜽 = 𝟎
Solution : 𝑐𝑜𝑠 2𝑥 = 1 − 3𝑠𝑖𝑛𝑥 Solution : 𝑠𝑖𝑛 2𝜃 − 𝑠𝑖𝑛𝜃 + 𝑐𝑜𝑠𝜃 − 𝑐𝑜𝑠2𝜃 = 0
1 − 2 sin 𝑥 − 1 = −3𝑠𝑖𝑛𝑥 2𝑐𝑜𝑠 𝑠𝑖𝑛 + 2 𝑠𝑖𝑛 𝑠𝑖𝑛 =0
2 sin 𝑥 + 3𝑠𝑖𝑛𝑥 = 0
𝑠𝑖𝑛𝑥 (2 sin 𝑥 − 3) = 0 2𝑐𝑜𝑠 𝑠𝑖𝑛 + 2𝑠𝑖𝑛 𝑠𝑖𝑛 =0
𝑎) 𝑠𝑖𝑛𝑥 = 0 𝑏) 2 sin 𝑥 − 3 = 0
2𝑠𝑖𝑛 𝑐𝑜𝑠 + 𝑠𝑖𝑛 =0
𝑥 = 𝑛𝜋 ; 𝑛 ∈ 𝑍 𝑠𝑖𝑛𝑥 =
If 𝑛 = 0,1 𝑥 = 0, 𝜋 ∈ [0,2𝜋) −1 ≤ 𝑠𝑖𝑛𝑥 ≤ 1 ∴No solution
a) 2𝑠𝑖𝑛 =0 𝑏)𝑐𝑜𝑠 + 𝑠𝑖𝑛 =0
3. Solve the following equations:
(i) 𝒔𝒊𝒏 𝟓𝒙 − 𝒔𝒊𝒏 𝒙 = 𝒄𝒐𝒔 𝟑𝒙 𝑠𝑖𝑛 =0 𝑠𝑖𝑛 = −𝑐𝑜𝑠
Solution : 𝑠𝑖𝑛 5𝑥 − 𝑠𝑖𝑛 𝑥 = 𝑐𝑜𝑠 3𝑥
= 𝑛𝜋 = −1
2 𝑐𝑜𝑠 𝑠𝑖𝑛 = cos 3𝑥
2 𝑐𝑜𝑠3𝑥 𝑠𝑖𝑛2𝑥 = cos 3𝑥 𝜃 = 2𝑛𝜋 𝑛∈𝑧 tan = −𝑡𝑎𝑛
2 𝑐𝑜𝑠3𝑥 𝑠𝑖𝑛2𝑥 − cos 3𝑥 = 0
tan = tan(− )
𝑐𝑜𝑠3𝑥 (2 𝑠𝑖𝑛2𝑥 − 1) = 0
a) 𝑐𝑜𝑠3𝑥 = 0 b) 2 𝑠𝑖𝑛2𝑥 − 1 = 0 Principal value 𝛼 = −
3𝑥 = (2𝑛 + 1) 𝑠𝑖𝑛2𝑥 = G.S ; = 𝑛𝜋 −
𝑥 = (2𝑛 + 1) 𝑊ℎ𝑒𝑟𝑒 𝑛 ∈ 𝑍 𝑠𝑖𝑛2𝑥 = 𝑠𝑖𝑛 𝜃= − 𝑛∈𝑍
Principal solution : 2x=
(vi)𝒔𝒊𝒏𝜽 + 𝒄𝒐𝒔𝜽 = √𝟐
Gen.sol : 2𝑥 = 𝑛𝜋 + (−1) Solution : 𝑠𝑖𝑛𝜃 + 𝑐𝑜𝑠𝜃 = √2
𝑥 = 𝑛𝜋 + (−1) Dividing by √2 We get,
𝟐
(ii)𝟐 𝐜𝐨𝐬 𝜽 + 𝟑𝒔𝒊𝒏𝜽 − 𝟑 = 𝟎 𝑠𝑖𝑛𝜃 + 𝑐𝑜𝑠𝜃 = 1
√ √
Solution : 2 cos 𝜃 + 3𝑠𝑖𝑛𝜃 − 3 = 0
𝑠𝑖𝑛𝜃 + 𝑐𝑜𝑠𝜃 = 1
2(1 − sin 𝜃) + 3𝑠𝑖𝑛𝜃 − 3 = 0 √ √
2 𝑠𝑖𝑛 𝑠𝑖𝑛𝜃 + 𝑐𝑜𝑠 𝑐𝑜𝑠𝜃 = 1
2 − 2 sin 𝜃 + 3𝑠𝑖𝑛𝜃 − 3 = 0
1  3  2
−2 sin 𝜃 + 3𝑠𝑖𝑛𝜃 − 1 = 0 1 2 𝑐𝑜𝑠𝜃 𝑐𝑜𝑠 + 𝑠𝑖𝑛𝜃 𝑠𝑖𝑛 =1
2 sin 𝜃 − 3𝑠𝑖𝑛𝜃 + 1 = 0  
2 2 Cos 𝜃 − =1
𝑠𝑖𝑛𝜃 − (sin 𝜃 − 1) = 0 1
 1
2 Cos 𝜃 − = cos 0
a) 𝑠𝑖𝑛𝜃 − = 0
b) 𝑠𝑖𝑛𝜃 − 1 = 0 Principal value 𝛼 = 0
1 𝑠𝑖𝑛𝜃 = 1
𝑠𝑖𝑛𝜃 = General solution : 𝜃− = 2𝑛𝜋 ± 0
2 𝑠𝑖𝑛𝜃 = sin
𝑠𝑖𝑛𝜃 = sin 𝜃 = 2𝑛𝜋 + ;
𝑃𝑟. 𝑆𝑜𝑙. 𝛼 = ;
𝑃𝑟. 𝑆𝑜𝑙. 𝛼 = ; G. Sol. 𝜃=𝑛𝜋 + (−1) 𝜃 = (8𝑛𝜋 + 1) ; 𝑛∈𝑍
G. Sol. 𝜃=𝑛𝜋 + (−1)
_________________________________________________________
(iii) 𝒄𝒐𝒔𝜽 + 𝐜𝐨𝐬 𝟑𝜽 = 𝟐 𝐜𝐨𝐬 𝟐𝜽 (vii) 𝒔𝒊𝒏 𝜽 + √𝟑 𝒄𝒐𝒔 𝜽 = 𝟏
Solution : 𝑐𝑜𝑠𝜃 + cos 3𝜃 = 2 cos 2𝜃 Solution : 𝑠𝑖𝑛 𝜃 + √3 𝑐𝑜𝑠 𝜃 = 1
𝑐𝑜𝑠3𝜃 + cos 𝜃 = 2 cos 2𝜃 √
Divided by 2 𝑠𝑖𝑛 𝜃 + 𝑐𝑜𝑠 𝜃 =
2 𝑐𝑜𝑠 𝑐𝑜𝑠 = 2 𝑐𝑜𝑠2𝜃 𝑠𝑖𝑛 𝑠𝑖𝑛 𝜃 + 𝑐𝑜𝑠 𝑐𝑜𝑠 𝜃 =
2 𝑐𝑜𝑠2𝜃 𝑐𝑜𝑠𝜃 = 2 𝑐𝑜𝑠2𝜃
𝑐𝑜𝑠 𝜃 𝑐𝑜𝑠 + 𝑠𝑖𝑛 𝜃 𝑠𝑖𝑛 =
2 𝑐𝑜𝑠2𝜃 𝑐𝑜𝑠𝜃 − 2 𝑐𝑜𝑠2𝜃 = 0
2 𝑐𝑜𝑠2𝜃 (𝑐𝑜𝑠𝜃 − 1) = 0 Cos 𝜃 − =
a) 2 𝑐𝑜𝑠2𝜃 = 0 b) 𝑐𝑜𝑠𝜃 − 1 = 0 Cos 𝜃 − = 𝑐𝑜𝑠
𝑐𝑜𝑠2𝜃 = 0 𝑐𝑜𝑠𝜃 = 1
2𝜃 = (2𝑛 + 1) Principal value 𝛼 =
𝑐𝑜𝑠𝜃 = cos 0
Principal Solution = 0 ; General solution : 𝜃− = 2𝑛𝜋 ±
𝜃 = (2𝑛 + 1) n∈ 𝑧
General Solution 𝜃=2𝑛𝜋 ± 0
𝜃 = 2𝑛𝜋 + ± ; 𝑛∈𝑍
𝜃 = 2𝑛𝜋 n ∈ 𝑧
_______________________________________________________
(viii) 𝒄𝒐𝒕𝜽 + 𝒄𝒐𝒔𝒆𝒄𝜽 = √𝟑
(iv)𝒔𝒊𝒏𝜽 + 𝒔𝒊𝒏𝟑𝜽 + 𝒔𝒊𝒏𝟓𝜽 = 𝟎
Solution : 𝑐𝑜𝑡𝜃 + 𝑐𝑜𝑠𝑒𝑐𝜃 = √3
Solution : 𝑠𝑖𝑛𝜃 + 𝑠𝑖𝑛3𝜃 + 𝑠𝑖𝑛5𝜃 = 0
𝑠𝑖𝑛5𝜃 + 𝑠𝑖𝑛𝜃 + 𝑠𝑖𝑛3𝜃 = 0 + = √3
2 𝑠𝑖𝑛 𝑐𝑜𝑠 + 𝑠𝑖𝑛3𝜃 = 0 cos 𝜃 + 1 = √3 sin 𝜃
2 sin 3𝜃 cos 2𝜃 + 𝑠𝑖𝑛3𝜃 = 0 √3 sin 𝜃 − cos 𝜃 = 1

sin 3𝜃 (2cos 2𝜃 + 1) = 0 Divide by 2 sin 𝜃 − cos 𝜃 =
a) sin 3𝜃 = 0 b) 2cos 2𝜃 + 1 = 0 1 √3 1 𝜋 𝜋 1
3𝜃 = 𝑛𝜋 cos 𝜃 − sin 𝜃 = − ⟹ cos 𝜃 cos − sin 𝜃 cos = −
cos 2𝜃 = − 2 2 2 3 3 2
𝑛𝜋 cos 𝜃 + = −𝑐𝑜𝑠 ⇒ cos 𝜃 + = cos 𝜋 − = 𝑐𝑜𝑠
𝜃= ; 𝑛 ∈ 𝑍 cos 2𝜃 = −𝑐𝑜𝑠 ⟹ cos 2𝜃 = cos(𝜋 − )
3 Principal value is 𝛼 = General solution : 𝜃 + = 2𝑛𝜋 ±
cos 2𝜃 = cos ;Principal solution𝛼 =
𝜃 = 2𝑛𝜋 − ± 𝑛∈𝑍
Gen. Solu : 2𝜃 = 2𝑛𝜋 ± ⟹ 𝜃 = 𝑛𝜋 ±
𝝅 𝟐𝝅
(ix) 𝒕𝒂𝒏𝜽 + 𝒕𝒂𝒏 𝜽 + 𝟑 + 𝒕𝒂𝒏 𝜽 + 𝟑
= √𝟑 (ii) 𝐜𝐨𝐬 𝟏 √𝟑
𝟐
Solution : 𝒕𝑎𝑛𝜃 + 𝑡𝑎𝑛 𝜃 + + 𝑡𝑎𝑛 𝜃 + = √3

𝑡𝑎𝑛𝜃 + + = √3
√3 √3 √
𝑡𝑎𝑛𝜃 + + = √3 Principal value of cos is .
√3 √3
𝑡𝑎𝑛𝜃 1 − 3 tan2 𝜃 + tan 𝜃 + √3 1 + √3 tan 𝜃 + (tan 𝜃 − √3) 1 − √3tan 𝜃
= √3 (iii) 𝐜𝐨𝐬𝐞𝐜 (−𝟏)𝟏
1 − √3tan 𝜃 (1 + √3tan 𝜃 )
𝑡𝑎𝑛𝜃 − 3 tan 𝜃 + tan 𝜃 + √3 tan 𝜃 + √3 + 3 tan 𝜃 + tan 𝜃 − √3 tan 𝜃 − √3 + 3 tan 𝜃
= √3
1 − 3 tan2 𝜃
6𝑡𝑎𝑛𝜃 − 3 tan 𝜃
= √3
1 − 3 tan2 𝜃
3(2𝑡𝑎𝑛𝜃 − tan 𝜃)
= √3
1 − 3 tan2 𝜃
2𝑡𝑎𝑛𝜃 − tan 𝜃 1 Principal value of cosec (−1) is .
2
=
1 − 3 tan 𝜃 √3
1 (iv) 𝐒𝐞𝐜 𝟏
(−√𝟐)
𝑡𝑎𝑛3𝜃 =
√3
𝜋
𝑡𝑎𝑛3𝜃 = 𝑡𝑎𝑛
6
Principal Value = ; General Solution : 3𝜃 = 𝑛𝜋 +

𝜃= + 𝑛∈𝑍
√𝟓 𝟏
(x)𝒄𝒐𝒔 𝟐𝜽 =
𝟒

Solution : 𝑐𝑜𝑠 2𝜃 =
𝑐𝑜𝑠 2𝜃 = cos 36° Principal value of Sec (−√2) is .
𝑐𝑜𝑠 2𝜃 = cos
(v) 𝐭𝐚𝐧 𝟏 (√𝟑)
Principal Value is 𝛼 =
General solution : 2𝜃 = 2𝑛𝜋 ±
𝜃 = 𝑛𝜋 ± 𝑛∈𝑍

(xi) 𝟐 𝐜𝐨𝐬𝟐 𝒙 − 𝟕𝒙 + 𝟑 = 𝟎 6


Solution : 2 cos 𝑥 − 7𝑥 + 3 = 0
(𝑐𝑜𝑠𝑥 − 3) 𝑐𝑜𝑠𝑥 − =0  6 7  1
6 1
 
a) cos 𝑥 − 3 = 0 2 2
1
cos 𝑥 = 3 3 
2
Principal value of tan (√3) is .
Range of cosx is [-1,1] ∴ 𝑛𝑜𝑡 𝐴𝑑𝑚𝑖𝑠𝑠𝑎𝑏𝑙𝑒.
02.A man standing directly opposite to one side of a road
b) 𝑐𝑜𝑠𝑥 − = 0
of width x meter views a circular shaped traffic green
𝑐𝑜𝑠𝑥 = signal of diameter a meter on the other side of the road.
𝜋 The bottom of the green signal is b meter height from the
cos 𝑥 = cos horizontal level of viewer’s eye. If 𝛼 denotes the angle
3
𝜋 subtended by the diameter of the green signal at the
𝑃𝑟𝑖𝑛𝑐𝑖𝑝𝑎𝑙 𝑣𝑎𝑙𝑢𝑒 𝑖𝑠
3 viewer’s eye, then prove that 𝛼 = tan − tan .
𝜋
𝐺𝑒𝑛𝑒𝑟𝑎𝑙 𝑠𝑜𝑙𝑢𝑡𝑖𝑜𝑛 𝑥 = 2𝑛𝜋 ± , 𝑛 ∈ 𝑍 Solution :
3
C
EXERCISE 3.11 DC=a [diameter of the green signal]
a Given ∟𝐶𝐵𝐷 = 𝛼 ; 𝐿𝑒𝑡 ∟𝐷𝐵𝐴 = 𝛽
01.Find the principal value of
∟𝐶𝐵𝐴 = 𝛼 + 𝛽
(i) 𝐬𝐢𝐧 𝟏 𝟏 D
√𝟐 DA=b and AB=x

𝛼
b

Principal value of sin is . 𝛽


√ A x B
.
In ∆ ABD , 𝑡𝑎𝑛 𝛽 = = =
𝑏
𝑡𝑎𝑛 𝛽 =
𝑥
𝛽 = tan ---------------(1)
In ∆ ABC, tan(𝛼 + 𝛽) = =
𝛼 + 𝛽 = tan
𝛼 = tan −𝛽
𝛼 = tan − tan [From (1)
Hence Proved .
Creative Questions
1
tan 7 °
2
PV MATRIC HR SEC SCHOOL 𝟒. 𝑰𝒏 𝒂 ∆𝑨𝑩𝑪 , 𝑷𝒓𝒐𝒗𝒆 𝒕𝒉𝒂𝒕
𝒔𝒊𝒏 𝑩 𝒄 − 𝒂 𝒄𝒐𝒔 𝑩
=
𝒔𝒊𝒏 𝑪 𝒃 − 𝒂 𝒄𝒐𝒔 𝑪
MATHEMATICS - XI STD Solution : We know that the following Projection Formula,
𝑐 = 𝑎 𝑐𝑜𝑠𝐵 + 𝑏 𝑐𝑜𝑠 𝐴 ; 𝑏 = 𝑎 𝑐𝑜𝑠𝐶 + 𝑐 𝑐𝑜𝑠 𝐴
EXERCISE 3.9 R.H.S : = [by using above
𝒔𝒊𝒏 𝑨 𝒔𝒊𝒏(𝑨 𝑩) 𝑏 𝑐𝑜𝑠 𝐴
01. In ∆ABC, if
𝒔𝒊𝒄 𝑪
= 𝒔𝒊𝒏(𝑩 𝑪)
, Prove that 𝒂𝟐 , 𝒃𝟐 , 𝒄𝟐 are in =
𝑐 𝑐𝑜𝑠 𝐴
Arithmetic Progression: =
𝒂 𝒃 𝒄
Solution : = = = 𝟐𝑹 = = = L.H.S Hence Proved.
𝑺𝒊𝒏 𝑨 𝑺𝒊𝒏 𝑩 𝑺𝒊𝒏 𝑪
Now 𝑎 = 2𝑅 𝑆𝑖𝑛 𝐴 ; 𝑏 = 2𝑅 𝑆𝑖𝑛 𝐵 ; 𝑐 = 2𝑅 𝑆𝑖𝑛 𝑐 --------(1) 5. 𝑰𝒏 𝒂 ∆𝑨𝑩𝑪 , 𝑷𝒓𝒐𝒗𝒆 𝒕𝒉𝒂𝒕
( ) 𝒂 𝒄𝒐𝒔 𝑨 + 𝒃 𝒄𝒐𝒔 𝑩 + 𝒄 𝒄𝒐𝒔𝑪 = 𝟐𝒂 𝒔𝒊𝒏 𝑩 𝒔𝒊𝒏 𝑪
= ( ) Solution : L.H.S : 𝑎 𝑐𝑜𝑠 𝐴 + 𝑏 𝑐𝑜𝑠 𝐵 + 𝑐 𝑐𝑜𝑠𝐶
𝑠𝑖𝑛(𝐵 − 𝐶) 𝑠𝑖𝑛 𝐴 = 𝑠𝑖𝑛(𝐴 − 𝐵)𝑠𝑖𝑛 𝐶 = (2𝑅𝑠𝑖𝑛 𝐴) 𝑐𝑜𝑠 𝐴 + (2𝑅𝑠𝑖𝑛 𝐵) 𝑐𝑜𝑠 𝐵 + (2𝑅𝑠𝑖𝑛 𝐶) 𝑐𝑜𝑠𝐶
𝑠𝑖𝑛(𝐵 − 𝐶) 𝑠𝑖𝑛 [180 − (𝐵 + 𝐶)] = 𝑠𝑖𝑛(𝐴 − 𝐵) 𝑠𝑖𝑛 [180 − (𝐴 + 𝐵)] = 𝑅[2 sin 𝐴 cos 𝐴 + 2 𝑠𝑖𝑛𝐵 𝐶𝑜𝑠 𝐵 + 2 𝑆𝑖𝑛 𝐶 cos 𝐶]
𝑠𝑖𝑛(𝐵 − 𝐶) 𝑠𝑖𝑛 (𝐵 + 𝐶) = 𝑠𝑖𝑛(𝐴 − 𝐵) 𝑠𝑖𝑛 (𝐴 + 𝐵) = R[ sin 2A+ sin 2B + sin 2C]
sin 𝐵 − sin 𝐶 = sin 𝐴 − sin 𝐵 =R[4 sin A sin B sin C] [By Using Exercise 3.7-1(i)
Multiply by 4R2 on both sides =2 [2R sin A] sin B sin C
4𝑅 sin 𝐵 − 4𝑅 sin 𝐶 = 4𝑅 sin 𝐴 − 4𝑅 sin 𝐵 = 2 a sin B sin C = R.H.S Hence Proved.
𝑩 𝑪
(2𝑅 sin 𝐵) − (2𝑅 sin 𝐶) = (2𝑅 sin 𝐴) − (2𝑅 sin 𝐵) 6. 𝑰𝒏 𝒂 ∆𝑨𝑩𝑪 , ∠𝑨 = 𝟔𝟎°. 𝑷𝒓𝒐𝒗𝒆 𝒕𝒉𝒂𝒕 𝒃 + 𝒄 = 𝟐𝒂 𝒄𝒐𝒔 .
𝟐
𝑏 −𝑐 =𝑎 −𝑏 [From (1) Solution : Given ∠𝐴 = 60° , In ∆𝐴𝐵𝐶 𝐴 + 𝐵 + 𝐶 = 180
𝑖𝑓 𝑡 − 𝑡 = 𝑡 − 𝑡 , 𝑡ℎ𝑒𝑛 𝑡 , 𝑡 , 𝑡 𝑎𝑟𝑒 𝑖𝑛 𝐴. 𝑃 60°+B+C=180
𝑏 −𝑐 =𝑎 −𝑏 B+C=180-60

𝟐
𝒂𝟐 , 𝒃 , 𝒄𝟐 𝒂𝒓𝒆 𝒊𝒏 𝑨𝒓𝒊𝒕𝒉𝒎𝒆𝒕𝒊𝒄 𝑷𝒓𝒐𝒈𝒓𝒆𝒔𝒔𝒊𝒐𝒏 . B+C=120°
𝑏 + 𝑐 = 2𝑅 𝑠𝑖𝑛𝐵 + 2𝑅 𝑠𝑖𝑛𝐶
02. The angles of a triangle ABC, are in Arithmetic Progression = 2𝑅 [𝑠𝑖𝑛𝐵 + 𝑠𝑖𝑛𝐶]
and if b : c =√𝟑:√𝟐 fin ∟𝑨 =?
= 2𝑅 [2 𝑠𝑖𝑛 cos ]
Solution : Given b : c =√3:√2
√ = 2𝑅 [2 𝑠𝑖𝑛 cos ]
=
√ = 4𝑅 𝑠𝑖𝑛 cos
√ = 4𝑅 𝑠𝑖𝑛
°
cos
=

= 4𝑅 𝑠𝑖𝑛(60°) cos

= 4𝑅 𝑠𝑖𝑛𝐴 cos
= -----------(1)
√ = 2 [2𝑅 𝑠𝑖𝑛𝐴] cos
Sine formula = = 2 a cos R.H.S
= -----------(2) 7. In a ∆ ABC , Prove that the following ,
𝑨 𝑨
From (1) and (2) (i) a sin + 𝑩 = (𝒃 + 𝒄)𝒔𝒊𝒏
𝟐 𝟐

𝑆𝑖𝑛 𝐵 =

𝑆𝑖𝑛 𝐶 = L.H.S : a sin + 𝐵 = 2 R sin A sin +𝐵

𝐵 = 60° 𝐶 = 45° = 2 R 2 sin cos sin +𝐵
= 4 R sin sin + 𝐵 cos
Now A+B+C= 180°
𝐴 1 𝐴 𝐴 𝐴 𝐴
A+60°+45°= 180° = 4 R sin [𝑠𝑖𝑛 +𝐵+ + 𝑠𝑖𝑛 +𝐵+ ]
2 2 2 2 2 2
A=180°- 105° 𝐴 1
A= 75° = 2 R sin [𝑠𝑖𝑛(𝐴 + 𝐵) + 𝑠𝑖𝑛𝐵]
2 2
Where Angles C,B,A are in A.P = 2R sin {𝑠𝑖𝑛(𝐴 + 𝐵) + 𝑠𝑖𝑛𝐵}
𝒔𝒊𝒏 𝑨
03. In a ∆ABC, if cos C= , show that the triangles are = 2R sin {𝑠𝑖𝑛(180° − 𝐶) + 𝑠𝑖𝑛𝐵}
𝟐 𝒔𝒊𝒏 𝑩
isosceles.
= 2R sin {sin 𝐶 + 𝑠𝑖𝑛𝐵}
Solution : cos C=
In a ∆ABC, AB=c ; BC=a ; CA= b = 2R sin +
Applying cosine and sin formulas to each terms , we get 𝐴 𝑐 𝑏
= 2R sin +
2 2𝑅 2𝑅
= = sin [𝑏 + 𝑐] R.H.S Hence Proved.
𝑨
(ii) 𝒂(𝒄𝒐𝒔𝑩 + 𝒄𝒐𝒔 𝑪) = 𝟐(𝒃 + 𝒄) 𝐬𝐢𝐧𝟐 𝟐
= Solution :
𝑎 +𝑏 −𝑐 R.H.S 2(𝑏 + 𝑐) sin
= 2(𝑏 + 𝑐)
=𝑎
𝑎 = (𝑏 + 𝑐) (1 − cos 𝐴)
𝑎 +𝑏 −𝑐 =𝑎 = b - b cos A + c – c cos A
𝑏 −𝑐 =0 = 𝑐 𝑐𝑜𝑠𝐴 + 𝑎 cos 𝐶 − 𝑏 cos 𝐴 + 𝑎 𝑐𝑜𝑠𝐵 + 𝑏 𝑐𝑜𝑠𝐴 − 𝑐𝑐𝑜𝑠 𝐴
𝑏 =𝑐 [ By applying Projection Formula
𝑏=𝑐 = 𝑎 cos 𝐶 + 𝑎 𝑐𝑜𝑠𝐵
Where two sides of the triangle b & c are Equal.
= a (cos 𝐶 + 𝑐𝑜𝑠𝐵) = L.H.S
So that triangle ABC is a isosceles triangle.
Hence Proved.
𝒂𝟐 𝒄𝟐 𝒔𝒊𝒏 (𝑨 𝑪) 10.A rope of length 12 m is given. Find the largest area of the
(iii) =
𝒃𝟐 𝒔𝒊𝒏 (𝑨 𝑪) triangle formed by this rope and find the dimensions of the
L.H.S : =
( ) ( ) triangle so formed.
( )
Solution :
= The equilateral triangle has the maximum area for any fixed
perimeter. The rope to be formed must be of maximum area.
= So we have to formed equilateral shaped triangle.
( ) ( )
=
( ) Let consider x metre is the side of the equilateral triangular rope.
( ) ( )
=
( ( ( ))
( ) ( ) Now Perimeter a + b + c = 12 m [ where a=b=c= x m
= X + X + X= 12
( ( ))
( )
= = R.H.S Hence Proved. 3X=12
( )
𝒂 𝐬𝐢𝐧(𝑩 𝑪) 𝒃 𝐬𝐢𝐧(𝑪 𝑨) 𝒄 𝐬𝐢𝐧(𝑨 𝑩)
X=4m
(iv) = = Largest Area : Method 2 :
𝒃𝟐 𝒄𝟐 𝒄 𝟐 𝒂𝟐 𝒂 𝟐 𝒃𝟐
𝒂 𝐬𝐢𝐧(𝑩 − 𝑪)
=
2𝑅 𝑆𝑖𝑛 𝐴 𝑠𝑖𝑛 (𝐵 − 𝐶) Method 1 : Area of a Equilateral
𝒃𝟐 − 𝒄𝟐 (2𝑅𝑠𝑖𝑛 𝐵) − (2𝑅𝑠𝑖𝑛 𝐶) Area of a Equilateral
=
( ( )) ( ) Triangle is =
(2𝑅𝑠𝑖𝑛 𝐵) (2𝑅𝑠𝑖𝑛 𝐶) √ √
Triangle is = 𝑎 𝑆𝑞. 𝑢𝑛𝑖𝑡𝑠
=
( ) ( ) Perimetre 2s = 12; s=6
2𝑅 {sin2 𝐵−sin2 𝐶}

= (4) =4√3𝑆𝑞. 𝑈𝑛𝑖𝑡𝑠 =4√3𝑆𝑞. 𝑚

= = ----------------------(1)
4𝑅2 sin2 𝐵−sin2 𝐶 11. Derive Projection formula from
( ) 1
Similarly = ----------------------------------(2) (i) Law of sines, (ii) Law of cosines
2𝑅
( ) 1 (i) To Prove 𝑎 = 𝑏 𝑐𝑜𝑠𝐶 + 𝑐 𝑐𝑜𝑠 𝐵 by using Law of sines
= ----------------------------------(3) R.H.S : 𝑏 𝑐𝑜𝑠𝐶 + 𝑐 𝑐𝑜𝑠 𝐵
2𝑅
( ) ( ) ( )
𝐹𝑟𝑜𝑚 (1), (2)𝑎𝑛𝑑(3) = = = 2𝑅 𝑠𝑖𝑛𝐵 𝑐𝑜𝑠𝐶 + 2𝑅 𝑠𝑖𝑛𝐶 𝑐𝑜𝑠 𝐵
𝒂 𝒃 𝑨 𝑩 𝑨 𝑩 = 2𝑅 [𝑠𝑖𝑛𝐵 𝑐𝑜𝑠𝐶 + 𝑠𝑖𝑛𝐶 𝑐𝑜𝑠 𝐵]
(v) 𝒂 = 𝒕𝒂𝒏 𝒄𝒐𝒕
𝒃 𝟐 𝟐 = 2𝑅 [𝑠𝑖𝑛𝐵 𝑐𝑜𝑠𝐶 + 𝑐𝑜𝑠 𝐵 𝑠𝑖𝑛𝐶]
= 2𝑅 [sin(𝐵 + 𝐶)]
= = 2𝑅 [sin(180 − 𝐴)]
( ) = 2R sin A
= =2R[ ]=a [ L.H.S Hence Proved.
( )
𝑠𝑖𝑛𝐴+ 𝑠𝑖𝑛 𝐵 (i) To Prove 𝑎 = 𝑏 𝑐𝑜𝑠𝐶 + 𝑐 𝑐𝑜𝑠 𝐵 by using Law of cosines
= 𝑠𝑖𝑛𝐴− 𝑠𝑖𝑛 𝐵
R.H.S : 𝑏 𝑐𝑜𝑠𝐶 + 𝑐 𝑐𝑜𝑠 𝐵
= =𝑏 +𝑐
= +
= 𝑡𝑎𝑛 𝑐𝑜𝑡 Hence Proved.
=
9. In ∆𝑨𝑩𝑪,Prove that 𝒂𝟐 − 𝒃𝟐 + 𝒄𝟐 tan B= 𝒂𝟐 + 𝒃𝟐 − 𝒄𝟐 𝒕𝒂𝒏𝑪
To Prove (𝑎 − 𝑏 + 𝑐 )tan B=(𝑎 + 𝑏 − 𝑐 ) 𝑡𝑎𝑛𝐶 = =a [ L.H.S Hence Proved.
It is enough to prove =
EXERCISE 3.10
L.H.S : = = 𝑋 01. Determine whether the following measurements produce
one triangle, two triangles or no triangle: ∠𝑩 = 𝟖𝟖°, a=23 , b=2.
= 𝑋 Solve if solution exist. A
Solution : By using sine formula
= . . . 𝑎 𝑏 𝑐
= =
= R.H.S Hence Proved . sin 𝐴 sin 𝐵 sin 𝐵
b=2
10. An Engineer has to develop a triangular shaped park with 23 2 =
a perimeter 120 m in a village. The park to be developed must =
sin 𝐴 sin 88°
be of maximum area. Find out the dimensions of the park.
Solution : [from the Example 3.63 we conclude as below] B C
° a=23
The equilateral triangle has the maximum area for any fixed sin 𝐴 =
perimeter. The park to be developed must be of maximum area. .
So we have to design equilateral shaped triangle. Sin A = = 11.41
Sin A = 11.41 This is not possible
Let consider x metre is the side of the equilateral triangular park. Because −1 ≤ 𝑠𝑖𝑛𝑥 ≤ 1 . Solution of the given triangle does not
exist.
Now Perimeter a + b + c = 120 m [ where a=b=c= x m
02.If the sides of a ∆ABC are a = 4; b = 6 and c = 8, then show
X + X + X= 120
that 4 cosB + 3 cosC = 2.
3X=120
Solution : By using cosine formula,
X = 40 m
b cos C + c cos B = a
Hence the dimensions of the park are 40m, 40m, and 40m. 6 cos C + 8 cos B =4
÷ by 2
3 cos C + 4 cos B= 2 Hence Proved.
03. In a ∆ ABC , if 𝒂 = √𝟑 -1 , b= √𝟑 +1 and ∠𝑪=60° , then find P
the other side and other two angles.
Solution : By Napier’s Formula , 45°
𝐴−𝐵 𝑎−𝑏 𝐶 √3 − 1 − √3 + 1 60 15°
𝑡𝑎𝑛 = 𝑐𝑜𝑡 = 𝑐𝑜𝑡
2 𝑎+𝑏 2 √3 − 1 + √3 + 1 2
1 x
= 𝑐𝑜𝑡 30° = 𝑋 3=1
√ 3
i.e., 𝑡𝑎𝑛 =1⇒ 𝑡𝑎𝑛 = tan 45° 30° 135° 45° 90°
= 45° A B C
𝐴 − 𝐵 = 90° ----------- (1)
We know that in a ∆ 𝐴 + 𝐵 = 180° − 𝐶 Let P be the intruder , A and B are the soldiers,
We know ∠𝑪=60° then 𝐴 + 𝐵 = 180° − 60° Let x be the distance between the intruder and soldier B.
𝐴 + 𝐵 = 120°---------- (2) In ∆ PBC , ∠𝑃𝐵𝐶 = 45° 𝑎𝑛𝑑 ∠𝑃𝐶𝐵 = 90°
By solving (1) and (2) We get 𝐴 = 105° 𝑎𝑛𝑑 𝐵 = 15° Then ∠𝐵𝑃𝐶 = 45°
Now by using cosine formula, In ∆ APC , ∠𝑃𝐴𝐶 = 30° 𝑎𝑛𝑑 ∠𝑃𝐶𝐵 = 90°
𝑎 +𝑏 −𝑐 Then ∠𝐴𝑃𝐵 + ∠𝐵𝑃𝐶 = 180° − ∠𝑃𝐶𝐵 − ∠𝑃𝐴𝐶
cos 𝐶 =
2𝑎𝑏 ∠𝐴𝑃𝐵 + 45° = 180° − 90° − 30°
√3−1 √3+1 ∠𝐴𝑃𝐵 = 15°
cos 60° = In ∆ ABP , using sine formula,
√3−1 √3+1

=
√ √
°
= °
( )
1
2 =8−𝑐 5 . sin 30° 5.
2 = 5√2 𝑘𝑚.
𝑥= =
𝑐 =6 sin 15° √3 − 1 √3 − 1
𝑐 = √6 2√2

4.In a triangle ABC , Prove that area ∆= . 08. A researcher wants to determine the width of a pond from
Solution: Given R.H.S = east to west, which cannot be done by actual measurement.
From a point P, he finds the distance to the eastern-most point
of the pond to be 8 km, while the distance to the western most
= . sin 𝐴
point from P to be 6 km. If the angle between the two lines of
= . sin 𝐴 sight is 60°, find the width of the pond.
P
Solution :
= sin 𝐴 Let A be the point on the eastern side 60°
And B be the point on the western side.
1
= 𝑏𝑐 𝑆𝑖𝑛𝐴 = ∆ [𝐴𝑟𝑒𝑎] L.H.S Let a=6 , b=8 , ∠𝑃 = 60° a=6 b=8
2
Hence Proved. By using cosine formula
𝑐 = 𝑎 + 𝑏 − 2𝑎𝑏 cos 𝑃
5. In a ∆ ABC if a = 12 cm, b = 8 cm and C = 30°, then show that B c A
= 6 + 8 − 2(6)(8) cos 60°
its area is 24 sq.cm
= 36+64-2(6)(8)
Solution : Area of ∆ ABC = 𝑎𝑏 𝑆𝑖𝑛𝐶
= 100 – 48
= x 12 x 8 x sin 30° 𝑐 = 52
= x 12 x 8 x 𝑐 = √52 = 2√13 kms.
= 24 𝑠𝑞. 𝑐𝑚 Hence Proved. 09.Two Navy helicopters A and B are flying over the Bay of
06. In a ∆ ABC if a = 18 cm, b = 24 cm and c = 30 cm, then show Bengal at same altitude from the sea level to search a missing
that its area is 216 sq.cm. boat. Pilots of both the helicopters sight the boat at the same
time while they are apart 10 km from each other. If the
Solution : 𝑆𝑒𝑚𝑖 𝑃𝑒𝑟𝑖𝑚𝑒𝑡𝑟𝑒 = = = = 36 distance of the boat from A is 6 km and if the line segment AB
Bu using Heron’s Formula , subtends 60° at the boat, find the distance of the boat from B.
Area of A triangle : ∆= 𝑠(𝑠 − 𝑎)(𝑠 − 𝑏)(𝑠 − 𝑐) Solution : 10
A B
= 36(36 − 18)(36 − 24)(36 − 30)
𝐵𝑦 𝑢𝑠𝑖𝑛𝑔
= 36(36 − 18)(36 − 24)(36 − 30) 6 x
Cosine formula 60°
= 36(18)(12)(6)
= √36x9x2x4x3x2x3 = 6𝑥3x2x2x3
We get,
C
= 216 sq.cm 𝐴𝐵 = 𝐵𝐶 + 𝐴𝐶 − 2(𝐵𝐶)𝐴𝐶 cos 𝐶
07. Two soldiers A and B in two different underground 10 = 𝑥 + 6 − 2(𝑥)(6) cos 60°
bunkers on a straight road, spot an intruder at the top of a hill. 1
The angle of elevation of the intruder from A and B to the 100 = 𝑥 + 36 − 12𝑥
2
ground level in the eastern direction are 30° and 45° ± ( ) ± √
respectively. If A and B stand 5 km apart, find the distance of 𝑥 − 6𝑥 − 64 = 0 ⟹ 𝑥 = =
the intruder from B.
𝑥 = 3 ± √73 𝑊ℎ𝑒𝑟𝑒 𝑥 = 3 + √73 [ 3 + √73 𝑖𝑠 𝑛𝑒𝑔𝑎𝑡𝑖𝑣𝑒
10. A straight tunnel is to be made through a mountain. A 13. A plane is 1 km from one landmark and 2 km from another.
surveyor observes the two extremities A and B of the tunnel to From the planes point of view the land between them
be built from a point P in front of the mountain. If AP = 3km, subtends an angle of 45°. How far apart are the landmarks?
BP = 5 km And ∠𝑨𝑷𝑩 = 𝟏𝟐𝟎° then find the length of the Solution : Using cosine formula,
tunnel to be built. 𝑐 = 𝑎 + 𝑏 − 2𝑎𝑏 𝑐𝑜𝑠𝐶 C
Solution : P 𝑐 = 2 + 1 − 2(2)(1)𝑐𝑜𝑠45°
1km 45° 2km
1
120° 𝑐 = 4+1−4
3 5 √2
A c B
𝑐 = 5 − 2√2 𝑐 = 5 − 2√2 km
A p B 14. A man starts his morning walk at a point A reaches two
By using Cosine formula ,
points B and C and finally back to A such that ∠A = 60° and
∠B = 45°; AC = 4km in the ∆ABC. Find the total distance he
covered during his morning walk. C
Solution :
b 45° a
In triangle ABC ∠A = 60° and
∠B = 45°; so that ∠C = 75° 60° 75°
kms. Using sine formula , A 4 B
11.A farmer wants to purchase a triangular shaped land with
= = ⟹ = =
sides 120feet and 60feet and the angle included between these ° ° °
two sides is 60°. If the land costs Rs.500 per sq.ft, find the = =
amount he needed to purchase the land. Also find the ° ° ° °
. ° . °
perimeter of the land. 𝑎= °
𝑏= °
Solution : A √ √
. .

𝑎= 𝑏=
120 60° 60 √ √
𝑎 = 2√6 b=2(√3 + 1)
B a C Total distance covered by the man = a+b +c
Area ∆= 𝑏𝑐 sin 𝐴 = 2√6 + 2 √3 + 1 + 4
= x 120 x 60 x sin 60° =2√6 + 2√3 + 2 + 4
√ =2√6 + 2√3 + 6
= x 120 x 60 x = 1800√3 𝑓𝑒𝑒𝑡
=2√6 + 2(√3 + 3) kms
𝐶𝑜𝑠𝑡 𝑜𝑓 𝑡ℎ𝑒 𝑙𝑎𝑛𝑑 = 𝑅𝑠. 500 𝑋 1800√3 𝑓𝑒𝑒𝑡
15.Two vehicles leave the same place P at the same time
= 𝑅𝑠. 1558800
moving along two different roads. One vehicle moves at an
By using Cosine formula ,
average speed of 60km/hr and the other vehicle moves at an
𝑎 = 120 + 60 − 2(120)(60) cos 60°
average speed of 80 km/hr. After half an hour the vehicle reach
𝑎 = 14400 + 3600 − 7200
the destinations A and B. If AB subtends 60°at the initial point
𝑎 = 18000 − 7200 = 10800
P, then find AB.
𝑎 = 18000
Solution : P
𝑎 = √18000 = 60√3
𝑝 = 40 + 30 − 2(40)(30) cos 60° 40 60° 30
Perimeter = 120+60+60√3 = 180+ 60√3= 180+20√27 feet. 𝑝 = 1600 + 900 − 1200
12.A fighter jet has to hit a small target by flying a horizontal 𝑝 = 1300
distance. When the target is sighted, the pilot measures the 𝑝 = √1300 = 10√13 𝑘𝑚 A p B
angle of depression to be 30°. If after 100 km, the target has an 16. Suppose that a satellite in space, an earth station and the
angle of depression of 45°, how far is the target from the centre of earth all lie in the same plane. Let r be the radius of
fighter jet at that instant? earth and R be the distance from the centre of earth to the
Solution : A 100 B A’ satellite. Let d be the distance from the earth station to the
30° 45° satellite. Let 30° be the angle of elevation from the earth
station to the satellite. If the line segment connecting earth
station and satellite subtends angle 𝜶 at the centre of earth,
𝒓 𝟐
h h x then prove that d = 𝟏 + 𝑹
𝒓
− 𝟐 𝑹 𝒄𝒐𝒔𝜶 S
Solution :
45° Let s be the position of the satellite, R d
90° 90° 30° E be the position of the earth station
𝛼 30°
D 100 E h C And C be the centre of the earth .
C r E
From the diagram
From the diagram ∠BAC= ∠𝐴𝐶𝐷 = 30°
Also ∠A’BC= ∠𝐵𝐶𝐷 = 45° ⇒ 𝑎𝑙𝑠𝑜 𝐵𝐸 = 𝐸𝐶 = ℎ 𝑑 = 𝑟 + 𝑅 − 2𝑟𝑅 cos 𝛼
𝑟 2𝑟
In ∆ ADC , tan 30° = 𝑑 =𝑅 + 1 − 𝑐𝑜𝑠𝛼
𝑅 𝑅
= ⇒ √3ℎ = 100 + ℎ
𝒓 𝟐

𝒓
ℎ= d= 𝟏+ − 𝟐 𝒄𝒐𝒔𝜶
√ 𝑹 𝑹
In ∆ BEC sin 45°= ⇒ x =√


Now 𝑥= kms

3. Four children are running a race.
PV MATRIC HR SEC SCHOOL (i) In how many ways can the first two places be filled?
MATHEMATICS - XI STD (ii) In how many different ways could they finish the race?
(i)First place can be given to any one of the four children
EXERCISE 4.1 in 4 ways.
1(i)A person went to a restaurant for dinner. In the menu The second place can be given to any of the remaining
card, the person saw 10 Indian and 7 Chinese food three children in 3 ways.
items. In how many ways the person can select either an ∴ The number of ways = 4 x 3 = 12 ways.
Indian or a Chinese food? (ii)The winner may be any of the 4 children.
The number of ways of selecting Indian food = 10 The runner may be one of the remaining 3 children.
The number of ways of selecting Chinese food = 7 The Third person may be one of the remaining 2 children.
∴The number of ways of selecting Indian food or Chinese Number of children coming last is 1 child.
food = 10+7=17 ∴ The number of ways = 4 x 3 x 2 x 1= 24 ways
(ii) There are 3 types of toy car and 2 types of toy train 4. Count the number of three-digit numbers which can be
available in a shop. Find the number of ways a baby can formed from the digits 2,4,6,8 if (i) repetitions of digits is
buy a toy car and a toy train? allowed.(ii) Repetitions of digits is not allowed
(ii) Number of ways of buying a toy car = 3 (i) Repetitions of digits is allowed
Number of ways of buying a toy train = 2 Hundreds Tens Unit
∴the number of ways a baby can buy a toy car and a toy 4 4 4
train = 3 x 2 = 6 Every places can be filled in 4 ways.
∴Total number of three digit numbers=4 x4 x4=64 .
(iii)How many two-digit numbers can be formed using
(ii) Repetitions of digits is not allowed
1,2,3,4,5 without repetition of digits?
Hundreds Tens Unit
tens One’s
2 3 4
4 5
The one’s place can filled up in 5 ways The one’s place can filled up in 4 ways
Then ten place can filled up in 4 ways The tens place can filled up in 3 ways
∴No of ways of forming two digits = 5 x 4 = 20 ways The Hundreds place can filled up in 2 ways
∴Total number of three digit numbers=4 x3 x2=24 .
(iv) Three persons enter in to a conference hall in which
5.How many three-digit numbers are there with 3 in the unit
there are 10 seats. In how many ways they can take their
place? (i) with repetition (ii) without repetition.
seats?
Solution :
Number of ways of getting a seat for 1st Person = 10 (i) with repetition
Number of ways of getting a seat for 2nd Person = 9 The given digits are 0,1,2,3,4,5,6,7,8,9
Number of ways of getting a seat for 3rd Person = 8 The Unit place can be filled in only one way of using 3.
∴ The number of ways of getting seats for 3 persons in the Since repetition is allowed , the tens place can be filled in
conference Hall = 10 x 9 x 8 = 720 ten ways using any one of the digits from
(V)In how many ways 5 persons can be seated in a row? 0,1,2,3,4,5,6,7,8,9
The number of ways of 1st person can be seated = 5 The hundreds place can be filled in 9 ways (excluding 0)
The number of ways of 2nd person can be seated = 4 Hundreds Tens Unit
The number of ways of 3rd person can be seated = 3 9 10 1
The number of ways of 4th person can be seated = 2 By fundamental principle of multiplication , total number
The number of ways of 5th person can be seated = 1 of 3 digit numbers = 9 x 10 x 1 = 90 Ways
∴ The number of ways of 5 persons can be seated in a row (ii) without repetition :
=5x4x3x2x1 The Unit place can be filled in only one way of using 3.
= 120 Ways The hundreds place can be filled in 8 ways (excluding 0,3)
2. (i) A mobile phone has a passcode of 6 distinct digits. The tens place can be filled in 8 ways (including 0,3)
What is the maximum number of attempts one makes to Hundreds Tens Unit
retrieve the passcode? 8 8 1
Since the pass code has 6 distinct digits, The first digit can By fundamental principle of multiplication, total number
be tried in 10 ways using, 0,1,2,3,4,5,6,7,8,9 respectively. of 3 digit numbers = 8 x 8 x 1 = 64 Ways.
Similarly 2nd, 3rd, 4th, 5th ,6th digits can also be tried in 6.How many numbers are there between 100 and 500 with the
9, 8,7,6,5 ways respectively. digits 0, 1, 2, 3, 4, 5 ? if (i) repetition of digits allowed (ii) the
∴Maximum number of Attempts = 10 x 9 x 8 x 7 x 6 x 5 repetition of digits is not allowed.
= 1,51,200 Ways Solution :
(ii) Given four flags of different colours, how many (i) repetition of digits allowed :
different signals can be generated if each signal requires Hundreds Tens Unit
the use of three flags, one below the other? 4 6 6
The upper place flag can filled in 4 ways We have to form number between 100 to 500 it has 3 digit.
The middle place of the flag can filled in 3 Ways The unit place can be filled in 6 ways using 0,1,2,3,4,5
The Bottom place of the flag can filled in 2 Ways Tenth place also can be filled in 6 ways using 0,1,2,3,4,5
∴Number of ways = 4 x 3 x 2 = 24 ways The hundreds place can be filled in 4 ways (excluding 0,5)
Required number of 3 digit numbers= 6x6x4 = 144
(ii) repetition of digits is not allowed : Case (i) Let unit place fixed by 0:
Hundreds Tens Unit Hundreds Tens Unit
4 5 4 0
The hundreds place can be filled in 4 ways (excluding 0,5) 5 4 1
Tenth place also can be filled in 5 ways Hundreds place can filled by any one of the digit 1,2,3,4,5
The unit place can be filled in 4 ways using 0,1,2,3,4,5 in 5 ways. Having filled this tenth place can filled up by
Required number of 3 digit numbers= 4x5x4 = 80 remaining numbers by 4 ways.
07.How many three-digit odd numbers can be formed by using 𝑁𝑜 𝑜𝑓 3 𝑑𝑖𝑔𝑖𝑡 𝑛𝑢𝑚𝑏𝑒𝑟 𝑤ℎ𝑖𝑐ℎ
= 5 x 4 x 1 = 20 -------(1)
the digits 0, 1, 2, 3, 4, 5 ? if (i) the repetition of digits is not 𝑑𝑖𝑣𝑖𝑠𝑎𝑏𝑙𝑒 𝑏𝑦 5 𝑎𝑛𝑑 𝑒𝑛𝑑 𝑤𝑖𝑡ℎ 0
allowed (ii) the repetition of digits is allowed. Case (ii) Let unit place fixed by 5:
(i) repetition of digits is not allowed : Hundreds Tens Unit
Hundreds Tens Unit 5
4 4 3 4 4 1
Since we need three digit odd numbers, so that Hundreds place can filled by any one of the digit 1,2,3,4
The unit place can be filled in 3 ways using 1,3,5 in 4 ways (Excluding 0). Having filled this tenth place can
The hundreds place can be filled in 4 ways (excluding 0) filled up by remaining numbers by 4 ways.
Tenth place also can be filled in 4 ways (including 0) 𝑁𝑜 𝑜𝑓 3 𝑑𝑖𝑔𝑖𝑡 𝑛𝑢𝑚𝑏𝑒𝑟 𝑤ℎ𝑖𝑐ℎ
= 4 x 4 x 1 = 16 -------(2)
Required number of 3 digit numbers= 4x4x3 =48 𝑑𝑖𝑣𝑖𝑠𝑎𝑏𝑙𝑒 𝑏𝑦 5 𝑎𝑛𝑑 𝑒𝑛𝑑 𝑤𝑖𝑡ℎ 0
(ii) repetition of digits allowed From (1) and (2)
Hundreds Tens Unit Number of number divisible by 5 = 20 + 16 = 36
5 6 3 (ii) repetition of digits are allowed:
The unit place can be filled in 3 ways using 1,3,5 Hundreds Tens Unit
The hundreds place can be filled in 5 ways (excluding 0) 5 6 2
Tenth place also can be filled in 6 ways (including 0) The unit place can be filled in 2 ways using 0,5
Required number of 3 digit numbers= 5x6x3 = 90 The hundreds place can be filled in 5 ways (excluding 0)
Tenth place can be filled in 6 ways (including 0)
09.Count the numbers between 999 and 10000 subject to
Number of number divisible by 5 = 2 x 5 x 6 =60.
the condition that there are (i) no restriction.
(ii) no digit is repeated. (iii) at least one of the digits is 10.To travel from a place A to place B, there are two
repeated. different bus routes B1,B2, two different train
Solution : routes T1, T2 and one air route A1. From place B to place
(i) no restriction. C there is one bus route say B’1, two different train
Given digits are 0,1,2,3,4,5,6,7,8,9 routes say T’1, T’2 and one air route A’1. Find the number
We need numbers from 999 to 10000. It has 4 digits. of routes of commuting from place A to place C via place
Thousand Hundreds Tens Unit B without using similar mode of transportation.
9 10 10 10 A’1
A1
Thousands place can be filled in 9 ways (Excluding 0)
Hundreds, Tens ,Units places can be filled in 10 ways
B1 B’1
Required number of 4 digit numbers = 9 x 10 x 10 x 10 B2
0A T1 B T1 C
= 9000
(ii) no digit is repeated. B B
Thousand Hundreds Tens Unit
T2 T2
9 9 8 7
Condition is one should not use same mode of transport.
Thousands place can be filled in 9 ways (Excluding 0)
Hundreds place can be filled in 9 ways (including 0) Selection No of Selection No of Total
tens place can be filled in 8 ways Of Mode Routes Of Mode Routes No
ones place can be filled in 7 ways Transport Available Transport Available Routes
Required number of 4 digit numbers = 9 x 9 x 8 x 7 To travel For To travel For To
= 4536 A to B A to B B to C B to C A to C
(iii) at least one of the digits is repeated. Bus 2 Air/Train 3 6
Required number of numbers = Train 2 Air/Bus 2 4
Total number of No of 4 digit number Air 1 Bus/Train 3 3

4 digit Number when no digit is repeated Total Number of Routes 13
= 9000 – 4536 = 4464 11. How many numbers are there between 1 and 1000 (both
9. How many three-digit numbers, which are divisible inclusive) which are divisible neither by 2 nor by 5?
by 5, can be formed using the digits 0, 1, 2, 3,4, 5 if n(A)= Number of numbers divisible by 2 = 500
(i) repetition of digits are not allowed? (ii) repetition of n(B)= Number of numbers divisible by 5 = 200
digits are allowed? 𝑛(𝐴 ∩ 𝐵) = No. of numbers divisible by 2&5=100
Solution : No of numbers divisible by 2 Or 5 =
(i) repetition of digits are not allowed: 𝑛(𝐴𝑈𝐵) = 𝑛(𝐴) + 𝑛(𝐵) − 𝑛(𝐴 ∩ 𝐵)=500+200-100=600
The given digits are 0,1,2,3,4,5 No of numbers divisible neither by 2 nor by 5
A number divisible by 5 if end with 0 or 5. = 1000 -600 = 400
12.How many strings can be formed using the letters of (iv) 3! X 4! = (3x2x1)+(4x3x2x1) = 3+24= 144
the word LOTUS if the word (i) either starts with L or (v)
!
=
!
= 220
ends with S? (ii) neither starts with L nor ends with S? !
(𝒏 𝟑)!
! !
(𝒏 𝟑)(𝒏 𝟐)(𝒏 𝟏)!
Solution : (vi) (𝒏 𝟏)!
= (𝒏 𝟏)!
= (𝒏 + 𝟑)(𝒏 + 𝟐)
(i) either starts with L or ends with S 𝒏!
a) The words starts with L 15. Evaluate
𝒓!(𝒏 𝒓)!
L (i) n=6 , r = 2
1 4 3 2 1 6! 6! 6x5x4x3x2x1
= = = 15
Since the words starts with L, The remaining 4 boxes can 2! (6 − 2)! 2! 4! 1x2x1x2x3x4
filled in 4x3x2x1 by the remaining letters O,T,U,S . (ii) n=10 , r = 3
∴ Number of words starting with L = 1 x 2 x 3 x 4 = 24 10! 10! 10𝑥9𝑥8𝑥7𝑥6𝑥5𝑥4𝑥3𝑥2𝑥1
= = = 120
b) The words ending with S 3! (10 − 3)! 3! 7! 3𝑥2𝑥1𝑥3𝑥2𝑥1
S (ii) For any n with r = 2
1 4 3 2 1 𝑛! 𝑛(𝑛 − 1)(𝑛 − 2)! 𝑛(𝑛 − 1)
= =
Since the words ending with S, The remaining 4 boxes can 2! (𝑛 − 2)! 2! (𝑛 − 2) 2
filled in 4x3x2x1 by the remaining letters L,O,T,U . 16. Find the value of n if
∴ Number of words ending with S = 1 x 2 x 3 x 4x1 = 24 (i) (n + 1)! = 20(n − 1)!
c) The words starting with L & ending with S Given (n + 1)! = 20(n − 1)!
L S (n + 1) n (n-1)! = 20(n − 1)!
1 3 2 1 1 (n + 1) n=20
Since the words starting with L and ending with S, The 𝑛 + 𝑛 − 20 = 0
remaining 3 boxes can filled in 3x2x1 by the remaining (n+5)(n-4)=0
letters O,T,U . ∴ Number of words starting with L and 𝑛=4 n=-5 Not Possible ;
𝟏 𝟏 𝒏
ending with S = 1 x 2 x 3 = 6 (ii) + 𝟗! = 𝟏𝟎!
𝟖!
+ =
Number of Words either starts with L ! ! !
Multiply by 10!
nor End with S = 24 +24-6 = 42 ! !
(ii) neither starts with L nor ends with S + =n
! !
! !
+ =n
! !
5 4 3 2 1 (10x9) + 10 = n
Total number of words formed by the letters of the word 90 + 10 = n
LOTUS is 5x4x3x2x1 = 120 words 𝑛 = 100
Number of words neither starts with L nor end with S
=
𝑇𝑜𝑡𝑎𝑙 𝑛𝑢𝑚𝑏𝑒𝑟

𝑁𝑢𝑚𝑏𝑒𝑟 𝑜𝑓 𝑤𝑜𝑟𝑑𝑠 𝑒𝑖𝑡ℎ𝑒𝑟 𝑠𝑡𝑎𝑟𝑡𝑠 EXERCISE 4.3
𝑜𝑓 𝑤𝑜𝑟𝑑𝑠 𝑤𝑖𝑡ℎ 𝐿 𝑎𝑛𝑑 𝑒𝑛𝑑 𝑤𝑖𝑡ℎ 𝑆 𝟎𝟏. If nC = nC
12 9 find 21Cn
= 120 – 42 = 78 Solution :
RECALL : n C = nC
12 9
if n different objects are to be placed in m places, nC12= nCn-9
then the number of ways of placing is 𝑚𝑛
12 = n-9 ⟹ 𝑛 = 21
13. 21 C
n = 21C21=1
(i) Count the total number of ways of answering 6
objective type questions, each question 𝟎𝟐. If 15C2r-1= 15C2r+4 find r.
having 4 choices. Solution :
15C
Number of ways of answering 6 objective type 2r-1= C2r+4
15

questions , each question has four choices= 4 15C2r-1= 15 C15-2r-4


(ii) In how many ways 10 pigeons can be placed in 3 2𝑟 − 1 = 15 − 2𝑟 − 4
different pigeon holes ? 2𝑟 + 2𝑟 = 15 + 1 − 4
Number of ways that 10 pigeons can be placed in 4𝑟 = 12 ⟹ 𝑟 = 3
3 different pigeon holes = 3
03.If nPr=720 , and nCr=120 , find n,r.
(iii) Find the number of ways of distributing 12 distinct
Solution :
prizes to 10 students?
nP =720 , and nC =120
r r
Number of ways in 12 ‘distinct prizes’ can be
distributed among 10 students = 10 We know that 𝑃 = 𝐶 x 𝑟!
___________________________________________________ 720 = 120 𝑋 𝑟!
14. Find the value of 𝑟! = 6
(i) 6! (ii) 4! + 5! (iii) 3! − 2! 𝑟=3
𝟏𝟐! (𝒏 𝟑)!
(iv) 3! × 4! (v) (vi) (𝒏 𝑎𝑙𝑠𝑜 𝑃 = 720
𝟗!𝒙 𝟑! 𝟏)!
(i) 6! = 6x5x4x3x2x1 = 720 𝑃 = 720
(ii) 4!+5!=(4x3x2x1)+(5x4x3x2x1)=24+120=144 𝑛(𝑛 − 1)(𝑛 − 2) = 10x9x8
(iii) 3!-2! = (3x2x1)-(2x1)=6-2=4 𝑛 = 10
04.Prove that 15C3+ 2x15C4 + 15C5 = 17C5. =
!
𝑥
( )
(𝒏 𝒓)! (𝒓 𝟏)! ( )
Solution : 𝑛!
= (𝑛 − 𝑟 + 1) (𝑛−𝑟+1)(𝑛−𝑟
L.H.S : 15C3+ 2x15C4 + 15C5 )! 𝑟−1 !
𝑛!
=15C3+ 15C4+15C4 + 15C5 = (𝑛 − 𝑟 + 1)
(𝑛−𝑟+1)! 𝑟−1 !
𝑛!
𝐶 + 𝐶 = 𝐶 ------------(1) = (𝑛 − 𝑟 + 1)
(𝑛−(𝑟−1))! 𝑟−1 !
=(15C4 + 15C3)+(15C5+15C4) = (𝑛 − 𝑟 + 1) 𝐶 R.H.S Hence Proved
= 16C4 + 16C5 [∵ 𝐹𝑟𝑜𝑚 (1) 9. (i) A Kabaddi coach has 14 players ready to play. How
= 16C5 + 16C4 many different teams of 7 players could the coach put on
= 17C5 [∵ 𝐹𝑟𝑜𝑚 (1) the court?
= R.H.S Hence Proved Solution : Here 7 players must be selected from 14 players
05. Prove that 35C5 + ∑𝟒𝒓 𝟎 (𝟑𝟗 𝒄) 𝑪𝟒 =35C5 This can be done 14C7 ways.
Solution : Hence, number of different team of players = 14C7
L.H.S : 35C5 + ∑ ( )
𝐶 =
= 35C5 + 39C4 +38C4 +37C4 +36C4 +35C4 = 13 x 11 x 2 x 3 x 4 = 3432
𝑢𝑠𝑖𝑛𝑔 𝑡ℎ𝑒 𝑟𝑒𝑠𝑢𝑙𝑡 𝐶 + 𝐶 = 𝐶 (ii) There are 15 persons in a party and if each 2 of them
= (35C5+35C4)+ 39C4 +38C4 +37C4 +36C4 shakes hands with each other, how many handshakes
= 36C5+39C4 +38C4 +37C4 +36C4 happen in the party?
By continuing this Process, Solution : The total number of handshakes is same as the
= (36C5+36C4)+ 39C4 +38C4 +37C4 number of ways of selecting two persons among 15
= 37C5+ 39C4 +38C4 +37C4 persons. This can be done in 15C2 Ways.
= (37C5+37C4) + 39C4 +38C4 Number of Handshakes =15C2 = = 15 𝑥 7 = 105.
= 38C5+ 39C4 +38C4 (iii) How many chords can be drawn through 20 points
= (38C5+38C4)+ 39C4 on a circle?
= 39C5 + 39C4 Solution : A Chord is obtained by joining any two points
= 40C5 R.H.S Hence Proved On a circle. Number of chords drawn through 20 points is
06. If (n+1)C8 : (n-3)P4 = 57 : 16 , find the value of n. same as the number of ways selecting 2 points out of 20
Solution : points. This can be done in 20C2 Ways.
(𝒏 𝟏) 𝑪
𝟖
=
𝟓𝟕
Total Number of Chords =20C2= =190
(𝒏 𝟑) 𝑷 𝟏𝟔
𝟒
16 ( )
𝐶 = 57 ( )
𝑃 (iv) In a parking lot one hundred , one year old cars, are
16
( )!
= 57 (
( )! parked. Out of them five are to be chosen at random for
!( )! )! to check its pollution devices. How many different set of
( ) ( )( )( )! ( )!
( )! !
= ( )!
five cars can be chosen?
16 (𝑛 + 1)𝑛(𝑛 − 1)(𝑛 − 2) = 57 𝑋 8! Solution :
57 𝑋 8! 5cars can be chosen out of 100 cars in 100C5 Ways
(𝑛 + 1)𝑛(𝑛 − 1)(𝑛 − 2) =
16 100C =
5 = 451725120
(𝑛 + 1)𝑛(𝑛 − 1)(𝑛 − 2) = 21 𝑥 20 𝑥 19 𝑥 18
𝑛 = 20 (v) How many ways can a team of 3 boys,2 girls and 1
𝟐𝒏 𝒙 𝟏 𝒙 𝟑 𝒙………(𝟐𝒏 𝟏) transgender be selected from 5 boys, 4 girls and
07. Prove that 2nCn=
𝒏! 2 trans genders?
2nC =
!
Solution : L.H.S n
)!
Solution :
!(
3 boys can be selected from 5 Boys in 5C3 ways.
=
( )( )( )( )…… . . .
2 girls can be selected from 4 Boys in 4C2 ways.
! !
1 trans gender can be selected from 2 in 2C1 ways.
=
{ ( )( )…. . } {( )( )……. . } Total number of selection = 5C3 x 4C2 x 2C1
! !
= 𝑥 𝑥 2 = 120
2 {𝑛 (𝑛 − 1)(𝑛 − 2) … .2.1} {(2𝑛 − 1)(2𝑛 − 3) … … .3.1}
=
𝑛! 𝑛! 10. Find the total number of subsets of a set with
2𝑛 𝑛! {(2𝑛 − 1)(2𝑛 − 3) … … .3.1} (i) 4 elements (ii) 5 elements (iii) n elements.
=
𝑛! 𝑛! Solution :
2
𝑛
{( )( )……. . }
=
! We know that nC0+ nC1+ nC2+……..+ nCn=2
2 𝑥 1 𝑥 3 𝑥…..(2𝑛−3)𝑥 (2𝑛−1)
𝑛

= (i) 4 Elements
!
= ………( )
R.H.S 2 = 16
! (ii) 5 Elements
Hence Proved. 2 = 32
08.Prove that 𝟏 ≤ 𝒓 ≤ 𝒏 then n x (n-1)Cr-1=(n-r+1)nCr-1. (ii) n Elements
Solution : nC + nC + nC +……..+ nC =2
0 1 2 n
( )!
L.H.S n x (n-1)Cr-1 = 𝑛
𝒏 𝟏 (𝒓 𝟏) ! (𝒓 𝟏)!
𝒏( )!
= (𝒏 𝟏 𝒓 𝟏)! (𝒓 𝟏)!
11.A trust has 25 members. 16.Determine the number of 5 card combinations out of
(i) How many ways 3 officers can be selected? a deck of 52 cards if there is exactly three aces in each
(ii) In how many ways can a President, Vice President combination.
and a Secretary be selected? Solution : Total Number of cards = 52
Solution : No of ace cards =4
(i) 3 officers can be selected from 25 members in No of non ace cards = 48
25C =
3 = 2300 Number of selection of 5 cards combination from 52 cards
(ii) A president can be selected from 25 members with 3 aces in each combination is
48 C x 4 C ⟹ 48C x 4C =
𝟒𝟖 𝒙 𝟒𝟕
in 25 ways. A Vice president can be selected 2 3 2 1 𝒙 𝟒 = 𝟒𝟓𝟏𝟐
𝟐
from 24 members in 24 ways, and a secretary 17. Find the number of ways of forming a committee of 5
can be selected from 23 ways. members out of 7 Indians and 5 Americans,
25P =25 x 24 x 23 = 13800
3 so that always Indians will be the majority in the
12. How many ways a committee of six persons from 10 committee.
persons can be chosen along with a chair person and a Solution :
secretary? Condition is Indians will be the majority in the committee.
Solution : Possibilities Indians Americans Combinations
Required number of selection = 10C1 x 9C1x 8C6 (7) (5)
= 10C1 x 9C1x 8C6 (1) 3 2 7C
3 x 5C2 = 350
= 10 x 9 x = 2520 (2) 4 1 7C
4 x 5C1 = 175
(3) 5 - 7C = 021
5
13.How many different selections of 5 books can be
Total 546
made from 12 different books if,
18. A committee of 7 peoples has to be formed from 8
(i) Two particular books are always selected?
men and 4 women. In how many ways can this
(ii) Two particular books are never selected?
be done when the committee consists of
Solution :
(i) exactly 3 women? (ii) at least 3 women?
There are 12 Books , we have to select 5 Books.
(iii) at most 3 women?
(i) Two particular books are always selected , the
Solution :
remaining 3 books can be selected from 10 (i) exactly 3 women
books in 10C3 ways. That means 4 men in the committee
10C = = 120 Ways
3 Number of combinations = 8C4 x 4C3 = 280
(ii) Since two books are never to be selected , The (ii) at least 3 women
selection of 5 boos from 10 books are done in Men (8) Women (4) Combinations
10C ways.
5 4 3 8C x 4 C = 280
4 3
10C =
5 = 252 Ways. 3 4 8C x 4C = 56
3 4

14. There are 5 teachers and 20 students. Out of them a Total 336
committee of 2 teachers and 3 students is to be formed. (iii) at most 3 women
Find the number of ways in which this can be done. Men (8) Women (4) Combinations
Further find in how many of these committees 7 0 8C x 4C =
7 0 8
(i) a particular teacher is included? 6 1 8 C x 4 C = 112
6 1
(ii) a particular student is excluded? 5 2 8 C x 4 C = 336
5 2
Solution: 4 3 8 C x 4 C = 280
4 3
The number of selection of 2 teachers and 3 students from Total 736
5 teachers and 20 students. 19. 7 relatives of a man comprises 4 ladies and 3
5 C x 20C =
2 3 𝑥 = 11400 gentlemen, his wife also has 7 relatives; 3 of them
(i) Let Particular teacher be Included : are ladies and 4 gentlemen. In how many ways can they
Numbers of selections = 4C1 x 20C3 invite a dinner party of 3 ladies and 3 gentlemen so that
there are 3 of man’s relative and 3 of the wife’ s
=4x
relatives?
= 4560 Solution :
(ii) Let Particular student be excluded : Condition is 3 Gentlemen and 3 ladies are invited
Numbers of selections = 5C2 x 19C3 Husband Wife
Possibilities

= x
= 9690 Combinations
Ladies

Ladies
Gents

Gents
3

15.In an examination a student has to answer 5


questions, out of 9 questions in which 2 are compulsory.
In how many ways a student can answer the questions?
(1) 3 - - 3 3C
3x4C0 x 4C0 x 3C3 = 1
Solution:
(2) 2 1 1 2 3C
2x4C1 x 4C1 x 3C2 = 144
(3) 1 2 2 1 3C
1x4C2 x 4C2 x 3C1 = 324
Number of selection is 7C3 = = 35
(4) - 3 3 - 3C
0x4C3 x 4C3 x 3C0 = 16
Total = 485
20. A box contains two white balls, three black balls and 24. There are 11 points in a plane. No three of these lies
four red balls. In how many ways can three balls be in the same straight line except 4 points, which are
drawn from the box, if at least one black ball is to be collinear. Find,
included in the draw? (i) the number of straight lines that can be obtained
Solution : from the pairs of these points?
Condition is one black ball must be included out of 3 Balls (ii) the number of triangles that can be formed for which
W(2) B(3) R(4) Combinations the points are their vertices?
(1) 2 1 - 2C x3 C x 4 C = 3
2 1 0 Solution :
(2) 1 1 1 2C x3 C x 4 C =24
2 1 1
(3) - 1 2 2C x3 C x 4 C =18
0 1 2
(4) 1 2 - 2C x3 C x 4 C = 6
1 2 0
(5) - 2 1 2C x3 C x 4 C =12
0 2 1
(6) - 3 3 2C x3 C x 4 C = 1
0 3 3
Total = 64
21. Find the number of strings of 4 letters that can be (i)To get a straight line we need two points,
formed with the letters of the word Number of straight lines from 11 points in a plane is 11C2.
EXAMINATION?
But 4 points are collinear (lie in a same line )
Solution : 4C lines must be substrated. At the same time these four
2
Total number of letters in “Examination” – 11 Letters
points creating a line.
‘A’ , ‘I’ & ‘N’ occur twice - 3 Letters
Number of lines = 11C2 - 4C2 +1 = 55 – 6 +1 = 50 lines
Other letters are distinct E,X,M,T,O - 5 Letters
(ii) To get a triangle we need three points.
(a) All four letter are different from each other
No of triangle to be formed by 11 points are = 11C3
i.e., Letters are distinct [E,X,A,M,I,N,T,O]
But the 4 collinear points does not make any triangle .
= 8C4 x 4! = 1680 ----------------(1) 4C must be substrated.
3
(b) 2 Letters are same , other 2 letters are distinct
Number of triangle formed = 11C3 - 4C3 = 165 – 4 =161
i.e., Example : AAMT , EXNN
! 25. A polygon has 90 diagonals.
= 3C1 x 7C2 x = 756 -----------(2)
! Find the number of its sides?
(C) 2 letters are same and other two letters are same Solution :
i.e., Example : AANN , IINN Let there are n sides of a polygon.
!
= 3C2 x = 18 -----------(3) We know that the number of diagonals of n sided
! !
Total number of combination ( )
polygon is .
(1)+(2)+(3) = 1680 + 756 + 18 = 2454 ( )
22. How many triangles can be formed by joining 15 Given = 90
points on the plane, in which no line joining any 𝑛(𝑛 − 3) = 180
three points? 𝑛 − 3𝑛 − 180 = 0
Solution : (𝑛 − 15)(𝑛 + 12) = 0
To form a triangle we need 𝑛 = 15 𝑜𝑟 𝑛 = −12 (𝑛𝑜𝑡 𝑉𝑎𝑙𝑖𝑑)
minimum 3 non collinear points.
𝑛 = 15
3 points can be selected 15 non
There are 15 sides for the polygon which has 90
collinear points in 5C3 Ways.
5C =
diagonals.
3 = 455
23. How many triangles can be formed by 15 points, in
which 7 of them lie on one line and the remaining 8 on
another parallel line?
Solution :

To form a triangle we need 3 points .


Line 1 Line 2
Combination
(8 points) (7 points)
(1) 1 2 8C x7 C = 168
1 2
(2) 2 1 8C x7 C = 196
2 1
364
We may form 364 triangles in total.
PV MATRIC HR SEC SCHOOL 5. A test consists of 10 multiple choice questions. In how
many ways can the test be answered if
MATHEMATICS - XI STD (i) Each question has four choices?
Permutation to answer Every Question= 𝑃 =4
EXERCISE 4.2 No of ways to answer all the 10 Questions =
01.If 𝒏 𝟏 𝑷𝟑 : 𝒏
𝑷𝟒 = 𝟏: 𝟏𝟎 , 𝒇𝒊𝒏𝒅 𝒏. 4 x 4 x 4 x 4 x 4 x 4 x 4 x 4 x 4x 4 = 410
Solution : 𝑃: 𝑃 = 1: 10 (ii) The first four questions have three choices
𝑃 1 and the remaining have five choices?
= Permutation to answer every First 4 Question= 𝑃 =3
𝑃 10
(𝑛 − 1)(𝑛 − 2) (𝑛 − 3) 1 Permutation to answer every another 6 Question= 𝑃 =4
= No of ways to answer all the 10 Questions =
𝑛(𝑛 − 1)(𝑛 − 2)(𝑛 − 3) 10
1 1 3 x 3 x 3 x 3 x 4 x 4 x 4 x 4 x 4x 4 =34 x 46
= (iii) Question number n has n + 1 choices?
𝑛 10
𝑛 = 10 According to given question,
First question has 2 choice
02. 𝑰𝒇 𝟏𝟎 𝑷𝒓 𝟏 = 𝟐 𝑿 𝟔 𝑷𝒓 , 𝒇𝒊𝒏𝒅 𝒓. Second question has 3 choices
Solution : 𝑃 = 2𝑋 𝑃 Third question has 4 choices
𝑛! …….
𝐵𝑦 𝑢𝑠𝑖𝑛𝑔 𝑡ℎ𝑒 𝑓𝑜𝑟𝑚𝑢𝑙𝑎 𝑃 =
(𝑛 − 𝑟)! 10th question has 11 choices
10! 6! Number of ways to answer = 1 x 2 x 3 x ……x 11 = 11!
= 2𝑋
10 − (𝑟 − 1) ! (6 − 𝑟)! 6. A student appears in an objective test which contain 5
10! 6! multiple choice questions. Each question has four
= 2𝑋
(11 − 𝑟)! (6 − 𝑟)! choices out of which one correct answer.
10 𝑋 9 𝑋 8 𝑋 7𝑋 6! 6! (i) What is the maximum number of different answers
= 2𝑋
(11 − 𝑟)(10 − 𝑟)(9 − 𝑟)(8 − 𝑟)(7 − 𝑟)(6 − 𝑟)! (6 − 𝑟)! can the students give?
10 𝑋 9 𝑋 8 𝑋 7 5 multiple choice questions each has 4 choices.
=2
(11 − 𝑟)(10 − 𝑟)(9 − 𝑟)(8 − 𝑟)(7 − 𝑟) The maximum number of answer = 45
10 𝑋 9 𝑋 8 𝑋 7
(11 − 𝑟)(10 − 𝑟)(9 − 𝑟)(8 − 𝑟)(7 − 𝑟) = (ii) How will the answer change if each question may
2
Rewrite the above in descending order, have more than one correct answers?
(11 − 𝑟)(10 − 𝑟)(9 − 𝑟)(8 − 𝑟)(7 − 𝑟) = 7 𝑋 6 𝑋 5𝑋 4 𝑋 3 The questions may have 1 correct answer , or
𝑟=4 The questions may have 2 correct answer or
3 or 4 or 5 correct answer
03. (i) Suppose 8 people enter an event in a swimming
Number of correct answers = 1 + 2 + 3 + 4 + 5 = 15
meet. In how many ways could the gold, silver and
Maximum number of correct answers = 155.
bronze prizes be awarded?
Solution : 7. How many strings can be formed from the letters of
Total Number of Peoples = 8 the word ARTICLE, so that vowels occupy the even
Total number of Prizes = 3 [gold,silver,bronze] places?
Number of ways of awarding = 𝑃 = 8𝑥7𝑥6 Solution : Number of letters = 7
= 336 Ways. Vowels are A , I , E . Other Letters are R , T , C , L
(ii) Three men have 4 coats, 5 waist coats and 6 caps. In Condition : vowels occupy the even places
how many ways can they wear them? V V V
Total number of mens = 3 Available even places are 3 for three vowels
Number of coats = 4 Let the 3 vowels occupy 3 even places in 3! Ways
Number of Waist= 5 Remaining 4 letters occupy 4 odd places in 4! Ways
Number of Caps = 6 Number of strings = 3! X 4! = 6 x 24 = 144
Number of ways of wearing Coats = 𝑃 8 women and 6 men are standing in a line.
Number of ways of wearing Coats = 𝑃 (i) How many arrangements are possible if any
Number of ways of wearing Coats = 𝑃 individual can stand in any position?
Total Number of ways of wearing Since any individual can stand in any positions , 8 women
them = 𝑃 𝑥 𝑃𝑥 𝑃 and 6 men can be arrange in 𝑃 = 14!
= 24 x 60 x 120 (ii) In how many arrangements will all 6 men be
= 1,72,800 standing next to one another?
04. Determine the number of permutations of the letters Considering six men as one unit, we have 9 peoples and
of the word SIMPLE if all are taken at a time? they can arranged in 9! ways . these six men can arrange
Solution : SIMPLE among themselves in 6! Ways
Number of Letters = 6 Total number of arrangements = 9! X 6!
The number of permutations of the letters of (iii) In how many arrangements will no two men be
the word SIMPLE if all are taken at a time = 𝑃 standing next to one another?
= 6! According to the given condition let we arrange men and
= 720 women as follows
x w x w x w x w x w x w x w x w x
x ⟹ Seats for men (ii) How many different sequences containing six heads
and two tails are possible?
⟹9 seats available for 6 men= 𝑃 Number of possible ways =
!
= 28
! !

W ⟹ Seats for women 14.How many strings are there using the letters of the
word INTERMEDIATE, if
⟹8 seats available for 8 women= 𝑃 = 8! (i) The vowels and consonants are alternative
(ii) All the vowels are together (iii) Vowels are never
Required number of Arrangements = 8! X 𝑃
together (iv) No two vowels are together.
9.Find the distinct permutations of the letters of the Solution : INTERMEDIATE
word MISSISSIPPI? Total number of letters = 12
Solution : MISSISSIPPI Vowels = I E E I A E [ here 3 E’s , 2 I’s & 1 A ]
Total number of letters = 11 Constants = N T R M D T [ Here 2 T’s ]
Number of S’s = 4 (i) The vowels and consonants are alternative
Numbers of I’s = 4 A) If first starts with Vowel
Number of P’s = 2 V C V C V C V C V C V C
!
Required numbers of arrangements = = 34650 Number of 𝑉𝑜𝑤𝑙𝑒𝑠 𝐶𝑜𝑛𝑠𝑡𝑎𝑛𝑡𝑠
! ! ! = 𝑥
arrangements 𝐴𝑟𝑟𝑒𝑛𝑔𝑒𝑚𝑒𝑛𝑡𝑠 𝐴𝑟𝑟𝑒𝑛𝑔𝑒𝑚𝑒𝑛𝑡𝑠
10.How many ways can the product a2b3c4
be expressed ! !
= 𝑋
without exponents? ! ! ! !

Solution : a2b3c4 Means 2 a’s , 3 b’s and 4 c’s . B) If first starts with Constants
Total number of exponents = 9 C V C V C V C V C V C V
! Number of 𝑉𝑜𝑤𝑙𝑒𝑠 𝐶𝑜𝑛𝑠𝑡𝑎𝑛𝑡𝑠
Number of arrangements = = 1260 = 𝑥
! ! ! arrangements 𝐴𝑟𝑟𝑒𝑛𝑔𝑒𝑚𝑒𝑛𝑡𝑠 𝐴𝑟𝑟𝑒𝑛𝑔𝑒𝑚𝑒𝑛𝑡𝑠
! !
11. In how many ways 4 mathematics books, 3 physics = 𝑋
! ! ! !
books, 2 chemistry books and 1 biology book can be Total Number of ! ! ! !
arranged on a shelf so that all books of the same = 𝑋 + 𝑋
arrangements ! ! ! ! ! ! ! !
subjects are together. ! !
=2x 𝑋 = 43200
Solution : As per the given condition ! ! ! !

Take 4 maths books as 1 unit, (ii) All the vowels are together
Take all the vowels as I Unit, Then we have seven
3 Physics books as 1 unit,
units for arrangements .
2 Chemistry books as 1 unit, !
Number of arrangements = ( 𝑇ℎ𝑒𝑟𝑒 𝑎𝑟𝑒 2 𝑇 𝑠)
1 biology books a 1 unit !
!
For example,(below diagram represent 1 way of arrangements) Among the vowels number of arrangements =
! !
( 𝑇ℎ𝑒𝑟𝑒 𝑎𝑟𝑒 3 𝐸 𝑠 𝑎𝑛𝑑 2𝐼′𝑠)
! !
Required number of permutations = 𝑋
M1

M2

M3

M4

B
P1

P2

P3

C1

C2

! ! !
= 2520 x 60
= 151200 -----(1)
(iii) Vowels are never together
Vowels are 𝑇𝑜𝑡𝑎𝑙 𝑉𝑜𝑤𝑒𝑙𝑠 𝑎𝑟𝑒
There are 4 kinds of books arranged in 4! Ways. = −
𝑛𝑒𝑣𝑒𝑟 𝑡𝑜𝑔𝑒𝑡ℎ𝑒𝑟 𝐴𝑟𝑟𝑒𝑛𝑔𝑒𝑚𝑒𝑛𝑡𝑠 𝑡𝑜𝑔𝑒𝑡ℎ𝑒𝑟
4 maths books arranged among themselves in 4! Ways !
= – 151200 From (1)
3 physics books arranged among themselves in 3! Ways ! ! !
= 19958400 – 151200 = 19807200
2 chemistry books arranged among themselves in 2! Ways
(iv) No two vowels are together.
1 biology book arranged among themselves in 1 Way
Similar to (i) = 43200.
Total number of arrangements = 4! X 4! X 3! X 2! X 1
15.Each of the digits 1, 1, 2, 3, 3 and 4 is written on a
= 24 x 24 x 6 x 2 x 1
separate card. The six cards are then laid out
= 6912 Ways in a row to form a 6-digit number.
12.In how many ways can the letters of the word (i) How many distinct 6-digit numbers are there?
SUCCESS be arranged so that all Ss are together? (ii) How many of these 6-digit numbers are even?
Solution : SUCCESS (iii)How many of these 6-digit numbers are divisible by 4?
Let all the 3 Ss consider as 1 unit. Solution :
There will be 5 Units with 2 C’s . The digits are 1,1,2,3,3 and 4
!
Number of arrangements =
!
= = 60 (i) Number of distinct 6 digits Numbers = = 180
! !
!
(ii) If a number is even,The unit Place must be either 2 or 4
13.A coin is tossed 8 times, !
Number of cards = if 2 is in the unit place and
(i) How many different sequences of heads and tails are ! !
!
possible? = if 4 is in the unit place
! !
if n coins are tossed then the number of outcomes is 2n. Total number of cards =
!
+
!
= 𝑥
! ! ! !
No of possible = 28 = 30 + 30 = 60
(iii) If a number is divisible by 4 , then the last two digit
number must be divisible by 4. Therefore the last two 18.If the letters of the word FUNNY are permuted in all
digits should be 12 , 24 or 32 . possible ways and the strings thus formed
Number of Cards =
!
+
!
+
!
= 12 + 6 + 12 = 30 are arranged in the dictionary order, find the rank of the
! ! ! ! word FUNNY.
16.If the letters of the word GARDEN are permuted in Solution :
all possible ways and the strings thus formed FUNNY
are arranged in the dictionary order, then find the ranks
Alphabetical Order : 𝑭 N N U Y
of the words (i) GARDEN (ii) DANGER.
F Starts the word
Solution :
F N _ _ _ 3! 6
(i) GARDEN
F U Starts
Alphabetical Order : A D E 𝑮 N R
F U N Starts
A _ _ _ _ _ 5! 120
F U N N starts
D _ _ _ _ _ 5! 120
F U N N Y 1! 1
E _ _ _ _ _ 5! 120
7
G Starts the Word
G A Starts the Word [Rectified Solution for the TEXT BOOK- Example 4.42 ]
G A D _ _ _ 3! 6
If the letters of the word IITJEE are permuted in all
G A E _ _ _ 3! 6 possible ways and the strings thus formed are arranged
G A N _ _ _ 3! 6 in the lexicographic order, find the rank of the word
G A R Starts the Word IITJEE
G A R D Starts the Word IITJEE
G A R D E Starts
Alphabetical Order : E E 𝑰 I J T
G A R D E N 1! 1 𝟓!
Total 379 E - - - - - 60
𝟐!
I Starts the word
(ii) DANGER I E - - - - 4! 24
Alphabetical Order : A 𝑫 E G N R I I Starts the word
A _ _ _ _ _ 5! 120 I I E - - - 3! 6
D Starts the Word 𝟑!
I I J - - - 3
D A Starts the Word 𝟐!
D A E - - - 3! 6 I I T Starts
D A G - - - 3! 6 I I T E - - 2! 2
D A N Starts the Word I I T J Starts
D A N E - - 2! 2 I I T J E Starts
D A N G Starts I I T J E E 1! 1
D A N G E Starts TOTAL 96
D A N G E R 1! 1
Total 135 19. Find the sum of all 4-digit numbers that can be
17.Find the number of strings that can be made using all formed using digits 1, 2, 3, 4, and 5 repetitions not
letters of the word THING. If these words are written as allowed?
in a dictionary, what will be the 85th string? Solution : The number of 4-digit numbers that can be
Solution : formed using the given 5 digits is 5P4 = 120.
(i)The number of strings that can be made using all We first find the sum of the digits in the unit place of all
letters of the word THING = 5! = 120 these 120 numbers. By filling the 1 in unit place the
(ii)THING remaining three places can be filled with remaining 4
Alphabetical Order : G H I N T digits in 4P3 = 24 ways. This means, the number of 4-digit
Total Words numbers having 1 in units place is 4P3 = 24. Similarly, each
G - - - - 4! 24 24 of the digits 2, 3, 4, 5 appear 24 times in units place. An
H - - - - 4! 24 48 [24+24] addition of all these digits gives the sum of all the unit
I - - - - 4! 24 72 [24+24+24] digits of all 120 numbers. Therefore,
N Starts the Word (4P3 × 1) + (4P3 × 2) + (4P3 × 3) + (4P3 × 4) + (4P3 × 5)
N G - - - 3! 6 78 [ 72+6]
= 4P3 × (1 + 2 + 3 + 4 + 5)
N H - - - 3! 6 84 [ 78+6]
N I Starts = 4P3 × (sum of the digits)
Next Word will be the 85th Word = 4P3 × 15.
We need the word in alphabetical Similarly, we get the sum of the digits in
order which starts with “NI” . 10th place as 4P3 × 15. Since it is in
Arrange other letters in alphabetical order as“GHT”
N I G H T 1! 1 85th Word 10th place, its value is 4P3 ×15 × 10.
Similarly, the values of the sum of the digits in 100th place ------ Space for Study Notes ----
and 1000thplace are
4P3 ×15×100 and 4P3 ×15×1000 respectively.
Hence the sum of all the 4 digit numbers formed
by using the digits 1, 2, 3, 4, 5 is
(4P3 × 15) + (4P3 × 15 × 10)
+ (4P3 × 15 × 100) + (4P3 × 15 × 1000)
= 4P3 x 15 ( 1+10+100+1000)
= 24× 15 × 1111
= 399960.
Alternative Method :
RECALL:
𝑻𝒉𝒆𝒐𝒓𝒆𝒎𝟒. 𝟏: 𝑇ℎ𝑒 𝑠𝑢𝑚 𝑜𝑓 𝑎𝑙𝑙 𝑟 − 𝑑𝑖𝑔𝑖𝑡 𝑛𝑢𝑚𝑏𝑒𝑟𝑠 𝑡ℎ𝑎𝑡
𝑐𝑎𝑛 𝑏𝑒𝑓𝑜𝑟𝑚𝑒𝑑 𝑢𝑠𝑖𝑛𝑔 𝑡ℎ𝑒 𝑔𝑖𝑣𝑒𝑛 𝑛 𝑛𝑜𝑛 𝑧𝑒𝑟𝑜 𝑑𝑖𝑔𝑖𝑡𝑠 𝑖𝑠
( )
𝑃( ) 𝑋 (𝑆𝑢𝑚 𝑜𝑓 𝑡ℎ𝑒 𝐷𝑖𝑔𝑖𝑡𝑠)𝑋 111. . (𝑟 𝑇𝑖𝑚𝑒𝑠)
Solution : From Give Data ,
r = 4 ( Number of Digits )
n = 5 (Number of non zero numbers given 1,2,3,4,5)
Sum of All Numbers =
( )
𝑃( ) 𝑋 (𝑆𝑢𝑚 𝑜𝑓 𝑡ℎ𝑒 𝐷𝑖𝑔𝑖𝑡𝑠)𝑋 111. . (𝑟 𝑇𝑖𝑚𝑒𝑠)
= ( ) 𝑃( ) 𝑋(1 + 2 + 3 + 4 + 5)𝑋 1111
= 4P3× 15 × 1111
= 24× 15 × 1111
= 399960.
20. Find the sum of all 4-digit numbers that can be
formed using digits 0, 2, 5, 7, 8 without repetition?
Solution :
The given digits are 0, 2, 5, 7, 8
Number of four digit numbers that can be formed is
= 4 x 4 x 3 x 2 = 96
Out of 96,
24 Numbers are ending with 0
18 Numbers are ending with 2
18 Numbers are ending with 5
18 Numbers are ending with 7
18 Numbers are ending with 8
Which means total for unit place is
= (24x0) + (18x2) +(18x5) +(18x7) +(18x8)
= 0+ 18 ( 2 + 5 + 7+ 8 )
= 18 x 22 = 396
Total =
396 Units + 396 tens + 396 Hundreds + (24x22) Thousands
= 396 + 3960 + 39600 + 528000 = 571956
Alternative Method :
RECALL:
𝑻𝒉𝒆𝒐𝒓𝒆𝒎𝟒. 𝟐: 𝐼𝑓 0 𝑖𝑠 𝑜𝑛𝑒 𝑑𝑖𝑔𝑖𝑡 𝑎𝑚𝑜𝑛𝑔 𝑡ℎ𝑒 𝑔𝑖𝑣𝑒𝑛 𝑛 𝑑𝑖𝑔𝑖𝑡 ,
𝑡ℎ𝑒𝑛 𝑤𝑒 𝑔𝑒𝑡 𝑡ℎ𝑎𝑡 𝑡ℎ𝑒 𝑠𝑢𝑚 𝑜𝑓 𝑡ℎ𝑒 𝑟 𝑑𝑖𝑔𝑖𝑡 𝑛𝑢𝑚𝑏𝑒𝑟𝑠 𝑡ℎ𝑎𝑡 𝑐𝑎𝑛
𝑏𝑒 𝑓𝑜𝑟𝑚𝑒𝑑 𝑢𝑠𝑖𝑛𝑔 𝑡ℎ𝑒 𝑔𝑖𝑣𝑒𝑛 𝑛 𝑑𝑖𝑔𝑖𝑡 (𝐼𝑛𝑐𝑙𝑢𝑑𝑖𝑛𝑔 0)𝑖𝑠
( )
𝑃( ) 𝑋 (𝑆𝑢𝑚 𝑜𝑓 𝑡ℎ𝑒 𝐷𝑖𝑔𝑖𝑡𝑠)𝑋 111. . (𝑟 𝑇𝑖𝑚𝑒𝑠) −
( )
𝑃( ) 𝑋 (𝑆𝑢𝑚 𝑜𝑓 𝑡ℎ𝑒 𝐷𝑖𝑔𝑖𝑡𝑠)𝑋 111. . (𝑟 − 1 𝑇𝑖𝑚𝑒𝑠)
Solution : From Give Data ,
r = 4 ( Number of Digits )
n = 5 (Number of numbers given 0,2,5,7,8)
Sum of All Numbers =
( )
𝑃( ) 𝑋 (𝑆𝑢𝑚 𝑜𝑓 𝑡ℎ𝑒 𝐷𝑖𝑔𝑖𝑡𝑠)𝑋 111. . (𝑟 𝑇𝑖𝑚𝑒𝑠) −
( )
𝑃( ) 𝑋 (𝑆𝑢𝑚 𝑜𝑓 𝑡ℎ𝑒 𝐷𝑖𝑔𝑖𝑡𝑠)𝑋 111. . (𝑟 − 1 𝑇𝑖𝑚𝑒𝑠)
= { 𝑃 𝑋 22 𝑋 1111} − { 𝑃 𝑋 22 𝑋 111}
= (24 x 22 x 1111)-(6x 22 x 111) =586608-14652=571956.
Binomial Theorem, Sequences and Series 1

PV MATRIC HIGHER SECONDARY SCHOOL

Binomial Theorem, Sequences and Series


Solution Book
5
Points To Remember

Binomial Theorem :
(𝑎 + 𝑏) = nC0 an b0 + nC1 an-1 b1+……….+ nCr an-r br+………+ nCn a0 bn
Observations :
 There are (n+1) terms in the expansion of (a+b)n.
 First term is always an and last term is always bn.
 The coefficients at equidistance from the centre are equal. That is coefficient of
first and last term are equal, coefficient of 2nd and that of last but one are equal
and so on.
 Replacing b by –b we get, alternatively positive and negative terms.
 If a=1 , b=x gives (1+x)n= 1 + nC1 x + nC2 x2+ nC3 x3+…………..+ nCn xn.
 If x=1 , in the above equation 2n=1+ nC1+ nC3+ nC3…..+ nCn
C0+C1+C2+………+Cn=2n
[Notation Cr replaces the notation nCr When n is fixed]
 Changes x by –x,we get(1-x)n= 1 - nC1 x + nC2 x2- nC3 x3+…………..+ (-1)nxn
 Put x=1 in above We get 0 = 1- nC1+ nC2- nC3+ nC4- ………….
nC1+ nC3+ nC5 …… = 1+ nC2+ nC4+ …….
nC1+ nC3+ nC5…… = nC0+ nC2+ nC4+ …….
Sum of odd coefficients = Sum of Even Coefficients
 General term of the expression of the expression (x+a)n is Tr+1= nCr xn-r ar
 Middle Term of (x+y)n is
 If n is even , the middle Term is 𝑇
 If n is odd, The middle terms are 𝑇 and 𝑇
Greatest coefficient : 𝑛𝐶 [if n is even] . 𝑛𝐶 and 𝑛𝐶 [if n is odd]

PVMHSS |
Binomial Theorem, Sequences and Series 2

Exercise 5.1
𝟑 𝟑
01.Expand (i) 𝟐𝒙𝟐 − (ii) (𝟐𝒙𝟐 − 𝟑√𝟏 − 𝒙𝟐 )𝟒 + (𝟐𝒙𝟐 + 𝟑√𝟏 − 𝒙𝟐 )𝟒
𝒙

Solution : (𝑥 + 𝑎) = xn + nC1 xn-1 a1+………….+ nCr xn-r ar+…………+ an

(𝑥 − 𝑎) = xn - nC1 xn-1 a1+……….+ nCr xn-r ar+………+ (-1)n an


(i) 2𝑥 − = (2𝑥 )3 - 3C1 (2𝑥 )2 ( )1 + 3C2 (2𝑥 )1 ( )2 - 3C3 (2𝑥 )0 ( )3
= 8𝑥 - 3 (4𝑥 ) ( ) + 3 (2𝑥 ) ( )-
= 8𝑥 − 36𝑥 + 54 − .
(ii) (2𝑥 − 3√1 − 𝑥 ) + (2𝑥 + 3√1 − 𝑥 )
Put √1 − 𝑥 = 𝑘
(2𝑥 − 3𝑘) + (2𝑥 + 3𝑘)
= {(2𝑥 )4 - 4C1 (2𝑥 )3 (3𝑘) + 4C2 (2𝑥 )2 (3𝑘)2 -4C3 (2𝑥 ) (3𝑘)3+(3𝑘)4 } +
{(2𝑥 )4 + 4C1 (2𝑥 )3 (3𝑘) + 4C2 (2𝑥 )2 (3𝑘)2 +4C3 (2𝑥 ) (3𝑘)3+(3𝑘)4 }
= 2 [16𝑥 + 24 𝑥 (9k2)+81𝑘 ]
= 2 [16𝑥 + 216 𝑥 k2 + 81 𝑘 ]
= 2 [16𝑥 + 216 𝑥 (1 − 𝑥 )+81 (1 − 𝑥 )2]
= 2 [16𝑥 + 216 𝑥 (1 − 𝑥 )+81 (1 − 2𝑥 + 𝑥 ) ]
= 2 [16𝑥 + 216 𝑥 - 216 𝑥 + 81 - 162𝑥 +81𝑥 ]
=2 [16𝑥 - 216 𝑥 +297 𝑥 - 162𝑥 +81]
= 32𝑥 - 512 𝑥 +594 𝑥 - 324𝑥 +162
02. Compute (i)1024 (ii) 994 (iii) 97
Solution : `
(i) 1024 = (100+2)4= 1004 + 4C1 (1003) (2) +4C2 (1002) (2)2+4C1 (100) (2)3 +(2)4
= 1004 + 8 (1003) +24 (1002) +32 (100) +16
= 10,82,43,216
(ii) 994 = (100-1)4= 1004 - 4C1 (1003) (1) +4C2 (1002) (1)2-4C1 (100) (1)3 +(1)4
= 1004 - 4 (1003) +6 (1002) - 4 (100) +1
= 9,60,59,601
(iii)97=(10-1)7= 107 - 7C1 106 (1) +7C2 105 (1)2 - 7C3 104 (1)3+7C4 103 (1)4
-7C5 102 (1)5+7C6 101 (1)6-(1)7
= 107 - 7C1 106 +7C2 105 - 7C3 104 +7C4 103 -7C5 102 +7C6 101-1
= 10000000-7000000+2100000-350000+35000-2100+70-1
= 47,82,969.

PVMHSS |
Binomial Theorem, Sequences and Series 3
3. Using binomial theorem, indicate which of the following two number is larger:
(1.01)1000000, 10000.
Solution : (1.01)1000000 = (1+0.01)1000000
= 11000000+1000000C1 1999999 (0.01)1+ 1000000C2 1999998 (0.01)2......
= 1+ (1000000)(0.01)+ 1000000C2 1999998 (0.01)2……….
= 1+ 10000+ 1000000C2 1999998 (0.01)2+………..
(1.01)1000000 = 10001+1000000C2 1999998 (0.01)2+………..
It is clear that (1.01)1000000 > 10000
𝟏 𝟏𝟎
4.Find the coefficient of x15 in (𝒙𝟐 + ) .
𝒙𝟑
Solution : Tr+1= nCr xn-r ar
= 10Cr (𝑥 )10-r ( )r
= 10Cr x20-2r
= 10Cr x20-2r-3r
=10Cr x20-5r
We have to find the co-efficient of x15 so that we have compare 20-5r with 15.
Now 20-5r = 15
-5r=-5
𝑟=1
Tr+1= nCr xn-r ar
If r=1 T1+1= 10C1 (𝑥 )10-1 ( )1 =10 (𝑥 )9
T1+1 = 10 (𝑥 ) = 10 𝑥 . ∴ Coefficient of x15 is 10.
𝟏 𝟔
5.Find the coefficient of x6 and the coefficient of x2 in (𝒙𝟐 − )
𝒙𝟑
Solution : Tr+1= nCr xn-r ar
= 6Cr (𝑥 )6-r ( )r
( )
= 6Cr x12-2r
= 6Cr (−1) x12-2r-3r
= 6Cr (−1) x12-5r
a) Finding Coefficient of x6 b) Finding Coefficient of x2
We have to find the co-efficient of x6 We have to find the co-efficient of x2
so that we have compare 12-5r with 6. so that we have compare 12-5r with 2.
12-5r=6 12-5r=2
-5r=-6 -5r=-10
r= r=2
r cannot be a fraction . If r=2 ; T3=6C2 (−1) x12-10
There is no term containing x6 = 15 x2
Co-efficient of x2 is 15.

PVMHSS |
Binomial Theorem, Sequences and Series 4
𝟏 𝟓
6.Find the co-efficient of x4 in the expansion of (𝟏 + 𝒙𝟑 )𝟓𝟎 𝒙𝟐 +
𝒙

Solution : (1 + 𝑥 ) 𝑥 +
= {1+50C1 x3+50C2 𝑥 +50C3 𝑥 +…..+𝑥 } {(x2)5 +5C1 (x2)4 ( )+ 5C2 (x2)3 ( )2+…..+( )5}
= {1+50C1 x3+50C2 𝑥 +50C3 𝑥 +…..+𝑥 } { x10+5C1 x8 ( )+ 5C2 (x6) ( ) +…..+( ) }
= {1+50C1 x3+50C2 𝑥 +50C3 𝑥 +…..+𝑥 } { x10 + 5 x7 + 10 x4 + 10 x + + }
Coefficient of x4 ⇒ (1 x 10 ) + (50C1x10) + (50C2 x 5) + (50C3 x 1)
⇒ 10 + 500 +6125 +19600 = 26235 .
𝟏 𝟓
7.Find the constant term of 𝟐𝒙𝟑 −
𝟑𝒙𝟐
Solution : Tr+1= nCr xn-r ar
= 5Cr (2𝑥 )5-r ( )r
( )
= 5Cr (2)5-r (x3)5-r
( )
= 5Cr (2)5-r (x)15-3r
( )
= 5Cr (2)5-r (x)15-3r-2r
( )
= 5Cr (2)5-r (x)15-5r
We have to find the co-efficient of x0 for getting constant term.
so that we have compare 15-5r with 0.
If 15-5r = 0 ⟹ 𝑟 = 3
( )
If r=3 Constant term is T4= 5C3 (2)5-3 (x)15-5(3) = 10 x 22 x =
8.Find the last two digit of the number 3600.
Solution : 3600= (32)300= 9300 = (10-1)300
= 10300 - 300C1 (10)299(1)1+300C2 (10)298(1)2-…….-300C299 (10)1(1)299+(1)300
3600 =10300 - 300C1 (10)299(1)1+300C2 (10)298(1)2-…….-3000+1
When simplifying above expansion we may get the value of 3600 .
The last two digit of 3600 will be 01.
9.If n is a Positive Integer , Show that 9n+1-8n-9 is always divisible by 64.
Solution : Given : 9n+1-8n-9
9n+1 = (1+8)n+1= 1n+1 + n+1C1 (1)n (8)1 + n+1C2 (1)n-1 (8)2+ n+1C3 (1)n-2 (8)3+…………+ 8n+1
= 1+(n+1)8+n+1C2 (8)2+ n+1C3 (8)3+ ……….+8n+1
= 1+8n+8+n+1C2 (8)2+ n+1C3 (8)3+ ……….+8n+1
= 8n+9+n+1C2 (8)2+ n+1C3 (8)3+……….+8n+1
9n+1-8n-9 = {8n+9+n+1C2 (8)2+ n+1C3 (8)3+……….+8n+1}-8n-9
= n+1C2 (8)2+ n+1C3 (8)3+……….+8n+1
= (8)2 [n+1C2 + n+1C3 (8) +……….+8n-1] = 64 [n+1C2 + n+1C3 (8) +……….+8n-1]
∴ 9n+1-8n-9 is divisible by 64.

PVMHSS |
Binomial Theorem, Sequences and Series 5
10. If n is an odd positive integer, prove that the coefficients of the middle terms in
the expansion of (x + y)n are equal.
Solution : Here If n is odd. So that middle terms are 𝑇 and 𝑇

middle terms are 𝑇 = 𝐶 𝑥 𝑦 and 𝑇 = 𝐶 𝑥 𝑦


Coefficient of 𝑇 = 𝐶
= 𝐶 [∵ nCr = nCn-r
= 𝐶
= 𝐶
= Coefficient of 𝑇
∴ Coefficient of 𝑇 = Coefficient of 𝑇 Hence Proved.
11. If n is a positive integer and r is a nonnegative integer, prove that
the coefficients of xr and xn−r in the expansion of (1 + x)n are equal.
Solution : Tr+1= nCr xn-r ar
Term xr of (1+x)n is Tr+1= nCr 1n-r xr
Tr+1 = nCr xr ∴Co-efficient of xr is nCr .
Term x n-r of (1+x)n is T n-r+1 = nC n-r 1n-n-r xn-r
T n-r+1 = nC n-r xn-r ∴Co-efficient of x n-r is nC n-r.
But nC n-r = nCr using property of combination.
∴Co-efficient of xr and Co-efficient of x n-r are equal.
12. If a and b are distinct integers, prove that a − b is a factor of an - bn,
whenever n is a positive integer. [Hint: write an = (a − b + b)n and expand]
Solution : 𝑎 − 𝑏 = (𝑎 + 𝑏 − 𝑏) − 𝑏
= {[(a − b) + b] } − 𝑏
={(a − b) + 𝑛𝐶 (a − b) 𝑏 + 𝑛𝐶 (a − b) 𝑏 + ⋯ . + 𝑛𝐶 (a − b) 𝑏 +𝑏 }−𝑏
= (a − b) + 𝑛𝐶 (a − b) 𝑏 + 𝑛𝐶 (a − b) 𝑏 + ⋯ . + 𝑛𝐶 (a − b) 𝑏 +𝑏 −𝑏
= (a − b) + 𝑛𝐶 (a − b) 𝑏 + 𝑛𝐶 (a − b) 𝑏 + ⋯ . + 𝑛𝐶 (a − b) 𝑏
𝑎 − 𝑏 = (𝑎 − 𝑏) {(a − b) + 𝑛𝐶 (a − b) 𝑏 + 𝑛𝐶 (a − b) 𝑏 + ⋯ . + 𝑛𝐶 𝑏 }
∴ a − b is a factor of an - bn
13.In the binomial expansion of (a + b)n, the coefficients of the 4th and 13th terms
are equal to each other, find n.
Solution : 𝑇 =𝑇 = 𝑛𝐶 𝑎 𝑏 𝑎𝑛𝑑
𝑇 =𝑇 = 𝑛𝐶 𝑎 𝑏
Given that co-efficients are equal 𝑛𝐶 = 𝑛𝐶
𝑛𝐶 = 𝑛𝐶
𝑛 − 3 = 12
𝑛 = 12 + 3
𝑛 = 15

PVMHSS |
Binomial Theorem, Sequences and Series 6
14.If the binomial coefficients of three consecutive terms in the expansion of (a + x)n
are in the ratio 1 : 7 : 42, then find n.
Solution : Let 𝑇 , 𝑇 , 𝑇 be the terms whose coefficients are in the ratio 1: 7 :42
𝑛𝐶 ∶ 𝑛𝐶 ∶ 𝑛𝐶 = 1 ∶ 7 ∶ 42
𝑛𝐶𝑟+1
Case (i) : = Case (ii) : =
𝑛𝐶𝑟+2
! !
!( )! ( )!( )!
! = ! =
( )!( )! ( )!( )!

! ( )!( )! ! ( )!( )!
𝑥 = 𝑥 =
!( )! ! ( )!( )! !

( )!( )! ( )!( )!
= =
!( )! ( )!( )!

( ) ! ( )! ( )( )! ( )!
= =
!( )( )! ( )! ( )( )!

= =

𝑛 − 8𝑟 = 7 − − − (1) 𝑛 − 7𝑟 = 13 − − − (2)

By solving (1) and (2) we get n=55 and r= 6


15. In the binomial coefficients of (1 + x)n, the coefficients of the 5th, 6th and 7th
terms are in AP. Find all values of n.
Solution : Given Coefficients of T4,T5,T6 , in the expression of (1+x)n are in A.P .

i.e., nC4 , nC5 , nC6 are in A.P.


in A.P Second Term – First Term = Third Term – Second Term
nC
5 - nC4 = nC6 - nC5
2(nC5) = nC6 + nC4
2 𝑥 𝑛! 𝑛! 𝑛!
= +
5! (𝑛 − 5)! 6! (𝑛 − 6)! 4! (𝑛 − 4)!
!( )!
Multiply by ⇒ = +
! ( ) ( )( )
( )( )
=
( ) ( )( )
=
𝑛 − 21𝑛 + 98 = 0
(𝑛 − 7)(𝑛 − 14) = 0
𝑛 = 7 𝑜𝑟 𝑛 = 14

PVMHSS |
Binomial Theorem, Sequences and Series 7
𝟐𝒏!
16. 𝑷𝒓𝒐𝒗𝒆 𝒕𝒉𝒂𝒕 𝑪𝟐𝟎 + 𝑪𝟐𝟏 + 𝑪𝟐𝟐 + ⋯ … … … . + 𝑪𝟐𝒏 =
(𝒏!)𝟐
Here Notation Cr replaces the notation nCr When n is fixed.
We know that (1+x)n= 1 + nC1 x + nC2 x2+ nC3 x3+…………..+ nCn xn
Let we written as (1 + 𝑥) = 𝐶 + 𝐶 𝑥 + 𝐶 𝑥 + ⋯ … . +𝐶 𝑥
Similarly (𝑥 + 1) = 𝐶 𝑥 + 𝐶 𝑥 +𝐶 𝑥 + ⋯….𝐶 𝑥 + 𝐶 𝑥 + 𝐶
Now (1 + 𝑥) (𝑥 + 1) =
{𝐶 + 𝐶 𝑥 + 𝐶 𝑥 + ⋯ … . +𝐶 𝑥 }{𝐶 𝑥 + 𝐶 𝑥 + ⋯+ 𝐶 𝑥 + 𝐶 }
Now (1 + 𝑥) = {𝐶 + 𝐶 𝑥 + 𝐶 𝑥 + ⋯ … . +𝐶 𝑥 }{𝐶 𝑥 + 𝐶 𝑥 + ⋯+ 𝐶 𝑥 + 𝐶 }
Take coefficients of xn on both sides ,
General term in
2𝑛𝐶 = 𝐶 + 𝐶 + 𝐶 + ⋯ … … … . + 𝐶
! (1 + 𝑥) is Tr+1
)!
= 𝐶 + 𝐶 + 𝐶 + ⋯ … … … . + 𝐶 T =2nC (1)2n-r xr
!( r+1 n
!
= 𝐶 + 𝐶 + 𝐶 +⋯……….+ 𝐶 Put r=n
! !
2𝑛! Tn+1=2nCn (1)2n-n xr
= 𝐶 + 𝐶 + 𝐶 + ⋯……….+ 𝐶 Where coefficient is
(𝑛!)
2nCn
Hence Proved.

Points To Remember

Arithmetico-
Arithmetic Geometric Harmonic
Geometric
Progression Progression Progression
Progression
A.P GP AGP HP
a,a+d,a+2d,………. a,ar,ar2,ar3,…...... a,(a+d)r,(a+2d)r ,.
2 1
,
1
,
1
..
…a+(n-1)d,a+nd,.. ……...arn-1,arn,... …(a+(n-1)d)r …
n-1 𝑎 𝑎 + 𝑑 𝑎 + 2𝑑
…. ( ) , … ..
Example: Example: Example: Example:
3,7,11,15…….. 2,6,18,54,…….. 4,14,40,104,…… 1, , … … …
Tn= a+(n-1)d Tn= arn-1 Tn=(a+(n-1)d)rn-1 Tn= ( )
If a,b,c are in AP If a,b,c are in G.P If a,b,c are inH.P
2𝑏 = 𝑎 + 𝑐 𝑏 = 𝑎𝑐 2𝑎𝑐
𝑏=
𝑎+𝑐
Means of two positive real numbers
A.M : Arithmatic mean of two positive real numbers a & b is .
G.M : Geometric Mean of two positive real numbers a & b is +√𝑎𝑏 .
H.M : Harmonic Mean of two positive real numbers a & b is .
Relation between A.M, G.M , and H.M
(i) A.M > G.M > H.M (ii) G.M = (𝐴. 𝑀)(𝐻. 𝑀)
(iii) A.M,G.M,H.M forms a decreasing G.P.

PVMHSS |
Binomial Theorem, Sequences and Series 8

Exercise 5.2
01. Write the first 6 terms of the sequences whose nth terms are given below and
classify them as arithmetic progression, geometric progression, arithmetico-
geometric progression, harmonic progression and none of them.
𝟏
(i) 𝒏 𝟏
𝟐
𝟏
Here nth term is
𝟐𝒏 𝟏
The First six terms are , , , , , …….
Which is G.P . where a = and r = .
(𝐧 𝟏)(𝐧 𝟐)
(ii)
(𝐧 𝟑)(𝐧 𝟒)
( )( )
Here nth term is
( )( )
( )( )
The First six terms are , , , , , ……..
( )( )
It is Not an AP/GP/HP
𝟏 𝒏
(iii) 𝟒
𝟐

Here nth term is 4 .

The First six terms are 4 ,4 ,4 ,4 ,4 ,4 … . .4


i.e., 2 ,1 , , , , … … … . .4
Which is a G.P . Where a = 2 and r = .
( 𝟏)𝒏
(iv)
𝒏
( 𝟏)𝐧
Here nth term is .
𝐧
( 𝟏)𝐧
The First six terms are -1 , , , , , ………..
𝐧
It is Not an AP/GP/HP
𝟐𝒏 𝟑
(v)
𝟑𝒏 𝟒
Here nth term is .
The First six terms are , , , , , …….
It is Not an AP/GP/HP
(vi) 2018
Here tn=2018 . Which brings a constant sequence.
The First six terms are 2018, 2018, 2018, 2018, 2018, 2018,……….. 2018,
Constant sequence has same common ratio and same common difference.
Hence it is an A.P , G.P and AGP.
𝟑𝒏 𝟐 𝟑𝒏 𝟐
(vii) 𝒏 𝟏 Here nth term is 𝒏 𝟏 .
𝟑 𝟑
𝟑𝒏 𝟐
The First six terms are 1, , , , , ,….. . Which is AGP.
𝟑𝒏 𝟏

PVMHSS |
Binomial Theorem, Sequences and Series 9
2.Write the first 6 terms of the sequences whose nth term an is given below.
𝒏 + 𝟏 𝑰𝒇 𝒏 𝒊𝒔 𝒐𝒅𝒅
(i) 𝒂𝒏 =
𝒏 𝑰𝒇 𝒏 𝒊𝒔 𝒆𝒗𝒆𝒏
Solution : If n is odd an= n+1 and If n is even an=n
The First six terms are 2,2,4,4,6,6,……
𝟏 𝑰𝒇 𝒏 = 𝟏
(ii) 𝒂𝒏 = 𝟐 𝑰𝒇 𝒏 = 𝟐
𝒂𝒏 𝟏 + 𝒂𝒏 𝟐 𝑰𝒇 𝒏 > 𝟐
Solution : According to the given condition ,
The First six terms are 1,2,3,5,8,13……..
𝒏 𝑰𝒇 𝒏 = 𝟏, 𝟐, 𝟑
(iii) 𝒂𝒏 = 𝒂
𝒏 𝟏 + 𝒂𝒏 𝟐 + 𝒂𝒏 𝟑 𝑰𝒇 𝒏 > 𝟑
Solution : According to the given condition ,
The First six terms are 1,2,3,6,11,20……..
3. Write the nth term of the following sequences.
(i) 2,2,4,4,6,6,…………
Solution : Given sequence is 2,2,4,4,6,6….
The odd terms are 2,4,6 ….. The Even terms are 2,4,6,…..
𝑛 + 1 𝐼𝑓 𝑛 𝑖𝑠 𝑜𝑑𝑑
𝑎 =
𝑛 𝐼𝑓 𝑛 𝑖𝑠 𝑒𝑣𝑒𝑛
𝟏 𝟐 𝟑 𝟒 𝟓
(ii) , , , , …
𝟐 𝟑 𝟒 𝟓 𝟔
𝟏 𝟐 𝟑 𝟒 𝟓
Solution : Given sequence is , , , , …
𝟐 𝟑 𝟒 𝟓 𝟔
Consider the numerator 1,2,3,4,5….. a =1 and d = 2-1 = 1
Now an= a+(n-1)d = 1+(n-1)1=1+n-1=n
Consider the denominator 2,3,4,5,6….. a =2 and d =3-2 = 1
Now an= a+(n-1)d = 2+(n-1)1=2+n-1=n+1
Now 𝑎 = for all n∈ 𝑁
𝟏 𝟑 𝟓 𝟕 𝟗
(iii) , , , , …
𝟐 𝟒 𝟔 𝟖 𝟏𝟎
𝟏 𝟑 𝟓 𝟕 𝟗
Solution : Given sequence is , , , , … ..
𝟐 𝟒 𝟔 𝟖 𝟏𝟎
Consider the numerator 1,3,5,7,9….. a =1 and d = 3-1 = 2
Now an= a+(n-1)d = 1+(n-1)2=1+2n-2=2n-1
Consider the denominator 2,4,6,8,10….. a =2 and d =4-2 = 2
Now an= a+(n-1)d = 2+(n-1)2=2+2n-2=2n
Now 𝑎 = for all n∈ 𝑁.
(iv) 6,10,4,12,2,14,0,16,-2,……
Solution : Given sequence is 6,10,4,12,2,14,0,16,-2,……
Consider the odd terms are 6,4,2,0,-2 ….. a=6 and d= 4-6=-2
Now an= a+(n-1)d = 6+(n-1)(-2)=6-2n+2=8-2n
Consider the Even terms are 10,12,14,16,…….a=10 and d= 12-10=2

PVMHSS |
Binomial Theorem, Sequences and Series
1
0
Now an= a+(n-1)d = 10+(n-1)(2)=10+2n-2=8+2n
8 − 2𝑛 𝐼𝑓 𝑛 𝑖𝑠 𝑜𝑑𝑑
𝑎 = for all n∈ 𝑁
8 + 2𝑛 𝐼𝑓 𝑛 𝑖𝑠 𝑒𝑣𝑒𝑛
4.The product of three increasing numbers in GP is 5832. If we add 6 to the second
number and 9 to the third number, then resulting numbers form an AP.
Find the numbers in GP.
Solution : Let the three numbers are in G.P be, , 𝑎, 𝑎𝑟
Given : Product of these 3 numbers = 5832
𝑥 𝑎 𝑥 𝑎𝑟 = 5832
a3 = 5832
𝑎 = 18
Also give that , 18 + 6 , 18𝑟 + 9 𝑎𝑟𝑒 𝑖𝑛 𝐴. 𝑃
2(24) = + 18𝑟 + 9
48 = + 18𝑟 + 9
48r=18+18𝑟 +9r
6𝑟 − 13𝑟 + 18 = 0
(3r-2)(2r-3)=0
𝑟= 𝑎𝑛𝑑
Required three numbers are in Increasing G.P. so that r= .
The numbers in G.P are 12,18,27.

𝟑 𝟓 𝟕
5. Write the nth term of the sequence 𝟏𝟐𝟐𝟐 , 𝟐𝟐𝟑𝟐 , 𝟑𝟐𝟒𝟐 … … . . 𝒂𝒔 𝒂 𝒅𝒊𝒇𝒇𝒆𝒓𝒆𝒏𝒄𝒆 𝒐𝒇 𝒕𝒘𝒐 𝒕𝒆𝒓𝒎𝒔.
𝟑 𝟓 𝟕
Solution : Given sequence is𝟏𝟐 𝟐𝟐 , 𝟐𝟐 𝟑𝟐 , 𝟑𝟐 𝟒𝟐 … … ..
Consider the numerator 3,5,7….. a =3 and d = 5-3 = 2
Now an= a+(n-1)d = 3+(n-1)2=3+2n-2=2n+1.
By observing the denominator n2 (n+1)2 is the general term.
Now general tem of the given sequence is tn = +
( )

= By adding and subtracting 𝑛


( )
( )
=
( )

= −
( ) ( )
( )
= −
( ) ( )
= −
( )

PVMHSS |
Binomial Theorem, Sequences and Series
1
1

6. If tk is the kth term of a GP, then show that tn−k, tn, tn+k also form a GP for any
positive integer k.
Solution : kth term in GP = tk= a rk-1
Then tn-k= a rn-k-1 ; tn= a rn-1 ; tn+k= a rn+k-1
To prove tn−k, tn, tn+k are in G.P b2 = ac
(tn)2 = (tn−k)( tn+k )
(a rn-1)2 = (a rn-k-1)( a rn+k-1 )
L.H.S : (a rn-1)2 = a2 r2n-2 ----(1)
R.H.S: (a rn-k-1)( a rn+k-1 ) = a2 r2n-2 ----(2)
From (1) and (2) L.H.S = R.H.S Hence tn−k, tn, tn+k are in G.P.
7.If a, b, c are in geometric progression, and if 𝒂𝟏/𝒙 = 𝒃𝟏/𝒚 = 𝒄𝟏/𝒛 then prove that
x, y, z are in arithmetic progression.
Solution : Given 𝑎 / = 𝑏 / = 𝑐 / = 𝑘
Then 𝑎 / = 𝑘 ; 𝑏 / = 𝑘 ; 𝑐 / = 𝑘
𝑎 =𝑘 ; 𝑏 =𝑘 ; 𝑐 = 𝑘
Given that a, b, c are in geometric progression then b2 = ac .
i.e., ( 𝑘𝑦 ) = 𝑘𝑥 . 𝑘𝑧
𝑘 =𝑘
i.e., 2y = x+z
y+y = x+z
y-x = z-y ⟹ t2-t1=t3-t2 then, t1,t2,t3 are in A.P
i.e., x,y,z are inA.P.
8.The AM of two numbers exceeds their GM by 10 and HM by 16.
Find the numbers.
Solution : AM-GM=10 and AM – HM = 16
Let A.M = =x
G.M = √𝑎𝑏 = 𝑦
H.M = =
𝑥 − 𝑦 = 10 ⟹ 𝑥 = 10 + 𝑦
𝑥− = 16 ⟹ (10 + 𝑦) − = 16
100 + 2𝑦 + 𝑦 − 𝑦 = 160 + 16𝑦
4𝑦 = 60 ⟹ 𝑦 = 15
Here 𝑥 = 10 + 15 = 25
⟹ 𝑎 + 𝑏 = 50 ⟹ 𝑎 = 50 − 𝑏
√𝑎𝑏 = 15 ⟹ 𝑎𝑏 = 225
⟹ (50 − 𝑏)𝑏 = 225

PVMHSS |
Binomial Theorem, Sequences and Series
1
2
⟹ 𝑏 − 25 + 225 = 0
⟹ (𝑏 − 5)(𝑏 − 45) = 0
𝑏 = 5 𝑜𝑟 𝑏 = 45 𝑎𝑙𝑠𝑜 𝑎 = 45 𝑜𝑟 𝑎 = 5
Hence the numbers are 45 and 5.
9. If the roots of the equation (q − r)x2 + (r − p)x + p − q = 0 are equal, then show that
p, q and r are in AP.
Solution : (q − r)x2 + (r − p)x + p − q = 0
(q − r)x2 + (r − p+q-q)x + p – q = 0
(q − r)x2 ((q-r)+(p-q))x + (p – q) = 0
By factorising (q-r)x [x-1] + (p-q)[x-1]=0
(𝑥 − 1)[ (𝑞 − 𝑟)𝑥 − (𝑝 − 𝑞)] = 0
X= 1 and x=
Where roots are equal x=
𝑞−𝑟 = 𝑝−𝑞
⟹ t2-t1=t3-t2 then, t1,t2,t3 are in A.P
so that p,q,r are in A.P.
10.If a, b, c are respectively the pth, qth and rth terms of a GP, show that
(q − r) log a + (r − p) log b + (p − q) log c = 0.
Solution : A , AR , AR2 , ……ARn-1 are in G.P
𝑎 = 𝑡𝑒𝑟𝑚 𝑝 ⟹ 𝑡 = 𝑎 = 𝐴 𝑅
𝑏 = 𝑡𝑒𝑟𝑚 𝑞 ⟹ 𝑡 = 𝑎 = 𝐴 𝑅
𝑐 = 𝑡𝑒𝑟𝑚 𝑟 ⟹ 𝑡 = 𝑎 = 𝐴 𝑅
Let A be the first term . R be the common ratio of G.P

Prepared By
S.Manikandan.,M.Sc.,B.Ed.,
PG.Teacher in Mathematics
PVMHSS |
Binomial Theorem, Sequences and Series
1
3

Points To Remember
Sum of Arithmetic, Geometric and Arithmetico-Geometric Progressions
Series Definitions Sum formula

Arithmetic
A series is said to be an arithmetic series if S = [2𝑎 + (𝑛 − 1)𝑑] [or]
the terms of the series form an arithmetic 𝑛
series S = [𝑎 + 𝑙 ]
sequence. 2
( )
A series is said to be a geometric series if S = if r > 1
Geometric
the terms of the series form a geometric S =
( )
if r > 1
series
sequence.
Sn = na if r=1
Arithmetico- A series is said to be an arithmetico- 𝑎 − (𝑎 + (𝑛 − 1)𝑑)𝑟 1−𝑟
Geometric Geometric series if the terms of the series 𝑆 = + 𝑑𝑟
1−𝑟 (1 − 𝑟)
series form an arithmetico-Geometric sequence.

Exercise 5.3
01.Find the sum of the first 20-terms of the arithmetic progression having the sum
of first 10 terms as 52 and the sum of the first 15 terms as 77.
Solution :
Given 𝑆 = 52 , 𝑆 = 77, 𝑆 =?
𝑆 = [2𝑎 + (𝑛 − 1)𝑑]
10
𝑆 = [2𝑎 + (10 − 1)𝑑] = 52 𝑆 = [2𝑎 + (15 − 1)𝑑] = 77
2
2𝑎 + 9𝑑 = ------(1) 2𝑎 + 14𝑑 = ------(2)

By using elimination method solving (1) and (2) we get 𝑎 = 𝑎𝑛𝑑 𝑑 =


Now 𝑆 = [2𝑎 + (20 − 1)𝑑]
= 10 2 + 19
= 10 −
= 10 = =
𝑆 =
𝟏𝟑 𝟏𝟑 𝟐𝟑 𝟏𝟑 𝟐𝟑 𝟑𝟑
02. Find the sum up to the 17th term of the series + + + ⋯ … … ..
𝟏 𝟏 𝟑 𝟏 𝟑 𝟓
Solution : We may write the series with nth term as follows,
1 1 +2 1 +2 +3 1 + 2 + 3 + ⋯ … … . . +𝑛
+ + + ⋯……..+
1 1+3 1+3+5 1 + 3 + 5 + ⋯ … … + (2𝑛 − 1)
𝑛(𝑛 + 1) 𝑛 (𝑛 + 1)
1 + 2 + 3 + ⋯ … … . . +𝑛 2 𝑛 (𝑛 + 1)
𝑊ℎ𝑒𝑟𝑒 𝑡 = ⟹𝑛 ⟹ 𝑛 4 ⟹
1 + 3 + 5 + ⋯ … … + (2𝑛 − 1) [1 + (2𝑛 − 1)] [2𝑛] 4𝑛
2 2

PVMHSS |
Binomial Theorem, Sequences and Series
1
4

Now the give series written as


1 1 +2 1 +2 +3 𝑛 + 2𝑛 + 1
+ + + ⋯……..+
1 1+3 1+3+5 4
Now Sn = ∑ 2 𝑛
(𝑛 + 2𝑛 + 1) ⟹ ∑1(𝑛 + 2𝑛 + 1) ⟹ [∑ 𝑛 + 2 ∑ 𝑛 + ∑ 1]
( )( ) ( )
Finally we get 𝑆 = +2 +𝑛
( )( ) ( )
𝑆 = +2 + 17
⟹𝑆 = [1785 + 306 + 17] = 527
03.Compute the sum of first n terms of the following series:
i) 8 + 88 + 888 + 8888 +……………
solution : 8 + 88 + 888 + 8888 +……………
= 8[1 + 11 + 111 + ⋯ … … … … . ]
9
= 𝑋 8[1 + 11 + 111 + ⋯ … … … … . ]
9
8
= [9 + 99 + 999 + ⋯ … … … … . ]
9
8
= [(10 − 1) + (100 − 1) + (1000 − 1) + ⋯ … … … … . ]
9
8
= [(10 + 100 + 1000 + ⋯ … … ) − (1 + 1 + 1 + ⋯ … . 𝑛 𝑡𝑖𝑚𝑒𝑠)]
9
8 10(10 − 1)
= −𝑛
9 9
= (10 − 1) −
ii) 6 + 66 + 666 + 6666 +……………
solution : 6 + 66 + 666 + 6666 +……………
= 6[1 + 11 + 111 + ⋯ … … … … . ]
9
= 𝑋 6[1 + 11 + 111 + ⋯ … … … … . ]
9
6
= [9 + 99 + 999 + ⋯ … … … … . ]
9
6
= [(10 − 1) + (100 − 1) + (1000 − 1) + ⋯ … … … … . ]
9
6
= [(10 + 100 + 1000 + ⋯ … … ) − (1 + 1 + 1 + ⋯ … . 𝑛 𝑡𝑖𝑚𝑒𝑠)]
9
6 10(10 − 1)
= −𝑛
9 9
= (10 − 1) − ⟹ (10 − 1) −
04.Compute the sum of first n terms of 1 + (1 + 4) + (1 + 4 + 42) + (1 + 4 + 42 + 43) +…
Solution : We may write the series with nth term as follows,
1 + (1 + 4) + (1 + 4 + 42) + (1 + 4 + 42 + 43) +…………+(1 + 4 + 42 + 43+……….+4n)
( )
𝑊ℎ𝑒𝑟𝑒 𝑡 =1 + 4 + 42 + 43+……….+4n-1 ⟹ ⟹ ⟹ −

PVMHSS |
Binomial Theorem, Sequences and Series
1
5
Now the give series written as
1 + (1 + 4) + (1 + 4 + 42) + (1 + 4 + 42 + 43) +…………+(1 + 4 + 42 + 43+……….+4n-1)
1+5+21+85+……………+ −
∑ ∑ ⋯….. ( ⋯… )( )
Now 𝑆 = ∑ − ⟹ − ⟹ −
( )
⟹ − ⟹ (4 − 1) −
𝟒 𝟕 𝟏𝟎
5.Find the general term and sum to n terms of the sequence 𝟏, , , … ….
𝟑 𝟗 𝟐𝟕
Solution :
Numerator is A.P with a=1,d=3 Denominator is G.P with a=1,r=3
tn= 1+(n-1)3 = 1+3n-3 tn= 1(3)n-1=3n-1
tn = 3n-2 tn = 3n-1
General term of the given sequence = 𝑡 =
( ( ) )
𝑆 = +𝑑𝑋𝑟𝑋 ( )
Where a=1 , d=3 , r=
1 1
1 − (1 + (𝑛 − 1)3) 1 1−
𝑆 = 3 +3𝑋 𝑋 3
1 3 1
1− 1−
3 3
1 1
1 − (1 + 3𝑛 − 3) 1−
𝑆 = 3 + 3
2 4
3 9
3 − 3𝑛 + 2 3 −1
𝑆 = 3 + 3
2 4
3 9
3 − 3𝑛 + 2 3 −1
𝑆 = +
2(3 ) 2(3 )
6.Find the value of n, if the sum to n terms of the series √𝟑 + √𝟕𝟓 + √𝟐𝟒𝟑 + ⋯ . . 𝒊𝒔 𝟒𝟑𝟓√𝟑
Solution : √3 + √75 + √243 + ⋯ … … … . 𝑛 𝑡𝑒𝑟𝑚𝑠 = 435√3
= √3 + √25√3 + √81√3 + ⋯ . . … . 𝑛 𝑡𝑒𝑟𝑚𝑠 = 435√3
= √3 1 + √25 + √81 + ⋯ . . … . 𝑛 𝑡𝑒𝑟𝑚𝑠 = 435√3
1 + √25 + √81 + ⋯ . . … . 𝑛 𝑡𝑒𝑟𝑚𝑠 = 435
1 + 5 + 9 + ⋯. … … . . . … . 𝑛 𝑡𝑒𝑟𝑚𝑠 = 435 Which is in A.P with a= 1 and d=4
[2(1) + (𝑛 − 1)4] = 870
𝑛(2 + 4𝑛 − 4) = 870
𝑛(4𝑛 − 2) = 870
4𝑛 − 2𝑛 − 870 = 0
(𝑛 − 15)(2𝑛 + 29) = 0
𝑛 = 15 𝑜𝑟
n is a positive integer so that suitable value for n = 15.
𝑛 = 15

PVMHSS |
Binomial Theorem, Sequences and Series
1
6
7.Show that the sum of (m + n)th and (m − n)th term of an AP. is equal to twice
the mth term.
Solution : T = 𝑎 + (𝑚 − 1)𝑑
T = 𝑎 + (𝑚 + 𝑛 − 1)𝑑
T = 𝑎 + (𝑚 − 𝑛 − 1)𝑑
To Prove : T +T = 2T
L.H.S : T +T = 𝑎 + (𝑚 + 𝑛 − 1)𝑑 + 𝑎 + (𝑚 − 𝑛 − 1)𝑑
= 𝑎 + 𝑚𝑑 + 𝑛𝑑 − 𝑑 + 𝑎 + 𝑚𝑑 − 𝑛𝑑 − 𝑑
= 2𝑎 + 2𝑚𝑑 − 2𝑑
= 2(𝑎 + 𝑚𝑑 − 𝑑)
= 2(𝑎 + (𝑚 − 1)𝑑)
= 2T R.H. S Hence Proved.
8. A man repays an amount of Rs.3250 by paying Rs.20 in the first month and then
increases the payment by Rs.15 per month. How long will it take him to clear the
amount?
Solution : Where a= 20 , d =15 , and Sn=3250
𝑆 = [2(20) + (𝑛 − 1)15] = 3250
𝑆 = 𝑛[40 + 15𝑛 − 15] = 6500
𝑛[25 + 15𝑛] = 6500
15𝑛 + 25𝑛 − 6500 = 0
3𝑛 + 5𝑛 − 1300 = 0
(𝑛 − 20)(3𝑛 + 65) = 0
𝑛 = 20 𝑜𝑟
n is a positive integer so that suitable value for n = 20.
𝑛 = 20
9.In a race, 20 balls are placed in a line at intervals of 4 meters, with the first ball 24
meters away from the starting point. A contestant is required to bring the balls back
to the starting place one at a time. How far would the contestant run to bring back
all balls?
Solution :
1 2 3 4 5 6 ………………. 20
24 m 4m 4m 4m 4m 4m
To bring the first ball he will walk 48m. (24+24 i.e., including return )
To bring the second ball he will walk (24+4)x2 = 56 m
By continuing similarly,
Required series is 48+56+64+……….20 terms
20
𝑆 = [2(48) + (20 − 1)8]
2
= 10[96 + (19)8]
= 10[96 + 152]
= 10 x 248
= 2480𝑚

PVMHSS |
Binomial Theorem, Sequences and Series
1
7
10.The number of bacteria in a certain culture doubles every hour. If there were 30
bacteria present in the culture originally, how many bacteria will be present at the
end of 2nd hour, 4th hour and nth hour?
Solution : From Given data we observe that a=30 , r= 2
Required sequence is 30 , 60 ,120 , 240, ……
No of bacteria present at the end of 2nd hour = T3 = 30 (2)3-1 = 30 x 4 =120
No of bacteria present at the end of 4th hour = T5 = 30 (2)5-1 = 30 x 16 =480
No of bacteria present at the end of nth hour = Tn+1 = 30 (2)n+1-1 =30 (2)n
11. What will Rs.500 amounts to in 10 years after its deposit in a bank which pays
annual interest rate of 10% compounded annually?
Solution : From Given data we observe that a= 500 , r = 10% = = , n=10

Amount = P 1 + = 500 1 + = 500(1.1) = 1296.87


Note :

Take log on both sides

12. In a certain town, a viral disease caused severe health hazards upon its people
disturbing their normal life. It was found that on each day, the virus which caused
the disease spread in Geometric Progression. The amount of infectious virus
particle gets doubled each day, being 5 particles on the first day. Find the day when
the infectious virus particles just grow over 1,50,000 units?
Solution : From Given data we observe that a= 5 , r = 2 , Tn=1,50,000 then n=?
𝑇 = 𝑎𝑟 > 150000
5𝑥2 > 150000
2 > 30000
2
> 30000
2
2 > 60000
log 2 > log 60000
𝑛 log 2 > log 60000
𝑛>
.
𝑛>
.
𝑛 > 15.8740
So that the suitable value of n, to satisfy the above in equation is 𝑛 = 15 .
On the 15th day , the infection virus particle just grow over 1500000 .

PVMHSS |
Binomial Theorem, Sequences and Series
1
8
Points To Remember
Binomial Series for rational exponent: [for all real numbers x satisfying |x| < 1]
𝑛(𝑛 − 1) 𝑛(𝑛 − 1)(𝑛 − 2)
(1 + 𝑥) 1 + 𝑛𝑥 + 𝑥 + 𝑥 + ⋯ … … … … … … ..
2! 3!
𝑛(𝑛 − 1) 𝑛(𝑛 − 1)(𝑛 − 2)
(1 − 𝑥) 1 − 𝑛𝑥 + 𝑥 − 𝑥 + ⋯ … … … … … … ..
2! 3!
𝑛(𝑛 + 1) 𝑛(𝑛 + 1)(𝑛 + 2)
(1 + 𝑥) 1 − 𝑛𝑥 + 𝑥 − 𝑥 + ⋯ … … … … … … ..
2! 3!
𝑛(𝑛 + 1) 𝑛(𝑛 + 1)(𝑛 + 2)
(1 − 𝑥) 1 + 𝑛𝑥 + 𝑥 + 𝑥 + ⋯ … … … … … … ..
2! 3!
Exercise 5.4
1. Expand the following in ascending powers of x and find the condition on x for
which the binomial expansion is valid.
𝟏 𝟏 𝟏 𝟏
(i) ⟹ 𝒙 ⟹ (𝟏 + ) 𝟏
𝟓 𝒙 𝟓(𝟏 ) 𝟓 𝒙
𝟓

⟹ 1+ = 1− + − + ⋯….
This expansion is Valid only if < 1 ⟹ |𝑥| < 5.

(ii) ( )
= = ⟺ 1+
( )

⟹ 1+
2 4𝑥 2 𝑋 3 4𝑥 2 𝑋 3 𝑋 4 4𝑥
= 1 − (2) + − + ⋯……..
9 3 2 3 6 3
2 4𝑥 4𝑥 4𝑥
= 1 − (2) + (3) − (4) + ⋯……..
9 3 3 3
4𝑥
This expansion is Valid only if < 1 ⟹ |𝑥 | < .
3

(iii) (5 + 𝑥 ) = 5 1 +
2 2 2 2 2
2 𝑥 −1 𝑥 −1 −2 𝑥
=5 1+ + 3 3 + 3 3 3 + ⋯………..
3 5 2 5 6 5

=5 1+ 𝑥 − 𝑥 + 𝑥 − ⋯………..
𝑥2
This expansion is Valid only if < 1 ⟹ 𝑥2 < 5.
5

(iv) (𝑥 + 2) = (2) 1+

= (2) 1− + − + ⋯….

= (2) 1− 𝑥+ 𝑥 − 𝑥 + ⋯..
𝑥
This expansion is Valid only if < 1 ⟹ |𝑥| < 2.
2

PVMHSS |
Binomial Theorem, Sequences and Series
1
9
𝟑
2.Find √𝟏𝟎𝟎𝟏 approximately (Two decimal Places)
Solution : √1001 =(1 + 1000)

= (1000) 1 +

= 10 1 + + + ⋯..

= 10 1 + + + ⋯..

= 10 1 + − + ⋯..
= 10[1 + 0.000333111] = 10[1.000333111] = 10.00333111
𝟑 𝟑 𝟏
3.Prove that √𝒙𝟑 + 𝟔 − √𝒙𝟑 + 𝟑 is approximately equal to 𝟐 when x is sufficiently
𝒙
large.
Solution : √𝑥 + 6 − √𝑥 + 3 x is large = (𝑥 + 6) − (𝑥 + 3)

=(𝑥 ) 1+ − (𝑥 ) 1+

= 𝑥 1+ − 𝑥 1+

= 𝑥 1+ + + ⋯… − 𝑥 1 + + + ⋯…

= 𝑥 1+ − + ⋯… − 𝑥 1 + − + ⋯…
=𝑥 1+ − + ⋯… 1+ − + ⋯…
= 𝑥{ + ⋯.}
= . (approx.) Hence Proved .
𝟏 𝒙 𝒙𝟐
4. Prove that is approximately equal to 𝟏 − 𝒙 + When x is very Small.
𝟏 𝒙 𝟐
𝟏 𝟏
𝟏 𝒙
Solution : = (𝟏 − 𝒙)𝟐 (𝟏 + 𝒙) 𝟐
𝟏 𝒙
𝟏 𝟏 𝟏 𝟏
𝟏 𝟏 𝟏 𝟏
= 𝟏− 𝒙+ 𝟐 𝟐 𝟐
𝒙 − ⋯.. 𝟏− 𝒙+ 𝟐 𝟐
𝒙𝟐 − ⋯ . .
𝟐 𝟐 𝟐 𝟐
𝒙 𝒙𝟐 𝒙 𝟑𝒙𝟐
= 𝟏− − − ⋯.. 𝟏− + − ⋯..
𝟐 𝟖 𝟐 𝟖
𝒙 𝒙 𝒙𝟐 𝒙𝟐 𝟑𝒙𝟐
= 𝟏− − − + + + ⋯..
𝟐 𝟐 𝟖 𝟒 𝟖
𝟒𝒙𝟐
=𝟏−𝒙+ + ⋯..= 1 − 𝑥 + (approx.)
𝟖
Hence Proved.
RECALL : Exponential Series : ∑ Is called exponential series.
!

𝑒 =1+ + + +⋯ 𝑒 =1+ + + +⋯ 𝑒 =1− + − +⋯


! ! ! ! ! ! ! ! !

PVMHSS |
Binomial Theorem, Sequences and Series
2
0
RECALL : Logarithmic Series : ∑ (−1) Is called Logarithmic series.
!
𝑥 𝑥 𝑥
log(1 + 𝑥) = 𝑥 − + − +⋯
2 2 2
𝑥 𝑥 𝑥
log(1 − 𝑥) = −𝑥 − − − − ⋯
2 2 2
1+𝑥 𝑥 𝑥
𝑙𝑜𝑔 = 2 𝑥 + + +. .
1−𝑥 3 5

5.Find the six terms of the exponential series


( ) ( ) ( ) ( ) ( )
(i) 𝒆𝟓𝒙 = 1+ + + + + + ⋯ ….
! ! ! ! !
= 1+ 5𝑥 + + + + + ⋯ ….
( ) ( ) ( ) ( ) ( )
(ii) 𝒆 𝟐𝒙
= 1- + − + − + ⋯ ….
! ! ! ! !
= 1- 2𝑥 + 2𝑥 − + − + ⋯ ….
𝟏 𝒙 𝒙 𝒙 𝒙 𝒙
(ii) 𝒆𝟐𝒙 = 1- 𝟐
+ 𝟐
− 𝟐
+ 𝟐
− 𝟐
+ ⋯ ….
! ! ! ! !
=1+ + + + + + ⋯.

6.Write the first four terms of the logarithmic series


(i) 𝒍𝒐𝒈(𝟏 + 𝟒𝒙)
( ) ( ) ( )
𝑙𝑜𝑔(1 + 4𝑥) = 4𝑥 − + − + ⋯ … ….
= 4𝑥 − + − + ⋯ … ….
= 4𝑥 − 8𝑥 + − 64𝑥 + ⋯ … …. Valid if |4𝑥| < 1 ⟹ |𝑥| <
(i) 𝒍𝒐𝒈(𝟏 − 𝟐𝒙)
( ) ( ) ( )
𝑙𝑜𝑔(1 − 2𝑥) = −2𝑥 − − − − ⋯ … ….
= −2𝑥 − − − − ⋯ … ….
= −2𝑥 − 2𝑥 − − 4𝑥 + ⋯ … …. Valid if |2𝑥| < 1 ⟹ |𝑥| <
𝟏 𝟑𝒙
(iii) 𝒍𝒐𝒈
𝟏 𝟑𝒙
( ) ( ) ( )
𝑙𝑜𝑔 = 2 (3𝑥) + + + +⋯
= 2 3𝑥 + + + +⋯ Valid if |3𝑥| < 1 ⟹ |𝑥| <
𝟏 𝟐𝒙
(ii) 𝒍𝒐𝒈 =
𝟏 𝟐𝒙
1 − 2𝑥 (2𝑥) (2𝑥) (2𝑥)
𝑙𝑜𝑔 = −2 (2𝑥) + + + +⋯
1 + 2𝑥 3 5 7
= 2 2𝑥 + + + +⋯ Valid if |2𝑥| < 1 ⟹ |𝑥| <

PVMHSS |
Binomial Theorem, Sequences and Series
2
1
𝒙𝟐 𝒙𝟑 𝒙𝟒 𝒚𝟐 𝒚𝟑 𝒚𝟒
7.If 𝒚 = 𝒙 + + + + ⋯ … , then show that 𝒙 = 𝒚 − + − + ⋯ ..
𝟐 𝟑 𝟒 𝟐! 𝟑! 𝟒!

Solution : Given 𝑦 = 𝑥 + + + + ⋯ … = −𝑙𝑜𝑔(1 − 𝑥)

𝑖. 𝑒. , 𝑦 = 𝑙𝑜𝑔(1 − 𝑥)

⟹ 𝑒 = (1 − 𝑥)

⟹𝑒 = (1 − 𝑥)

Now 𝑥 = 1−𝑒

𝑥 = 1 − (1 − 𝑦 + − + − ⋯)
! ! !

𝒚𝟐 𝒚𝟑 𝒚𝟒
𝒙= 𝒚− + − + ⋯.
𝟐! 𝟑! 𝟒!

8. If 𝒑 − 𝒒 is small compared to either p or q , then show that


𝒏 𝒑 (𝒏 𝟏)𝒑 (𝒏 𝟏)𝒒 𝟖 𝟏𝟓
= (𝒏 . Hence find
𝒒 𝟏)𝒑 (𝒏 𝟏)𝒒 𝟏𝟔

Solution : =

=
( ) ( )
= ( ) ( )
( )
( )
=
( )
( )
( )
( )

( )

=
( )
( )
( )

( ) ( )
≅ ≅ ≅( ) ( )
. Hence Proved.

( ) ( )
=( ) ( )
= = = 0.99196

PVMHSS |
Binomial Theorem, Sequences and Series
2
2
𝟑 𝟒𝒙 𝒙𝟐
9.Find the coefficient of x4 in the expansion of .
𝒆𝟐𝒙
𝟐𝒙
Solution : = (3 − 4𝑥 + 𝑥 ) 𝒆
2𝑥 4𝑥2 8𝑥3 16𝑥4
= (3 − 4𝑥 + 𝑥 ) 1 − + − + −⋯
1! 2! 3! 4!
. . .
𝑡ℎ𝑒 𝑐𝑜𝑒𝑓𝑓𝑖𝑐𝑖𝑒𝑛𝑡 𝑜𝑓 𝑥 = + + ⟹2+ + 2= .
! ! !
𝟏 𝟏 𝟏
10.Find the value of ∑𝒏 𝟏 𝟐𝒏 𝟏 𝟗𝒏 𝟏 +
𝟗𝟐𝒏 𝟏
Solution :

**************************

**************************

PVMHSS |
(ii) 𝐿𝑒𝑡 𝑡ℎ𝑒 𝑔𝑖𝑣𝑒𝑛 𝑝𝑜𝑖𝑛𝑡 𝑃(ℎ, 𝑘)𝑚𝑜𝑣𝑒𝑠 3 𝑢𝑛𝑖𝑡𝑠 equidistant

PV
from y axis.
Where h=3 P(h,k)
MATRIC HR. SEC. SCHOOL 3 Units
Equation of a line with parallel to y axis
k
MATHEMATICS - XI STD is x = h ⟹ 𝑥 = 3 ⟹ 𝑥 − 3 = 0
h
Points To Remember 03.If θ is a parameter, find the equation of the locus of a
Definition 6.1 moving point, whose coordinates are
A point is an exact position or location on a plane surface. x = a cos3θ, y = a sin3θ.
Definition 6.2 𝑥 = 𝑎 cos 𝜃 𝑦 = 𝑎 sin 𝜃
The path traced out by a moving point under certain 𝑥 𝑦
= cos 𝜃 = sin 𝜃
conditions is called the locus of that point. Alternatively, 𝑎 𝑎
when a point moves in accordance with a geometrical 𝑥 / 𝑦 /
= cos 𝜃 = sin 𝜃
law, its path is called locus. The plural of locus is loci. 𝑎 𝑎
𝑥 / 𝑦 /
A moving point P = cos 𝜃 = sin 𝜃
Name 𝑎 𝑎
under the given Graph
of the path cos 𝛼 + sin 𝛼 = 1
condition 𝑥 / 𝑦 /
A point P moves + =1
Perpendicular 𝑎 𝑎
such that it is 4.Find the value of k and b, if the points P(−3, 1) and
bisector of the
equidistant from Q(2,b) lie on the locus of x2 − 5x + ky = 0.
line segment
two fixed points A
AB Let P(-3,1) is lies on the line x2 − 5x + ky = 0.
and B
(-3)2-5(-3)+k(1)=0
A point P moves
Angle 9 +15+k=0 ⟹ 𝑘 = −24
such that it is
bisector Let Q(2,b) is lies on the line x2 − 5x -24y = 0.
equidistant from
of the (2)2-5(2)-24(b)=0
two fixed lines ox
angle∠xoy 4 -10-24b=0 ⟹ 𝑏 = −
and oy
A point P moves 5.A straight rod of length 8 units slides with its ends A
Circle and B always on the x and y axes respectively.
equidistant from a
Find the locus of the mid point of the line segment AB.
fixed point O
Let A(a,0) and B(0,b) lies on the ends of the rod.
Where A is on the x axis and b is on the y axis.
Exercise 6.1 Let P(h,k) is the midpoint of AB.

01. Find the locus of P, if for all values of 𝜶, the co- Now P(h,k) is , B(0,b)
ordinates of a moving point P is i.e., P(h,k) is P , . 𝑎 𝑏
(i) (9cos 𝜶 , 9 sin 𝜶) ,
Where ℎ = ⟹ 𝑎 = 2ℎ P(ℎ, 𝑘) 2 2
Let 𝑥 = 9 cos 𝛼 ⟹ = cos 𝛼
𝑘 = ⟹ 𝑎 = 2𝑘
𝑦 = 9 sin 𝛼 ⟹ = sin 𝛼
From triangle OAB, we have,
We know that cos 𝛼 + sin 𝛼 = 1 𝐴𝐵 = 𝑂𝐴 + 𝑂𝐵
8 =𝑎 +𝑏 A(a,0)
+ =1
8 = (2ℎ) + (2𝑘)
+ = 1 ⟹ 𝑥 + 𝑦 = 81 4ℎ + 4𝑘 = 64
ℎ + 𝑘 = 16
(ii) (9cos 𝜶 , 6 sin 𝜶)
Locus of P(h,k) is 𝑥 + 𝑦 = 16 .
Let 𝑥 = 9 cos 𝛼 ⟹ = cos 𝛼 6.Find the equation of the locus of a point such that the
𝑦 = 6 sin 𝛼 ⟹ = sin 𝛼 sum of the squares of the distance from the points (3, 5),
We know that cos 𝛼 + sin 𝛼 = 1 (1,−1) is equal to 20.
P(h,k)
+ =1
+ =1
A(3,5) B(1,-1)
2.Find the locus of a point P that moves at a constant From given data
distant of (i) two units from the x-axis 𝑃𝐴 + 𝑃𝐵 = 20
(ii) three units from the y-axis.
(i) 𝐿𝑒𝑡 𝑡ℎ𝑒 𝑔𝑖𝑣𝑒𝑛 𝑝𝑜𝑖𝑛𝑡 𝑃(ℎ, 𝑘)𝑚𝑜𝑣𝑒𝑠 2 𝑢𝑛𝑖𝑡𝑠 equidistant (ℎ − 3) + (𝑘 − 5) + (ℎ − 1) + (𝑘 + 1) = 20
from x axis. ℎ − 6ℎ + 9 + 𝑘 − 10𝑘 + 25 + ℎ − 2ℎ + 1 + 𝑘 + 2ℎ + 1 = 20
P(h,k)
Where k=2 2ℎ + 2𝑘 − 8ℎ − 8𝑘 + 36 = 0
Equation of a line with parallel to x axis k 2 Units ℎ + 𝑘 − 4ℎ − 4𝑘 + 18 = 0
is y = k ⟹ 𝑦 = 2 ⟹ 𝑦 − 2 = 0 Locus of P(h,k) is 𝑥 + 𝑦 − 4𝑥 − 4𝑦 + 18 = 0
h
7.Find the equation of the locus of the point P such that =ℎ ; =𝑘
the line segment AB, joining the points A(1,−6) and
2 + 𝑎 = 2ℎ -7+b= 2𝑘
B(4,−2), subtends a right angle at P. B(4,-2)
𝑎 = 2ℎ-2 b= 2𝑘 + 7
Substitute on (1)
Given PAB is a right angled triangle.
Slope of PA x Slope of PB = -1 2(2ℎ − 2) + 9(2𝑘 + 7) =18
2(4ℎ − 8ℎ + 4) + 9(4𝑘 + 24𝑘 + 49) = 18
𝑥 = −1 8ℎ − 16ℎ + 8 + 36𝑘 + 252𝑘 + 441 = 18
𝑥 = −1 8ℎ − 16ℎ + 8 + 36𝑘 + 252𝑘 + 441 = 18
8ℎ + 36𝑘 − 16ℎ + 252𝑘 + 431 = 0
= −1 Locus of A(h,k) is
P(h,k) A(1,-6) 8𝑥 + 36𝑦 − 16𝑥 + 252𝑦 + 431 = 0
= −1 11.If R is any point on the x-axis and Q is any point on
ℎ + 𝑘 − 5ℎ + 8𝑘 + 16 = 0 the y-axis and P is a variable point on RQ with RP = b,
Locus of P(h,k) is 𝑥 + 𝑦 − 5𝑥 + 8𝑦 + 16 = 0 PQ = a. then find the equation of locus of P.
8.If O is origin and R is a variable point on y2 = 4x, then Let P (h,k) br the point on RQ such
find the equation of the locus of the mid-point of the That Angle ORQ is 𝜃 Q a
line segment OR. R(2h,2k) In Triangle PLR sin 𝜃 = 𝜃 P(h,k)
M
O is the Origin (0,0) h
In Triangle PMQ cos 𝜃 =
Let P(h,k) be the moving point of OA. k k b
Since O is (0,0) and P (h,k). P(h,k) We know that 𝜃
Let A must be (2h,2k). sin 𝜃 + sin 𝜃 = 1 O h L R
A lies on the parabola 𝑦 = 4𝑥
O 𝑘 ℎ
+ =1
i.e., (2𝑘) = 4(2ℎ) 𝑏 𝑎
4𝑘 = 8ℎ ℎ 𝑘
+ =1
𝑘 = 2ℎ 𝑎 𝑏
Locus of P(h,k) is 𝑦 = 2𝑥 . Locus of P(h,k) is + = 1.
9.The coordinates of a moving point p are 12.If the points P(6, 2) and Q(−2, 1) and R are the vertices
𝒂 𝒃
(𝒄𝒐𝒔𝒆𝒄𝜽 + 𝒔𝒊𝒏𝜽), (𝒄𝒐𝒔𝒆𝒄𝜽 − 𝒔𝒊𝒏𝜽) where θ is a of a ΔPQR and R is the point on the locus y = x2 − 3x + 4,
𝟐 𝟐
variable parameter. Show that the equation of the locus then find the equation of the locus of centroid of ΔPQR
Let P(6,2) , Q(-2,1) and R(a,b) be the given points,
P is b2x2 − a2y2 = a2b2 .
Given R is on y = x2 − 3x + 4
𝑎 𝑏 i.e., b = a2-3a+4 --------(1)
(ℎ, 𝑘) = (𝑐𝑜𝑠𝑒𝑐𝜃 + 𝑠𝑖𝑛𝜃), (𝑐𝑜𝑠𝑒𝑐𝜃 − 𝑠𝑖𝑛𝜃)
2 2 The centroid of the triangle PQR is P(h,k)
Where ℎ = (𝑐𝑜𝑠𝑒𝑐𝜃 + 𝑠𝑖𝑛𝜃) i.e., , = (ℎ, 𝑘)
= (𝑐𝑜𝑠𝑒𝑐𝜃 + 𝑠𝑖𝑛𝜃) ---------(1) 6−2+𝑎
= ℎ ⟹ 𝑎 = 3ℎ − 4
𝑏 3
𝑘 = (𝑐𝑜𝑠𝑒𝑐𝜃 − 𝑠𝑖𝑛𝜃) 2+1+𝑏
2 = 𝑘 ⟹ 𝑏 = 3𝑘 − 3
3
= (𝑐𝑜𝑠𝑒𝑐𝜃 − 𝑠𝑖𝑛𝜃) ---------(1) Substitute in (1)
Squaring and subtracting (1) & (2) 3𝑘 − 3 = (3ℎ − 4 ) − 3(3ℎ − 4 ) + 4
2ℎ 2𝑘 3𝑘 − 3 = 9ℎ − 24ℎ + 16 − 9ℎ + 12 + 4
- = (𝑐𝑜𝑠𝑒𝑐𝜃 + 𝑠𝑖𝑛𝜃) − (𝑐𝑜𝑠𝑒𝑐𝜃 − 𝑠𝑖𝑛𝜃) 9ℎ − 33ℎ − 3𝑘 + 35 = 0
𝑎 𝑏
4ℎ 4𝑘 Locus of P(h,k) is 9𝑥 − 33𝑥 − 3𝑦 + 35 = 0
− = 4𝑐𝑜𝑠𝑒𝑐𝜃 𝑠𝑖𝑛𝜃 13.If Q is a point on the locus of x2 + y2 + 4x − 3y + 7 = 0,
𝑎 𝑏
4ℎ 4𝑘 then find the equation of locus of P which divides
− =4 segment OQ externally in the ratio 3:4,where O is origin.
𝑎 𝑏
ℎ 𝑘 Let Q be (a,b) lies on x2 + y2 + 4x − 3y + 7 = 0
− =1 a2 + b2 + 4a – 3b + 7 = 0
𝑎 𝑏
O is origin (0,0) ,Which divides OQ externally in
Locus of P(h,k) is − =1 the ratio 3:4 is
b2x2 − a2y2 = a2b2 . ( )
,
( )
= (ℎ, 𝑘)
10.If P(2,−7) is a given point and Q is a point on -3a=h ⟹ 𝑎 =
2x2 + 9y2 = 18, then find the equations of the
locus of the mid-point of PQ. 3b=-k ⟹ 𝑏 =
Let (2,7) be the given points , Let Q(a,b) be a point on −ℎ −𝑘 −ℎ −𝑘
+ +4 −3 +7 = 0
2x2 + 9y2 = 18 3 3 3 3
ℎ + 𝑘 -12h+9k+63=0
2a2 + 9b2 = 18
Q(a,b) Locus of (h,k) is x + y -12x+9y+63=0
Let A(h,k) be the mid point of PQ.
To find the locus of A A(h,k)
P(2,-7)
14.Find the points on the locus of points that are 3 units
from x-axis and 5 units from the point (5, 1).
Let P be (h,k) and Q(5,1)
Distance between P & x axis k=3
P(h,k)⟹(h,3)
𝑃𝑄 = 5
𝑃𝑄 = 25
(ℎ − 5) + (𝑘 − 1) = 25
(ℎ − 5) + (3 − 1) = 25 Q(5,1)
ℎ − 10ℎ + 25 + 4 = 25
ℎ − 10ℎ + 4 = 0
10 ± √100 − 16 10 ± √84
ℎ= = = 5 ± √21
2 2
The points are (5 + √21,3) and (5 − √21,3) .
15.The sum of the distance of a moving point from the
points (4, 0) and (−4, 0) is always 10 units. Find the
equation of the locus of the moving point.

P(h,k)

B(-4,0) A(4,0)
Let A and B be the fixed points (4, 0) and (− 4, 0)
respectively and
P(h,k) be the moving point.
It is given that AP + BP = 10.
(ℎ − 4) + (𝑘 − 0) + (ℎ + 4) + (𝑘 − 0) = 10
(ℎ − 4) + (𝑘 − 0) = 10 − (ℎ + 4) + (𝑘 − 0)
Squaring on both sides
(ℎ − 4) + (𝑘 − 0) = 10 − (ℎ + 4) + (𝑘 − 0)
By simplifying above we get
9𝑘 + 25ℎ = 225
+ =1
Locus of P (h,k) is + =1
Points To Remember
Straight Lines
4.Determine the matrices A and B If they satisfy

PV
𝟔 −𝟔 𝟎 𝟑 𝟐 𝟖
𝟐𝑨 − 𝑩 + = 𝟎 𝒂𝒏𝒅 𝑨 − 𝟐𝑩 =
−𝟒 𝟐 𝟏 −𝟐 𝟏 −𝟕
MATRIC HR. SEC. SCHOOL Let 2𝐴 − 𝐵 = −6 6 0
--------------(1)
4 −2 −1
3 2 8
MATHEMATICS - XI STD 𝐴 − 2𝐵 =
−2 1 −7
--------------- (2)
2x(1) – (2)
Exercise 7.1 ⟹ [4A-2B] – [A-2B]= −12
8
12 0
−4 −2

3 2 8
−2 1 −7

1.Construct m x n matrix A= 𝒂𝒊𝒋 , Where 𝒂𝒊𝒋 is given by 4A-2B –A+2B = −12 12 0 + −3 −2 −8


8 −4 −2 2 −1 7
(i) 𝒂𝒊𝒋 =
(𝒊 𝟐𝒋)𝟐
𝒘𝒊𝒕𝒉 𝒎 = 𝟐 , 𝒏 = 𝟑 3A= −15 10 −8
𝟐
10 −5 5
𝑎 𝑎 𝑎 −15 10 −8
A= 𝑎 𝑎 𝑎
A=
10 −5 5
𝑎 =
( )
= ;𝑎 =
( )
= ;𝑎 =
( )
= (1) – 2 x (2)
( ) ( ) ( ) ⟹ [2A-B] – [2A-4B]= −6 6 0

6 4 16
𝑎 = =0 ;𝑎 = =2 ;𝑎 = =8 4 −2 −1 −4 2 −14
2A-B -2A+4B = −6 6 0
+
−6 −4 −16
1 9 25 4 −2 −1 4 −2 14
Now A= ⟹
0 8 20 4 16 3B = −12 2 −16
|𝟑𝒊 𝟒𝒋| 8 −4 13
(i) 𝒂𝒊𝒋 = 𝒘𝒊𝒕𝒉 𝒎 = 𝟑 , 𝒏 = 𝟒 −12 2 −16
𝟒 𝐵=
𝒂𝟏𝟏 𝒂𝟏𝟐 𝒂𝟏𝟑 𝒂𝟏𝟒 8 −4 13
A= 𝒂𝟐𝟏 𝒂𝟐𝟐 𝒂𝟐𝟑 𝒂𝟐𝟒 5.If A=
𝟏 𝒂
then compute A4.
𝒂𝟑𝟏 𝒂𝟑𝟐 𝒂𝟑𝟑 𝒂𝟑𝟒 𝟎 𝟏
|𝟑 𝟒| |𝟑 𝟖| 1 𝑎 1 𝑎 1+0 𝑎+𝑎 1 2𝑎
𝑎 = = ; 𝑎 = = ; A2 = A x A = = =
𝟒 𝟒 0 1 0 1 0+0 0+1 0 1
|𝟑 𝟏𝟐| |𝟑 𝟏𝟔| 1 2𝑎 1 2𝑎 1 + 0 2𝑎 + 2𝑎
𝑎 = = ; 𝑎 = = ; 1 4𝑎
𝟒 𝟒 A4 = A2 x A2 = = =
0 1 0 1 0+0 0+1 0 1
|𝟔 𝟒| |𝟔 𝟖|
𝑎 = = ; 𝑎 = = ; 1 4𝑎
𝟒 𝟒 𝐴 =
|𝟔 𝟏𝟐| |𝟔 𝟏𝟔| 0 1
𝑎 = = ; 𝑎 = = ;
𝟒 𝟒 𝒄𝒐𝒔 𝜶 −𝒔𝒊𝒏 𝜶
|𝟗 𝟒| |𝟗 𝟖| 6.Consider the matrix 𝑨𝜶 =
𝑎 = = ; 𝑎 = = ; 𝒔𝒊𝒏 𝜶 𝒄𝒐𝒔 𝜶
𝟒 𝟒
|𝟗 𝟏𝟐| |𝟗 𝟏𝟔| (i) Show that 𝑨𝜶 𝑨𝜷 = 𝑨(𝜶 𝜷)
𝑎 = = ; 𝑎 = = ;
𝟒 𝟒 (ii)Find all the possible values of 𝜶 satisfying the
⎡ ⎤ condition 𝑨𝜶 + 𝑨𝜶 𝑻 = 𝑰
1 5 9 13 𝑐𝑜𝑠 𝛼 −𝑠𝑖𝑛 𝛼
⎢ ⎥ (i)Given 𝐴 =
A=⎢ ⎥ ⟹ 2 2 6 10 𝑠𝑖𝑛 𝛼 𝑐𝑜𝑠 𝛼
⎢ ⎥ 5 1 3 7 𝑐𝑜𝑠 𝛽 −𝑠𝑖𝑛 𝛽
⎣ ⎦ Then 𝐴 =
𝑠𝑖𝑛 𝛽 𝑐𝑜𝑠 𝛽
2.Find the values of p,q,r and s if
𝑐𝑜𝑠 𝛼 −𝑠𝑖𝑛 𝛼 𝑐𝑜𝑠 𝛽 −𝑠𝑖𝑛 𝛽
𝟏 𝟎 −𝟒 𝐴 𝐴 =
𝒑𝟐 − 𝟏 𝟎 −𝟑𝟏 − 𝒒𝟑 𝑠𝑖𝑛 𝛼 𝑐𝑜𝑠 𝛼 𝑠𝑖𝑛 𝛽 𝑐𝑜𝑠 𝛽
𝟑
𝟕 𝒓+𝟏 𝟗 = 𝟕 𝟗 𝑐𝑜𝑠 𝛼 𝑐𝑜𝑠 𝛽 − 𝑠𝑖𝑛 𝛼 𝑠𝑖𝑛 𝛽 −𝑐𝑜𝑠 𝛼 𝑠𝑖𝑛 𝛽 − 𝑠𝑖𝑛 𝛼 𝑐𝑜𝑠𝛽
𝟐 =
−𝟐 𝟖 𝒔−𝟏 𝑠𝑖𝑛 𝛼 𝑐𝑜𝑠 𝛽 + 𝑐𝑜𝑠 𝛼 𝑠𝑖𝑛 𝛽 −𝑠𝑖𝑛 𝛼 𝑠𝑖𝑛 𝛽 + 𝑐𝑜𝑠 𝛼 𝑐𝑜𝑠 𝛽
−𝟐 𝟖 −𝝅 𝑐𝑜𝑠 𝛼 𝑐𝑜𝑠 𝛽 − 𝑠𝑖𝑛 𝛼 𝑠𝑖𝑛 𝛽 −(𝑐𝑜𝑠 𝛼 𝑠𝑖𝑛 𝛽 + 𝑠𝑖𝑛 𝛼 𝑐𝑜𝑠𝛽)
1 0 −4 =
𝑝 −1 0 −31 − 𝑞 𝑠𝑖𝑛 𝛼 𝑐𝑜𝑠 𝛽 + 𝑐𝑜𝑠 𝛼 𝑠𝑖𝑛 𝛽 𝑐𝑜𝑠 𝛼 𝑐𝑜𝑠 𝛽 − 𝑠𝑖𝑛 𝛼 𝑠𝑖𝑛 𝛽
Given 7 𝑟+1 9 = 7 9 𝑐𝑜𝑠 (𝛼 + 𝛽) −𝑠𝑖𝑛 (𝛼 + 𝛽)
= = 𝑨(𝜶 𝜷) Hence Proved
−2 8 𝑠−1 −2 8 −𝜋 𝑠𝑖𝑛 (𝛼 + 𝛽) 𝑐𝑜𝑠 (𝛼 + 𝛽)
By the property of Equality of Matrices, (ii) Given 𝐴 + 𝐴 =𝐼
𝑝 −1=1 −31 − 𝑞 = −4 𝑟+1= 𝑠 − 1 = −𝜋 𝑐𝑜𝑠 𝛼 −𝑠𝑖𝑛 𝛼 𝑐𝑜𝑠 𝛼 𝑠𝑖𝑛 𝛼 1 0
+ =
𝑠𝑖𝑛 𝛼 𝑐𝑜𝑠 𝛼 −𝑠𝑖𝑛 𝛼 𝑐𝑜𝑠 𝛼 0 1
𝑝 = 1+1 −𝑞 = −4 + 31 𝑟 = −1 𝑠 = 1−𝜋 2 𝑐𝑜𝑠 𝛼 0 1 0
=
0 2 𝑐𝑜𝑠 𝛼 0 1
𝑝 =2 𝑞 = −27 𝑟= Where 2 𝑐𝑜𝑠 𝛼 = 1 ⟹ 𝑐𝑜𝑠 𝛼 =
𝑝 = ±√2 𝑞 = −3 Principal Value is
𝟐𝒙 + 𝒚 𝟒𝒙 𝟕 𝟕𝒚 − 𝟏𝟑
3.Determine the value of x+y if =
𝒚 𝒙+𝟔
𝟓𝒙 − 𝟕 𝟒𝒙
2𝑥 + 𝑦 4𝑥 7 7𝑦 − 13 𝑐𝑜𝑠 𝛼 = 𝑐𝑜𝑠 ⟹ 𝛼 = 2𝑛𝜋 ± , 𝑛∈𝑍
Given =
𝑦 𝑥+6
5𝑥 − 7 4𝑥
By the property of Equality of Matrices, 𝟒 𝟐
7.If A= and such that (A-2I)(A-3I)=0 ,
4𝑥 = 𝑥 + 6 2𝑥 + 𝑦 = 7 −𝟏 𝒙
find the value of x.
4𝑥 − 𝑥 = 6 Where x=2
Given (A-2I)(A-3I)=0
3𝑥 = 6 2(2) + 𝑦 = 7
4 2 1 0 4 2 1 0
x=2 𝑦 = 7−4⟹ 𝑦 =3 −2 −3 =0
−1 𝑥 0 1 −1 𝑥 0 1
Then the value of 𝑥 + 𝑦 = 2 + 3 = 5 ⟹ 𝑥 + 𝑦 = 5
4 2 2 0 4 2 3 0 3 0 0 0
− − =0 (iii) If A= and B=
−1 𝑥 0 2 −1 𝑥 0 3 3 0 3 3
2 2 1 2 0 0 3 0 0 0 0 0
= AB = =
−1 𝑥 − 2 −1 𝑥 − 3 0 0 3 0 3 3 0 0
2−2 4 + 2𝑥 − 6 0 0 0 0 3 0 0 0 0 0
= BA = = =
−1 − 𝑥 + 2 −2 + (𝑥 − 2)(𝑥 − 3) 0 0 3 3 3 0 9+9 0 18 0
0 2𝑥 − 2 0 0 Where AB=0 But BA≠0.
=
−𝑥 + 1 −2 + (𝑥 − 2)(𝑥 − 3) 0 0 11.Show that 𝒇(𝒙)𝒇(𝒚) = 𝒇(𝒙 + 𝒚)
2𝑥 − 2 = 0 𝒄𝒐𝒔 𝒙 −𝒔𝒊𝒏 𝒙 𝟎
𝑥=1 Where 𝒇(𝒙) = 𝒔𝒊𝒏 𝒙 𝒄𝒐𝒔 𝒙 𝟎
𝟏 𝟎 𝟎 𝟎 𝟎 𝟏
𝑐𝑜𝑠 𝑥 −𝑠𝑖𝑛 𝑥 0
8.If A= 𝟎 𝟏 𝟎 , Show that A2 is a unit matrix.
Given 𝑓(𝑥) = 𝑠𝑖𝑛 𝑥 𝑐𝑜𝑠 𝑥 0
𝒂 𝒃 −𝟏
1 0 0 1 0 0 0 0 1
𝑐𝑜𝑠 𝑦 −𝑠𝑖𝑛 𝑦 0
A2 = A x A = 0 1 0 0 1 0
Also 𝑓(𝑦) = 𝑠𝑖𝑛 𝑦 𝑐𝑜𝑠 𝑦 0
𝑎 𝑏 −1 𝑎 𝑏 −1
1+0+0 0+0+0 0+0+0 0 0 1
= 0+0+0 0+1+0 0+0+0 𝑐𝑜𝑠 𝑥 −𝑠𝑖𝑛 𝑥 0 𝑐𝑜𝑠 𝑦 −𝑠𝑖𝑛 𝑦 0
𝑓(𝑥) 𝑓(𝑦) = 𝑠𝑖𝑛 𝑥 𝑐𝑜𝑠 𝑥 0 𝑠𝑖𝑛 𝑦 𝑐𝑜𝑠 𝑦 0
𝑎+0−𝑎 0+𝑏−𝑏 0+0+1
1 0 0 0 0 1 0 0 1
𝑐𝑜𝑠 𝑥 𝑐𝑜𝑠𝑦 − 𝑠𝑖𝑛𝑥 𝑠𝑖𝑛𝑦 − cos 𝑥 𝑠𝑖𝑛𝑦 − sin 𝑥 cos 𝑦 0
= 0 1 0 ∴ 𝐴 𝑖𝑠 𝑎 𝑢𝑛𝑖𝑡 𝑀𝑎𝑡𝑟𝑖𝑥.
= 𝑠𝑖𝑛𝑥 𝑐𝑜𝑠𝑦 + cos 𝑥 𝑠𝑖𝑛𝑦 − sin 𝑥 sin 𝑦 + 𝑐𝑜𝑠𝑥 𝑐𝑜𝑠𝑦 0
0 0 1 0+0+0 0+0+0 1
𝟏 𝟎 𝟐
9. If A= 𝟎 𝟐 𝟏 and A3-6A2+7A+kI=0,find the value of k. 𝑐𝑜𝑠 𝑥 𝑐𝑜𝑠𝑦 − 𝑠𝑖𝑛𝑥 𝑠𝑖𝑛𝑦 −(cos 𝑥 𝑠𝑖𝑛𝑦 + sin 𝑥 cos 𝑦) 0
𝟐 𝟎 𝟑 = 𝑠𝑖𝑛𝑥 𝑐𝑜𝑠𝑦 + cos 𝑥 𝑠𝑖𝑛𝑦 𝑐𝑜𝑠𝑥 𝑐𝑜𝑠𝑦 − sin 𝑥 sin 𝑦 0
1 0 2 1 0 2 0+0+0 0+0+0 1
A2 = A x A = 0 2 1 0 2 1 𝑐𝑜𝑠 (𝑥 + 𝑦) −𝑠𝑖𝑛 (𝑥 + 𝑦) 0
2 0 3 2 0 3 = 𝑠𝑖𝑛 (𝑥 + 𝑦) 𝑐𝑜𝑠 (𝑥 + 𝑦) 0 = f(x,y) Hence Proved.
1+0+4 0+0+0 2+0+6 5 0 8
= 0+0+2 0+4+0 0+2+3 = 2
0 0 1
4 5
12.If A is a square matrix such that A2=A then find the
2+0+6 0+0+0 4+0+9 8 0 13
5 0 8 1 0 2 value of 7A-(I+A)3.
A3= A2 x A = 2 4 5 0 2 1 Given A2=A ; also A3=A2 A = A A = A2 A
8 0 13 2 0 3
5 + 0 + 16 0 + 0 + 0 10 + 0 + 24 21 0 34 Also I3 = I ; I2 = I ; IA= I ; I2A=A [∵ I is the Identity Matrix
= 2 + 0 + 10 0 + 8 + 0 4 + 4 + 15 = 12 8 23 7A-(I+A)3 = 7 A – (I3+A3+3I2A+3IA2)
8 + 0 + 26 0 + 0 + 0 16 + 0 + 39 34 0 55 = 7 A – (I + A + 3 A + 3A)
Given A3-6A2+7A+kI=0 = 7 A – I + 7A = - I
21 0 34 5 0 8 1 0 2 1 0 0
12 8 23 − 6 2 4 5 + 7 0 2 1 + 𝑘 0 1 0 =0 13.Verify the Property A(B+C)=AB+AC When the matrix
34 0 55 8 0 13 2 0 3 0 0 1 𝟑 𝟏 𝟒 𝟕
𝟐 𝟎 −𝟑
21 0 34 −30 0 −48 7 0 14 𝑘 0 0 A,B,C is given by A= B= −𝟏 𝟎 C= 𝟐 𝟏
12 8 23 + −12 −24 −30 + 0 14 7 + 0 𝑘 0 =0 𝟏 𝟒 𝟓
𝟒 𝟐 𝟏 −𝟏
34 0 55 −48 0 −78 14 0 21 0 0 𝑘 3 1 4 7
−2 + 𝑘 0 0 0 0 0 2 0 −3
0 −2 + 𝑘 0 = 0 0 0 L.H.S : A (B+C) = −1 0 + 2 1
1 4 5
0 0 −2 + 𝑘 0 0 0 4 2 1 −1
−2 + 𝑘 = 0 7 8
2 0 −3
𝑘=2 = 1 1
1 4 5
10.Give your own examples of matrices satisfying the 5 1
14 + 0 − 15 16 + 0 − 3
following conditions in each case: =
7 + 4 + 25 8+4+5
(i) A and B such that . AB≠ BA =
−1 13
--------(1)
(ii) A and B such that AB=0=BA , A≠ 0 and B ≠ 0 36 17
(iii) A and B such that , AB=0 and BA≠0 R.H.S :
3 1 4 7
Solution : 2 0 −3 2 0 −3
AB+AC = −1 0 + 2 1
1 2 4 3 1 4 5 1 4 5
(i) If A= and B= 4 2 1 −1
3 4 2 1 6 + 0 − 12 2 + 0 − 6 8 + 0 − 3 14 + 0 + 3
1 2 4 3 4+4 3+2 8 5 = +
AB = = = 3 − 4 + 20 1 + 0 + 10 4+8+5 7+4−5
3 4 2 1 12 + 8 9 + 4 20 13 −6 −4 5 17 −1 13
4 3 1 2 4 + 9 8 + 12 13 20 = + = -----------(2)
BA = = = 19 11 17 6 36 17
2 1 3 4 2+3 4+4 5 8
From (1) & (2) L.H.S = R.H.S Hence Proved .
Where AB ≠ BA
2 2 5 5 14. Find the matrix A which satisfies the matrix relation
(ii)If A= and B= 𝟏 𝟐 𝟑 −𝟕 −𝟖 −𝟗
2 2 5 5 A =
2 2 5 5 0 0 𝟒 𝟓 𝟔 𝟐 𝟒 𝟔
AB= = Where order of A Must be 2 x 2
−2 −2 −5 −5 0 0
5 5 2 2 0 0 𝑎 𝑏
BA= = Let A=
−5 −5 −2 −2 0 0 𝑐 𝑑
Where AB=BA=0 𝑎 𝑏 1 2 3 −7 −8 −9
=
𝑐 𝑑 4 5 6 2 4 6
𝑎 𝑏 1 2 3
=
−7 −8 −9 17. Express the following matrices as the sum of a
𝑐 𝑑 4 5 6 2 4 6 symmetric matrix and a skew-symmetric matrix:
𝑎 + 4𝑏 2𝑎 + 5𝑏 3𝑎 + 6𝑏 −7 −8 −9 𝟒 −𝟐
= (i)
𝑐 + 4𝑑 2𝐶 + 5𝑑 3𝑐 + 6𝑑 2 4 6 𝟑 −𝟓
𝑎 + 4𝑏 = −7 --------(1) 4 −2 4 3
If A= Now AT=
2a+5b = -8 ----------(2) 3 −5 −2 −5
c+4d=2 -----------(3) 8 1
Let P= (𝐴 + 𝐴 ) =
1 −10
2c+5d=4----------(4) 8 1
Now PT= Where 𝑃 = 𝑃
When solving (1) & (2) 𝑎 = 1 𝑎𝑛𝑑 𝑏 = −2 1 −10
When solving (3) & (4) 𝑐 = 2 𝑎𝑛𝑑 𝑑 = 0 Thus P= (𝐴 + 𝐴 ) is a Symmetric Matrix.
𝑎 𝑏 1 −2 0 −5
A= = Let Q= (𝐴 − 𝐴 ) =
𝑐 𝑑 2 0 5 0
4 5 0 5
𝟐 −𝟏 𝟏 Now Q =T Where −𝑄 = 𝑄
15.If AT= −1 0 and B= , Verify the −5 0
𝟕 𝟓 −𝟐
2 3 Thus Q= (𝐴 − 𝐴 ) is a Skew - Symmetric Matrix.
following (i) (A+B)T=AT+BT=BT+AT (ii)(A-B)T=AT-BT
Also P+Q=A ;
(iii) (BT)T=B.
𝟑 𝟑 −𝟏
4 5 (ii) −𝟐 −𝟐 𝟏
4 −1 2
Solution : AT= −1 0 then A=
5 0 3 −𝟒 −𝟓 𝟐
2 3 3 3 −1 3 −2 −4
2 7 Let B= −2 −2 1 and BT= 3 −2 −5
2 −1 1
B= then BT= −1 5
7 5 −2 −4 −5 2 −1 1 2
1 −2 6 1 −5
(i) (A+B)T=AT+BT=BT+AT Let R= (𝐵 + 𝐵 ) = 1 −4 −4
4 −1 2 2 −1 1 6 −2 3
A+B= + = −5 −4 4
5 0 3 7 5 −2 12 5 1 6 1 −5
6 12 Now RT= 1 −4 −4 Where 𝑅 = 𝑅
(A+B)T = −2 5 -----------------------------------------(1) −5 −4 4
3 1
4 5 2 7 6 12 Thus R= (𝐵 + 𝐵 ) is a Symmetric Matrix.
AT+BT = −1 0 + −1 5 = −2 5 -----------(2) 0 5 3
2 3 1 −2 3 1 Let S= (𝐵 − 𝐵 ) = −5 0 6
2 7 4 5 6 12 −3 −6 0
BT+AT = −1 5 + −1 0 = −2 5 ------------(3) 0 5 3
1 −2 2 3 3 1 Now ST= −5 0 6 Where −𝑆 = 𝑆
From (1) , (2) and (3) (A+B)T=AT+BT=BT+AT −3 −6 0
(ii)(A-B)T=AT-BT Thus S= (𝐵 − 𝐵 ) is a Skew - Symmetric Matrix.
4 −1 2 2 −1 1 Also R+S=B ;
A-B = −
5 0 3 7 5 −2 18.Find the Matrix A such that
4 −1 2 −2 1 −1 2 0 1
= + = 2 −1 −1 −8 −10
5 0 3 −7 −5 2 −2 −5 5
2 −2 1 0 𝐴 = 1 2 −5
(A-B)T = 0 −5 --------------------------------------------(4) −3 4 9 22 15
1 5 From Given data, We may assume the order of 𝐴 is 2 x 3
4 5 2 7 𝑎 𝑏 𝑐
Let AT =
AT+BT = −1 0 − −1 5 𝑑 𝑒 𝑓
2 3 1 −2 2 −1 −1 −8 −10
4 5 −2 −7 2 −2 1 0 𝐴 = 1 2 −5
= −1 0 + 1 −5 = 0 −5 ------------(5) −3 4 9 22 15
2 3 −1 2 1 5 2 −1 −1 −8 −10
𝑎 𝑏 𝑐
From (4) & (5) (A-B)T=AT-BT. 1 0
𝑑 𝑒 𝑓
= 1 2 −5
(iii) (BT)T=B −3 4 9 22 15
2 −1 1 2𝑎 − 𝑑 2𝑏 − 𝑒 2𝑐 − 𝑓 −1 −8 −10
B= ---------------------------------------------(6) 𝑎 𝑏 𝑐 = 1 2 −5
7 5 −2
2 7 −3𝑎 + 4𝑑 −3𝑏 + 4𝑒 −3𝑐 + 4𝑓 9 22 15
BT= −1 5 When Equating above matrices ,
1 −2 𝑎=1 𝑏=2 𝑐 = −5
2 −1 1
(BT)T = ----------------------------------------(7) 2𝑎 − 𝑑 = −1 2𝑏 − 𝑒 = −8 2𝑐 − 𝑓 = −10
7 5 −2
From (6) & (7) (BT)T=B . 2(1) − 𝑑 = −1 2(2) − 𝑒 = −8 2(−5) − 𝑓 = −10
16. If A is a 3 × 4 matrix and B is a matrix such that ATB and BAT 𝑑=3 𝑒 = 12 𝑓=0
both are defined, what is the order of the matrix B? 𝑎 𝑏 𝑐 1 2 −5
Now AT= =
Order of A is 3 x 4 ; then Order of AT is 4 x 3 . 𝑑 𝑒 𝑓 3 12 0
AT B is defined, so that number of rows in B is 3. Also BAT 1 3
Then A= 2 12
is defined, so that number of column in B is 4.
−5 0
Order of B is 3 x 4
𝟏 𝟐 𝟐 Converse Proof :
19.If A= 𝟐 𝟏 −𝟐 is a matrix such that AAT=9I , Then To Prove : AB = BA
𝒙 𝟐 𝒚 Given : A & B are symmetric . i.e., A=AT ; B=BT
find the values of x & y.
And also AB is symmetric. i.e., AB = (AB)T
Given A AT=9I
Proof : AB = (AB)T [ ∵ AB is symmetric(Given)
1 2 2 1 2 𝑥 1 0 0
2 1 −2 2 1 2 = 9 0 1 0 = B T AT [ ∵ Reversal law
𝑥 2 𝑦 2 −2 𝑦 0 0 1 =BA [ ∵ A=AT ; B=BT (Given)
1+4+4 2+2−4 𝑥 + 4 + 2𝑦 9 0 0 AB = BA . Hence Proved.
2+2−4 4+1+4 2𝑥 + 2 − 2𝑦 = 0 9 0 23. If A and B are symmetric matrices of same order,
𝑥 + 4 + 2𝑦 2𝑥 + 2 − 2𝑦 𝑥 + 4 + 𝑦 0 0 9 prove that (i) AB + BA is a symmetric matrix.
9 0 𝑥 + 4 + 2𝑦 9 0 0 (ii) AB - BA is a skew-symmetric matrix.
0 9 2𝑥 + 2 − 2𝑦 = 0 9 0 (i) To Prove : AB + BA is a symmetric
𝑥 + 4 + 2𝑦 2𝑥 + 2 − 2𝑦 𝑥 + 4 + 𝑦 0 0 9 i.e., AB + BA=( AB + BA)T
𝑥 + 4 + 2𝑦 = 0 ⇒ 𝑥 + 2𝑦 = −4 − − − − − (1) Given : A & B are symmetric . i.e., A=AT ; B=BT
2𝑥 + 2 − 2𝑦 = 0 ⇒ 𝑥 − 𝑦 = −1 − − − − − (2) Proof : (AB+BA)T = (AB)T+(BA)T
When solving (1) and (2) by elimination method we get = B T A T + AT BT
𝑥 = −2 𝑎𝑛𝑑 𝑦 = −6 = BA + AB
0 1 −2 = AB + BA
20.(i) for What value of x, The matrix A= −1 0 𝑥 is (AB + BA)T = AB + BA
2 −3 0 ∴ 𝐴𝐵 + 𝐵𝐴 is a symmetric matrix. Hence Proved.
Skew symmetric .
(ii) To Prove : AB - BA is a skew symmetric matrix
If A is Skew symmetric –A=AT
i.e., - (AB – BA)=( AB + BA)T
0 −1 2 0 −1 2
1 0 −𝑥 = 1 0 −3 Given : A & B are symmetric . i.e., A=AT ; B=BT
−2 3 0 −2 𝑥 0 Proof : (AB-BA)T = (AB)T-(BA)T
If we equating on both sides 𝑥 = 3 𝑡ℎ𝑒𝑛 𝑥 = 3 / = B T AT - A T BT
0 𝑝 3 = BA - AB
(ii)If 2 𝑞 −1 is skew symmetric , then find the = -(AB – BA)
𝑟 1 0 (AB + BA)T = -(AB – BA)
values of p,q,r . ∴ 𝐴𝐵 − 𝐵𝐴 is a semi symmetric matrix. Hence Proved.
0 𝑝 3 24. A shopkeeper in a Nuts and Spices shop makes
Let B= 2 𝑞 −1 . B is skew symmetric –B=BT
gift packs of cashew nuts, raisins and almonds. Pack I
𝑟 1 0
0 −𝑝 −3 0 2 𝑟 contains 100 gm of cashew nuts, 100 gm of raisins and
−2 −𝑞 1 = 𝑝 𝑞 1 50 gm of almonds. Pack-II contains 200 gm of cashew
−𝑟 −1 0 3 1 0 nuts, 100 gm of raisins and 100 gm of almonds. Pack-III
If we equating on both sides contains 250 gm of cashew nuts, 250 gm of raisins and
𝑝 = −2 and 𝑟 = −3 150 gm of almonds. The cost of 50 gm of cashew nuts is
Rs.50, 50 gm of raisins is Rs.10, and 50 gm of almonds is
−𝑞 = 𝑞 is only satisfy the valve of q=0 , 𝑞 = −2
Rs.60. What is the cost of each gift pack?
21.Construct the Matrix A= 𝒂𝒊𝒋 , Where 𝒂𝒊𝒋 = 𝒊 − 𝒋.
𝟑𝒙𝟑 Solution :Given data expressed in the following table
State Whether A is symmetric or skew symmetric.
𝑎 𝑎 𝑎 Pack I Pack II Pack II
A= 𝑎 𝑎 𝑎 Cashew nuts 100gm 200gm 250gm
𝑎 𝑎 𝑎 Raisins 100gm 100gm 250gm
Where 𝑎 = 𝑖 − 𝑗 Almonds 50gm 100gm 150gm
0 −1 −2 0 1 2 Let convert every quantity into 50 gm units.
A= 1 0 −1 also AT= −1 0 1 Cost of
2 1 0 −2 −1 0 Pack I Pack II Pack II 50gm unit
Where –A=AT . So that A is Skew symmetric . Cashew nuts 2 4 5 Rs.50
22. Let A and B be two symmetric matrices. Prove that Raisins 2 2 5 Rs.10
AB = BA if and only if AB is a symmetric matrix. Almonds 1 2 3 Rs.60
To Prove : AB is symmetric. i.e., AB = (AB)T Cost matrix = A= [50 10 60]
Given : A & B are symmetric . i.e., A=AT ; B=BT 𝑃𝐼 𝑃𝐼𝐼 𝑃𝐼𝐼𝐼
And also AB = BA 2 4 5 Cashew nuts
Proof : (AB)T = BTAT [ ∵ Reversal law Nuts & spices matrix=B = 2 2 5 Raisins
=BA [ ∵ B=BT and A=AT (Given) 1 2 3 Almonds
2 4 5
= AB [ ∵ AB = BA (Given) AB = [50 10 60] 2 2 5 = [180 340 480]
(AB)T = AB . ∴ AB is symmetric. 1 2 3 3𝑥3
Cost of Pack I , Pack II & pack III is Rs.180 , Rs.340 and Rs.480
Respectively.
𝟏+𝒂 𝟏 𝟏
Exercise 7.2 4.Prove that 𝟏 𝟏+𝒃
𝟏 𝟏 𝟏
𝟏 = 𝒂𝒃𝒄 𝟏 + 𝒂 + 𝒃 + 𝒄
𝟏 𝟏 𝟏+𝒄
1+𝑎 1 1
01.Without expanding the determinant , Prove that
L.H.S 1 1+𝑏 1
𝒔 𝒂𝟐 𝒃𝟐 + 𝒄𝟐 1 1 1+𝑐
𝒔 𝒃𝟐 𝒄𝟐 + 𝒂𝟐 = 𝟎 𝑎 −𝑏 0 𝑅 ⟶𝑅 −𝑅
𝒔 𝒄𝟐 𝒂𝟐 + 𝒃𝟐 = 0 𝑏 −𝑐 𝑅 ⟶ 𝑅 − 𝑅
Solution : L.H.S 1 1 1+𝑐 𝑅 ⟶𝑅
𝑠 𝑎 𝑏 +𝑐 𝑠 𝑎 𝑎 +𝑏 +𝑐 = 𝑎[𝑏(1 + 𝑐) + 𝑐] + 𝑏[0 + 𝑐]
𝑠 𝑏 𝑐 +𝑎 = 𝑠 𝑏 𝑎 +𝑏 +𝑐 𝐶 ⟶𝐶 +𝐶 = 𝑎[𝑏 + 𝑏𝑐 + 𝑐] + 𝑏𝑐
𝑠 𝑐 𝑎 +𝑏 𝑠 𝑐 𝑎 +𝑏 +𝑐 = 𝑎𝑏 + 𝑎𝑏𝑐 + 𝑎𝑐 + 𝑏𝑐
1 𝑎 1
= 𝑠(𝑎 + 𝑏 + 𝑐 ) 1 𝑏 1 = 𝑎𝑏𝑐 + 𝑎𝑏 + 𝑏𝑐 + 𝑎𝑐
1 𝑐 1 = 𝑎𝑏𝑐 1 + + + R.H.S Hence Proved.
= 𝑠(𝑎 + 𝑏 + 𝑐 ){0} [∵ 𝐶 ≡ 𝐶 𝐬𝐞𝐜 𝟐 𝜽 𝐭𝐚𝐧𝟐 𝜽 𝟏
=0 R.H.S . Hence Proved 5.Prove that 𝐭𝐚𝐧𝟐 𝜽 𝐬𝐞𝐜 𝟐 𝜽 −𝟏 = 𝟎
𝒃 + 𝒄 𝒃𝒄 𝒃𝟐 𝒄𝟐 𝟑𝟖 𝟑𝟔 𝟐
2.Show that 𝒄 + 𝒂 𝒄𝒂 𝒄𝟐 𝒂𝟐 = 𝟎 sec 𝜃 tan 𝜃 1
𝒂 + 𝒃 𝒂𝒃 𝒂𝟐 𝒃𝟐 𝐿. 𝐻. 𝑆 = tan 𝜃 sec 𝜃 −1
Solution : L.H.S 38 36 2
𝑏 + 𝑐 𝑏𝑐 𝑏 𝑐 sec 𝜃 sec 𝜃 − tan 𝜃 1
𝑎𝑏𝑐 𝑏 + 𝑐 𝑏𝑐 𝑏 𝑐 = tan 𝜃 tan 𝜃 − sec 𝜃 −1 𝐶 ⟶ 𝐶 − 𝐶
𝑐 + 𝑎 𝑐𝑎 𝑐 𝑎 = 𝑐 + 𝑎 𝑐𝑎 𝑐 𝑎
𝑎𝑏𝑐 38 38 − 36 2
𝑎 + 𝑏 𝑎𝑏 𝑎 𝑏 𝑎 + 𝑏 𝑎𝑏 𝑎 𝑏
Multiply R1,R2,& R3 by a,b & c respectively sec 𝜃 1 1
= tan 𝜃 −1 −1
𝑎𝑏 + 𝑎𝑐 𝑎𝑏𝑐 𝑎𝑏 𝑐
= 𝑏𝑐 + 𝑎𝑏 𝑎𝑏𝑐 𝑏𝑐 𝑎 38 2 2
=0 R.H.S [∵ 𝐶 ≡ 𝐶
𝑎𝑐 + 𝑏𝑐 𝑎𝑏𝑐 𝑐𝑎 𝑏
𝑎𝑏 + 𝑎𝑐 1 𝑏𝑐 𝒙 + 𝟐𝒂 𝒚 + 𝟐𝒃 𝒛 + 𝟐𝒄
=
( )
𝑏𝑐 + 𝑎𝑏 1 𝑎𝑐 6.Show that 𝒙 𝒚 𝒛 =𝟎
𝑎𝑐 + 𝑏𝑐 1 𝑎𝑏 𝒂 𝒃 𝒄
𝑎𝑏 + 𝑏𝑐 + 𝑐𝑎 1 𝑏𝑐 𝑥 + 2𝑎 𝑦 + 2𝑏 𝑧 + 2𝑐
= 𝑎𝑏𝑐 𝑎𝑏 + 𝑏𝑐 + 𝑐𝑎 1 𝑎𝑐 𝐶 ⟶ 𝐶 + 𝐶 L.H.S 𝑥 𝑦 𝑧
𝑎𝑏 + 𝑏𝑐 + 𝑐𝑎 1 𝑎𝑏 𝑎 𝑏 𝑐
1 1 𝑏𝑐 2𝑥 + 2𝑎 2𝑦 + 2𝑏 2𝑧 + 2𝑐
= 𝑎𝑏𝑐(𝑎𝑏 + 𝑏𝑐 + 𝑐𝑎) 1 1 𝑎𝑐 = 𝑥 𝑦 𝑧 𝐶 ⟶𝐶 +𝐶
1 1 𝑎𝑏 𝑎 𝑏 𝑐
= 𝑎𝑏𝑐(𝑎𝑏 + 𝑏𝑐 + 𝑐𝑎){0} [∵ 𝐶 ≡ 𝐶 2(𝑥 + 𝑎) (𝑦 + 𝑏) (𝑧 + 𝑐)
= 𝑥 𝑦 𝑧
= 0 R.H.S Hence Proved
𝑎 𝑏 𝑐
𝒂𝟐 𝒃𝒄 𝒂𝒄 + 𝒄𝟐 𝑥+𝑎 𝑦+𝑏 𝑧+𝑐
3.Prove that 𝒂𝟐 + 𝒂𝒃 𝒃 𝟐
𝒂𝒄 = 𝟒𝒂𝟐 𝒃𝟐 𝒄𝟐 =2 𝑥 𝑦 𝑧
𝒂𝒃 𝒃𝟐 + 𝒃𝒄 𝒄𝟐 𝑎 𝑏 𝑐
𝑎 𝑏𝑐 𝑎𝑐 + 𝑐 0 0 0
L.H.S 𝑎 + 𝑎𝑏 𝑏 𝑎𝑐 =2 𝑥 𝑦 𝑧 𝐶 ⟶ 𝐶 −𝐶 −𝐶
𝑎𝑏 𝑏 + 𝑏𝑐 𝑐 𝑎 𝑏 𝑐
2𝑎 + 2𝑎𝑏 2𝑏 + 2𝑏𝑐 2𝑎𝑐 + 2𝑐 = 2(0) = 0 𝑅. 𝐻. 𝑆 Hence Proved.
= 𝑎 + 𝑎𝑏 𝑏 𝑎𝑐 𝑅 ⟶𝑅 +𝑅 +𝑅
7.Write the general form of a 3x3 skew-symmetric
𝑎𝑏 𝑏 + 𝑏𝑐 𝑐
matrix and prove that its determinant is zero.
2(𝑎 + 𝑎𝑏) 2(𝑏 + 𝑏𝑐) 2(𝑎𝑐 + 𝑐 )
= 𝑎 + 𝑎𝑏 Method 1: Condition for skew symmetric matrices :
𝑏 𝑎𝑐
𝑎𝑏 𝑏 + 𝑏𝑐 𝑐 AT = -A
𝑎 + 𝑎𝑏 𝑏 + 𝑏𝑐 𝑎𝑐 + 𝑐 |𝐴 | = |−𝐴| Take modulus on both sides
=2 𝑎 + 𝑎𝑏 𝑏 𝑎𝑐 |𝐴| = (-1)n |𝐴| [By Property 1 & 3
𝑎𝑏 𝑏 + 𝑏𝑐 𝑐 Where n is the order of the square matrix, i.e.,n=3
𝑎 + 𝑎𝑏 𝑏 + 𝑏𝑐 𝑎𝑐 + 𝑐 𝑅 ⟶ 𝑅 − 𝑅 |𝐴| = - |𝐴|
=2 0 𝑏𝑐 𝑐 𝑅 ⟶𝑅 −𝑅 |𝐴| + |𝐴| = 0
𝑎 0 𝑎𝑐
𝑎(𝑎 + 𝑏) 𝑏(𝑏 + 𝑐) 𝑐(𝑎 + 𝑐) 2|𝐴| = 0
=2 0 𝑏𝑐 𝑐 |𝐴| = 0 Hence Proved.
𝑎 0 𝑎𝑐 𝟎 𝒉 𝒈
(𝑎 + 𝑏) (𝑏 + 𝑐) (𝑎 + 𝑐) Method 1: skew symmetric form A= − 𝒉 𝟎 𝒇
=2𝑎𝑏𝑐 0 𝑐 𝑐 −𝒈 −𝒇 𝟎
𝑎 0 𝑎
|𝐴| =−ℎ(𝑔𝑓) + 𝑔(𝑓ℎ)
= 2𝑎𝑏𝑐 { (𝑎 + 𝑏)(𝑎𝑐) − (𝑏 + 𝑐)(−𝑎𝑐) + (𝑎 + 𝑐)(−𝑎𝑐)}
= 2𝑎𝑏𝑐 { 𝑎 𝑐 + 𝑎𝑏𝑐 + 𝑎𝑏𝑐 + 𝑎𝑐 − 𝑎 𝑐 − + 𝑎𝑐 } = −𝑓𝑔ℎ + 𝑓𝑔ℎ = 0
= 2𝑎𝑏𝑐 { 2𝑎𝑏𝑐} = 4 𝑎 𝑏 𝑐 R.H.S Hence Proved.
𝒂 𝒃 𝒂𝜶 + 𝒃 𝒂𝟐 + 𝒙𝟐 𝒂𝒃 𝒂𝒄
8.If 𝒃 𝒄 𝒃𝜶 + 𝒄 = 𝟎 11.Show that 𝒂𝒃 𝒃𝟐 + 𝒙𝟐 𝒃𝒄 is
𝒂𝜶 + 𝒃 𝒃𝜶 + 𝒄 𝟎 𝒂𝒄 𝒃𝒄 𝒄𝟐 + 𝒙𝟐
𝑷𝒓𝒐𝒗𝒆 𝒕𝒉𝒂𝒕 𝒂, 𝒃, 𝒄 𝒂𝒓𝒆 𝒊𝒏 𝑮. 𝑷 𝒐𝒓 𝜶 𝒊𝒔 𝒂 𝒓𝒐𝒐𝒕 𝒐𝒇 divisible by 𝒙𝟒 .
𝒂𝒙𝟐 + 𝟐𝒃𝒙 + 𝒄 = 𝟎
𝑎𝑏𝑐 𝑎 + 𝑥 𝑎𝑏 𝑎𝑐
𝑎 𝑏 𝑎𝛼 + 𝑏 = 𝑎𝑏 𝑏 +𝑥 𝑏𝑐
𝑏 𝑐 𝑏𝛼 + 𝑐 = 0 𝑎𝑏𝑐
𝑎𝑐 𝑏𝑐 𝑐 +𝑥
𝑎𝛼 + 𝑏 𝑏𝛼 + 𝑐 0 Multiply R1 , R2 and R3 by a,b,c respectively
Expanding about R3 𝑎(𝑎 + 𝑥 ) 𝑎 𝑏 𝑎 𝑐
(𝑎𝛼 + 𝑏)[𝑏(𝑏𝛼 + 𝑐) − 𝑐(𝑎𝛼 + 𝑏)] 1
= 𝑎𝑏 𝑏(𝑏 + 𝑥 ) 𝑏 𝑐
− (𝑏𝛼 + 𝑐)[𝑎(𝑏𝛼 + 𝑐) − 𝑏(𝑎𝛼 + 𝑏)] = 0 𝑎𝑏𝑐
𝑎𝑐 𝑏𝑐 𝑐(𝑐 + 𝑥 )
(𝑎𝛼 + 𝑏)[𝑏 𝛼 + 𝑏𝑐 − 𝑎𝑐𝛼 − 𝑏𝑐]
(𝑎 + 𝑥 ) 𝑎 𝑎
− (𝑏𝛼 + 𝑐)[𝑎𝑏𝛼 + 𝑎𝑐 − 𝑎𝑏𝛼 − 𝑏 ] = 0
= 𝑏 (𝑏 + 𝑥 ) 𝑏
(𝑎𝛼 + 𝑏)[𝑏 𝛼 − 𝑎𝑐𝛼] − (𝑏𝛼 + 𝑐)[𝑎𝑐 − 𝑏 ] = 0
𝑐 𝑐 (𝑐 + 𝑥 )
(𝑎𝛼 + 𝑏)𝛼 [𝑏 − 𝑎𝑐] + (𝑏𝛼 + 𝑐)[𝑏 − 𝑎𝑐] = 0
𝑥 +𝑎 +𝑏 +𝑐 𝑥 +𝑎 +𝑏 +𝑐 𝑥 +𝑎 +𝑏 +𝑐
[𝑏 − 𝑎𝑐]{(𝑎𝛼 + 𝑏)𝛼 + (𝑏𝛼 + 𝑐)} = 0 = 𝑏 𝑏 +𝑥 𝑏
[𝑏 − 𝑎𝑐]{𝑎𝛼 + 𝑏𝛼 + 𝑏𝛼 + 𝑐} = 0 𝑐 𝑐 𝑐 +𝑥
[𝑏 − 𝑎𝑐]{𝑎𝛼 + 2𝑏𝛼 + 𝑐} = 0 𝑅 ⟶ 𝑅 +𝑅 +𝑅
𝑏 − 𝑎𝑐 = 0 or 𝑎𝛼 + 2𝑏𝛼 + 𝑐 = 0 1 1 1
= (𝑥 + 𝑎 + 𝑏 + 𝑐 ) 𝑏 𝑏 +𝑥 𝑏
𝑏 = 𝑎𝑐 𝛼 𝑖𝑠 𝑎 𝑟𝑜𝑜𝑡 𝑜𝑓
𝑐 𝑐 𝑐 +𝑥
i.e., a,b,c 𝑎𝑥 + 2𝑏𝑥 + 𝑐 = 0 1 0 0 𝐶 ⟶𝐶 −𝐶
are in G.P Hence Proved . = (𝑥 + 𝑎 + 𝑏 + 𝑐 ) 𝑏 𝑥 0 𝐶 ⟶𝐶 −𝐶
𝟏 𝒂 𝒂𝟐 − 𝒃𝒄 𝑐 0 𝑥
9.Prove that 𝟏 𝒃 𝒃𝟐 − 𝒄𝒂 = 𝟎 =(𝑥 + 𝑎 + 𝑏 + 𝑐 ) 1(𝑥 − 0)
𝟏 𝒄 𝒄𝟐 − 𝒂𝒃 =(𝑥 + 𝑎 + 𝑏 + 𝑐 ) 𝑥
1 𝑎 𝑎 − 𝑏𝑐 1 𝑎 𝑎 1 𝑎 𝑏𝑐 ∴ Given determinant divisible by 𝑥 .
L.H.S 1 𝑏 𝑏 − 𝑐𝑎 = 1 𝑏 𝑏 − 1 𝑏 𝑐𝑎 12.If a,b,c are all positive , and are pth, qth and rth terms
1 𝑐 𝑐 − 𝑎𝑏 1 𝑐 𝑐 1 𝑐 𝑎𝑏 𝒍𝒐𝒈 𝒂 𝒑 𝟏
1 𝑎 𝑎 1 𝑎 𝑏𝑐
of a G.P , Show that 𝒍𝒐𝒈 𝒃 𝒒 𝟏 = 𝟎
= 1 𝑏 𝑏 − 1 𝑏 𝑐𝑎
𝒍𝒐𝒈 𝒄 𝒓 𝟏
1 𝑐 𝑐 1 𝑐 𝑎𝑏
1 𝑎 𝑎 𝑎 𝑎 𝑎𝑏𝑐 From given data
= 1 𝑏 𝑏 − 𝑏 𝑏 𝑎𝑏𝑐 𝑡 = 𝑎 = 𝐴𝑅 ⇒ log 𝑎 = log 𝐴𝑅
1 𝑐 𝑐 𝑐 𝑐 𝑎𝑏𝑐 𝑙𝑜𝑔 𝑎 = 𝑙𝑜𝑔 𝐴 + (𝑝 − 1)𝑙𝑜𝑔 𝑅
1 𝑎 𝑎 𝑎 𝑎 1
= 1 𝑏 𝑏 − 𝑏 𝑏 1 𝑡 = 𝑏 = 𝐴𝑅 ⇒ log 𝑏 = log 𝐴𝑅
1 𝑐 𝑐 𝑐 𝑐 1 𝑙𝑜𝑔 𝑏 = 𝑙𝑜𝑔 𝐴 + (𝑞 − 1)𝑙𝑜𝑔 𝑅
1 𝑎 𝑎 1 𝑎 𝑎 𝑡 = 𝑐 = 𝐴𝑅 ⇒ log 𝑐 = log 𝐴𝑅
= 1 𝑏 𝑏 + 1 𝑏 𝑎 𝑐 → 𝑐
𝑙𝑜𝑔 𝑐 = 𝑙𝑜𝑔 𝐴 + (𝑟 − 1)𝑙𝑜𝑔 𝑅
1 𝑐 𝑐 1 𝑐 𝑎
1 𝑎 𝑎 1 𝑎 𝑎 𝑙𝑜𝑔 𝑎 𝑝 1 𝑙𝑜𝑔 𝐴 + (𝑝 − 1)𝑙𝑜𝑔 𝑅 𝑝 1
= 1 𝑏 𝑏 − 1 𝑏 𝑏 𝑐 → 𝑐 L.H.S 𝑙𝑜𝑔 𝑏 𝑞 1 = 𝑙𝑜𝑔 𝐴 + (𝑞 − 1)𝑙𝑜𝑔 𝑅 𝑞 1
1 𝑐 𝑐 1 𝑐 𝑐 𝑙𝑜𝑔 𝑐 𝑟 1 𝑙𝑜𝑔 𝐴 + (𝑟 − 1)𝑙𝑜𝑔 𝑅 𝑟 1
= 0 [ R.H.S ] Hence Proved 𝑙𝑜𝑔 𝐴 𝑝 1 (𝑝 − 1)𝑙𝑜𝑔 𝑅 𝑝 1
10.If a,b,c are pth,qth & rth terms of an A.P , Find the Value = 𝑙𝑜𝑔 𝐴 𝑞 1 + (𝑞 − 1)𝑙𝑜𝑔 𝑅 𝑞 1
𝒂 𝒃 𝒄 𝑙𝑜𝑔 𝐴 𝑟 1 (𝑟 − 1)𝑙𝑜𝑔 𝑅 𝑟 1
of 𝒑 𝒒 𝒓 .
1 𝑝 1 𝑝−1 𝑝 1
𝟏 𝟏 𝟏 = 𝑙𝑜𝑔 𝐴 1 𝑞 1 + 𝐿𝑜𝑔𝑅 𝑞 − 1 𝑞 1
Given : a,b,c are pth,qth & rth terms of an A.P
1 𝑟 1 𝑟−1 𝑟 1
tp=A+(p-1)d 1 𝑝 1 𝑝−1 𝑝−1 1
tq=A+(q-1)d = 𝑙𝑜𝑔 𝐴 1 𝑞 1 + 𝐿𝑜𝑔𝑅 𝑞 − 1 𝑞 − 1 1 𝑐 → 𝑐 − 𝑐
tr=A+(r-1)d 1 𝑟 1 𝑟−1 𝑟−1 1
𝑎 𝑏 𝑐 A + (p − 1)d A + (q − 1)d A + (r − 1)d = 𝑙𝑜𝑔 𝐴(0) + 𝐿𝑜𝑔𝑅(0) = 0 + 0 = 0 R.H.S
𝑝 𝑞 𝑟 = 𝑝 𝑞 𝑟 𝟏 𝐥𝐨𝐠 𝒙 𝒚 𝐥𝐨𝐠 𝒙 𝒛
1 1 1 1 1 1 13.Find the value of 𝐥𝐨𝐠 𝒚 𝒙 𝟏 𝐥𝐨𝐠 𝒙 𝒚 𝒊𝒇 𝒙, 𝒚, 𝒂 ≠ 𝟏
𝐴 𝐴 𝐴 (p − 1)d (q − 1)d (r − 1)d 𝐥𝐨𝐠 𝒛 𝒙 𝐥𝐨𝐠 𝒛 𝒚 𝟏
= 𝑝 𝑞 𝑟 + 𝑝 𝑞 𝑟 1 log 𝑦 log 𝑧
1 1 1 1 1 1
1 1 1 p−1 q−1 r−1 = log 𝑥 1 log 𝑦
=A 𝑝 𝑞 𝑟 + 𝑑 𝑝 − 1 𝑞 − 1 𝑟 − 1 𝑅 ⟶ 𝑅 − 𝑅 log 𝑥 log 𝑦 1
1 1 1 1 1 1 log 𝑥 log 𝑥 log 𝑦 log 𝑥 log 𝑧
1
= A (0) + d( 0 ) = 0 = log 𝑦 log 𝑥 log 𝑦 log 𝑦 log 𝑦
log 𝑥 log 𝑦 log 𝑧
log 𝑧 log 𝑥 log 𝑧 log 𝑦 log 𝑧
log 𝑥 log 𝑦 log 𝑧 𝟒 𝟑 −𝟐
log 𝑥 log 𝑦 log 𝑧 20.Verify det(AB) = det(A) det(B) for A= 𝟏 𝟎
= log 𝑥 log 𝑦 log 𝑧 𝟕 and
log 𝑥 log 𝑦 log 𝑧 𝟐 𝟑 −𝟓
log 𝑥 log 𝑦 log 𝑧
𝟏 𝟑 𝟑
1 1 1 B= −𝟐 𝟒 𝟎
= log 𝑥 log 𝑦 log 𝑧 1 1 1
𝟗 𝟕 𝟓
1 1 1 To Prove : |AB | = |A| |B|
= log 𝑥 log 𝑦 log 𝑧 {0} = 0
𝟏
4 3 −2 1 3 3
𝟐
𝜶 𝟏 𝟏 AB= 1 0 7 −2 4 0
14. If A= 𝟏
, Prove that ∑𝒏𝒌 𝟏 𝒅𝒆𝒕 𝑨𝒌 = 𝟏− . 2 3 −5 9 7 5
𝟑 𝟒𝒏
𝟎 4 − 6 − 18 12 + 12 − 14 12 + 0 − 10
𝟐

𝛼 = 1 + 0 + 63 3 + 0 + 49 3 + 0 + 35
A= then |𝐴| = − 0 = 2 − 6 − 45 6 + 12 − 35 6 + 0 − 25
0 −20 10 2
= 64 52 38
𝛼 𝛼 𝛼
𝐴 = = ; |𝐴2 | =
1
−0 =
1 −49 −17 −19
0 0 0 16 16 = −20(−988 + 646) − 10(−1216 + 1862) + 2(−1088 + 2548)
=-20(-342)-10(646)+2(1460)= 6840-6460+2920 = 3300 --------(1)
𝛼 𝛼 1 1 |A| = 4(0 − 21) − 3(−5 − 14) − 2(3 − 0)
𝐴 = = ; |𝐴3 | = −0=
0 0 0 64 64
= 4(−21) − 3(−19) − 2(3) = −84 + 57 − 6 = −33
∑ 𝑑𝑒𝑡(𝐴 ) = +
1
+
1
+ ⋯ … … … ..(n terms) |B|= 1(20 − 0) − 3(−10 − 0) + 3(−14 − 36)
16 64
Above series is like a G.P = 1(20) − 3(−10) + 3(−50) = 20 + 30 − 150 = −100
1
1−
1 𝑛 1
1−
1
1
|A| |B| = -33 (-100) = 3300 -------------------------------- (2)
𝑆 =
4 4
1 = 4 4𝑛
=
3
1 − 41𝑛 Hence Proved From (1) and (2) |AB | = |A| |B|
4
15.With out expanding , Evaluate the following determinants 21.Using co factors element second row, Evaluate |A|
𝟐 𝟑 𝟒 5 3 8
(i) 𝟓 𝟔 𝟖 where A= 2 0 1 .
𝟔𝒙 𝟗𝒙 𝟏𝟐𝒙 1 2 3
2 3 4 2 3 4 |A|= 𝑎 𝐴 + 𝑎 𝐴 + 𝑎 𝐴
5 6 8 = 3𝑥 5 6 8 = 3𝑥(0) = 0 [∵ 𝑅 ≡ 𝑅 3 8 5 8 5 3
6𝑥 9𝑥 12𝑥 2 3 4 = -2 +0 −1
2 3 1 3 1 2
𝒙+𝒚 𝒚+𝒛 𝒛+𝒙
= -2(9 − 16) − 1(10 − 3)
(ii) 𝒛 𝒙 𝒚
= -2(-7) -1(7) = 14-7 = 7
𝟏 𝟏 𝟏
𝑥+𝑦+𝑧 𝑥+𝑦+𝑧 𝑥+𝑦+𝑧
= 𝑧 𝑥 𝑦 𝑅1 ⟶ 𝑅1 + 𝑅2 Exercise 7.3
1 1 1
1 1 1
= (𝑥 + 𝑦 + 𝑧) 𝑧 𝑥 𝑦 = (𝑥 + 𝑦 + 𝑧){0} = 0 [∵ 𝑅 ≡ 𝑅 𝒙 𝒂 𝒂
1 1 1 01.Show that 𝒂 𝒙 𝒂 = (𝒙 − 𝒂)𝟐 (𝒙 + 𝟐𝒂).
16.If A is a square matrix and |𝑨| = 𝟐,find the value of 𝑨𝑨𝑻 𝒂 𝒂 𝒙
𝑥 𝑎 𝑎
|𝐴𝐴 | = |𝐴| |𝐴 | = |𝐴| |𝐴| = 2 𝑥 2 = 4 [∵ |𝐴𝑇 | = |𝐴| Let |𝐴| = 𝑎 𝑥 𝑎
17.If A and B are square matrices of order 3 such that 𝑎 𝑎 𝑥
|𝑨| = −𝟏 and |𝑩| = 𝟑 , find the value of |𝟑𝑨𝑩| . 𝑎 𝑎 𝑎
|3𝐴𝐵| = 3 |𝐴𝐵| [∵ |𝑘𝐴| = 𝑘 |𝐴| Putting x=a we get |𝐴| = 𝑎 𝑎 𝑎 = 0
= 3 |𝐴||𝐵| 𝑎 𝑎 𝑎
Since all the three rows are identical (𝑥 − 𝑎) is
= 27(−1)(3) = −81
0 2𝜆 1 a factor of |𝐴|.
18. If 𝝀 = −𝟐, Determine the value of 𝜆 0 3𝜆 + 1 −2𝑎 𝑎 𝑎
−1 6𝜆 − 1 0 Putting x=-2a we get |𝐴| = 𝑎 −2𝑎 𝑎
0 2𝜆 1 0 −4 1 𝑎 𝑎 −2𝑎
If 𝝀 = −𝟐; 𝜆 0 3𝜆 + 1 = 4 0 13 0 0 0
−1 6𝜆 − 1 0 −1 −13 0
= 4(0 − 13) + 1(−52 − 0) = 52 – 52 =0
= 𝑎 −2𝑎 𝑎 𝑅 ⟶ 𝑅 +𝑅 +𝑅
1 4 20 𝑎 𝑎 −2𝑎
19.Determine the roots of the equation 1 −2 5 =0 |𝐴| = 0 , (𝑥 + 2𝑎) is a factor of |𝐴|.
1 2𝑥 5𝑥
1 4 20 The product of (𝑥 − 𝑎) (𝑥 + 2𝑎) is a factor of |𝐴|.
1 −2 5 =0 Now determinant is a cubic polynomial in x.
1 2𝑥 5𝑥
1 4 20 Degree of the product of known factors are 3.
𝑅 ⟶ 𝑅 −𝑅
0 −6 −15 𝑅 ⟶ 𝑅 −𝑅
=0 Degree of the leading diagonal elements are 3.
0 2𝑥 − 4 5𝑥 − 20
2
1(−30𝑥 + 120 + 30𝑥 − 60) = 0 Difference between them m=3-3=0
−30𝑥2 + 30𝑥 + 60 = 0 so that remaining factor must be a constant ‘k’.
𝑥 −𝑥−2=0 𝑥 𝑎 𝑎
(𝑥 − 2)(𝑥 + 1) = 0 𝑎 𝑥 𝑎 = 𝑘(𝑥 − 𝑎) (𝑥 + 2𝑎)
𝑥 = 2 𝑎𝑛𝑑 𝑥 = −1 𝑎 𝑎 𝑥
If a=0 𝑥 = 𝑘𝑥 ⟹ 𝑘 = 1
Thus |A|=(𝑥 − 𝑎) (𝑥 + 2𝑎) .
𝒃+𝒄 𝒂−𝒄 𝒂−𝒃 𝒃+𝒄 𝒂 𝒂𝟐
2.Show that 𝒃 − 𝒄 𝒄 + 𝒂 𝒃 − 𝒂 = 𝟖𝒂𝒃𝒄 4.Show that 𝒄 + 𝒂 𝒃 𝒃𝟐 = (𝒂 + 𝒃 + 𝒄)(𝒂 − 𝒃)(𝒃 − 𝒄)(𝒄 − 𝒂)
𝒄−𝒃 𝒄−𝒂 𝒂+𝒃 𝒂 + 𝒃 𝒄 𝒄𝟐
𝑏+𝑐 𝑎−𝑐 𝑎−𝑏 𝑏+𝑐 𝑎 𝑎
Let |𝐴| = 𝑏 − 𝑐 𝑐 + 𝑎 𝑏 − 𝑎 Let |A|= 𝑐 + 𝑎 𝑏 𝑏
𝑐−𝑏 𝑐−𝑎 𝑎+𝑏 𝑎+𝑏 𝑐 𝑐
𝑏 + 𝑐 −𝑐 −𝑏
Put a=0 |𝐴| = 𝑏 − 𝑐 𝑐 𝑏
𝑐−𝑏 𝑐 𝑏 𝑏+𝑐 𝑏 𝑏
𝑏 + 𝑐 −1 −1 Put a=b |A|= 𝑐 + 𝑏 𝑏 𝑏 =0 [∵ 𝑅 ≡ 𝑅
|𝐴| = 𝑏𝑐 𝑏 − 𝑐 1 1 =0 2𝑏 𝑐 𝑐
𝑐−𝑏 1 1 So that (a-b) is a factor .Similarly (b-c) (c-a) also factors of |A|.
⟹ a is a factor. Degree of the product of known factors are 3.
Similarly , b and c are also factors. Degree of the leading diagonal elements are 4.
Degree of the product of known factors are 3. Difference between them m=4-3=1
Degree of the leading diagonal elements are 3. so that remaining factor must be ‘k(a+b+c)’.
Difference between them m=3-3=0 𝑏+𝑐 𝑎 𝑎
so that remaining factor must be a constant ‘k’. 𝑐 + 𝑎 𝑏 𝑏 = 𝑘(𝑎 + 𝑏 + 𝑐)(𝑎 − 𝑏)(𝑏 − 𝑐)(𝑐 − 𝑎)
𝑏+𝑐 𝑎−𝑐 𝑎−𝑏 𝑎+𝑏 𝑐 𝑐
|𝐴| = 𝑏 − 𝑐 𝑐 + 𝑎 𝑏 − 𝑎 = 𝑘𝑎𝑏𝑐 Put a=0 , b=1, c=2
𝑐−𝑏 𝑐−𝑎 𝑎+𝑏 3 0 0
Put 𝑎 = 1 , 𝑏 = 1 , 𝑐 = 1 |A|= 2 1 1 = 𝑘(0 + 1 + 2)(0 − 1)(1 − 2)(2 − 0)
1 2 4
2 0 0
3(4 − 2) = 3𝑘(−1)(−1)(2)
|𝐴| = 0 2 0 = 𝑘(1)(1)(1)
0 0 2 6𝑘 = 6 ⟹ 𝑘 = 1
8=𝑘 𝑏+𝑐 𝑎 𝑎
Thus |A|= 8abc 𝑐 + 𝑎 𝑏 𝑏 = (𝑎 + 𝑏 + 𝑐)(𝑎 − 𝑏)(𝑏 − 𝑐)(𝑐 − 𝑎)
𝒙+𝒂 𝒃 𝒄 𝑎+𝑏 𝑐 𝑐
𝟒−𝒙 𝟒+𝒙 𝟒+𝒙
3.Solve 𝒂 𝒙+𝒃 𝒄 =𝟎
5.Solve 𝟒 + 𝒙 𝟒 − 𝒙 𝟒 + 𝒙 = 𝟎
𝒂 𝒃 𝒙+𝒄 𝟒+𝒙 𝟒+𝒙 𝟒−𝒙
𝑥+𝑎 𝑏 𝑐 4−𝑥 4+𝑥 4+𝑥
Let |𝐴| = 𝑎 𝑥+𝑏 𝑐
Let |A|= 4 + 𝑥 4 − 𝑥 4 + 𝑥
𝑎 𝑏 𝑥+𝑐
𝑎 𝑏 𝑐 4+𝑥 4+𝑥 4−𝑥
Put x=0 |𝐴| = 𝑎 𝑏 𝑐 = 0 4 4 4
𝑎 𝑏 𝑐 Put x=0 |A|= 4 4 4
Since all the three rows are identical 𝑥 is a factor of |𝐴|. 4 4 4
Again Put 𝑥 = −𝑎 − 𝑏 − 𝑐 Since all the three rows are identical 𝑥 is a factor of |𝐴|.
−𝑏 − 𝑐 𝑏 𝑐 16 −8 −8
|𝐴| = 𝑎 −𝑐 − 𝑎 𝑐 Put x=-12 |A|= −8 16 −8
𝑎 𝑏 −𝑎 − 𝑏 −8 −8 16
0 𝑏 𝑐 0 −8 −8
|𝐴| = 0 −𝑐 − 𝑎 𝑐 𝐶1 ⟶ 𝐶1 + 𝐶2 + 𝐶3 |A|= 0 16 −8 𝐶 ⟶ 𝐶 + 𝐶 + 𝐶
0 𝑏 −𝑎 − 𝑏 0 −8 16
|𝐴| = 0 |A|=0 , so that (x+12) is a factor.
(𝑥 + 𝑎 + 𝑏 + 𝑐) is a factor of |𝐴|. Degree of the product of known factors are 3.
Degree of the product of known factors are 3. Degree of the leading diagonal elements are 3.
Degree of the leading diagonal elements are 3. Difference between them m=3-3=0
Difference between them m=3-3=0 so that remaining factor must be ‘k’.
so that remaining factor must be a constant ‘k’ 4−𝑥 4+𝑥 4+𝑥
𝑥+𝑎 𝑏 𝑐 4 + 𝑥 4 − 𝑥 4 + 𝑥 = 𝑘𝑥 (𝑥 + 12)
𝑎 𝑥+𝑏 𝑐 = 𝑘𝑥 (𝑥 + 𝑎 + 𝑏 + 𝑐) 4+𝑥 4+𝑥 4−𝑥
𝑎 𝑏 𝑥+𝑐 8 0 0
If 𝑎 = 0 , 𝑏 = 1 , 𝑐 = 2 𝑎𝑛𝑑 𝑥 = 1 Put k=-4 0 8 0 = 𝑘(−4) (−4 + 12)
1 1 2 0 0 8
0 2 2 = 𝑘(1)(1 + 0 + 1 + 2) 8(64 − 0) = 16𝑘(8)
0 1 3 512 = 128𝑘 ⟹ 𝑘 = 4
1(6 − 2) = 4𝑘 ⟹ 𝑘 = 1 4−𝑥 4+𝑥 4+𝑥
Thus |A|=𝑥 (𝑥 + 𝑎 + 𝑏 + 𝑐) 4 + 𝑥 4 − 𝑥 4 + 𝑥 = 4𝑥 (𝑥 + 12)
𝑥+𝑎 𝑏 𝑐 4+𝑥 4+𝑥 4−𝑥
Given 𝑎 𝑥+𝑏 𝑐 =0 4−𝑥 4+𝑥 4+𝑥
𝑎 𝑏 𝑥+𝑐 Given 4 + 𝑥 4 − 𝑥 4 + 𝑥 = 0
𝑥 (𝑥 + 𝑎 + 𝑏 + 𝑐) = 0 4+𝑥 4+𝑥 4−𝑥
4𝑥 (𝑥 + 12) = 0
𝑥 = 0 (𝑡𝑤𝑖𝑐𝑒) 𝑥 = −(𝑎 + 𝑏 + 𝑐)
𝑥 = 0 (𝑡𝑤𝑖𝑐𝑒) 𝑥 = −12
𝟏 𝟏 𝟏 3.Idendify singular and non-singular matrices:
6.Show that 𝒙 𝒚 𝒛 = (𝒙 − 𝒚)(𝒚 − 𝒛)(𝒛 − 𝒙) 𝟏 𝟐 𝟑
𝒙𝟐 𝒚𝟐 𝒛𝟐 (i) 𝟒 𝟓 𝟔
1 1 1 𝟕 𝟖 𝟗
1 2 3
Let |A|= 𝑥 𝑦 𝑧 Let A= 4 5 6
𝑥 𝑦 𝑧 7 8 9
1 1 1 |𝐴| = 1(45 − 48) − 2(36 − 42) + 3(32 − 35)
Put x=y |A|= 𝑦 𝑦 𝑧 =0 = 1(−3) − 2(−6) + 3(−3)
𝑦 𝑦 𝑧 = −3 + 12 − 9 = 0
So that (x-y) is a factor. |𝐴| = 0 so that A is a Singular Matrix.
Similarly (y—z)(z-x) also factors of |A|. 𝟐 −𝟑 𝟓
Degree of the product of known factors are 3. (ii) 𝟔 𝟎 𝟒
𝟏 𝟓 −𝟕
Degree of the leading diagonal elements are 3. 2 −3 5
Difference between them m=3-3=0 Let B= 6 0 4
so that remaining factor must be ‘k’. 1 5 −7
1 1 1 |𝐵| = 2(0 + 7) + 3(−42 − 4) + 5(30 − 0)
|A|= 𝑥 𝑦 𝑧 = 𝑘(𝑥 − 𝑦)(𝑦 − 𝑧)(𝑧 − 𝑥) = 2(7) + 3(−46) + 5(30)
𝑥 𝑦 𝑧 = 14 − 138 + 150 = 26
Put X=0 , y=1 , z=2 |𝐵| ≠ 0 so that B is a Non Singular Matrix.
1 1 1 𝟎 𝒂−𝒃 𝒌
0 1 2 = 𝑘(0 − 1)(1 − 2)(2 − 0) (iii) 𝒃 − 𝒂 𝟎 𝟓
0 1 4 −𝒌 −𝟓 𝟎
0 𝑎−𝑏 𝑘
1(4 − 2) = 2𝑘 ⟹ 𝑘 = 1
Let C= 𝑏 − 𝑎 0 5
1 1 1
−𝑘 −5 0
Thus |A|= 𝑥 𝑦 𝑧 = (𝑥 − 𝑦)(𝑦 − 𝑧)(𝑧 − 𝑥) |𝐶| = −(𝑎 − 𝑏)(0 + 5𝑘) + 𝑘(−5𝑏 + 5𝑎)
𝑥 𝑦 𝑧 = −5𝑎𝑘 + 5𝑏𝑘 − 5𝑏𝑘 + 5𝑎𝑘
=0
Exercise 7.4 |𝐶| = 0 so that C is a Singular Matrix.
4.Determine the values of a and b so that the following
01.Find the area of the triangle whose vertices are (0,0) , matrices are singular.
(1,2) and (4,3). 𝟕 𝟑
(i)
𝑥 𝑦 1 −𝟐 𝒂
7 3
Area of the triangle = 𝑥 𝑦 1 Let A=
−2 𝑎
𝑥 𝑦 1
Given : A is a singular matrix. i.e.,|𝐴| = 0
0 0 1 −6
= 1 2 1 |𝐴| = 7𝑎 + 6 = 0 ⟹ 𝑎 =
7
4 3 1
𝒃−𝟏 𝟐 𝟑
= (1(3 − 8) (ii) 𝟑 𝟏 𝟐
𝟏 −𝟐 𝟒
= (−5) 𝑏−1 2 3
= = = 2.5 𝑠𝑞. 𝑈𝑛𝑖𝑡𝑠 Let B = 3 1 2
1 −2 4
2.If (k,2),(2,4) and (3,2) are vertices of the triangle of area Given : B is a singular matrix. i.e.,|𝐵| = 0
4 sq.units then determine the value of k. |𝐵| = (𝑏 − 1)(4 + 4) − 2(12 − 2) + 3(−6 − 1) = 0
𝑥 𝑦 1 (𝑏 − 1)(8) − 2(10) + 3(−7) = 0
Area of the triangle = 𝑥 𝑦 1 = 4 8𝑏 − 8 − 20 − 21 = 0
𝑥 𝑦 1 8𝑏 − 49 = 0
𝑘 2 1 49
= 2 4 1 =4 ⟹ 𝑏=
8
3 2 1
𝟎 𝒄𝒐𝒔𝜽 𝒔𝒊𝒏𝜽 𝟐
1 5.If Cos𝟐𝜽=0 , determine 𝒄𝒐𝒔𝜽 𝒔𝒊𝒏𝜽
(𝑘(4 − 2) − 2(2 − 3) + 1(4 − 12) = 4 𝟎
2 𝒔𝒊𝒏𝜽 𝟎 𝒄𝒐𝒔𝜽
(2𝑘 + 2 − 8) = 4 If Cos 2𝜃=0 ⟹ 2𝜃 = ⇒ 𝜃 =
|𝑘 − 3| = 4
If 𝜃 =
𝐼𝑓 𝑘 − 3 = 4 𝑘 = 7
𝐼𝑓 − 𝑘 + 3 = 4 𝑘 = −1 0
0 𝑐𝑜𝑠𝜃 𝑠𝑖𝑛𝜃 √ √

𝑐𝑜𝑠𝜃 𝑠𝑖𝑛𝜃 0 = 0
√ √
𝑠𝑖𝑛𝜃 0 𝑐𝑜𝑠𝜃
0
√ √
0
√ √ 0 1 1
0 ={ . . } 1 1 0
√ √ √ √ √
1 0 1
0
√ √

= {−1(1) + 1(−1)} = (4) =


𝐥𝐨𝐠 𝟑 𝟔𝟒 𝐥𝐨𝐠 𝟒 𝟑 𝐥𝐨𝐠 𝟐 𝟑 𝐥𝐨𝐠 𝟖 𝟑
6.Find the Product of
𝐥𝐨𝐠 𝟑 𝟖 𝐥𝐨𝐠 𝟒 𝟗 𝐥𝐨𝐠 𝟑 𝟒 𝐥𝐨𝐠 𝟑 𝟒
log 64 log 3 log 3 log 3
log 8 log 9 log 4 log 4
log 64 log 3 + log 3 log 4 log 64 log 3 + log 3 log 4
= log 8 log 3 + log 9 log 4 log 8 log 3 + log 9 log 4
log2 64 + log3 3 log8 64 + log 3
3
=
log2 8 + log3 9 log8 8 + log3 9
6 log2 2 + log3 3 2log8 8 + log 3
3
=
3 log2 2 + 2log3 3 log8 8 + 2 log3 3
6 +1 2+1
=
3+2 1+2
7 3
= = 21-15 = 6
5 3
___________________________________________________

.
𝐵𝐸⃗ + 𝐷𝐶⃗ = 𝑂𝐸⃗ − 𝑂𝐵⃗ + 𝑂𝐶⃗ − 𝑂𝐷⃗
PV MATRIC HR. SEC. SCHOOL =
⃗ ⃗ ⃗ ⃗
+ − 𝑏⃗ + 𝑐⃗ − ( + )

MATHEMATICS - XI STD ⃗ ⃗ ⃗ ⃗
= + − 𝑏⃗ + 𝑐⃗ − −

EXERCISE 8.1
⃗ ⃗ ⃗
= + − 𝑏⃗ + 𝑐⃗ − −
⃗ ⃗ ⃗ ⃗ ⃗ ⃗ ⃗ ⃗
01.Represent graphically the displacement of = − 𝑏⃗ + 𝑐⃗ − = =
(i) 45 cm 300 north of east. (ii) 80 km 600 south of west (⃗ ) ⃗
Solution : = ⟹ 𝑂𝐶⃗ − 𝑂𝐵⃗ = 𝐵𝐶⃗ Hence Proved
(i) N (ii) N 5.Prove that the line segment joining the midpoints of
two sides of a triangle is parallel to the third side whose
45 cm length is half of the length of the third side.
300 Solution : Let 𝑂𝐴⃗ = 𝑎⃗ ; 𝑂𝐵⃗ = 𝑏⃗ ; 𝑂𝐶⃗ = 𝑐⃗ A(𝑎⃗)
W E W E
600 D and E are the midpoints of the sides
AB and AC
80 km
By section formula,
𝑎⃗ + 𝑏⃗ 𝑎⃗ 𝑏⃗ D E
S S 𝑂𝐷⃗ = = +
2 2 2
02.Prove that the relation R defined on the set V of all vectors 𝑎⃗ + 𝑐⃗ 𝑎⃗ 𝑐⃗
by ′ 𝒂⃗ R 𝒃⃗ 𝒊𝒇 𝒂⃗ = 𝒃⃗ ′ is an equivalence relation on V. 𝑂𝐸⃗ = = +
2 2 2
B(𝑏⃗) C(𝑐⃗)
Solution :
To Prove : 𝐷𝐸⃗ = 𝐵𝐶⃗
Reflexive :
Every vectors is equal to itself. 𝑎⃗ 𝑐⃗ 𝑎⃗ 𝑏⃗ 𝑎⃗ 𝑐⃗ 𝑎⃗ 𝑏⃗
𝑎⃗ = 𝑎⃗ ; so that ′ 𝑎⃗ R 𝑎⃗ 𝑖𝑠 true. ∴ It is reflexive. 𝐷𝐸⃗ = 𝑂𝐸⃗ − 𝑂𝐷⃗ = + − + = + − −
2 2 2 2 2 2 2 2
Symmetric : ⃗ ⃗ ⃗ ⃗
If 𝑎⃗ R 𝑏⃗ , 𝑎⃗ = 𝑏⃗ ;
= − = 𝑂𝐶⃗ − 𝑂𝐵⃗ = 𝐵𝐶⃗
=
Also , 𝑏⃗ = 𝑎⃗ ; so that ′ ∴It is symmetric. 𝐷𝐸⃗ = 𝐵𝐶⃗ Hence Proved.
Transitive :
𝐴𝑙𝑠𝑜 𝐷𝐸⃗ = 𝐵𝐶⃗ ⟹ 𝐷𝐸 = 𝐵𝐶. ∴ 𝐷𝐸 ∥ 𝐵𝐶
If 𝑎⃗ R 𝑏⃗ , 𝑎⃗ = 𝑏⃗ ---------(1)
If 𝑏⃗ R 𝑐⃗ , 𝑏⃗ = 𝑐⃗ ---------(2)
From (1) and (2) 𝑎⃗ = 𝑐⃗ . 6.Prove that the line segments joining the midpoints of
So that 𝑎⃗ R 𝑐⃗ also true. ∴ It is Transitive. the adjacent sides of a quadrilateral form a
Set V satisfied all the three conditions. parallelogram. D(𝑑⃗)
∴ ′ 𝑎⃗ R 𝑏⃗ 𝑖𝑓 𝑎⃗ = 𝑏⃗ ′ is an equivalence relation on V Solution: R
3.Let 𝒂⃗ and 𝒃⃗ be the position vectors of the points A and B. Let 𝑂𝐴⃗ = 𝑎⃗ ; 𝑂𝐵⃗ = 𝑏⃗ ; C(𝑐⃗)
Prove that the position vectors of the points which trisects the 𝑂𝐶⃗ = 𝑐⃗ ; 𝑂𝐶⃗ = 𝑐⃗ . S
𝒂⃗ 𝟐𝒃⃗
line segment AB are 𝟑 and 𝟑 .
𝒃⃗ 𝟐𝒂⃗
P,Q,R and S are the mid points of Q
Solution: AB, BC, CD and DA respectively.
A(𝑎⃗) P Q B(𝑏⃗) By the section formula, A(𝑎⃗) P B(𝑏⃗ )
Let AP = PQ = QB = 𝜆(𝑠𝑎𝑦)
P divides AB in the ratio 1 : 2 𝑎⃗ + 𝑏 ⃗ 𝑎⃗ 𝑏 ⃗
𝑂𝑃⃗ = = +
2 2 2
Q divides AB in the ratio 2 : 1 𝑏⃗ + 𝑐⃗ 𝑏⃗ 𝑐⃗
. ⃗ . ⃗ 𝑂𝑄⃗ = = +
P.V of Q = 𝑂𝑄⃗ = 2 2 2
𝑐⃗ + 𝑑⃗ 𝑐⃗ 𝑑⃗
=
.⃗ .⃗

⃗ ⃗
o 𝑂𝑅⃗ = = +
2 2 2
𝑑⃗ + 𝑎⃗ 𝑑⃗ 𝑎⃗
. ⃗ . ⃗ 𝑂𝑆⃗ = = +
P.V of P = 𝑂𝑃⃗ = 2 2 2
.⃗ ⃗ 𝑏⃗ 𝑐⃗ 𝑎⃗ 𝑏⃗ 𝑏⃗ 𝑐⃗ 𝑎⃗ 𝑏⃗
=
.⃗


Hence Proved. 𝑃𝑄⃗ = 𝑂𝑄⃗ − 𝑂𝑃⃗ = + − + = + − −
2 2 2 2 2 2 2 2
𝑐⃗ 𝑎⃗ 𝑐⃗ − 𝑎⃗
4.If D and E are the midpoints of the sides AB and AC of = − = − − − − − − − (1)
2 2 2
𝟑
a triangle ABC, prove that 𝑩𝑬⃗ + 𝑫𝑪⃗ = 𝑩𝑪⃗ 𝑐⃗ 𝑑⃗ 𝑑⃗ 𝑎⃗ 𝑐⃗ 𝑑⃗ 𝑑⃗ 𝑎⃗
𝟐
A(𝑎 ⃗) 𝑆𝑅⃗ = 𝑂𝑅⃗ − 𝑂𝑆⃗ = + − + = + − −
Solution : Let 𝑂𝐴⃗ = 𝑎⃗ ; 𝑂𝐵⃗ = 𝑏⃗ ; 𝑂𝐶⃗ = 𝑐⃗ 2 2 2 2 2 2 2 2
D and E are the midpoints of the sides 𝑐⃗ 𝑎⃗ 𝑐⃗ − 𝑎⃗
= − = − − − − − − − (2)
AB and AC 2 2 2
By section formula, From (1) and (2)
D E
𝑎⃗ + 𝑏⃗ 𝑎⃗ 𝑏⃗ 𝑃𝑄⃗ = 𝑆𝑅⃗ ⟹ 𝑃𝑄 = 𝑆𝑅 𝐴𝑛𝑑 𝑃𝑄 ∥ 𝑆𝑅.
𝑂𝐷⃗ = = + Similarly we can prove
2 2 2
𝑎⃗ + 𝑐⃗ 𝑎⃗ 𝑐⃗ 𝑃𝑆⃗ = 𝑄𝑅⃗ ⟹ 𝑃𝑆 = 𝑄𝑅 𝐴𝑛𝑑 𝑃𝑆 ∥ 𝑄𝑅.
𝑂𝐸⃗ = = + ⃗
B(𝑏 ) C(𝑐⃗) ∴ PQRS is a parallelogram .
2 2 2
7.If 𝒂⃗ and 𝒃⃗ represent a side and a diagonal of a 11.Let A,B and C be the vertices of a triangle. Let D,E
parallelogram, find the other sides and the other and F be the mid points of the sides BC, CA and AB
diagonal. D −𝑎⃗ C respectively. Show that 𝑨𝑫⃗ + 𝑩𝑬⃗ + 𝑪𝑭⃗ = 𝟎⃗.
Solution : Solution:
A(𝑎⃗)
From given data Let 𝑂𝐴⃗ = 𝑎⃗ ; 𝑂𝐵⃗ = 𝑏⃗ ; 𝑂𝐶⃗ = 𝑐⃗
In the Parallelogram ABCD ⃗ Let D,E and F be the mid points
𝑏⃗𝑏
𝐴𝐵⃗ = 𝑎⃗ 𝑎𝑛𝑑 𝐴𝐶⃗ = 𝑏⃗ of the sides BC, CA and
Where 𝐴𝐵⃗ ∥ 𝐶𝐷⃗ but A 𝑎⃗ B AB respectively. F E
𝐴𝐵⃗ = −𝐶𝐷⃗ [∴ 𝐸𝑞𝑢𝑎𝑙 𝑙𝑒𝑛𝑔𝑡ℎ 𝑏𝑢𝑡 𝑜𝑝𝑝𝑜𝑠𝑖𝑡𝑒 𝑑𝑖𝑟𝑒𝑐𝑡𝑖𝑜𝑛 . 𝑏⃗ + 𝑐⃗ 𝑏⃗ 𝑐⃗
𝑂𝐷⃗ = = + G
𝑎⃗ = −𝐶𝐷⃗ ⟹ 𝐶𝐷⃗ = −𝑎⃗ 2 2 2
𝑎⃗ + 𝑐⃗ 𝑎⃗ 𝑐⃗
In ∆𝐴𝐵𝐶 𝐴𝐵⃗ + 𝐵𝐶⃗ = 𝐴𝐶⃗ 𝑂𝐸⃗ = = +
2 2 2 B(𝑏⃗) D C(𝑐⃗)
𝑎⃗ + 𝐵𝐶⃗ = 𝑏⃗ 𝑎⃗ + 𝑏⃗ 𝑎⃗ 𝑏⃗
𝐵𝐶⃗ = −𝑎⃗ + 𝑏⃗ 𝑂𝐹⃗ = = +
2 2 2
Where 𝐵𝐶⃗ ∥ 𝐴𝐷⃗ but L.H.S : 𝐴𝐷⃗ + 𝐵𝐸⃗ + 𝐶𝐹⃗ = 𝑂𝐷⃗ − 𝑂𝐴⃗ + 𝑂𝐸⃗ − 𝑂𝐵⃗ + 𝑂𝐹⃗ − 𝑂𝐶⃗
𝐵𝐶⃗ = −𝐴𝐷⃗ [∴ 𝐸𝑞𝑢𝑎𝑙 𝑙𝑒𝑛𝑔𝑡ℎ 𝑏𝑢𝑡 𝑜𝑝𝑝𝑜𝑠𝑖𝑡𝑒 𝑑𝑖𝑟𝑒𝑐𝑡𝑖𝑜𝑛 . ⃗ ⃗ ⃗ ⃗ ⃗ ⃗
= + − 𝑎⃗ + + − 𝑏⃗ + + − 𝑐⃗
−𝑎⃗ + 𝑏⃗ = −𝐴𝐷⃗ ⟹ 𝐶𝐷⃗ = 𝑎⃗ − 𝑏⃗
𝑏⃗ + 𝑐⃗ − 2𝑎⃗ + 𝑎⃗ + 𝑐⃗ − 2𝑏⃗ + 𝑎⃗ + 𝑏⃗ − 2𝑐⃗
In ∆𝐵𝐶𝐷 𝐵𝐷⃗ = 𝐵𝐶⃗ + 𝐶𝐷⃗ =
2
𝐵𝐷⃗ = −𝑎⃗ + 𝑏⃗ − 𝑎⃗ ⃗
= = 0⃗ R.H.S Hence Proved
𝐵𝐶⃗ = −2𝑎⃗ + 𝑏⃗
12.If ABCD is a quadrilateral and E and F are the mid
Other sides 𝑏⃗ − 𝑎⃗ , −𝑎⃗ , 𝑎⃗ − 𝑏⃗ and other diagonal 𝑏⃗ − 2𝑎⃗ points of AC and BD respectively, then Prove that
8.If 𝑷𝑶⃗ + 𝑶𝑸⃗ = 𝑸𝑶⃗ + 𝑶𝑹⃗ , Prove that the points P,Q,R 𝑨𝑩⃗ + 𝑨𝑫⃗ + 𝑪𝑩⃗ + 𝑪𝑫⃗ = 𝟒𝑬𝑭⃗
are collinear. P Q R Solution : C
Solution : In ∆ABD,
Given 𝑃𝑂⃗ + 𝑂𝑄⃗ = 𝑄𝑂⃗ + 𝑂𝑅⃗ F is the midpoint of BD.
By the triangular law of Addition, By the section formula, D F B
𝑃𝑄⃗ = 𝑄𝑅⃗ 𝐴𝐵⃗ + 𝐴𝐷⃗ E
⃗ ⃗
𝑃𝑄 ∥ 𝑄𝑅 𝑎𝑛𝑑 𝑄 𝑖𝑠 𝑎 𝑐𝑜𝑚𝑚𝑜𝑛 𝑝𝑜𝑖𝑛𝑡. 𝐴𝐹⃗ =
2
Hence P,Q,R lie on a line. O 𝐴𝐵⃗ + 𝐴𝐷⃗ = 2𝐴𝐹⃗ --------------(1)
⟹ 𝑃, 𝑄, 𝑅 𝑎𝑟𝑒 𝑐𝑜𝑙𝑙𝑖𝑛𝑒𝑎𝑟.
9.If D is the mid point of the side BC of a triangle ABC, A
In ∆BCD,
Prove that 𝑨𝑩⃗ + 𝑨𝑪⃗ = 𝟐𝑨𝑫⃗. A(𝑎⃗) F is the midpoint of BD.
Solution : Let 𝑂𝐴⃗ = 𝑎⃗ ; 𝑂𝐵⃗ = 𝑏⃗ ; 𝑂𝐶⃗ = 𝑐⃗ By the section formula,
D is the mid point of BC. 𝐶𝐵⃗ + 𝐶𝐷⃗
By the section formula, 𝐶𝐹⃗ =
2
𝑏⃗ + 𝑐⃗ 𝑏⃗ 𝑐⃗ 𝐶𝐵⃗ + 𝐶𝐷⃗ = 2𝐶𝐹⃗ --------------(2)
𝑂𝐷⃗ = = +
2 2 2
𝐴𝐵⃗ + 𝐴𝐶⃗ = 𝑂𝐵⃗ − 𝑂𝐴⃗ + 𝑂𝐶⃗ − 𝑂𝐴⃗ In ∆ACF,
= 𝑏⃗ − 𝑎⃗ + 𝑐⃗ − 𝑎⃗ E is the midpoint of AC.
B(𝑏⃗) D C(𝑐⃗) By the section formula,
= 𝑏⃗ + 𝑐⃗ − 2𝑎⃗ ---------(1)
2𝐴𝐷⃗ = 2 𝑂𝐷⃗ − 𝑂𝐴⃗ 𝐴𝐹⃗ + 𝐶𝐹⃗
𝐹𝐸⃗ =
⃗ ⃗ 2
= 2 + − 𝑎⃗ 𝐴𝐹⃗ + 𝐶𝐹⃗ = 2𝐸𝐹⃗ --------------(3)
= 𝑏⃗ + 𝑐⃗ − 2𝑎⃗ ---------(2) (1)+(2) is L.H.S
From (1) and (2) 𝐴𝐵⃗ + 𝐴𝐶⃗ = 2𝐴𝐷⃗ 𝐴𝐵⃗ + 𝐴𝐷⃗ + 𝐶𝐵⃗ + 𝐶𝐷⃗ = 2𝐴𝐹⃗ + 2𝐶𝐹⃗
10.If G is the centroid of a triangle ABC, Prove that = 2(𝐴𝐹⃗ + 𝐶𝐹⃗ )
𝑮𝑨⃗ + 𝑮𝑩⃗ + 𝑮𝑪⃗ = 𝟎⃗ = 2 2𝐸𝐹⃗ [By using (3)
Solution : Let 𝑂𝐴⃗ = 𝑎⃗ ; 𝑂𝐵⃗ = 𝑏⃗ ; 𝑂𝐶⃗ = 𝑐⃗ = 4𝐸𝐹⃗ R.H.S Hence Proved.
⃗ ⃗ ⃗
If G is the centroid of a triangle, then 𝑂𝐺⃗ =
L.H.S : 𝐺𝐴⃗ + 𝐺𝐵⃗ + 𝐺𝐶⃗ = 𝑂𝐴⃗ − 0𝐺⃗ + 𝑂𝐵⃗ − 𝑂𝐺⃗ + 𝑂𝐶⃗ − 𝑂𝐺⃗
= 𝑂𝐴⃗ + 𝑂𝐵⃗ + 𝑂𝐶⃗ − 3𝑂𝐺⃗
⃗ ⃗ ⃗
= 𝑎⃗ + 𝑏⃗ + 𝑐⃗ − 3
= 𝑎⃗ + 𝑏⃗ + 𝑐⃗ − (𝑎⃗ + 𝑏⃗ + 𝑐⃗)
= 0⃗ R.H.S
𝐺𝐴⃗ + 𝐺𝐵⃗ + 𝐺𝐶⃗ = 0⃗
Hence Proved
EXERCISE 8.2 (ii) 𝟑⃗ + ⃗ + 𝒌⃗
Direction ratios (3,1,1)

𝐼𝑓 𝑎⃗ = 𝑥𝚤⃗ + 𝑦𝚥⃗ + 𝑧𝑘⃗ r= 3 + (1) + 1 = √9 + 1 + 1 = √11


𝐷𝑖𝑟𝑒𝑟𝑐𝑡𝑖𝑜𝑛 𝑟𝑎𝑡𝑖𝑜 𝑜𝑓 𝑎⃗ 𝑖𝑠 (𝑥, 𝑦, 𝑧) Direction cosines , ,
√11 √11 √11
(iii) ⃗
Direction ratios (0,1,0)
|𝑎⃗| = 𝑟 = 𝑥 +𝑦 +𝑧
r= 0 + (1) + 0 = √0 + 1 + 0 = 1

𝐼𝑓 𝛼, 𝛽, 𝛾 𝑎𝑟𝑒 𝑡ℎ𝑒 𝑎𝑛𝑔𝑙𝑒 𝑜𝑓 𝑖𝑛𝑐𝑙𝑖𝑛𝑎𝑡𝑖𝑜𝑛 Direction cosines , , ⟹ (0,1,0)


1 1 1
𝑤𝑖𝑡ℎ 𝑐𝑜𝑜𝑟𝑑𝑖𝑛𝑎𝑡𝑒 𝑎𝑥𝑖𝑠, 𝑡ℎ𝑒𝑛 (iv) 𝟓⃗ − 𝟑⃗ − 𝟒𝟖𝒌⃗
Direction ratios (5,-3,-48)
𝐷𝑖𝑟𝑒𝑐𝑡𝑖𝑜𝑛 𝐶𝑜𝑠𝑖𝑛𝑒𝑠 𝑜𝑓 𝑎⃗ 𝑖𝑠
𝑥 𝑦 𝑧 r= 5 + (−3) + (−48) = √25 + 9 + 2304 = √2338
(cos 𝛼 , cos 𝛽 , 𝑐𝑜𝑠𝛾) (𝑜𝑟) , , Direction cosines , ,
𝑟 𝑟 𝑟 √2338 √2338 √2338
𝑥 𝑦 𝑧 (v) 𝟑⃗ − 𝟑𝒌⃗ + 𝟒⃗ ⟹ 𝟑⃗ + 𝟒𝒌⃗ − 𝟑𝒌⃗
𝑊ℎ𝑒𝑟𝑒 + + =1 Direction ratios (3,4,-3)
𝑟 𝑟 𝑟
cos 𝛼 + cos 𝛽 + cos 𝛾 = 1 r= 3 + (4) + (−3) = √9 + 16 + 9 = √34

sin 𝛼 + sin 𝛽 + sin 𝛾 = 2 Direction cosines , ,


√34 √34 √34
01.Verify whether the following ratios are direction (vi) ⃗ − 𝒌⃗
cosines of some vector or not. Direction ratios (1,0,-1)
𝟏 𝟑 𝟒
(i) , , r= 1 + (0) + (−1) = √1 + 1 = √2
𝟓 𝟓 𝟓
Sum of the squares of the direction cosines is 1. Direction cosines , , ⟹ , 0,
√2 √2 √2 √2 √2
1 3 4 1 9 16 26
+ + = + + = ≠1 4. A triangle is formed by joining the points (1, 0, 0),
5 5 5 25 25 25 25
Give ratio is not the direction cosines of a vector. (0, 1, 0) and (0, 0, 1). Find the direction cosines of the
(ii)
𝟏 𝟏 𝟏 medians.
, ,
√𝟐 𝟐 𝟐 A(𝑎⃗)(1,0,0)
Sum of the squares of the direction cosines is 1. Let 𝑂𝐴⃗ = 𝑎⃗ = 𝚤⃗;
𝟏 𝟏 𝟏 1 1 1 2+1+1 4 𝑂𝐵⃗ = 𝑏⃗ = 𝚥⃗ ;
+ + = + + = = =1 𝑂𝐶⃗ = 𝑐⃗ = 𝑘⃗ ;
𝟐 𝟐 2 4 4 4 4
√𝟐 Let D,E and F be the mid points F E
Give ratio is the direction cosines of a vector.
of the sides BC, CA and
𝟒
(ii) , 𝟎,
𝟑
AB respectively. G
𝟑 𝟒
Sum of the squares of the direction cosines is 1. ⃗
𝑏 + 𝑐⃗ 𝚥⃗ 𝑘 ⃗
𝑂𝐷⃗ = = + B(𝑏⃗)(0,1,0) D C(𝑐⃗)(0,0,1)
𝟒 𝟑 16 9 256 + 81 337 2 2 2
+ (𝟎) + = +0+ = = ≠1 𝑎⃗ + 𝑐⃗ 𝚤⃗ 𝑘⃗
𝟑 𝟒 9 16 144 144 𝑂𝐸⃗ = = +
Give ratio is not the direction cosines of a vector. 2 2 2
𝑎⃗ + 𝑏⃗ 𝚤⃗ 𝚥⃗
𝑂𝐹⃗ = = +
Find the direction cosines of a vector whose direction 2 2 2
⃗ ⃗ ⃗ ⃗
ratios are Median 𝐴𝐷⃗ = 𝑂𝐷⃗ − 𝑂𝐴⃗ = + − 𝚤⃗ = −𝚤⃗ + +
(i) 1,2,3 ⃗ ⃗ ⃗ ⃗
Median 𝐵𝐸⃗ = 𝑂𝐸⃗ − 𝑂𝐵⃗ = + − 𝚥⃗ = − 𝚥⃗ +
Where 𝑟 = √1 + 2 + 3 = √1 + 4 + 9 = √14
⃗ ⃗ ⃗ ⃗
Median 𝐶𝐹⃗ = 𝑂𝐹⃗ − 𝑂𝐶⃗ = + − 𝑘⃗ = + − 𝑘⃗
Direction Cosine is , ,
√14 √14 √14 ⃗ ⃗
(ii) 3,-1,3 Directions cosines of the median 𝐴𝐷⃗ = −𝚤⃗ + + is
Where 𝑟 = 3 + (−1) + 3 = √9 + 1 + 9 = √19 √
√6
, √6
, √6
= , , [Where 𝑟 = 1 + + =
√ √ √
Direction Cosine is , , 2 2 2
√19 √19 √19 ⃗ ⃗
(iii) 0,0,7 Directions cosines of the median 𝐵𝐸⃗ = − 𝚥⃗ + is
Where 𝑟 = √0 + 0 + 7 = √49 = 7 √
√6
, √6
, √6
= , , [Where 𝑟 = +1+ =
√ √ √
Direction Cosine is , , ⟹ (0,0,1) 2 2 2
7 7 7 ⃗ ⃗
Directions cosines of the median 𝐶𝐹⃗ = + − 𝑘⃗ is
Find the direction cosines and direction ratios for the
following vectors. , , = , , [Where 𝑟 = + +1=

√6 √6 √6
(i) 𝟑⃗ − 𝟒⃗ + 𝟖𝒌⃗ 2 2 2
√ √ √
Direction ratios (3,-4,8)
r= 3 + (−4) + 8 = √9 + 16 + 64 = √89
Direction cosines , ,
√89 √89 √89
𝟏
05.If ,
𝟏
, 𝒂 are the direction cosines of some vector, 9.Show that the following vectors are coplanar
𝟐 √𝟐
then fine a. (i) ⃗ − 𝟐⃗ + 𝟑𝒌⃗ , −𝟐⃗ + 𝟑⃗ − 𝟒𝒌⃗, −⃗ + 𝟐𝒌⃗
1 −2 3
Given , , 𝑎 are the direction cosines of some vector, −2 3 −4 = 1(6 − 4) + 2(−4 − 0) + 3((2 − 0)

then Sum of the squares of the direction cosines is 1. 0 −1 2
= 1(2) + 2(−4) + 3(2) = 2 − 8 + 6 = 0
1 1
+ + (𝑎) = 1 Determinant value is zero. Given vectors are coplanar.
2 √2 (ii) 𝟓⃗ + 𝟔⃗ + 𝟕𝒌⃗ , 𝟕⃗ − 𝟖⃗ + 𝟗𝒌⃗, 𝟑⃗ + 𝟐𝟎 ⃗ + 𝟓𝒌⃗
+ +𝑎 =1 5 6 7
7 −8 9 = 5(−40 − 180) + 6(35 − 27) + 7(140 + 24)
=1 3 20 5
= −1100 + 48 + 1148 = 0
1 + 2 + 4𝑎 = 4
Determinant value is zero. Given vectors are coplanar.
4𝑎 = 1
𝑎 = 10. Show that the points whose position vectors
𝑎=± 𝟒⃗ + 𝟓⃗ + 𝒌⃗ , −⃗ − ⃗ , 𝟑⃗ + 𝟗⃗ + 𝟒𝒌⃗ and −𝟒⃗ + 𝟒⃗ + 𝟒𝒌⃗ are
6. If (𝒂, 𝒂 + 𝒃, 𝒂 + 𝒃 + 𝒄) is one set of direction ratios of coplanar.
the line joining (1,0,0) and (0,1,0) , then find a set of 𝑂𝐴⃗ = 4𝚤⃗ + 5𝚥⃗ + 𝑘⃗
values of a, b, c. 𝑂𝐵⃗ = −𝚥⃗ − 𝑘⃗
Let A(1,0,0) and B(0,1,0) 𝑂𝐶⃗ = 3𝚤⃗ + 9𝚥⃗ + 4𝑘⃗
Now position vectors of A & B are 𝑂𝐴⃗ and 𝑂𝐵⃗ 𝑂𝐷⃗ = −4𝚤⃗ + 4𝚥⃗ + 4𝑘⃗
respectively. 𝐴𝐵⃗ = 𝑂𝐵⃗ − 𝑂𝐴⃗ = −4𝚤⃗ − 6𝚥⃗ − 2𝑘⃗
𝑂𝐴⃗ = 𝚤⃗ and 𝑂𝐵⃗ = 𝚥⃗ 𝐴𝐶⃗ = 𝑂𝐶⃗ − 𝑂𝐴⃗ = −𝚤⃗ + 4𝚥⃗ + 3𝑘⃗
𝐴𝐷⃗ = 𝑂𝐷⃗ − 𝑂𝐴⃗ = −8𝚤⃗ − 𝚥⃗ + 3𝑘⃗
𝐴𝐵⃗ = 𝑂𝐵⃗ − 𝑂𝐴⃗ = 𝚥⃗ − 𝚤⃗ = −𝚤⃗ + 𝚥⃗ −4 −6 −2
Direction ratios of 𝐴𝐵⃗ = −𝚤⃗ + 𝚥⃗ is (−1,1,0) −1 4 3 = −4(12 + 3) + 6(−3 + 24) − 2(1 + 32)
From given data, −8 −1 3
(𝒂, 𝒂 + 𝒃, 𝒂 + 𝒃 + 𝒄) = (−1,1,0) = −60 + 126 − 66 = 0
Then 𝑎 = −1 𝑎+𝑏= 1 𝑎+𝑏+𝑐 = 0 Determinant value is zero. Given points are coplanar.
−1 + 𝑏 = 1 𝑎+𝑏+𝑐 = 0 11.If 𝒂⃗ = 𝟐⃗ + 𝟑⃗ − 𝟒𝒌⃗, 𝒃⃗ = 𝟑⃗ − 𝟒⃗ − 𝟓𝒌⃗ , 𝒄⃗ = −𝟑⃗ + 𝟐⃗ + 𝟑𝒌⃗
Find the magnitude and direction cosine of the following ,
𝑏=2 −1+2+𝑐 = 0
(i) 𝒂⃗ + 𝒃⃗ + 𝒄⃗ (ii) 𝟑𝒂⃗ − 𝟐𝒃⃗ + 𝟓𝒄⃗
𝒄 = −1
(𝑎, 𝑏, 𝑐) ⟹ (−1,2, −1) (i) 𝑎⃗ + 𝑏⃗ + 𝑐⃗ = 2𝚤⃗ + 3𝚥⃗ − 4𝑘⃗ + 3𝚤⃗ − 4𝚥⃗ − 5𝑘⃗ − 3𝑖⃗ + 2𝑗⃗ + 3𝑘⃗
7. Show that the vectors 𝟐⃗ − ⃗ + 𝒌⃗, 𝟑⃗ − 𝟒⃗ − 𝟒𝒌⃗, 𝑎⃗ + 𝑏⃗ + 𝑐⃗ = 2𝚤⃗ + 𝚥⃗ − 6𝑘⃗
𝑎⃗ + 𝑏⃗ + 𝑐⃗ = √4 + 1 + 36 = √41
⃗ − 𝟑⃗ − 𝟓𝒌⃗ form a right angled triangle.
Solution : Direction cosine , ,
√ √ √
Let 𝑎⃗, 𝑏⃗ 𝑎𝑛𝑑 𝑐⃗ are the sides of the triangle ABC. (ii) 3𝑎⃗ − 2𝑏⃗ + 5𝑐⃗ = 6𝚤⃗ + 9𝚥⃗ − 12𝑘⃗ − 6𝚤⃗ + 8𝚥⃗ + 10𝑘⃗ − 15𝑖⃗ + 10𝑗⃗ + 15𝑘⃗
3𝑎⃗ − 2𝑏⃗ + 5𝑐⃗ = −15𝑖⃗ + 27𝑗⃗ + 13𝑘⃗
𝐴𝐵⃗ = 𝑎⃗ = 2𝚤⃗ − 𝚥⃗ + 𝑘⃗
3𝑎⃗ − 2𝑏⃗ + 5𝑐⃗ = √225 + 729 + 169 = √1123
𝐶𝐵⃗ = 𝑏⃗ = 3𝚤⃗ − 4𝚥⃗ − 4𝑘⃗
Direction cosine , ,
𝐶𝐴⃗ = 𝑐⃗ = 𝚤⃗ − 3𝚥⃗ − 5𝑘⃗ √ √ √
12. The position vectors of the vertices of a triangle are
Where 𝑎⃗ + 𝑐⃗ = 𝑏⃗ . 𝑎⃗, 𝑏⃗ 𝑎𝑛𝑑 𝑐⃗ are forms a triangle.
⃗ + 𝟐⃗ + 𝟑𝒌⃗, 𝟑⃗ − 𝟒⃗ + 𝟓𝒌⃗, −𝟐⃗ + 𝟑⃗ − 𝟕𝒌⃗. Find the
𝐴𝐵⃗ = 2 + (−1) + 1 = √4 + 1 + 1 = √6
perimeter of the triangle.
𝐵𝐶⃗ = 3 + (−4) + (−4) = √9 + 16 + 16 = √41
𝑂𝐴⃗ = ⃗ + 𝟐⃗ + 𝟑𝒌⃗
𝐶𝐴⃗ = 1 + (−3) + (−5) = √1 + 9 + 25 = √35
𝑂𝐵⃗ = 𝟑⃗ − 𝟒⃗ + 𝟓𝒌⃗
𝑊ℎ𝑒𝑟𝑒 𝐴𝐵⃗ + 𝐶𝐴⃗ = 6 + 35 = 41 = 𝐵𝐶⃗ 𝑂𝐶⃗ = −𝟐⃗ + 𝟑⃗ − 𝟕𝒌⃗
𝑎⃗, 𝑏⃗ 𝑎𝑛𝑑 𝑐⃗ are forms a right angled triangle ABC. 𝐴𝐵⃗ = 𝑂𝐵⃗ − 𝑂𝐴⃗ = 2𝚤⃗ − 6𝚥⃗ + 2𝑘⃗
8. Find the value of 𝝀 for which 𝒂⃗ = 𝟑⃗ + 𝟐⃗ + 𝟗𝒌⃗ and 𝐵𝐶⃗ = 𝑂𝐶⃗ − 𝑂𝐵⃗ = −5𝚤⃗ + 7𝚥⃗ − 12𝑘⃗
𝒃⃗ = ⃗ + 𝝀⃗ + 𝟑𝒌⃗ are parallel. 𝐶𝐴⃗ = 𝑂𝐴⃗ − 𝑂𝐶⃗ = 3𝚤⃗ − 𝚥⃗ + 10𝑘⃗
𝐼𝑓 𝑎⃗ 𝑎𝑛𝑑 𝑏⃗ are parallel, 𝑎⃗ = 𝑡 𝑏⃗ 𝐴𝐵⃗ = √4 + 36 + +4 = √44
3𝚤⃗ + 2𝚥⃗ + 9𝑘⃗ = 𝑡(𝚤⃗ + 𝜆𝚥⃗ + 3𝑘⃗ ) 𝐵𝐶⃗ = √25 + 49 + 144 = √218
3𝚤⃗ + 2𝚥⃗ + 9𝑘⃗ = 𝑡𝚤⃗ + t𝜆𝚥⃗ + 3𝑡𝑘⃗ 𝐶𝐴⃗ = √9 + 1 + 100 = √110
Equating coefficients, Perimeter = 𝐴𝐵⃗ + 𝐵𝐶⃗ + 𝐶𝐴⃗ = √44 + √218 + √110
𝑡 = 3 𝑎𝑙𝑠𝑜 t𝜆 = 2
13.Find the unit vector parallel to 𝟑𝒂⃗ − 𝟐𝒃⃗ + 𝟒𝒄⃗
3𝜆 = 2
If 𝒂⃗ = 𝟑⃗ − ⃗ − 𝟒𝒌⃗, 𝒃⃗ = −𝟐⃗ + 𝟒⃗ − 𝟑𝒌⃗ , 𝒄⃗ = ⃗ + 𝟐⃗ − 𝒌⃗
𝜆=
3𝑎⃗ − 2𝑏⃗ + 4𝑐⃗ = 9𝚤⃗ − 3𝚥⃗ − 12𝑘⃗ + 4𝚤⃗ − 8𝚥⃗ + 6𝑘⃗ + 4⃗𝑖 + 8𝑗⃗ − 4𝑘⃗
3𝑎⃗ − 2𝑏⃗ + 4𝑐⃗ = 17𝚤⃗ − 3𝚥⃗ − 10𝑘⃗
unit vector parallel to 3𝑎⃗ − 2𝑏⃗ + 4𝑐⃗ =
⃗ ⃗ ⃗ (i) 𝑖𝑓 𝑎⃗ 𝑎𝑛𝑑 𝑏⃗ 𝑎𝑟𝑒 𝑝𝑒𝑟𝑝𝑒𝑛𝑑𝑖𝑐𝑢𝑙𝑎𝑟 𝑎⃗. 𝑏⃗ = 0
⃗ ⃗ ⃗
⃗ ⃗ ⃗ ⃗ ⃗ ⃗ (2𝚤⃗ + 𝜆𝚥⃗ + 𝑘⃗ ) . 𝚤⃗ − 2𝚥⃗ + 3𝑘⃗ = 0
= = 2-2 𝜆+3 = 0
√ √
14.The position vectors 𝒂⃗ , 𝒃⃗, 𝒄⃗ of three points satisfy the -2 𝜆 = -5
relation 𝟐𝒂⃗ − 𝟕𝒃⃗ + 𝟓𝒄⃗ = 𝟎⃗.Are these points collinear? 𝜆=
Solution : 𝟐𝒂⃗ − 𝟕𝒃⃗ + 𝟓𝒄⃗ = 𝟎⃗
(ii) 𝑖𝑓 𝑎⃗ 𝑎𝑛𝑑 𝑏⃗ 𝑎𝑟𝑒 𝑝𝑒𝑟𝑝𝑒𝑛𝑑𝑖𝑐𝑢𝑙𝑎𝑟 𝑎⃗. 𝑏⃗ = 0
2𝑎⃗ = 7𝑏⃗ − 5𝑐⃗
(2𝚤⃗ + 4𝚥⃗ − 𝑘⃗ ) . 3 𝚤⃗ − 2𝚥⃗ + 𝜆𝑘⃗ = 0
𝑎⃗ = 𝑏⃗ − 𝑐⃗
6-8+ 𝜆 = 0
Where 𝑎⃗ is a linear combination of other two vectors.
-2+ 𝜆 = 0
∴vectors 𝑎⃗ , 𝑏⃗ , 𝑐⃗ are collinear.
𝜆=2
15. The position Vectors P,Q,R,S are ⃗ + ⃗ + 𝒌⃗ , 𝟐⃗ + 𝟓⃗ ,
03. If 𝒂⃗ and 𝒃⃗ are two vectors such that |𝒂⃗| = 𝟏𝟎, that
𝟑⃗ + 𝟐⃗ − 𝟑𝒌⃗ and ⃗ − 𝟔⃗ − 𝒌⃗ respectively . Prove that the
𝒃⃗ = 𝟏𝟓 , and 𝒂⃗ . 𝒃⃗ = 75√𝟐 find the angle between if
lines PQ and RS are parallel.
𝒂⃗ and 𝒃⃗.
𝑂𝑃⃗ = 𝚤⃗ + 𝚥⃗ + 𝑘⃗
⃗. ⃗
𝑂𝑄⃗ = 2𝚤⃗ + 5𝚥⃗ The angle between if 𝑎⃗ and 𝑏⃗ ′𝜃′ = cos |𝒂⃗| 𝒃⃗
𝑂𝑅⃗ = 3𝚤⃗ + 2𝚥⃗ − 3𝑘⃗ 𝟕𝟓√𝟐
𝑂𝑆⃗ = 𝚤⃗ − 6𝚥⃗ − 𝑘⃗ 𝜃 = cos
10 𝑋 15
𝑃𝑄⃗ = 𝑂𝑄⃗ − 𝑂𝑃⃗ = 𝚤⃗ + 4𝚥⃗ − 𝑘⃗ 𝟏
𝜃 = cos
𝑅𝑆⃗ = 𝑂𝑆⃗ − 𝑂𝑅⃗ = −𝟐𝚤⃗ − 8𝚥⃗ + 2𝑘⃗ √𝟐
𝜋
= -2(𝚤⃗ + 4𝚥⃗ − 𝑘⃗ ) 𝜃=
𝑅𝑆⃗ = −2𝑃𝑄⃗ 4
𝟒.Find the angle between the vectors
∴ the lines PQ and RS are parallel.
(i) 2⃗ + 𝟑⃗ − 𝟔𝒌⃗ 𝒂𝒏𝒅 𝟔⃗ − 𝟑⃗ + 𝟐𝒌⃗
16.Find the value or values of m for which
Let 𝑎⃗ =2𝚤⃗ + 3𝚥⃗ − 6𝑘⃗ 𝑎𝑛𝑑 𝑏⃗ = 6𝚤⃗ − 3𝚥⃗ + 2𝑘⃗
m (⃗ + ⃗ + 𝒌⃗)is a unit vector. ⃗. ⃗
m 𝚤⃗ + 𝚥⃗ + 𝑘⃗ = 𝑚𝚤⃗ + 𝑚𝚥⃗ + 𝑚𝑘⃗ The angle between if 𝑎⃗ and 𝑏⃗ ′𝜃′ = cos |𝒂⃗| 𝒃⃗

𝑚𝚤⃗ + 𝑚𝚥⃗ + 𝑚𝑘 𝑖𝑠 𝑎 𝑢𝑛𝑖𝑡 𝑣𝑒𝑐𝑡𝑜𝑟. 2𝚤⃗ + 3𝚥⃗ − 6𝑘⃗ . (6𝚤⃗ − 3𝚥⃗ + 2𝑘⃗ )
𝜃 = cos
∴ 𝑚 +𝑚 +𝑚 =1 √4 + 9 + 36 √36 + 9 + 4
3𝑚 = 1 12 − 9 − 12
𝑚 = 𝜃 = cos
7.7
m= ± 𝜃 = cos

17.Show that the points A (1, 1, 1), B(1, 2, 3) and (ii) ⃗ − ⃗ 𝒂𝒏𝒅 ⃗ − 𝒌⃗
C(2, - 1, 1) are vertices of an isosceles triangle. Let 𝑎⃗ = 𝚤⃗ − 𝚥⃗ 𝑎𝑛𝑑 𝑏⃗ = 𝚥⃗ − 𝑘⃗
𝑂𝐴⃗ = 𝚤⃗ + 𝚥⃗ + 𝑘⃗ The angle between if 𝑎⃗ and 𝑏⃗ ′𝜃′ = cos
⃗. ⃗
|𝒂⃗| 𝒃⃗
𝑂𝐵⃗ = 𝚤⃗ + 2𝚥⃗ + 3𝚥⃗
𝑂𝐶⃗ = 2𝚤⃗ − 𝚥⃗ + 𝑘⃗ (𝚤⃗ − 𝚥⃗). (𝚥⃗ − 𝑘⃗ )
𝜃 = cos
𝐴𝐵⃗ = 𝑂𝐵⃗ − 𝑂𝐴⃗ = 𝚥⃗ + 2𝑘⃗ √1 + 1 √1 + 1
−1
𝐵𝐶⃗ = 𝑂𝐶⃗ − 𝑂𝐵⃗ = 𝚤⃗ − 3𝚥⃗ − 2𝑘⃗ 𝜃 = cos
2
𝐶𝐴⃗ = 𝑂𝐴⃗ − 𝑂𝐶⃗ = −𝚤⃗ + 2𝚥⃗
𝜃=
𝐴𝐵⃗ = √1 + 4 = √5
𝐵𝐶⃗ = √1 + 9 + 4 = √14 5.If 𝒂⃗, 𝒃, 𝒄⃗ are three vectors such that 𝒂⃗ + 𝟐𝒃⃗ + 𝒄⃗ = 𝒐⃗ and

|𝒂⃗| = 𝟑 , 𝒃⃗ = 𝟒 , |𝒄⃗| = 𝟕 find the angle between 𝒂⃗ & 𝒃⃗.
𝐶𝐴⃗ = √1 + 4 = √5
Where AB=CA . Given points are the vertices of an Given 𝑎⃗ + 2𝑏⃗ + 𝑐⃗ = 𝑜⃗
isosceles triangle. 𝑎⃗ + 2𝑏⃗ = −𝑐⃗
𝑎⃗ + 2𝑏⃗ = |−𝑐⃗|
EXERCISE 8.3 Squaring on both sides,
1.Find 𝒂⃗. 𝒃⃗ 𝒘𝒉𝒆𝒏
𝑎⃗ + 2𝑏⃗ = |−𝑐⃗|
(i) 𝒂⃗ = ⃗ − 𝟐⃗ + 𝒌⃗ 𝒂𝒏𝒅 𝒃⃗ = 𝟑⃗ − 𝟒⃗ − 𝟐𝒌⃗
|𝑎⃗| + 2(𝑎⃗. 2𝑏⃗ ) + 2𝑏⃗ = |𝑐⃗|
𝑎⃗. 𝑏⃗ =(𝚤⃗ − 2𝚥⃗ + 𝑘⃗ ).( 3𝚤⃗ − 4𝚥⃗ − 2𝑘⃗ ) = 3+8-2 = 9
|𝑎⃗| + 2.2𝑎⃗. 𝑏⃗ + 4 𝑏⃗ = |𝑐⃗|
(ii) 𝒂⃗ = 𝟐⃗ + 𝟐⃗ − 𝒌⃗ 𝒂𝒏𝒅 𝒃⃗ = 𝟔⃗ − 𝟑⃗ + 𝟐𝒌⃗
𝑎⃗. 𝑏⃗ =(2𝚤⃗ + 2𝚥⃗ − 𝑘⃗ ).( 6𝚤⃗ − 3𝚥⃗ + 2𝑘⃗ ) = 12-6-2 = 4 |𝑎⃗| + 4|𝑎⃗| 𝑏⃗ cos 𝜃 + 4 𝑏⃗ = |𝑐⃗|
9 + 4(3)(4) cos 𝜃 + 4(16) = 49
48 cos 𝜃 = −24
2. Find the value of 𝝀 for which the vectors 𝒂⃗ 𝒂𝒏𝒅 𝒃⃗ are −1
𝜃 = cos
perpendicular(i) 𝒂⃗ = 𝟐⃗ + 𝝀⃗ + 𝒌⃗ 𝒂𝒏𝒅 𝒃⃗ = ⃗ − 𝟐⃗ + 𝟑𝒌⃗ 2
(ii) 𝒂⃗ = 𝟐⃗ + 𝟒⃗ − 𝒌⃗ 𝒂𝒏𝒅 𝒃⃗ = 𝟑⃗ − 𝟐⃗ + 𝝀𝒌⃗ 𝜃=
6.Show that the vectors 𝒂⃗ = 𝟐⃗ + 𝟑⃗ + 𝟔𝒌⃗, 𝒃⃗ = 𝟔⃗ + 𝟐⃗ − 𝟑𝒌⃗ , 𝜃
𝑎⃗ + 𝑏⃗ = 4 cos
𝒄⃗ = 𝟑⃗ − 𝟔⃗ + 𝟐𝒌⃗ are mutually orthogonal. 2
Solution : 𝜃

𝑎⃗ + 𝑏 = 2𝑐𝑜𝑠
𝑎⃗. 𝑏⃗ = 2𝚤⃗ + 3𝚥⃗ + 6𝑘⃗ . 6𝚤⃗ + 2𝚥⃗ − 3𝑘⃗ = 12+6-18=0 2
𝑏⃗. 𝑐⃗ = 6𝚤⃗ + 2𝚥⃗ − 3𝑘⃗ . 3𝚤⃗ − 6𝚥⃗ + 2𝑘⃗ = 18-12-6=0 𝑐𝑜𝑠 = 𝑎⃗ + 𝑏⃗ ----------(2)
𝑐⃗. 𝑎⃗ = 3𝚤⃗ − 6𝚥⃗ + 2𝑘⃗ . 2𝚤⃗ + 3𝚥⃗ + 6𝑘⃗ = 6-18+12=0 (𝑖𝑖𝑖) Dividing (1) By (2)
𝑎⃗ , 𝑏⃗ , 𝑐⃗ are mutually perpendicular vectors. ( )
𝜃
𝑠𝑖𝑛2
1
𝑎⃗−𝑏⃗
2
7.Show that the vectors −⃗ − 𝟐⃗ − 𝟔𝒌⃗, 𝟐⃗ − ⃗ + 𝒌⃗ ,and ⟹ 𝜃 = 1
( ) 𝑐𝑜𝑠2 2
𝑎⃗+𝑏⃗
−⃗ + 𝟑⃗ + 𝟓𝒌⃗ forms a right angle triangle.
𝜃 𝑎⃗ − 𝑏⃗
Let 𝑎⃗ = −𝚤⃗ − 2𝚥⃗ − 6𝑘⃗ , 𝑏⃗ = 2𝚤⃗ − 𝚥⃗ + 𝑘⃗ 𝑎𝑛𝑑 𝑐⃗ = −𝚤⃗ + 3𝚥⃗ + 5𝑘⃗ 𝑡𝑎𝑛 =
Where 𝑎⃗ + 𝑏⃗ + 𝑐⃗ = 0⃗ . ∴ 𝑎⃗ , 𝑏⃗ 𝑎𝑛𝑑 𝑐⃗ are the sides of a triangle.
2 𝑎⃗ + 𝑏⃗
Also 𝑏⃗. 𝑐⃗ = 2𝚤⃗ − 𝚥⃗ + 𝑘⃗ . −𝚤⃗ + 3𝚥⃗ + 5𝑘⃗ = −2 − 3 + 5 = 0 11. Let 𝒂⃗ , 𝒃⃗, 𝒄⃗ be three vectors such that |𝒂⃗| = 𝟑, 𝒃⃗ = 𝟒 ,
So that 𝑏⃗ ⊥ 𝑐⃗ . ∴ 𝑎⃗ , 𝑏⃗ 𝑎𝑛𝑑 𝑐⃗ are the sides of a right angled triangle. |𝒄⃗| = 𝟓 and each one of them being perpendicular to
8. If |𝒂⃗| = 𝟓 , 𝒃⃗ = 𝟔 , |𝒄⃗| = 𝟕 and 𝒂⃗ + 𝒃⃗ + 𝒄⃗ = 𝟎⃗ find the sum of the other two , find 𝒂⃗ + 𝒃⃗ + 𝒄⃗ .
𝒂⃗. 𝒃⃗ + 𝒃⃗. 𝒄⃗ + 𝒄⃗. 𝒂⃗ Given that each vector is perpendicular to sum of other
𝑎⃗ + 𝑏⃗ + 𝑐⃗ = 0⃗ two vectors.
𝑎⃗ + 𝑏⃗ + 𝑐⃗ = 0⃗ 𝑎⃗. 𝑏⃗ + 𝑐⃗ = 0 ⟹ 𝑎⃗. 𝑏⃗ + 𝑎⃗. 𝑐⃗ = 0 − − − −(1)
Squaring on both sides 𝑏⃗ . (𝑐⃗ + 𝑎⃗) = 0 ⟹ 𝑏⃗ . 𝑐⃗ + 𝑏⃗ . 𝑎⃗ = 0 − − − −(2)
𝑎⃗ + 𝑏⃗ + 𝑐⃗ = 0 𝑐⃗. 𝑎⃗ + 𝑏⃗ = 0 ⟹ 𝑐⃗. 𝑎⃗ + 𝑐⃗. 𝑏⃗ = 0 − − − −(3)
(1)+(2)+(3)
|𝑎⃗| + 𝑏⃗ + |𝑐⃗| + 2 𝑎⃗. 𝑏⃗ + 𝑏⃗. 𝑐⃗ + 𝑐⃗. 𝑎⃗ = 0
𝑎⃗. 𝑏⃗ + 𝑎⃗. 𝑐⃗ + 𝑏⃗ . 𝑐⃗ + 𝑏⃗ . 𝑎⃗ + 𝑐⃗. 𝑎⃗ + 𝑐⃗. 𝑏⃗ = 0
25 + 36 + 49 +2 𝑎⃗. 𝑏⃗ + 𝑏⃗. 𝑐⃗ + 𝑐⃗. 𝑎⃗ = 0 2𝑎⃗. 𝑏⃗ + 2𝑏⃗ . 𝑐⃗ + 2𝑐⃗. 𝑎⃗ = 0
2 𝑎⃗. 𝑏⃗ + 𝑏⃗. 𝑐⃗ + 𝑐⃗. 𝑎⃗ = −110 2 𝑎⃗. 𝑏⃗ + 𝑏⃗ . 𝑐⃗ + 𝑐⃗. 𝑎⃗ = 0 ----(4)
𝑎⃗. 𝑏⃗ + 𝑏⃗ . 𝑐⃗ + 𝑐⃗. 𝑎⃗ = −55
𝑎⃗ + 𝑏⃗ + 𝑐⃗ = |𝑎⃗| + 𝑏⃗ + |𝑐⃗| + 2 𝑎⃗. 𝑏⃗ + 𝑏⃗. 𝑐⃗ + 𝑐⃗. 𝑎⃗
9.Show that the points (2, - 1, 3), (4, 3, 1) and (3, 1, 2) are
= 3 +4 +5 +0 [From (4)
collinear.
𝑂𝐴⃗ = 2𝚤⃗ − 𝚥⃗ + 3𝑘⃗ 𝑂𝐵⃗ = 4𝚤⃗ + 3𝚥⃗ + 𝑘⃗ 𝑂𝐶⃗ = 3𝚤⃗ + 𝚥⃗ + 2𝑘⃗ 𝑎⃗ + 𝑏⃗ + 𝑐⃗ = 50
𝐴𝐵⃗ = 𝑂𝐵⃗ − 𝑂𝐴⃗ = 2𝚤⃗ + 4𝚥⃗ − 2𝑘⃗ 𝑎⃗ + 𝑏⃗ + 𝑐⃗ = 5√2
𝐵𝐶⃗ = 𝑂𝐶⃗ − 𝑂𝐵⃗ = 𝚤⃗ − 2𝚥⃗ + 𝑘⃗ 12.Find the projection of the vector ̂ + 𝟑 ̂ + 𝟕𝒌 on the
𝐶𝐴⃗ = 𝑂𝐴⃗ − 𝑂𝐶⃗ = −𝚤⃗ − 2𝚥⃗ + 𝑘⃗ vector 2 ̂ + 𝟔 ̂ + 𝟑𝒌.
Where 𝐴𝐵⃗ = −2𝐵𝐶⃗ ∴ 𝐴𝐵⃗ ∥ 𝐵𝐶⃗ Let 𝑎⃗ = 2 𝚤̂ + 6𝚥̂ + 3𝑘 and 𝑏⃗ = 𝚤̂ + 3𝚥̂ + 7𝑘
Also A is the common Point ∴ 𝐴, 𝐵, 𝐶 𝑎𝑟𝑒 𝑐𝑜𝑙𝑙𝑖𝑛𝑒𝑎𝑟. ⃗.⃗ ( ) ( ) ( )
Projection of 𝑏⃗ on 𝑎⃗ = = =
10. If 𝒂⃗, 𝒃⃗ are unit vectors and 𝜽 is the angle between | ⃗| √
𝜽 𝟏
them, show that (𝒊)𝒔𝒊𝒏 = 𝒂⃗ − 𝒃⃗ (𝒊𝒊)𝒄𝒐𝒔 = 𝒂⃗ + 𝒃⃗
𝜽 𝟏 13. Find𝝀, when the projection 𝒂⃗ = 𝝀 ̂ + ̂ + 𝟒𝒌 on
𝟐 𝟐 𝟐 𝟐
𝜽 𝒂⃗ 𝒃⃗
𝒃⃗ = 𝟐 ̂ + 𝟔 ̂ + 𝟑𝒌 is 4 units.
(𝒊𝒊𝒊)𝒕𝒂𝒏 = ⃗.⃗ 𝝀( ) ( ) ( )
𝟐 𝒂⃗ 𝒃⃗ Projection of 𝑎⃗ on 𝑏⃗ = ⃗ = = 4 (𝑔𝑖𝑣𝑒𝑛)

Given that 𝑎⃗, 𝑏⃗ are the unit vectors.i.e., |𝑎⃗| = 1; 𝑏⃗ = 1 2𝜆 + 18
=4⟹ 𝜆=5
(i) 𝑎⃗ − 𝑏⃗ = |𝑎⃗| + 𝑏⃗ − 2 𝑎⃗ . 𝑏⃗ 7
= |𝑎⃗| + 𝑏⃗ − 2|𝑎⃗| 𝑏⃗ 𝑐𝑜𝑠𝜃 14. 𝑻𝒉𝒓𝒆𝒆 𝒗𝒆𝒄𝒕𝒐𝒓𝒔 𝒂⃗, 𝒃, 𝒄⃗ are such that |𝒂⃗| = 𝟐, 𝒃⃗ = 𝟑,

= 1 + 1 − 2(1)(1) 𝑐𝑜𝑠𝜃 |𝒄⃗| = 𝟒 and 𝒂⃗ + 𝒃⃗ + 𝒄⃗ = 𝟎⃗ find 4𝒂⃗. 𝒃⃗ + 3𝒃⃗. 𝒄⃗ + 𝟑𝒄⃗. 𝒂⃗.

𝑎⃗ − 𝑏 = 2 − 2 𝑐𝑜𝑠𝜃 Given 𝑎⃗ + 𝑏⃗ + 𝑐⃗ = 0⃗
𝑎⃗ + 𝑏⃗ = −𝑐⃗
𝑎⃗ − 𝑏⃗ = 2(1 − 𝑐𝑜𝑠𝜃)
Multiply by 𝑐⃗
𝜃
𝑎⃗ − 𝑏⃗ = 2(2 sin ) 𝑎⃗. 𝑐⃗ + 𝑏⃗ . 𝑐⃗ = −𝑐⃗. 𝑐⃗
2
𝜃 𝑎⃗. 𝑐⃗ + 𝑏⃗ . 𝑐⃗ = −|𝑐⃗| = −4 = −16
𝑎⃗ − 𝑏⃗ = 4 sin
2 𝑎⃗. 𝑐⃗ + 𝑏⃗ . 𝑐⃗ = −16 ----------------(1)
𝜃

𝑎⃗ − 𝑏 = 2𝑠𝑖𝑛 Given 𝑎⃗ + 𝑏⃗ + 𝑐⃗ = 0⃗
2 𝑎⃗ + 𝑏⃗ + 𝑐⃗ = 0⃗
𝑠𝑖𝑛 = 𝑎⃗ − 𝑏⃗ ---------(1)
Squaring on both sides 𝑎⃗ + 𝑏⃗ + 𝑐⃗ =0
(𝑖𝑖) |𝑎⃗ + 𝑏| = |𝑎⃗| + 𝑏⃗ + 2 𝑎⃗ . 𝑏⃗
|𝑎⃗| + 𝑏⃗ + |𝑐⃗| + 2 𝑎⃗. 𝑏⃗ + 𝑏⃗. 𝑐⃗ + 𝑐⃗. 𝑎⃗ = 0
= |𝑎⃗| + 𝑏⃗ + 2|𝑎⃗| 𝑏⃗ 𝑐𝑜𝑠𝜃
2 + 3 + 4 + 2 𝑎⃗. 𝑏⃗ + 𝑏⃗. 𝑐⃗ + 𝑐⃗. 𝑎⃗ = 0
= 1 + 1 + 2(1)(1) 𝑐𝑜𝑠𝜃
2𝑎⃗. 𝑏⃗ + 2𝑏⃗ . 𝑐⃗ + 2𝑐⃗. 𝑎⃗ = −29
𝑎⃗ + 𝑏⃗ = 2 + 2 𝑐𝑜𝑠𝜃
4𝑎⃗. 𝑏 + 4𝑏⃗ . 𝑐⃗ + 4𝑐⃗. 𝑎⃗ = −58 ------------(2)

𝑎⃗ + 𝑏⃗ = 2(1 + 𝑐𝑜𝑠𝜃) (2)-(1) 4𝑎⃗. 𝑏⃗ + 4𝑏⃗. 𝑐⃗ + 4𝑐⃗. 𝑎⃗-𝑎⃗. 𝑐⃗ − 𝑏⃗ . 𝑐⃗ =-58+16
𝜃
𝑎⃗ + 𝑏⃗ = 2(2 𝑐𝑜𝑠 ) 4𝒂⃗. 𝒃⃗ + 3𝒃⃗. 𝒄⃗ + 𝟑𝒄⃗. 𝒂⃗ = -42
2
PV MATRIC HR. SEC. SCHOOL 3.(i) 𝟏𝟐𝟑 ∫ 12 𝑑𝑥 =12 ∫ 𝑑𝑥 = 12 𝑥 + 𝑐
𝒙𝟐𝟒 𝒙𝟐𝟒
MATHEMATICS - XI STD (ii)
𝒙𝟐𝟓
∫ 𝒙𝟐𝟓 𝑑𝑥 =∫ 𝑑𝑥 = log |𝑥| + 𝑐
IMPORTANT FORMULAS (iii) 𝒆 𝒙
∫ 𝑒 𝑑𝑥 = 𝑒 + 𝑐
0 𝑑𝑥 = 𝑐 𝑑𝑥 = 𝑥 + 𝑐 4. (𝒊)(𝟏 + 𝒙𝟐 ) 𝟏
∫(1 + 𝑥 ) 𝑑𝑥 = ∫ 𝑑𝑥
= tan 𝑥 + 𝑐
𝑥 1
𝑥 𝑑𝑥 = +𝑐 𝑑𝑥 = log |𝑥| + 𝑐 (𝒊𝒊)(𝟏 − 𝒙𝟐 ) 𝟏/𝟐
∫(1 − 𝑥 ) / 𝑑𝑥 = ∫ ( 𝑑𝑥
𝑛+1 𝑥 ) /
=∫ 𝑑𝑥
sin 𝑥 𝑑𝑥 = − cos 𝑥 + 𝑐 cos 𝑥 𝑑𝑥 = sin 𝑥 + 𝑐
= sin 𝑥+𝑐
sec 𝑥 𝑑𝑥 = tan 𝑥 + 𝑐 cosec 𝑥 𝑑𝑥 = −cot 𝑥 + 𝑐
𝑎
Integrals of the form ∫ 𝒇(𝒂𝒙 + 𝒃)𝒅𝒙
𝑒 𝑑𝑥 = 𝑒 + 𝑐 𝑎 𝑑𝑥 = +𝑐
log 𝑎 𝒊𝒇 𝒇(𝒙)𝒅𝒙 = 𝒈(𝒙) + 𝒄
1 1
𝑑𝑥 = sin 𝑥+𝑐 𝑑𝑥 = tan 𝑥+𝑐 𝟏
√1 − 𝑥 1+𝑥 𝒇(𝒂𝒙 + 𝒃)𝒅𝒙 = 𝒈(𝒂𝒙 + 𝒃) + 𝒄
𝒂
sec 𝑥 𝑡𝑎𝑛𝑥 𝑑𝑥 = sec 𝑥 + 𝑐
EXERCISE 11.2
cosec 𝑥 𝑐𝑜𝑡 𝑥 𝑑𝑥 = − cosec 𝑥 + 𝑐 Integrate the following with respect to x :
1.(i) (𝒙 + 𝟓)𝟔
EXERCISE 11.1 ( )
We know that ∫(𝑎𝑥 + 𝑏) 𝑑𝑥 = +𝑐
Integrate the following with respect to x :
( )
1.(i)𝒙𝟏𝟏 We know that ∫ 𝑥 𝑑𝑥 = +𝑐 ∫(𝑥 + 5) 𝑑𝑥 = +𝑐
7
(𝑥 + 5)
∫𝑥 𝑑𝑥 = +𝑐 =
7
+𝑐
𝟏 ( )
= +𝑐 (ii)
(𝟐 𝟑𝒙)𝟒
∫(−3𝑥 + 2) 𝑑𝑥 = +𝑐
−4+1
(ii)
𝟏
∫ 𝑑𝑥 = ∫ 𝑥 𝑑𝑥 = +𝑐 1 (−3𝑥 + 2)
𝒙𝟕
+𝑐
3 −4 + 1
= +𝑐 / ( )
(iii) √𝟑𝒙 + 𝟐 ∫(3𝑥 + 2) 𝑑𝑥 = +𝑐
= +𝑐 3
1 (3𝑥 + 2)2
𝟑 = +𝑐
(iii) √𝒙𝟒 ∫ √𝑥 𝑑𝑥 = ∫(𝑥 ) 𝑑𝑥 = +𝑐 3 3
2
3/2
2(3𝑥 + 2)
= +𝑐
= +𝑐 9
2.(i) sin 3x
= 𝑥 +𝑐 ∫ 𝑠𝑖𝑛 3𝑥 𝑑𝑥 = 𝑐𝑜𝑠 3𝑥 + 𝑐 ⟹ +c
𝟏 (ii) cos(5-11x)
(iv) 𝒙𝟓 𝟖
∫(𝑥 ) 𝑑𝑥 = +𝑐 ∫ cos(5 − 11𝑥) 𝑑𝑥 = − sin(5 − 11𝑥) + 𝑐 +
(iii) 𝐜𝐨𝐬𝐞𝐜 (𝟓𝒙 − 𝟕) 𝟐

1
= +𝑐 cosec2 (5𝑥 − 7) 𝑑𝑥 = cot(5𝑥 − 7) + 𝑐
5
1 3𝑥−6
= 𝑥 +𝑐 3.(i) 𝒆𝟑𝒙 𝟔
∫ 𝑒3𝑥−6 𝑑𝑥 =3
𝑒 +𝑐
−1
𝟏
(ii) 𝒆𝟖 𝟕𝒙
∫ 𝑒8−7𝑥 𝑑𝑥 = 7 𝑒8−7𝑥 + 𝑐
2.(i) ∫ 𝑑𝑥 = ∫ cosec 𝑥 𝑑𝑥 𝟏 1 1 −4 1
𝐬𝐢𝐧𝟐 𝒙 (iii) ∫ 6−4𝑥 𝑑𝑥 = −4 ∫ 6−4𝑥 𝑑𝑥 = −4 log|6 − 4𝑥| +𝑐
= 𝑐𝑜𝑡 𝑥 + 𝑐 𝟔 𝟒𝒙
𝒙
𝒕𝒂𝒏𝒙 4.(i) 𝐬𝐞𝐜 𝟐 ∫ sec 𝑑𝑥 = 𝑡𝑎𝑛 + 𝑐 ⟹ 5𝑡𝑎𝑛 + 𝑐
(ii) ∫ 𝑑𝑥 = ∫ tan 𝑥 sec 𝑥 𝑑𝑥 𝟓 /
𝒄𝒐𝒔 𝒙
(ii) cosec(5x+3) cot(5x+3)
= sec 𝑥 + 𝑐
𝒄𝒐𝒔 𝒙 ∫ 𝑐𝑜𝑠𝑒𝑐(5𝑥 + 3)𝑐𝑜𝑡 (5𝑥 + 3)𝑑𝑥 = − 𝑐𝑜𝑠𝑒𝑐(5𝑥 + 3) + 𝑐
(iii) ∫ 𝑑𝑥 = ∫ 𝑑𝑥
𝐬𝐢𝐧𝟐 𝒙 (iii) 30 sec(2-15x) tan(2-15x)
= ∫ cot 𝑥 𝑐𝑜𝑠𝑒𝑐 𝑥 𝑑𝑥
∫ 30 𝑠𝑒𝑐(2 − 15𝑥) 𝑡𝑎𝑛(2 − 15𝑥)𝑑𝑥
= −𝑐𝑜𝑠𝑒𝑐 𝑥 + 𝑐 = 30 ∫ 𝑠𝑒𝑐(2 − 15𝑥) 𝑡𝑎𝑛(2 − 15𝑥)𝑑𝑥
𝟏
(iv) ∫ 𝑑𝑥 = ∫ sec 𝑥 𝑑𝑥 1
𝐜𝐨𝐬𝟐 𝒙 = 30. 𝑠𝑒𝑐(2 − 15𝑥) + 𝑐
= 𝑡𝑎𝑛 𝑥 + 𝑐 −15
= −2 𝑠𝑒𝑐(2 − 15𝑥) + 𝑐
𝒙
𝟏 𝟏 𝒙 𝟕
5.(i) ∫ dx = sin 4𝑥 + 𝑐 6. 𝒄𝒐𝒔 −𝟒 + + 𝒆𝟓 𝟑
𝟏 (𝟒𝒙)𝟐 ( ) 𝟑 𝟑 𝟕𝒙 𝟗
𝟏 1 𝑥 7
(ii) ∫ dx = sin 9𝑥 + 𝑐 = 𝑐𝑜𝑠 −4 + +𝑒 𝑑𝑥
𝟏 𝟖𝟏𝒙𝟐 ( ) 3 3 7𝑥 + 9
𝟏 1 𝑥 7
(ii) ∫ dx = tan 6𝑥 + 𝑐 = 𝑐𝑜𝑠 − 4 𝑑𝑥 + 𝑑𝑥 + 𝑒 𝑑𝑥
𝟏 (𝟔𝒙)𝟐 3 3 7𝑥 + 9
1 𝑥 1
= 𝑐𝑜𝑠 − 4 𝑑𝑥 + 7 𝑑𝑥 + 𝑒 𝑑𝑥
3 3 7𝑥 + 9
1 𝑥 1
= 𝑐𝑜𝑠 − 4 𝑑𝑥 + 7 𝑑𝑥 + 𝑒 𝑑𝑥
3 3 7𝑥 + 9
1 1 𝑥 1 1
= 𝑐𝑜𝑠 − 4 + 7 log|7𝑥 + 9| + 𝑒 +𝑐
31 3 7 1/5
EXERCISE 11.3 = 𝑐𝑜𝑠
3
𝑥
− 4 + log|7𝑥 + 9| + 5 𝑒 +𝑐
𝟓 3
1.(𝒙 + 𝟒) + 𝟓
− 𝐜𝐨𝐬𝐞𝐜 𝟐 (𝟑𝒙 − 𝟏)
(𝟐 𝟓𝒙)𝟒
5 EXERCISE 11.4
(𝑥 + 4) + − cosec (3𝑥 − 1) 𝑑𝑥
(2 − 5𝑥) 01.If f’(x) = 4x-5 and f(2)=1 , find f(x).
5 We know that , ∫ 𝑓 (𝑥)𝑑𝑥 = 𝑓(𝑥)
= (𝑥 + 4) 𝑑𝑥 + 𝑑𝑥 − cosec (3𝑥 − 1) 𝑑𝑥
(2 − 5𝑥) 4𝑥
𝑓 (𝑥)𝑑𝑥 = (4𝑥 − 5) 𝑑𝑥 = − 5𝑥 + 𝑐 = 𝑓(𝑥)
= (𝑥 + 4) 𝑑𝑥 + 5 (2 − 5𝑥) 𝑑𝑥 − cosec (3𝑥 − 1) 𝑑𝑥 2
𝑓(𝑥) = 2𝑥 − 5𝑥 + 𝑐 − − − − − (1)
(𝑥 + 4) 1 (2 − 5𝑥) 1
= +5 − (−cot(3𝑥 − 1)) + 𝑐 Given that 𝑓(2) = 1
5+1 −5 −4 + 1 3
(𝑥 + 4) (2 − 5𝑥) 1 𝑓(2) = 2(2) − 5(2) + 𝑐 = 1 ⟹ 𝑐 = 3
= − + cot(3𝑥 − 1) + 𝑐 Substituting in (1)
6 −3 3
(𝑥 + 4) 1 1 𝑓(𝑥) = 2𝑥 − 5𝑥 + 3
= + + cot(3𝑥 − 1) + 𝑐 02. If f’(x) = 9x2-6x and f(0)=-3 , find f(x).
6 3(2 − 5𝑥) 3
𝟐𝟒 We know that , ∫ 𝑓 (𝑥)𝑑𝑥 = 𝑓(𝑥)
2. 𝟒𝒄𝒐𝒔(𝟓 − 𝟐𝒙) + 𝟗𝒆𝟑𝒙 𝟔
+
𝟔 𝟒𝒙 9𝑥 6𝑥
24 𝑓 (𝑥)𝑑𝑥 = (9𝑥 − 6𝑥) 𝑑𝑥 = − + 𝑐 = 𝑓(𝑥)
4𝑐𝑜𝑠(5 − 2𝑥) + 9𝑒 + 𝑑𝑥 3 2
6 − 4𝑥 𝑓(𝑥) = 3𝑥 − 3𝑥 + 𝑐 − − − − − (1)
=∫ 4𝑐𝑜𝑠(5 − 2𝑥) 𝑑𝑥 + ∫ 9𝑒 𝑑𝑥 -6 ∫ dx Given that 𝑓(0) = −3
=4 ∫ 𝑐𝑜𝑠(5 − 2𝑥) 𝑑𝑥 + 9∫ 𝑒 𝑑𝑥 - 6 log |6-4x|+c 𝑓(0) = 0 − 0 + 𝑐 = −3 ⟹ 𝑐 = −3
Substituting in (1)
=4 𝑠𝑖𝑛(5 − 2𝑥) + 9 𝑒 − 6 log |6 − 4x| + c
𝑓(𝑥) = 3𝑥 − 3𝑥 − 3
= - 2𝑠𝑖𝑛(5 − 2𝑥) + 3 𝑒 − 6 log |6 − 4x| + c 𝑓(𝑥) = 3(𝑥 − 𝑥 − 1)
𝒙
3.𝐬𝐞𝐜 𝟐 𝟓 + 𝟏𝟖 𝒄𝒐𝒔𝟐𝒙 + 𝟏𝟎 𝒔𝒆𝒄(𝟓𝒙 + 𝟑)𝒕𝒂𝒏 (𝟓𝒙 + 𝟑) 03. If f’’(x) = 12x-6 and f(1)=30, f’(1)=5 find f(x).
𝑥 We know that , ∫ 𝑓 (𝑥)𝑑𝑥 = 𝑓′(𝑥)
sec2 + 18 𝑐𝑜𝑠2𝑥 + 10 𝑠𝑒𝑐(5𝑥 + 3)𝑡𝑎𝑛 (5𝑥 + 3) 𝑑𝑥
5 12𝑥
=∫ sec2
𝑥
𝑑𝑥 + 18 ∫ cos 2𝑥 𝑑𝑥 + 10 ∫ 𝑠𝑒𝑐(5𝑥 + 3)𝑡𝑎𝑛 (5𝑥 + 3) 𝑑𝑥 𝑓 ′(𝑥)𝑑𝑥 = (12𝑥 − 6) 𝑑𝑥 = − 6𝑥 + 𝑐 = 𝑓′(𝑥)
5 2
1 𝑥 1 1 𝑓′(𝑥) = 6𝑥 − 6𝑥 + 𝑐 − − − − − (1)
= tan + 18 . sin 2𝑥 + 10. sec(5𝑥 + 3) + 𝑐 Given that 𝑓′(1) = 5
1/5 5 2 5
= 5 tan + 9 sin 2𝑥 + 2 sec(5𝑥 + 3) + 𝑐 𝑓′(1) = 6 − 6 + 𝑐 = 5 ⟹ 𝑐 = 5
𝑓′(𝑥) = 6𝑥 − 6𝑥 + 5
𝟖 𝟐𝟕 𝟏𝟓
4. + − We know that , ∫ 𝑓 (𝑥)𝑑𝑥 = 𝑓(𝑥)
𝟏 𝟗𝒙𝟐 𝟏 𝟐𝟓𝒙𝟐
𝟏 (𝟒𝒙)𝟐 6𝑥 6𝑥
𝑓 (𝑥)𝑑𝑥 = (6𝑥 − 6𝑥 + 5 ) 𝑑𝑥 = − + 5𝑥 + 𝑘 = 𝑓(𝑥)
8 15 27 3 2
= + − 𝑑𝑥 𝑓(𝑥) = 2𝑥 − 3𝑥 + 5𝑥 + 𝑘 − − − − − (2)
1 − (4𝑥) √1 − 9𝑥 1 + 25𝑥
1 1 1 Given that 𝑓(1) = 30
=8 𝑑𝑥 + 27 𝑑𝑥 − 15 𝑑𝑥 𝑓(1) = 2 − 3 + 5 + 𝑘 = 30 ⟹ 𝑐 = 26
1 − (4𝑥) 1 − (3𝑥) 1 + (5𝑥)
Substituting in (2)
=8 . sin (4𝑥) + 27. sin 3𝑥 – 15 . tan 5𝑥 +c 𝑓(𝑥) = 2𝑥 − 3𝑥 + 5𝑥 + 26
=2 sin (4𝑥) + 9sin 3𝑥 – 3 tan 5𝑥 +c 4. A ball is thrown vertically upward from the ground with
𝟔 𝟏𝟐
5. − an initial velocity of 39.2 m/sec. If the only force considered
𝟏 (𝟑𝒙 𝟐)𝟐
𝟏 (𝟑 𝟒𝒙))𝟐 is that attributed to the acceleration due to gravity, find
6 12 (i) how long will it take for the ball to strike the ground?
= − 𝑑𝑥 (ii) the speed with which will it strike the ground? and
1 + (3𝑥 + 2) 1 − (3 − 4𝑥)) (iii) how high the ball will rise?
6 12 Given initial velocity u= 39.2 m/sec
= 𝑑𝑥 − 𝑑𝑥
1 + (3𝑥 + 2) 1 − (3 − 4𝑥)) Distance 𝑥 = 𝑢𝑡 − 𝑔𝑡
=6. tan (3𝑥 + 2) − 12. sin (3 − 4𝑥) + 𝑐 1
𝑥 = 39.2𝑡 − (9.8)𝑡
=2 tan (3𝑥 + 2) + 3 sin (3 − 4𝑥) + 𝑐 2
𝑥 = 39.2𝑡 − 4.9𝑡 3.(𝟐𝒙 − 𝟓)(𝟑𝟔 + 𝟒𝒙)
(i) When the ball strikes the ground , x=0
(2𝑥 − 5)(36 + 4𝑥)𝑑𝑥 = (72𝑥 + 8𝑥 − 180 − 20𝑥)𝑑𝑥
39.2𝑡 − 4.9𝑡 = 0
−4.9𝑡 = −39.2𝑡 = ∫(8𝑥 + 52𝑥 − 180)𝑑𝑥
.
𝑡=
.
= + − 180𝑥 + 𝑐
t = 8 sec
= + 26𝑥 − 180𝑥 + 𝑐
(ii) To find the speed with which strike the
ground substitute t=8 sec in velocity. 4.𝐜𝐨𝐭𝟐 𝒙 + 𝐭𝐚𝐧𝟐 𝒙
𝑥 = 39.2𝑡 − 4.9𝑡 (cot 𝑥 + tan 𝑥)𝑑𝑥 = (coec 𝑥 − 1 + sec 𝑥 − 1) 𝑑𝑥
Velocity = = 39.2 − 9.8𝑡
=∫(coec 𝑥 + sec 𝑥 − 2) 𝑑𝑥
If t=8 sec V=39.2 – 9.8 (8) = 39.2 – 78.4 = - 39.2 m/sec
= -cot x+tanx -2x+c
(iii) At maximum height velocity is zero. 𝒄𝒐𝒔 𝟐𝒙 𝒄𝒐𝒔 𝟐𝜶
𝑉 = 39.2 − 9.8𝑡 = 0 ⟹ 𝑡 = 4 𝑠𝑒𝑐 5.
𝒄𝒐𝒔 𝒙 𝒄𝒐𝒔 𝜶
When t=4 ball reach the maximum height. 𝑑𝑥 = ∫ 𝑑𝑥

When t=4 sec x = 39.2(4) – 4.9(16)
= 156.8 - 78.4 = 78.4 m. =∫ 𝑑𝑥
5. A wound is healing in such a way that t days since =2 ∫ 𝑑𝑥
Sunday the area of the wound has been decreasing at a ( )( )
𝟑 =2 ∫ 𝑑𝑥
rate of 𝟐 cm per day. If on Monday the area of the
2
(𝒕 𝟐) =2 ∫(cos 𝑥 + cos 𝛼) 𝑑𝑥
wound was 2cm2. = 2 [2 sinx + xcos 𝛼]+c [Where cos 𝛼 is a constant
(i) What was the area of the wound on Sunday? 𝒄𝒐𝒔 𝟐𝒙
6.
(ii) What is the anticipated area of the wound on 𝐬𝐢𝐧𝟐 𝒙 𝐜𝐨𝐬𝟐 𝒙
Thursday if it continues to heal at the same rate? ∫ 𝑑𝑥 = 4 ∫ [ Multiply & divide by 4
Day t Area of the wound = 4∫( 𝑑𝑥
)
Sunday 0 (i)? = 4∫ 𝑑𝑥
Monday 1 2 cm2 ( )

Thursday 4 (ii)? = 4∫ 𝑑𝑥
Let A be the area of a healing wound in t days = 4 ∫ 𝑐𝑜𝑠𝑒𝑐 2𝑥 𝑐𝑜𝑡2𝑥 𝑑𝑥
𝟑 =4 𝑐𝑜𝑠𝑒𝑐 2𝑥 + 𝑐 ⟹ −2 𝑐𝑜𝑠𝑒𝑐 2𝑥 + 𝑐
Give that =
(𝒕 𝟐)𝟐 𝟑 𝟒 𝒄𝒐𝒔 𝒙
−3 7.
𝐬𝐢𝐧𝟐 𝒙
𝑑𝐴 = 𝑑𝑡 𝟑 + 𝟒 𝒄𝒐𝒔 𝒙 3 𝑐𝑜𝑠 𝑥
(𝑡 + 2) 𝑑𝑥 = 𝑑𝑥 + 4 𝑑𝑥
Integrating on both sides, 𝐬𝐢𝐧𝟐 𝒙 sin2 𝑥 sin2 𝑥
=∫ 3 cosec 𝑥 𝑑𝑥 + 4 ∫ 𝑐𝑜𝑡𝑥 𝑐𝑜𝑠𝑒𝑐 𝑥 𝑑𝑥
𝑑𝐴 = −3 (𝑡 + 2) 𝑑𝑡 = -3 cotx - 4cosec x + c
(𝑡+2) −2+1 𝐬𝐢𝐧𝟐 𝒙
𝐴 = −3 +𝑐 8.
𝟏 𝒄𝒐𝒔 𝒙
3 sin 𝑥 1 − cos 𝑥
𝐴= + 𝑐 -------(1) 𝑑𝑥 = 𝑑𝑥
𝑡+2
3 1 + 𝑐𝑜𝑠 𝑥 1 + cos 𝑥
If t=1 , A = 2 ⟹ 2 = +𝑐 ⟹ 𝑐 =1 ( )( )
3
1+2 =∫ 𝑑𝑥
(1) ⟹ 𝐴 = +1
𝑡+2 =∫(1 − cos 𝑥)𝑑𝑥
(i) If t=0 ; A=? = 𝑥 − sin 𝑥 + 𝑐
3
(1) ⟹ 𝐴 = + 1 ⟹ 2.5 𝑠𝑞. 𝑐𝑚 9.
𝒔𝒊𝒏 𝟒𝒙
0+2
𝒔𝒊𝒏 𝒙
(ii) If t=4 ; A=? 𝑠𝑖𝑛 4𝑥 2 sin 2𝑥 cos 2𝑥
(1) ⟹ 𝐴 =
3
+ 1 ⟹ 1.5 𝑠𝑞. 𝑐𝑚 𝑑𝑥 = 𝑑𝑥
4+2 𝑠𝑖𝑛 𝑥 sin 𝑥
( )
=∫ 𝑑𝑥
EXERCISE 11.5 = 2 ∫ 2 𝑐𝑜𝑠𝑥 𝑐𝑜𝑠2𝑥 𝑑𝑥
𝒙𝟑 𝟒𝒙𝟐 𝟑𝒙 𝟐
1. 2 𝐶𝑜𝑠𝐴 𝐶𝑜𝑠 𝐵 = cos(𝐴 + 𝐵) + 𝐶𝑜𝑠(𝐴 − 𝐵)
𝒙𝟐
𝑥 + 4𝑥 − 3𝑥 + 2 𝑥 4𝑥 3𝑥 2 =2 ∫ cos(𝑥 + 2𝑥) + cos(𝑥 − 2𝑥) 𝑑𝑥
𝑑𝑥 = ( + − + ) 𝑑𝑥 =2 ∫ cos 3𝑥 + cos(−𝑥) 𝑑𝑥
𝑥 𝑥 𝑥 𝑥 𝑥
=2 ∫ cos 3𝑥 + cos 𝑥 𝑑𝑥
=∫ 𝑥 + 4 − + 2𝑥 𝑑𝑥
=2 + 𝑠𝑖𝑛𝑥 + 𝑐
= + 4𝑥 − log|𝑥| − + 𝑐 10.cos 3x cos 2x
𝟐
2. √𝒙 +
𝟏 ∫(cos 3x cos 2x) dx = ∫ 2cos 3x cos 2x dx
√𝒙
2 𝐶𝑜𝑠𝐴 𝐶𝑜𝑠 𝐵 = cos(𝐴 + 𝐵) + 𝐶𝑜𝑠(𝐴 − 𝐵)
1 1 1
√𝑥 + 𝑑𝑥 = 𝑥 + 2√𝑥 + 𝑑𝑥 = ∫ cos(3𝑥 + 2𝑥) + cos(3𝑥 − 2𝑥) dx
√𝑥 √𝑥 𝑥
= ∫ cos 5𝑥 + cos 𝑥 dx
= ∫ 𝑥 + 2 + 𝑑𝑥
= + 𝑠𝑖𝑛𝑥 + 𝑐
= + 2𝑥 + log|𝑥| + 𝑐
11.𝐬𝐢𝐧𝟐 (𝟓𝒙) 17.
𝒙 𝟏
( ) (𝒙 𝟐)(𝒙 𝟑)
∫ sin (5𝑥) 𝑑𝑥 = ∫ 𝑑𝑥 [sin 𝑥 = ∫( 𝑑𝑥 = ∫ + 𝑑𝑥 [By using Partial fraction
)( )
= ∫(1 − 𝑐𝑜𝑠10𝑥)𝑑𝑥 = −∫ 𝑑𝑥 + 2 ∫ 𝑑𝑥
= 𝑥− +𝑐 ∫
( )
𝑑𝑥 = log|𝑓(𝑥)| + 𝑐
( )
𝟏 𝒄𝒐𝒔 𝟒𝒙
12. = - log|x+3|+2 log|x+3|+c
𝒄𝒐𝒕𝒙 𝒕𝒂𝒏 𝒙
𝟏
1 + 𝑐𝑜𝑠 4𝑥
𝑑𝑥 =
1 + 2 cos 2𝑥 − 1
𝑑𝑥
18. (𝒙
𝟏)(𝒙 𝟐)𝟐
𝑐𝑜𝑡𝑥 − 𝑡𝑎𝑛 𝑥 𝑐𝑜𝑠 𝑥 𝑠𝑖𝑛 𝑥 1 1 1 1

𝑠𝑖𝑛 𝑥 𝑐𝑜𝑠 𝑥 𝑑𝑥 = [ − − ]𝑑𝑥
(𝑥 − 1)(𝑥 + 2) 9(𝑥 − 1) 9(𝑥 + 2) 3(𝑥 + 2)
=∫ 𝑑𝑥 [By using Partial fraction
=∫ ( )
𝑑𝑥 − ∫ ( )
𝑑𝑥 − ∫ ( ) 𝑑𝑥
=∫ 𝑑𝑥 = ∫( 𝑑𝑥 − ∫ ( 𝑑𝑥 − ∫ 𝑑𝑥
) ) ( )
=∫ 𝑑𝑥 = ∫( 𝑑𝑥 − ∫ ( 𝑑𝑥 − ∫(𝑥 + 2) 𝑑𝑥
) )
= ∫ 𝑠𝑖𝑛 2𝑥 cos 2𝑥 𝑑𝑥 = log|𝑥 − 1| − log|𝑥 + 2| −
( )
+𝑐
= ∫ 2 𝑠𝑖𝑛 2𝑥 cos 2𝑥 𝑑𝑥 = log|𝑥 − 1| − log|𝑥 + 2| + +𝑐
( )
𝟑𝒙 𝟗
= ∫ 𝑠𝑖𝑛 4𝑥 𝑑𝑥 19.
(𝒙 𝟏)(𝒙 𝟐)(𝒙𝟐 𝟏)
3𝑥 − 9 −1 1 3
= +𝑐 𝑑𝑥 = + + 𝑑𝑥
(𝑥 − 1)(𝑥 + 2)(𝑥 + 1) 𝑥−1 𝑥+2 𝑥 +1
= +𝑐 [By using Partial fraction

13.𝒆 𝒙 𝒍𝒐𝒈 𝒂
𝒆 𝒙 =− ∫ 𝑑𝑥 + ∫ 𝑑𝑥 + 3 ∫ 𝑑𝑥
∫𝑒 𝑒 𝑑𝑥 = ∫ 𝑒 𝑒 𝑑𝑥 = − log|𝑥 − 1| + log|𝑥 + 2| + 3 tan 𝑥 + 𝑐
= ∫ 𝑎 𝑒 𝑑𝑥 = log|𝑥 + 2| − log|𝑥 − 1| + 3 tan 𝑥 + 𝑐
+ 3 tan−1 𝑥 + 𝑐
|𝒙 𝟐|
= ∫(𝑎𝑒) 𝑑𝑥 = 𝒍𝒐𝒈
|𝒙 𝟏|
( ) 𝒙𝟑
= +𝑐 20.
( ) (𝒙 𝟏)(𝒙 𝟐)
14.(𝟑𝒙 + 𝟒)√𝟑𝒙 + 𝟕
∫( 𝑑𝑥 = ∫ 𝑥 + 3 + 𝑑𝑥
∫(3𝑥 + 4)(3𝑥 + 7) / 𝑑𝑥 = ∫[(3𝑥 + 7) − 3](3𝑥 + 7) /
𝑑𝑥 )( )

= ∫ (3𝑥 + 7) / − 3(3𝑥 + 7) /
𝑑𝑥 [ When dividing𝑥 by (𝑥 − 1)(𝑥 − 2)
= ∫ (3𝑥 + 7) / − 3(3𝑥 + 7) /
𝑑𝑥 =∫ 𝑥 + 3 + + 𝑑𝑥
( ) ( ) [By using partial fraction
= − 3. +𝑐
/ = + 3𝑥 − log|𝑥 − 1| + 8 log|𝑥 − 2| + 𝑐

=
( )

( )
+𝑐 Note: For Question Number 17,18,19 and 20 refer
𝟖𝟏 𝒙 𝟒𝟏 𝒙 exercise 2.8 in Ist volume about partial fractions.
15.
𝟐𝒙
8 +4 (2 ) + (2 )
𝑑𝑥 = 𝑑𝑥
2 2
=∫ 𝑑𝑥
=∫ + 𝑑𝑥
= ∫[2 +2 ] 𝑑𝑥
= ∫[2 +2 ] 𝑑𝑥
= + +𝑐
( ) ( )

= − +𝑐
( ) ( )
𝟏
16.
√𝒙 𝟑 √𝒙 𝟒
√ √
∫√ 𝑑𝑥 = ∫ 𝑥 𝑑𝑥
√ √ √ √ √
√ √
= ∫( ) ( )
𝑑𝑥
√ √
=∫ 𝑑𝑥
= ∫(𝑥 + 3) − (𝑥 − 4) 𝑑𝑥
( ) ( )
= − +𝑐

= 𝑥 (𝑥 + 3) − (𝑥 − 4) + 𝑐
= (𝑥 + 3) − (𝑥 − 4) + 𝑐
PV MATRIC HR. SEC. SCHOOL 4.
𝟏𝟎𝒙𝟗 +𝟏𝟎𝒙 𝐥𝐨𝐠 𝒆 𝟏𝟎
𝟏𝟎𝒙 +𝒙𝟏𝟎
MATHEMATICS - XI STD ∫
10𝑥9 + 10𝑥 log𝑒 10
𝒅𝒙
10𝑥 + 𝑥10
1 Let u=10𝑥 + 𝑥 10
=∫ 𝑑𝑢 𝑑𝑢
EXERCISE 11.6 𝑢 𝑑𝑥
= 10𝑥9 + 10𝑥 log𝑒 10
= log|𝑢| + 𝑐 𝑑𝑢 = (10𝑥9 + 10𝑥 log𝑒 10) 𝑑𝑥
SOME IMPORTANT RESULTS: = log|10𝑥 + 𝑥10 | + 𝑐
𝒔𝒊𝒏√𝒙
5.
√𝒙 Let u=√𝑥
𝑠𝑖𝑛√𝑥 𝑑𝑥 𝑑𝑢
=2
1
∫ 𝑑𝑥 = ∫ 𝑠𝑖𝑛√𝑥 𝑑𝑥 √𝑥
√𝑥 √𝑥 1
= ∫ 𝑠𝑖𝑛𝑢 (2𝑑𝑢) 𝑑𝑢 = 𝑑𝑥
2√𝑥
= 2 ∫ 𝑠𝑖𝑛𝑢 𝑑𝑢 𝑑𝑥
= 2(−𝑐𝑜𝑠𝑢) + 𝑐 2𝑑𝑢 =
= −2 𝑐𝑜𝑠𝑢 + 𝑐 √𝑥
= −2 𝑐𝑜𝑠 √𝑥 + 𝑐
𝒄𝒐𝒕 𝒙
6. Let u=log(𝑠𝑖𝑛𝑥)
𝒍𝒐𝒈(𝒔𝒊𝒏𝒙)
𝑐𝑜𝑡 𝑥 1 𝑑𝑢 1
∫ 𝑙𝑜𝑔(𝑠𝑖𝑛𝑥) 𝑑𝑥 =∫ 𝑙𝑜𝑔(𝑠𝑖𝑛𝑥) 𝑐𝑜𝑡 𝑥 𝑑𝑥 = sin 𝑥 cos 𝑥
𝑑𝑥
1 𝑑𝑢 = cot 𝑥 𝑑𝑥
=∫ 𝑑𝑢
𝑢
= log|𝑢| + 𝑐
= log|log(𝑠𝑖𝑛𝑥)| + 𝑐
Integrate the following with respect to x 𝒄𝒐𝒔𝒆𝒄 𝒙
𝒙 7. 𝒙 𝑥
1. 𝒍𝒐𝒈(𝒕𝒂𝒏 )
𝟐 Let u=𝑙𝑜𝑔(tan 2)
√𝟏+𝒙𝟐
Let u=1 + 𝑥 2 𝑐𝑜𝑠𝑒𝑐 𝑥
𝑥 1 ∫ 𝑑𝑥
∫ 𝑑𝑥 = ∫ 𝑥 𝑑𝑥 𝑑𝑢
= 2𝑥 𝑙𝑜𝑔(tan 𝑥) 𝑑𝑢
=
1
sec 2 ( )
𝑥 1
√1 + 𝑥 2 √1 + 𝑥 2 𝑑𝑥 2 𝑑𝑥 tan
𝑥
2 2
1 𝑑𝑢 𝑑𝑢 = 2𝑥 𝑑𝑥 1 2
=∫ 𝑑𝑢
= ∫ 𝑑𝑥
𝑢 1
𝑑𝑢 = tan𝑥 sec 2 2 (2) 𝑑𝑥
𝑥 1
√𝑢 2 = 𝑥 𝑑𝑥
1 −
1
2 = log|𝑢| + 𝑐 2
= ∫𝑢 2 𝑑𝑢 𝑥 𝑑𝑢 = cot
𝑥 1 1
( ) 𝑑𝑥
2 = log |𝑙𝑜𝑔(tan )| + 𝑐 2 cos2
𝑥
2
1
1
𝑢2
2 𝑥
2

=2 [ 1 ]+𝑐 𝑑𝑢 =
cos
2
𝑥
1 1
𝑥( ) 𝑑𝑥
2 sin cos2 2
2 2
= √𝑢 + 𝑐 𝑑𝑢 = 2 sin𝑥 cos𝑥 𝑑𝑥
1

= √1 + 𝑥 2 + 𝑐 2 2

𝒙𝟐
1
2. 𝑑𝑢 = 𝑑𝑥
𝟏+𝒙𝟔 Let u=𝑥 3 sin 𝑥
𝑥2 1 𝑑𝑢 𝑑𝑢 = 𝑐𝑜𝑠𝑒𝑐 𝑥 𝑑𝑥
∫ 𝑑𝑥 = ∫ 𝑥2 𝑑𝑥 = 3𝑥 2
𝑑𝑥
1 + 𝑥6 1 + (𝑥3 )2 𝒔𝒊𝒏 𝟐𝒙
1 𝑑𝑢 𝑑𝑢 = 3𝑥 2 𝑑𝑥 8.
=∫ 𝑑𝑢
𝒂𝟐 +𝒃𝟐 𝐬𝐢𝐧𝟐 𝒙
𝑠𝑖𝑛 2𝑥 Let u=𝑎2 + 𝑏 2 sin2 𝑥
1+𝑢2 3 3
= 𝑥 2 𝑑𝑥 ∫ 𝑑𝑥 𝑑𝑢
1 1 𝑎2 + 𝑏2 sin2 𝑥 = 2
𝑏 2 𝑠𝑖𝑛 𝑥 𝑐𝑜𝑠 𝑥
= ∫ 𝑑𝑢 1 𝑑𝑢 𝑑𝑥
3 1+𝑢2
=∫
=
1
tan−1 𝑢 + 𝑐 𝑢 𝑏2 𝑑𝑢 = 𝑏2 𝑠𝑖𝑛 2𝑥 𝑑𝑥
3 1 𝑑𝑢 𝑑𝑢
1 =∫ = 𝑠𝑖𝑛2𝑥 𝑑𝑥
= 3 tan−1(𝑥 3 ) + 𝑐 𝑢 𝑏2 2
1 1 𝑏
𝒆𝒙 −𝒆−𝒙 = 2 ∫ 𝑑𝑢
3. 𝑏 𝑢
𝒆𝒙 +𝒆−𝒙 Let u=𝑒 𝑥 + 𝑒 −𝑥 1 1
𝒆𝒙 − 𝒆−𝒙 1 = 2 log|𝑢| + 𝑐 ⟹= log|𝑎2 + 𝑏 2 sin2 𝑥| + 𝑐
∫ 𝒙 𝑑𝑥 = ∫ 𝒙 𝒙 −𝒙
𝒆 − 𝒆 𝒅𝒙 𝑑𝑢 𝑏 𝑏2
𝒆 + 𝒆−𝒙 𝒆 + 𝒆−𝒙 = 𝑒𝑥 − 𝑒−𝑥 𝐬𝐢𝐧−𝟏 𝒙
𝑑𝑥 9.
1
= ∫ 𝑑𝑢 𝑑𝑢 = 𝑒𝑥 − 𝑒−𝑥 𝑑𝑥 √𝟏−𝒙𝟐 Let u=sin−1 𝑥
𝑢 −1
sin 𝑥 𝑑𝑢 1
= log|𝑢| + 𝑐 ∫ 𝑑𝑥 = ∫ 𝑢 𝑑𝑢 𝑑𝑥
= √1−𝑥 2
= log|𝑒𝑥 + 𝑒−𝑥 | + 𝑐 √1 − 𝑥 2 1
𝑢2 𝑑𝑢 = 𝑑𝑥
Alternative Method : =
2
+𝑐 √1−𝑥2
𝑓′(𝑥) (sin−1 𝑥)2
𝐵𝑦 𝑢𝑠𝑖𝑛𝑔 𝑡ℎ𝑒 𝑓𝑜𝑟𝑚𝑢𝑙𝑎 ∫ 𝑑𝑥 = log|𝑓(𝑥)| + 𝑐 = +𝑐
𝑓(𝑥) 2
𝒆𝒙 −𝒆−𝒙
∫ 𝒆𝒙+𝒆−𝒙 𝑑𝑥 = log |𝑒𝑥 + 𝑒−𝑥 | +𝑐
√𝒙 𝑢6 𝑢8
10.
𝟏+√𝒙
Let u=1 + √𝑥 ⟹ ∫(𝑢5 − 𝑢7 )𝑑𝑢 ⟹ − +𝑐
6 8
√𝑥 𝑢−1 Then u-1=√𝑥 sin6 𝑥 sin8 𝑥
∫ 1+ 𝑥 𝑑𝑥 = ∫ 𝑢 2(𝑢 − 1)𝑑𝑢 = − +𝑐
√ 𝑑𝑢 1
(𝑢−1)2 = 6 8
= 2∫ 𝑑𝑢 𝑑𝑥 2√𝑥 𝒄𝒐𝒔 𝒙
𝑢 16.
𝑢2 −2𝑢+1 1 𝒄𝒐𝒔(𝒙−𝒂)
= 2∫ 𝑑𝑢 𝑑𝑢 = 2 𝑑𝑥 𝑐𝑜𝑠 𝑥
𝑢 √𝑥
1 ∫ 𝑑𝑥
= 2 ∫ [𝑢 − 2 + ] 𝑑𝑢
𝑢
2√𝑥 𝑑𝑢 = 𝑑𝑥 𝑐𝑜𝑠(𝑥 − 𝑎)
𝑢2 2(𝑢 − 1) 𝑑𝑢 = 𝑑𝑥 𝑐𝑜𝑠 (𝑥 − 𝑎 + 𝑎)
= 2 [ − 2𝑢 + log |𝑢|] + 𝑐 =∫ 𝑑𝑥
2 (2𝑢 − 2) 𝑑𝑢 = 𝑑𝑥 𝑐𝑜𝑠(𝑥 − 𝑎)
= 𝑢2 − 4𝑢 + 2 log|𝑢| + 𝑐 𝑐𝑜𝑠 ((𝑥 − 𝑎) + 𝑎)
=∫ 𝑑𝑥
= (1 + √𝑥)2 − 4(1 + √𝑥) + 2 log|1 + √𝑥| + 𝑐 𝑐𝑜𝑠(𝑥 − 𝑎)
𝟏 𝑐𝑜𝑠(𝑥 − 𝑎)𝑐𝑜𝑠 𝑎 − 𝑠𝑖𝑛(𝑥 − 𝑎)𝑠𝑖𝑛 𝑎
11. =∫ 𝑑𝑥
𝒙 𝒍𝒐𝒈𝒙 𝒍𝒐𝒈(𝒍𝒐𝒈𝒙) Let u=𝑙𝑜𝑔(𝑙𝑜𝑔𝑥) 𝑐𝑜𝑠(𝑥 − 𝑎)
1
∫ 𝒅𝒙 𝑑𝑢 1 1 𝑐𝑜𝑠(𝑥 − 𝑎)𝑐𝑜𝑠 𝑎 𝑠𝑖𝑛(𝑥 − 𝑎)𝑠𝑖𝑛 𝑎
𝑥 𝑙𝑜𝑔𝑥 𝑙𝑜𝑔(𝑙𝑜𝑔𝑥) = = ∫[ − ] 𝑑𝑥
1 𝑑𝑥 𝑑𝑥 𝑙𝑜𝑔𝑥 𝑥 𝑐𝑜𝑠(𝑥 − 𝑎) 𝑐𝑜𝑠(𝑥 − 𝑎)
=∫
𝑙𝑜𝑔(𝑙𝑜𝑔𝑥) 𝑥 𝑙𝑜𝑔 𝑥 1
1 𝑑𝑢 = 𝑑𝑥 = ∫[𝑐𝑜𝑠 𝑎 − 𝑠𝑖𝑛 𝑎 𝑡𝑎𝑛(𝑥 − 𝑎)] 𝑑𝑥
𝑥 𝑙𝑜𝑔𝑥
= ∫ 𝑑𝑢
𝑢 𝑑𝑥
= log|𝑢| + 𝑐 𝑑𝑢 = = ∫ 𝑐𝑜𝑠 𝑎 𝑑𝑥 − ∫ 𝑠𝑖𝑛𝑎 𝑡𝑎𝑛(𝑥 − 𝑎) 𝑑𝑥
𝑥 𝑙𝑜𝑔𝑥
= log|𝑙𝑜𝑔(𝑙𝑜𝑔𝑥)| + 𝑐
𝜶 = 𝑐𝑜𝑠 𝑎 ∫ 𝑑𝑥 − 𝑠𝑖𝑛𝑎 ∫ 𝑡𝑎𝑛(𝑥 − 𝑎) 𝑑𝑥
12. 𝜶𝜷 𝒙𝜶−𝟏 𝒆−𝜷𝒙
∫ 𝛼𝛽 𝑥𝛼−1 𝑒−𝛽𝑥
𝛼
𝑑𝑥 Let u= 𝛽𝑥 𝛼 = 𝑐𝑜𝑠 𝑎 ( 𝑥 ) − 𝑠𝑖𝑛𝑎 [𝑙𝑜𝑔|𝑠𝑒𝑐𝑥|] + 𝑐
𝑑𝑢 Integration by parts
= ∫ 𝑒−𝛽𝑥
𝛼
(𝛼𝛽 𝑥𝛼−1 𝑑𝑥) = 𝛼𝛽 𝑥𝛼−1 If u and v are two differentiable functions, then
𝑑𝑥
𝑑𝑢 = 𝛼𝛽 𝑥𝛼−1 𝑑𝑥 ∫ 𝑢𝑑𝑣 = 𝑢𝑣 − ∫ 𝑣 𝑑𝑢
= ∫ 𝑒 −𝑢 𝑑𝑢
𝑒 −𝑢 𝛼 The success of this method depends on the choice of u.
= + 𝑐 ⟹ −𝑒 −𝑢 + 𝑐 ⇒ −𝑒 −𝛽𝑥 + 𝑐
−1 The choice of u is based on the code word ”ILATE”
13.𝒕𝒂𝒏𝒙 √𝒔𝒆𝒄 𝒙 2 I- For Inverse trigonometric functions
Let 𝑢 = sec 𝑥
∫ 𝑡𝑎𝑛𝑥 √𝑠𝑒𝑐 𝑥 𝑑𝑥 Examples: tan−1 𝑥 , sin−1 𝑥
then u=√𝑠𝑒𝑐 𝑥
𝑑𝑢 L- For logarithmic functions
= ∫ 2 𝑑𝑢 2𝑢 = sec 𝑥 tan 𝑥 Example: 𝑙𝑜𝑔𝑥
𝑑𝑥
2𝑢 𝑑𝑢 = sec 𝑥 tan 𝑥 𝑑𝑥 A- For Algebraic functions
= 2 ∫ 𝑑𝑢
2√𝑠𝑒𝑐 𝑥 𝑑𝑢 = sec 𝑥 tan 𝑥 𝑑𝑥 Examples: 𝑥, √𝑥
= 2𝑢 + 𝑐 2 𝑑𝑢 = √𝑠𝑒𝑐 𝑥 tan 𝑥 𝑑𝑥 T- For trigonometric functions
= 2√𝑠𝑒𝑐 𝑥 + 𝑐 Examples: 𝑠𝑖𝑛𝑥, cos 𝑥
14.𝒙(𝟏 − 𝒙)𝟏𝟕 E- For Exponential functions
Let 𝑢= 1 − 𝑥
∫ 𝒙(𝟏 − 𝒙) 𝟏𝟕
𝒅𝒙 Examples: 𝑠𝑖𝑛𝑥, cos 𝑥
Then x=1-u
= ∫(1 − 𝑢)𝑢17 (−𝑑𝑢)
𝑑𝑢 EXERCISE 11.7
= −1
𝑑𝑥 1.(i) 𝟗𝒙𝒆𝟑𝒙
= − ∫(𝑢17 − 𝑢18 ) 𝑑𝑢 𝑑𝑢 = − 𝑑𝑥 ∫ 9𝑥𝑒 3𝑥 𝑑𝑥 =9 ∫ 𝑥 𝑒 3𝑥 𝑑𝑥
−𝑑𝑢 = 𝑑𝑥
𝑢18 𝑢19 𝐼𝐿 𝐴 𝑇𝐸 ; 𝐿𝑒𝑡 𝑢 = 𝑥
= −[ − ]+𝑐
18 19 𝐿𝑒𝑡 𝑢=𝑥 𝑎𝑛𝑑 𝑑𝑣 = 𝑒 3𝑥 𝑑𝑥
(1 − 𝑥 )18 (1 − 𝑥 )19
= −[ − ]+𝑐 𝑑𝑢
18 19 𝑑𝑥
=1 ∫ 𝑑𝑣 = ∫ 𝑒 3𝑥 𝑑𝑥
(1 − 𝑥 )19 (1 − 𝑥 )18
= − +𝑐 𝑒 3𝑥
19 18 𝑑𝑢 = 𝑑𝑥 𝑣= 3
15.𝐬𝐢𝐧𝟓 𝒙 𝐜𝐨𝐬 𝟑 𝒙
Let 𝑢= sin 𝑥
∫ sin5 𝑥 cos 3 𝑥 𝑑𝑥 𝑑𝑢 9 ∫ 𝑥 𝑒 3𝑥 𝑑𝑥 = 9 [𝑢𝑣 − ∫ 𝑣𝑑𝑢]
= cos 𝑥
𝑑𝑥 𝑒3𝑥 𝑒3𝑥
= ∫ sin5 𝑥 cos 2 𝑥 𝑐𝑜𝑠𝑥 𝑑𝑥 𝑑𝑢 = cos 𝑥 𝑑𝑥
= 9 (𝑥 ( 3
) − ∫ 3
𝑑𝑥) + 𝑐
𝑒3𝑥 𝑒3𝑥
= ∫ sin5 𝑥 (1 − sin2 𝑥)𝑐𝑜𝑠𝑥 𝑑𝑥 = 9 (𝑥 ( 3 ) − 9 ) + 𝑐
= 3𝑥𝑒3𝑥 − 𝑒3𝑥 + 𝑐
= ∫(sin5 𝑥 − sin7 𝑥) 𝑐𝑜𝑠𝑥 𝑑𝑥 = 𝑒3𝑥 (3𝑥 − 1) + 𝑐
1.(ii) 𝒙 𝒔𝒊𝒏 𝟑𝒙 2.(ii) 𝟐𝟕 𝒙𝟐 𝒆𝟑𝒙
∫ 𝑥 𝑠𝑖𝑛3𝑥 𝑑𝑥 = ∫ 27 𝑥 2 𝑒 3𝑥 𝑑𝑥 = 27 ∫ 𝑥 2 𝑒 3𝑥 𝑑𝑥
𝐼𝐿 𝐴 𝑇𝐸 ; 𝐿𝑒𝑡 𝑢 = 𝑥
𝐼𝐿 𝐴 𝑇𝐸 ; 𝐿𝑒𝑡 𝑢 = 𝑥 2
𝐿𝑒𝑡 𝑢=𝑥 𝑎𝑛𝑑 𝑑𝑣 = 𝑠𝑖𝑛3𝑥 𝑑𝑥
𝑑𝑢 𝐿𝑒𝑡 𝑢 = 𝑥 2 𝐿𝑒𝑡 𝑑𝑣 = 𝑒 3𝑥 𝑑𝑥
=1 ∫ 𝑑𝑣 = ∫ 𝑠𝑖𝑛3𝑥 𝑑𝑥
𝑑𝑥 ∫ 𝑑𝑣 = ∫ 𝑒 3𝑥 𝑑𝑥
− cos 3𝑥 𝑢′ = 2𝑥 𝑒 3𝑥
𝑑𝑢 = 𝑑𝑥 𝑣= 𝑣=
3 3
𝑢′′ = 2
𝑒 3𝑥
∫ 𝑥 𝑠𝑖𝑛3𝑥 𝑑𝑥 = [𝑢𝑣 − ∫ 𝑣𝑑𝑢] 𝑣1 =
9
− cos 3𝑥 − cos 3𝑥
= (𝑥 ( )−∫ 𝑑𝑥) + 𝑐 𝑒 3𝑥
3 3 𝑣2 =
=
−𝑥 cos 3𝑥
( 3 ) + ∫ 3 𝑑𝑥
cos 3𝑥
+𝑐 27
27 ∫ 𝑥 2 𝑒 3𝑥 𝑑𝑥 = 27 ∫ 𝑢 𝑑𝑣 = 27[𝑢𝑣 − 𝑢 𝑣1 + 𝑢 𝑣2] + 𝑐
′ ′′
−𝑥 cos 3𝑥 sin 3𝑥
= 3
+ 9 +c [By using Bernoulli’s formula
2 𝑒3𝑥 𝑒3𝑥 𝑒3𝑥
1.(iii) 𝟐𝟓𝒙𝒆 −𝟓𝒙 = 27 [𝑥 (
3
) − 2𝑥 ( ) + 2 ( )] + 𝑐
9 27
∫ 25𝑥𝑒 −5𝑥 𝑑𝑥 =25 ∫ 𝑥 𝑒 −5𝑥 𝑑𝑥 = 9𝑥 2 𝑒3𝑥 − 6𝑥𝑒3𝑥 + 2𝑒3𝑥 + 𝑐
𝐼𝐿 𝐴 𝑇𝐸 ; 𝐿𝑒𝑡 𝑢 = 𝑥 = 𝑒 3𝑥 (9𝑥 2 − 6𝑥 + 2) + 𝑐
2.(iii) 𝒙 𝒄𝒐𝒔𝒙
𝟐
𝐿𝑒𝑡 𝑢=𝑥 𝑎𝑛𝑑 𝑑𝑣 = 𝑒 −5𝑥 𝑑𝑥
𝑑𝑢 ∫ 𝒙𝟐 𝒄𝒐𝒔𝒙 𝑑𝑥
𝑑𝑥
=1 ∫ 𝑑𝑣 = ∫ 𝑒 −5𝑥 𝑑𝑥
𝑒 −5𝑥 𝐼𝐿 𝐴 𝑇𝐸 ; 𝐿𝑒𝑡 𝑢 = 𝑥 2
𝑑𝑢 = 𝑑𝑥 𝑣=
−5 𝐿𝑒𝑡 𝑢 = 𝑥 2 𝐿𝑒𝑡 𝑑𝑣 = 𝑐𝑜𝑠𝑥 𝑑𝑥
25 ∫ 𝑥 𝑒 −5𝑥
𝑑𝑥 = 25 [𝑢𝑣 − ∫ 𝑣𝑑𝑢] ∫ 𝑑𝑣 = ∫ 𝑐𝑜𝑠𝑥 𝑑𝑥
𝑢′ = 2𝑥
𝑣 = 𝑠𝑖𝑛𝑥
𝑒−5𝑥 𝑒−5𝑥
= 25 (𝑥 ( −5 ) − ∫ −5
𝑑𝑥) + 𝑐 𝑢′′ = 2 𝑣1 = −𝑐𝑜𝑠𝑥
𝑒−5𝑥 𝑒−5𝑥 𝑣2 = −𝑠𝑖𝑛𝑥
= 25 (𝑥 ( )+ )+𝑐
−5 −25
∫ 𝒙𝟐 𝒄𝒐𝒔𝒙 𝑑𝑥 = ∫ 𝑢 𝑑𝑣 = [𝑢𝑣 − 𝑢′ 𝑣1 + 𝑢′′ 𝑣2] + 𝑐
= −5𝑥𝑒−5𝑥 − 𝑒−5𝑥 + 𝑐
= −𝑒−5𝑥 (5𝑥 + 1) + 𝑐 [By using Bernoulli’s formula
= [𝑥 2 (𝑠𝑖𝑛𝑥) − 2𝑥(−𝑐𝑜𝑠𝑥) + 2(−𝑠𝑖𝑛𝑥)] + 𝑐
1.(iv) 𝒙 𝒔𝒆𝒄 𝒙 𝒕𝒂𝒏 𝒙
= 𝑥 2 𝑠𝑖𝑛𝑥 + 2𝑥 𝑐𝑜𝑠𝑥 − 2𝑠𝑖𝑛𝑥 + 𝑐
∫ 𝑥 𝑠𝑒𝑐𝑥 𝑡𝑎𝑛𝑥 𝑑𝑥 =
2.(iv) 𝒙𝟑 𝒔𝒊𝒏𝒙
𝐼𝐿 𝐴 𝑇𝐸 ; 𝐿𝑒𝑡 𝑢 = 𝑥
∫ 𝒙𝟑 𝒔𝒊𝒏𝒙 𝑑𝑥
𝐿𝑒𝑡 𝑢=𝑥 𝑎𝑛𝑑 𝑑𝑣 = 𝑠𝑒𝑐𝑥 𝑡𝑎𝑛𝑥 𝑑𝑥
𝑑𝑢 𝐼𝐿 𝐴 𝑇𝐸 ; 𝐿𝑒𝑡 𝑢 = 𝑥 3
=1 ∫ 𝑑𝑣 = ∫ 𝑠𝑒𝑐𝑥 𝑡𝑎𝑛𝑥 𝑑𝑥
𝑑𝑥
𝑑𝑢 = 𝑑𝑥 𝑣 = 𝑠𝑒𝑐𝑥 𝐿𝑒𝑡 𝑢 = 𝑥 3 𝐿𝑒𝑡 𝑑𝑣 = 𝑠𝑖𝑛𝑥 𝑑𝑥
𝑢′ = 3𝑥 2 ∫ 𝑑𝑣 = ∫ 𝑠𝑖𝑛𝑥 𝑑𝑥
∫ 𝑥 𝑠𝑖𝑛3𝑥 𝑑𝑥 = [𝑢𝑣 − ∫ 𝑣𝑑𝑢] 𝑢′′ = 6𝑥 𝑣 = −𝑐𝑜𝑠𝑥
= (𝑥 𝑠𝑒𝑐𝑥 − ∫ 𝑠𝑒𝑐𝑥 𝑑𝑥) + 𝑐 𝑢′′′ = 6 𝑣1 = −𝑠𝑖𝑛𝑥
= 𝑥 𝑠𝑒𝑐𝑥 − log|𝑠𝑒𝑐𝑥 + 𝑡𝑎𝑛𝑥| + 𝑐 𝑣2 = 𝑐𝑜𝑠𝑥
2.(i) xlogx 𝑣3 = 𝑠𝑖𝑛𝑥
∫ 𝑥 𝑙𝑜𝑔𝑥 𝑑𝑥 = ∫ 𝒙𝟑 𝒔𝒊𝒏𝒙 𝑑𝑥 = ∫ 𝑢 𝑑𝑣 = [𝑢𝑣 − 𝑢′ 𝑣1 + 𝑢′′ 𝑣2 − 𝑢′′′ 𝑣3] + 𝑐
𝐼 𝐿 𝐴𝑇𝐸 ; 𝐿𝑒𝑡 𝑢 = 𝑙𝑜𝑔𝑥 [By using Bernoulli’s formula
𝐿𝑒𝑡 𝑢 = log 𝑥 𝑎𝑛𝑑 𝑑𝑣 = 𝑥 𝑑𝑥 = [𝑥3 (−𝑐𝑜𝑠𝑥) − 3𝑥2 (−𝑠𝑖𝑛𝑥) + 6𝑥(𝑐𝑜𝑠𝑥) − 6𝑠𝑖𝑛𝑥] + 𝑐
𝑑𝑢 1 =-𝑥 3 𝑐𝑜𝑠𝑥 + 3𝑥 2 𝑠𝑖𝑛𝑥 + 6𝑥 𝑐𝑜𝑠𝑥 − 6𝑠𝑖𝑛𝑥 + 𝑐
= ∫ 𝑑𝑣 = ∫ 𝑥 𝑑𝑥
𝑑𝑥 𝑥 𝒙 𝐬𝐢𝐧−𝟏 𝒙
1 𝑥2 3(i)
𝑑𝑢 = 𝑑𝑥 𝑣= √𝟏−𝒙𝟐
𝑥 2 𝒙 𝐬𝐢𝐧−𝟏 𝒙
Let I=∫ 𝑑𝑥
∫ log 𝑥 𝑥 𝑑𝑥 = [𝑢𝑣 − ∫ 𝑣𝑑𝑢] √𝟏−𝒙𝟐
Put sin−1 𝑥 = 𝑡 𝑥 = 𝑠𝑖𝑛𝑡
𝑥2 𝑥2 1
= (log 𝑥 2 − ∫ 2 𝑥 𝑑𝑥) + 𝑐 𝑑𝑡 1 𝑑𝑥
𝑥2 log 𝑥 𝑥2
𝑎𝑙𝑠𝑜 = ⟹ 𝑑𝑡 =
= − +𝑐 𝑑𝑥 √1 − 𝑥 2 √1 − 𝑥 2
2 4
Now I = ∫ 𝑡 sin 𝑡 𝑑𝑡 3(iv) 𝐬𝐢𝐧−𝟏 (
𝟐𝒙
)
𝟏+𝒙𝟐
𝐼𝐿 𝐴 𝑇𝐸 ; 𝐿𝑒𝑡 𝑢 = 𝑥 Let I=∫ 𝐬𝐢𝐧−𝟏 (𝟏+𝒙𝟐 ) 𝑑𝑥
𝟐𝒙

𝐿𝑒𝑡 𝑢=𝑡 𝑎𝑛𝑑 𝑑𝑣 = 𝑠𝑖𝑛𝑡 𝑑𝑡 𝑑𝜃


𝑑𝑢 Put 𝑥 = tan 𝜃 𝑎𝑙𝑠𝑜 sec 2 𝜃 =1
𝑑𝑥
𝑑𝑡
=1 ∫ 𝑑𝑣 = ∫ 𝑠𝑖𝑛𝑡 𝑑𝑡 ⟹ sec 2 𝜃 𝑑𝜃 = 𝑑𝑥
−1 2 tan 𝜃
𝑑𝑢 = 𝑑𝑡 𝑣 = −𝑐𝑜𝑠𝑡 Now I=∫ sin ( ) sec 2 𝜃 𝑑𝜃
1+tan2 𝜃

∫ 𝑥 𝑠𝑖𝑛𝑡 𝑑𝑥 = [𝑢𝑣 − ∫ 𝑣𝑑𝑢] ⟹ ∫ tan−1 (sin 2𝜃) sec 2 𝜃 𝑑𝜃


= (𝑡(−𝑐𝑜𝑠𝑡) − ∫(−𝑐𝑜𝑠𝑡) 𝑑𝑡) + 𝑐 = ∫ 2𝜃 sec 2 𝜃 𝑑𝜃 = 2 ∫ 𝜃 sec 2 𝜃 𝑑𝜃
= −𝑡𝑐𝑜𝑠𝑡 + 𝑠𝑖𝑛𝑡 + 𝑐 𝐼𝐿 𝐴 𝑇𝐸 ; 𝐿𝑒𝑡 𝑢 = 𝜃
[cost = √1 − sin2 𝑡 = √1 − 𝑥 2
𝐿𝑒𝑡 𝑢=𝜃 𝑎𝑛𝑑 𝑑𝑣 = sec2 𝜃 𝑑𝜃
= − sin−1 𝑥 √1 − 𝑥 2 + 𝑥 + 𝑐 𝑑𝑢
𝑑𝜃
=1 ∫ 𝑑𝑣 = ∫ sec2 𝜃 𝑑𝜃
𝟓 𝒙𝟐
3(ii) 𝒙 𝒆
𝟐 𝟐 𝑑𝑢 = 𝑑𝜃 𝑣 = 𝑡𝑎𝑛𝜃
Let I=∫ 𝒙𝟓 𝒆𝒙 𝑑𝑥 ⟹ ∫(𝒙𝟐 )𝟐 𝒆𝒙 𝒙 𝑑𝑥
𝑑𝑡 𝑑𝑡 2 ∫ 𝜃 sec 2 𝜃 𝑑𝜃 = 2[𝑢𝑣 − ∫ 𝑣𝑑𝑢]
Put 𝑥2 = 𝑡 𝑎𝑙𝑠𝑜 = 2𝑥 ⟹ = 𝑥 𝑑𝑥
𝑑𝑥 2
𝑑𝑡 1 = 2(𝜃(𝑡𝑎𝑛𝜃) − ∫(𝑡𝑎𝑛𝜃) 𝑑𝜃) + 𝑐
Now I=∫ 𝑡 2 𝑒𝑡 ⟹ ∫ 𝑡 2 𝑒𝑡 𝑑𝑡 = 2(𝜃 𝑡𝑎𝑛𝜃 − log |𝑠𝑒𝑐𝜃| + 𝑐
2 2
𝐼𝐿 𝐴 𝑇𝐸 ; 𝐿𝑒𝑡 𝑢 = 𝑡 2 [𝑠𝑒𝑐𝜃 = √1 + tan2 𝜃 = √1 + 𝑥 2 .
= 2[𝑥 tan−1(𝑥) − log |√1 + 𝑥 2 | + 𝑐
𝐿𝑒𝑡 𝑢 = 𝑡 2 𝐿𝑒𝑡 𝑑𝑣 = 𝑒 𝑡 𝑑𝑡
∫ 𝑑𝑣 = ∫ 𝑒 𝑡 𝑑𝑡
𝑢′ = 2𝑡
𝑣 = 𝑒𝑡
𝑢′′ = 2 𝑣1 = 𝑒 𝑡
𝑣2 = 𝑒 𝑡
1 1 1
⟹ ∫ 𝑡2 𝑒𝑡 𝑑𝑡 = ∫ 𝑢 𝑑𝑣 = [𝑢𝑣 − 𝑢′ 𝑣1 + 𝑢′′ 𝑣2] + 𝑐
2 2 2
[By using Bernoulli’s formula
1 2 𝑡
= [𝑡 (𝑒 ) − 2𝑡(𝑒𝑡 ) + 2(𝑒𝑡 )] + 𝑐
2
1 𝑡 2
=
2
𝑒 [𝑡 − 2𝑡 + 2]+c
1 2
= 2 𝑒 𝑥 [𝑥 4 − 2𝑥 2 + 2]+c
𝟖𝒙
3(iii) 𝐭𝐚𝐧−𝟏 ( )
𝟏−𝟏𝟔𝒙𝟐
𝟖𝒙 𝟐 . 𝟒𝒙
Let I=∫ 𝐭𝐚𝐧 (𝟏−𝟏𝟔𝒙𝟐 ) 𝑑𝑥 ⟹ 𝐭𝐚𝐧−𝟏 (𝟏−(𝟒𝒙)𝟐 ) 𝑑𝑥
−𝟏

𝑑𝜃
Put 4𝑥 = tan 𝜃 𝑎𝑙𝑠𝑜 sec 2 𝜃 𝑑𝑥 = 4
sec 2 𝜃 𝑑𝜃
⟹ = 𝑑𝑥
4
𝟐 𝐭𝐚𝐧 𝜽 sec2 𝜃 𝑑𝜃
Now I=∫ 𝐭𝐚𝐧−𝟏 ( )
𝟏−𝐭𝐚𝐧𝟐 𝜽 4
1
⟹ ∫ tan−1 (tan 2𝜃) sec 2 𝜃 𝑑𝜃
4
1 1
= 4 ∫ 2𝜃 sec 2 𝜃 𝑑𝜃 = 2 ∫ 𝜃 sec 2 𝜃 𝑑𝜃
𝐼𝐿 𝐴 𝑇𝐸 ; 𝐿𝑒𝑡 𝑢 = 𝜃
𝐿𝑒𝑡 𝑢=𝜃 𝑎𝑛𝑑 𝑑𝑣 = sec2 𝜃 𝑑𝜃
𝑑𝑢
𝑑𝜃
=1 ∫ 𝑑𝑣 = ∫ sec2 𝜃 𝑑𝜃
𝑑𝑢 = 𝑑𝜃 𝑣 = 𝑡𝑎𝑛𝜃
1 1
∫ 𝜃 sec 2 𝜃 𝑑𝜃 = [𝑢𝑣 − ∫ 𝑣𝑑𝑢]
2 2
1
=2 (𝜃(𝑡𝑎𝑛𝜃) − ∫(𝑡𝑎𝑛𝜃) 𝑑𝜃) + 𝑐
1
= 2 (𝜃 𝑡𝑎𝑛𝜃 − log |𝑠𝑒𝑐𝜃| + 𝑐
[𝑠𝑒𝑐𝜃 = √1 − tan2 𝜃 = √1 + (4𝑥)2
1
= [4𝑥 tan−1 (4𝑥) − log |√1 + (4𝑥)2 | + 𝑐
2

You might also like